Mcq Ophthalmology

Published on December 2016 | Categories: Documents | Downloads: 550 | Comments: 0 | Views: 1834
of 206
Download PDF   Embed   Report

Comments

Content

1) i i. LAMKIN LAMKIN A. . NEURO-OPHTHALMOLOGY NEURO-OPHTHALMOLOGY L1 . Photostress recovery test Photostress recovery test a. This simple clinical test may help to differentiate central visual loss cause d by a macular lesion from that derived from optic neuropathy. Each eye is tested separately. b. Best corrected visual acuity is measured (the test is accurate only with acui ty of 20/80 or better), after which the patient is instructed to gaze directly into a strong light (eg, direct ophth almoscope or slit-lamp beam) held 2-3 cm from the affected eye. c. The patient is then directed to read the previously measured best acuity line as soon as possible. d. Normal recovery time is typically 45-60 seconds, and this is maintained with optic nerve disease. e. In patients with maculopathy, however, recovery times are frequently 90-180 s econds or more. L2 A pattern visual evoked response (VER) a. The two crucial parameters used for functional evaluation include the height of the first positive or upward wave (amplitude) and the time between stimulus presentation and the appearance o f this wave (latency). Implicit time, which is the time it takes to reach peak amplitude after a stimulu s is applied. b. (As opposed to a flash VER) is required for visual acuity assessment in preve rbal children. c. Although the VER is useful in establishing factitious visual loss, its reliab ility is limited by the fact that patients can produce false readings by using accommodation to fog their vision. d. Abnormalities in VER latency and amplitude have been reported in various macu lopathies and retinopathies, and, therefore, these features cannot distinguish optic neuropath y from retinal disorders with complete reliability. L3 Although OKN, the rocking mirror test and base up prism can help discover fac titious monocular visual loss, these tests are not sensitive enough to diagnose factitious visual deficit at 20/100 l evel. For mild visual deficit, a fogging refraction, stereo acuity, and red-green glasses may be useful in diagnosing fac titious visual loss. L4 An optic tract lesion may cause unilateral decrease VA if the lesion is not c omplete or if the optic nerve or chiasm is also involved. If the optic nerve or chiasm is involved, the APD is typically ip silateral. Pure optic tract lesion can produce a contralateral APD as well as a complete homonymous hemianopia. L5 In the setting of upgaze paresis, upturning of the eyes on forceful opening o f closed eyelid (intact Bell's phenomenon) implies supranuclear lesion. Upturning of the eyes upon forceful ope ning of closed eyelids is known as the Bell phenomenon. If forcefully opening closed eyelids stimulates upgaze, the final common pathway (infranuclear) for upgaze must be intact. L6 Some tissue such as cortical bone, rapidly flowing fluid (blood) and air give s no signal at all on MRI {dark (hypointense)}. T1-weighted images tend to show the anatomy well, whereas T2-wei ghted images tend to show the

pathology well. L7 The macular fibers constitute a large portion of the chiasm and decussate in the posterior chiasm. The chiasm lies approximately 1 cm (not 1 mm) above the anterior pituitary gland. L8 Optokinetic nystagmus (OKN) abnormalities indicate lesions of the parietoocci pital (slow-phase pursuit abnormalities) or the frontal lobe (fast-phase recovery abnormalities). High con gruity of visual field deficits indicates a lesion in the occipital lobe. Both partial complex seizures and formed visual hallucinations may be seen with temporal lobe lesions. Inferior nerve fibers from the superior retina course ant eriorly in Meyer's loop; therefore, lesions affecting Meyer's loop will result in pie-in-the-sky defects contralater al to the lesion. L9 Although OKN abnormalities asymmetry may occur rarely with occipital lobe les ion, this finding is generally indicative of parietal lobe lesion. L10 Causative events in the pathophysiology of optic disc oedema include: swolle n axons, intracellular fluid accumulation, and interruption of axonal transport. L11 The most typical visual field finding in acute (not chronic) papilloedema is an enlarged blind spot. Although rare unilateral papilloedema may occur, for example, if contralateral optic atrophy e xists, papilloedema may be detectable only in a viable disc. L12 Pseudotumor cerebri is characterized by (i) increased intracranial pressure on lumbar puncture, (ii) normal neuroimaging studies (although the ventricles may be small), and (iii) normal ce rebrospinal fluid (CSF). Papilledema need not be present for the diagnosis. Although the neurologic exami nation is usually normal, sixth nerve palsy may occur with increased intracranial pressure of any etiology. L13 Indications of treatment of pseudotumour cerebri are severe headache and vis ual field loss. Obesity is not an indication for treatment, although weight loss (even as little as 6% of total bo dy weight) often improve the condition. L14 The most common cause of permanent visual loss in patients with cavernous si nus-dural fistulae is open-angle glaucoma. Studies have documented that up to 80% of patients with cavernous sinu s-dural fistulae will develop ocular hypertension. Twenty-five percent of patients will develop optic disc cup ping and 20% visual field defects. Any entity that raises episcleral venous pressure can cause secondary open-angle glaucoma. L15 GCA: a. 40% of untreated patients will develop some form of permanent visual loss.

b. 65% of untreated patients will have contralateral involvement after permanent visual loss in one eye. c. The second eye may still become involved in 25% of cases despite early and ad equate steroid administration, usually within 6 days of starting treatment. Visual loss is usua lly profound and is unlikely to improve even with immediate treatment. d. 20% of patients do not have systemic symptoms (i.e. occult GCA). L16 Nonarteritic anterior ischemic optic neuropathy (NAION) is far more common t han arteritic anterior ischemic optic neuropathy (AAION) (approximately 95% vs. 5%) and patients have a lower mean age at diagnosis than patients with AAION (60 years vs. 70 years). It usually occurs in a younger age group and may resemble optic neuritis. L17 The various ways to differentiate the NAION from optic neuritis include (i) the absence of pain with eye movement, (ii) the age group affected, and (iii) delayed optic disc filling present in 75% of NAION cases (whereas filling should be normal in optic neuritis). L18 Although aspirin effective in reducing systemic vascular events and is frequ ently prescribed in patients with NAION, it doesn t appear to reduce the risk of involvement of the fellow eye. L19 Nearly 100% of patients dying of multiple sclerosis had some degree optic ne rve demyelination. L20 Ninety percent of optic gliomas occur in the first two decades of life. The percentage of patients with optic nerve glioma that have associated neurofibromatosis (NF-1) ranges from 14% to 60%. Mal ignant gliomas of the visual pathways, although rare, occur more frequently in middle-aged adults than in chi ldren. Survival averages 6 to 12 months after diagnosis. L21 Yanoff and Kanski. Gliomas of the optic nerve develop in 10 15% of affected pa tients. They can occur unilaterally or bilaterally and frequently involve the optic chiasm. L22 Optociliary vessels occur in optic nerve meningioma, CRVO, sphenoid wing men ingioma, long standing POAG, optic glioma and chronic papilloedema. L23 In contrast to optic nerve gliomas, meningiomas occur primarily in adults an d are three times more common in women. Although persons with neurofibromatosis (NF-1) have a higher incidence of meningiomas than the general population, only a few patients with meningiomas have NF-1. With contrast comput ed tomography (CT) scanning, the peripheral part of the involved optic nerve may show enhancement, resulting in the railroad track sign ( kinking is specific for optic nerve glioma). L24 Dominant (Kjer) optic neuropathy (DOA) manifests between 5 and 10 years of a ge. Visual loss may progress until the mid-teens, at which point it usually stabilizes. Color defects are alm ost universally present, and tritanopia (which can be detected with the Farnsworth-100 hue testing) is suggestive of DOA . Inheritance is naturally autosomal dominant. L25 Leber's hereditary optic neuropathy (LHON). a. All offspring of a female carrier are either affected or carriers. b. Ten percent of female carriers will be affected. c. There is generally sequential asymmetric bilateral involvement. d. A small percentage of patients will enjoy partial or complete recovery late i n their course. The incidence of

spontaneous recovery has been reported to be as high as 10%. L26 Patients with papillophlebitis have normal or near normal visual acuity. It ma y be a form of incomplete central retinal vein occlusion (CRVO) and usually resolves spontaneously within 12 month s. L27 Optic disc drusen occurs almost exclusively in white individuals. They are g enerally bilateral. L28 Morning glory anomaly F: M is 2:1. L29 Non-secreting tumors often present with visual field loss, whereas secreting tumors present with endocrine dysfunction. An exception is prolactin-secreting tumor in male patients because the decreased libido and impotence are often not reported early in their course. L30 When the causative lesion is not located at the site responsible for a clini cal sign, it is described as false localizing. Bitemporal macular hemianopia can only arise from compression of the posterior chiasm. Although most lesions responsible for this compression are parasellar, a lesion distant f rom this site (e.g., a tumor at the base of the brainstem) also may lead to chiasmal compression. This occurs if the lesion causes obstructive hydrocephalus, which enlarges the third ventricle, compressing the chiasm. L31 Precipitous aneurysmal distention caused by hemorrhage of the posterior comm unicating artery at its junction with the internal carotid artery may occur, creating a third nerve palsy. Ninety -five percent include pupillary involvement, and pain is nearly always present, although not universally. Furthe rmore, many third nerve palsies resulting from diabetic vasculopathy also can be intensely painful. L32 Third nerve aberrant regeneration never occurs with diabetic oculomotor neur opathy. Aberrant regeneration of the third nerve implies another etiology, such as aneurysm, tumor, inflammation, or trauma. L33 The most common cause of acquired fourth nerve palsy in adult is trauma. L34 The three-step test is useful for diagnosis but does not differentiate betwe en congenital and acquired trochlear nerve palsy. Large vertical fusional amplitudes (>5 prism diopters) (greater tha n 3.. AAO) and facial asymmetry from childhood head tilting suggest a decompensated congenital lesion. L35 Congenital fourth nerve palsies may be revealed by old photographs that show head tilting or a large vertical fusional range.

L36 35-year-old Chinese man presents with right ptosis and an abduction deficit, facial hypesthesia, and keratoconjunctivitis sicca in the right eye. This picture is highly suggestive o f nasopharyngeal carcinoma. L37 Nasopharyngeal carcinoma can involve numerous cranial nerves because of its proximity to the prepontine basal cistern. Most frequently, the trigeminal nerve is involved, causing facial hypes thesia or facial pain. The abducens nerve is the second most common. The hallmark of nasopharyngeal carcinoma is its propensity to involve multiple cranial nerves non-contiguously. Nasopharyngeal carcinoma is common in Chinese men. The least differentiated forms are also known as Schmincke's and Regaud's tumors. L38 Horizontal saccades originate in the contralateral frontal lobe, but either hemisphere can produce ipsilateral saccades if the other hemisphere is damaged. Appropriate stimulation of the pari etal or occipital cortex can also produce contralateral saccades. L39 Oculomotor apraxia. a. Both congenital and acquired forms of oculomotor apraxia may be observed. b. Saccades are generally affected more than Pursuits. c. Horizontal movements are generally affected much more than vertical movements . d. In the acquired form, blinks are frequently used to break fixation. e. In the congenital form, children frequently use compensatory, exaggerated hea d turns to refixate. L40 Opsoclonus and ocular flutter, ocular motor disorders most commonly associat ed with malignancy (neuroblastoma in children and small cell lung carcinoma in adults), these movem ents may be the first presenting sign of malignancy. L41 MS may cause ocular flutter. L42 Ocular bobbing is most commonly seen in comatose or quadriplegic patient wit h large infarcts or brain stem haemorrhage. L43 In progressive supranuclear palsy downward are the first affected (saccades are affected than pursuit); in contrast to Parinaud dorsal midbrain lesion upward is firstly affected. L44 Latency for smooth pursuit is shorter (125-135 ms), than saccades which requ ire (150-200 ms). L45 Parinaud syndrome. a. Collier syndrome may worsen with attempted upgaze. b. Skew deviation and papilloedema may be seen depending on the etiology. c. Convergence-retraction nystagmus is also a response to an effort at upgaze th at triggers medial rectus contraction. Hence this form of nystagmus is worsened by upward OKN testing. L46 Monocular nystagmus in toddlers has been associated with chiasmal glioma and hypothalamic glioma. It is also seen in blind eyes, MS, and spasmus nutans. L47 According to Alexander's law, the nystagmus is more pronounced when gaze is directed toward the side of the fast-beating component. L48 Of various forms of nystagmus, downbeat nystagmus most frequently violates A lexander's law. L49 Downbeat nystagmus is almost always present in primary position, but in acco rdance with Alexander's law, the down beating movements are accentuated in downgaze (especially down gaze to eith er side). Patients usually report difficulty in reading and oscillopsia. The oscillopsia can be debilitatin

g. (AAO) L50 The nystagmus most likely to be localized is see-saw nystagmus. It results f rom third ventricle tumours or diencephalic lesion involving the connections to the interstitial nucleus of Caj al and is therefore most localized. L51 See-saw nystagmus is most frequently associated with bitemporal hemianopia, because posterior chiasm is most vulnerable to diencephalic tumour. L52 Vergence eye movements are felt to have a pathway of their own for stimulati ng the brain stem motor nuclei. L53 Upper motor neuron facial nerve paralysis usually leaves voluntary eyelid cl osure intact. Voluntary eyelid closure is preserved in upper motor facial nerve paralysis, because of bilateral supranucle ar input to the upper fascial muscles (orbicularis). L54 Spontaneous or reflexive blinking requires normal function of the basal gang lia, as do other nonvolitional facial expressions. Therefore, these are commonly affected in Parkinson's disease. Voli tional movements, in distinction, are generally not adversely influenced. L55 Marcus jaw winking reflex is an example of synkinesis (abnormal innervation connecting two groups of normally unrelated muscles, such that contract together). L56 Compression of the facial nerve at the cerebellopontine angle has been demon strated in 90% of cases of hemifacial spasm. L57 Approximately 85% of patients with Bell's palsy spontaneously recover, altho ugh some will have some degree of aberrant regeneration. Recovery usually begins within 3 weeks of diagnosis and u sually becomes complete by 2 to 3 months. L58 The facial nerve is the most frequently involved cranial nerve in neurosarco idosis. The parotid gland can become infiltrated with granulomas, and the facial nerve is involved at this site. Faci al nerve involvement in sarcoidosis is frequently bilateral but asymmetric. The second most commonly involved crania l nerve is the optic nerve. L59 Of the three most common causes of facial nerve overactivity, only essential blepharospasm is believed to be related to basal ganglia dysfunction. Compression of the facial nerve in the cer ebellopontine angle by anomalous vessels has been demonstrated in 90% of cases of hemifacial spasm. Tumors in the cerebellopontine angle also

can cause hemifacial spasm. Facial myokymia is caused by disease in the pons inv olving the facial nucleus or fascicle. The most common causes include multiple sclerosis (MS) in adults and p ontine glioma in children. L60 The following conditions may be confused with essential blepharospasm. a. Severe dry eye. b. Retained conjunctival foreign body. c. Tardive dyskinesia. L61 Hemifacial spasm is distinct from essential blepharospasm in that it is unil ateral, and multiple facial muscles are involved. L62 Pupillary dilatation may be the only sign of third nerve palsy in uncal hern iation and basilar meningitis. Total oculomotor nerve palsy sparing the pupil in elderly patients usually suggests mi crovascular aetiology. L63 DDX of tonic pupil includes herpes zoster, syphilis, and giant cell arteriti s. L64 Localization of the lesion in Horner's syndrome is part of the clinical work up and sometimes guided by the extent of anhidrosis. First-order lesions (central nervous system) cause ipsilateral anhid rosis of the entire body. Secondorder lesions (Pancoast's tumor, neck trauma) cause ipsilateral facial anhidrosi s. Third-order lesions cause anhidrosis only around the affected eye or none at all. L65 Painful Horner syndrome may be caused by many disorders (neck trauma, migrai ne, cluster headache), but spontaneous dissection of the common carotid artery must be ruled out with angio graphy or MRI/MRA. L66 After determination of which pupil is abnormal, the next step in evaluating a patient with anisocoria is slit-lamp examination. Most patients with nonphysiologic anisocoria have sphincter muscle dysfunction visible at the slit lamp. Trauma is most common, followed by Adie's pupil. L67 The risk of developing multiple sclerosis (MS) for a relative of a patient w ith MS is nearly 20 folds higher than the general population. MS is a multifactorial disease with both genetic and environ mental risk factors. L68 The most common cranial mononeuropathy in MS is optic neuritis. The most fre quently affected motor nerve is the sixth nerve. L69 5-10% of patients with MS will have findings of posterior uveitis, including pars planitis or retinal periphlebitis. L70 Pheochromocytomas produce, secrete, and store catecholamines. They are most often derived from the adrenal medulla but may arise in any of the sympathetic ganglia. Two phakomatoses, neuro fibromatosis, and von HippelLindau disease, are associated with the tumor. L71 Astrocytic hamartomas can be seen in tuberous sclerosis or neurofibromatosis , or they can be sporadic. They are located in the nerve fiber layer of the retina and sometimes can be associated w ith the optic disc. L72 The triad of adenoma sebaceum, mental retardation, and seizures is considere d pathognomonic for tuberous sclerosis. This is known as Vogt's triad and is present in 30% of patients with tuberous sclerosis. Facial angiofibromas (adenoma sebaceum) are present in at least 75% of adults with tube rous sclerosis. L73 Seizures are part of the classic triad in tuberous sclerosis (80% of patient s have seizures). Patients with Sturge-

Weber syndrome and meningeal hemangioma may have seizure disorders. Patients wit h neurofibromatosis also have an increased risk of seizures. L74 Unilateral congenital glaucoma is seen in 25% of cases of Sturge Weber syndr ome. NF1 associated juvenile glaucoma. Glaucoma may be associated with V. H. L. disease. L75 Heart block in patient with Kerns Sayre syndrome is lethal and among the las t signs to develop. Some studies indicate that elevated cerebrospinal fluid (CSF) protein levels correlates with the presence of heart block. L76 A diagnostic feature of myotonic dystrophy is the presence of polychromatic ( Christmas tree ) cataracts, which may present even in the amyotonic forms. Wilson's disease is associated with the s unflower cataract. L77 Lid retraction and lag is the most common sign of Graves's disease. L78 The most frequently involved muscle in dysthyroid orbitopathy is the inferio r rectus. The medial rectus is the second most frequently affected muscle and may simulate a sixth nerve palsy. The least frequently involved muscle in Graves' ophthalmopathy is the lateral rectus. L79 Myasthenia gravis. a. 75% of MG patients will have eye findings at presentation. b. Only 33-50% will have ocular myasthenia only. c. 90% will have ocular symptoms during the course of the disease. d. Ptosis is the most common. e. 5% will develop Graves' disease. f. 10% will have thymus enlargement on CT scan. g. Ocular myasthenia should remain localized for 2 years to be reassured that sy stemic disease is unlikely to develop. L80 A tiny fraction of patients with Graves' disease will develop MG. L81 IOP after high grade carotid stenosis are low (caused by ciliary hypoperfusi on), normal, or elevated (caused by neovascularization of the iris and angle). Unilateral arcus is highly suggestive of contralateral carotid disease. Carotid stenosis protects the ipsilateral cornea from serum lipid deposition. L82 The etiology of Coats' disease is unknown, and there does not appear to be a ny genetic, familial, racial, or ethnic predisposition. However, Coats'-type retinal vascular changes have been noted in patients with facioscapulohumeral muscular dystrophy, Turner's syndrome, Senior-Loken syndrome , and one variant of the

epidermal nevus syndrome. In addition, Coats'-like retinopathy has been noted in up to 3.6% of patients with retinitis pigmentosa. L83 Downbeat nystagmus in primary position is localized to the craniocervical ju nction (or certain intoxications). This patient's clinical symptoms of intermittent occipital headaches with sudden head movements or anger suggest the diagnosis of Arnold-Chiari malformation. Arnold-Chiari malformation is one of th e most common causes of downbeat nystagmus. L84 Clonazepam and suboccipital craniotomy have been used in the treatment of do wnbeat nystagmus in patients with Arnold-Chiari malformation. By contrast, carbamazepine toxicity has been associa ted with downbeat nystagmus. L85 A 9-year-old boy presents to the ophthalmologist with the complaint that he has lost his position on the school basketball team because he cannot see the basket. He notes occasional morning he adaches but denies any nausea or vomiting. Examination reveals visual acuity of 20/40 in the right eye and 20/25 in the left eye. The patient has marked symmetric weakness of upgaze bilaterally. His pupils are 7 mm and are poorly reactive to light, with better reaction to a near target. There is approximately 2 mm of superior sclera l show bilaterally. Fundus examination suggests optic atrophy in both eyes. A review of the systems reports increased consumption of water, with frequent urination at night. The most likely diagnosis is: pinealoma. a. This patient appears to have Parinaud's dorsal midbrain syndrome, which may i nclude the following findings: pupillary light-near dissociation, lid retraction (the Collier sign), upgaze paresis, convergenceretraction nystagmus, fixation instability, small-amplitude skew deviation, and papilledema (if ventricular outflow has been compromised). The most common cause in this age group would be a pinealoma. Other causes include stroke, hydrocephalus, and multiple sclerosis (MS). b. After neuroimaging has been obtained, a lumbar puncture would be an important step in the diagnostic evaluation of this patient because pinealoma classically sheds cells into the ce rebrospinal fluid (CSF). L86 Optic neuritis in childhood is more commonly bilateral. Visual loss can be s evere, although intravenous corticosteroids can improve visual function. Diffuse enlargement of the optic ne rve on computed tomography (CT) scan may be seen in this condition, mimicking a neoplasm of the optic nerve sheath. The demyelination typically follows a viral illness or vaccination by 10 to 14 days. L87 The differential diagnosis for posterior ischemic optic neuropathy (PION) sh ould include radiation optic neuropathy, status post-coronary artery bypass graft, anemia, acute systemic hypotension, gi ant cell arteritis (GCA), and syphilis. Well-controlled essential hypertension is associated with anterior isc hemic optic neuropathy (AION) but not PION. L88 Bilateral disc oedema and headache may be caused by several reasons, but hyp ertension should be the first to be excluded because it is quite easy to do so. After checking blood pressure, a neu roimaging should be obtained

immediately. L89 Meningothelial (syncytial) meningioma is the most common histopathologic typ e of meningioma seen within the orbit. Pilocytic describes the cell type of gliomas of the visual pathways. L90 A meningioma arising in optic nerve sheath is most likely to lead to optic d isc edema. L91 In general, optic nerve compression >1 cm posterior to the globe does not ca use disc edema. L92 Tobacco-alcohol amblyopia seems to be seen only in heavy smokers/drinkers wi th poor nutrition. This has led many to believe that a combination of toxic plus nutritional insults must be nec essary for the development of the disorder. L93 The most common etiologies of bilateral central or cecocentral scotomas incl ude hereditary optic neuropathy and nutritional optic neuropathy (vitamin B12 and folate deficiency), drug toxicity, tobacco-alcohol amblyopia, and infiltrative disorders such as syphilis and tuberculosis. Cyanide levels are not helpful in suspected tobacco-alcohol amblyopia. L94 Although disc edema, disc hemorrhages, and choroidal rupture may be seen in acute traumatic optic neuropathy, the most common finding is a normal fundus. Disc pallor would be unusual in the acute setting but present in all cases after several weeks. L95 The development of diabetic papillopathy appears to be independent of serum glucose levels. Diabetic papillopathy is classically seen in young adults with type 1 diabetes with moderate to severe retinopathy (although it can be seen in patients with type 2 diabetes as well). It is painless, and associated v isual loss is generally mild. The disorder generally resolves spontaneously. L96 Thyroid optic neuropathy is considered to be a compressive optic neuropathy because of enlargement of extraocular muscles at the orbital apex. Treatment of thyroid optic neuropathy m ay include orbital radiation (usually 1,500 to 2,500 rad over a 10-day period) and orbital decompression, which provid es the most potential for decompression of the optic nerve. Systemic corticosteroids are thought to be eff ective only in the acute congestive phase and not in the fibrotic period. Therefore, if no response is noted within 3 weeks, systemic steroids should be tapered and another modality should be considered. Although a subtotal thyroidec tomy may provide primary treatment of dysthyroid state, it will have no effect on the eye findings (excep t, perhaps, lid retraction). L97 Bromocriptine has been shown to be effective primarily in the management of prolactin-secreting pituitary tumors and is less effective or ineffective with other types of pituitary tumors. L98 Acute visual loss associated with severe headache is a typically a sign of p ituitary apoplexy.

L99 Fortification spectra that precede visual loss suggest a diagnosis of migrai ne headaches. A fixed visual defect suggests a cerebral arteriovenous malformation (AVM). A family history of headac hes and nausea associated with visual disturbances are more suggestive of a migraine. Field defects may be foun d during acute migraine headaches, but those present interictally suggest another diagnosis. L100 A skew deviation is a motility disturbance with a vertical component that d oes not have a pattern consistent with a discrete muscle underaction or nerve palsy. They are generally due to supranucle ar or vestibulo-ocular dysfunction and generally reflect brainstem disease. They are typically comitant but not alw ays. L101 Small vessel disease is the most common cause of unilateral sixth nerve pal sy. More common causes of bilateral sixth nerve palsy include increased intracranial pressure, head trauma, and tumo rs of the ventral brainstem. Congenital bilateral sixth and seventh nerve palsies are characteristic of M.biu s' syndrome. L102 Isolated sixth nerve palsy in children is most commonly attributable to pos tviral inflammation occurring 1 to 3 weeks following a nonspecific viral illness of the upper respiratory tract. Recovery i s generally complete and occurs within 10 to 12 weeks. L103 Medial rectus contracture is distinctly uncommon in Duane's syndrome. In co ngenital sixth nerve palsy, it is quite common and results in esotropia in primary position. To differentiate between Du ane syndrome and congenital sixth nerve palsy is orthotropia in primary position because medial rectus contracture is uncommon in Duane syndrome, while sixth nerve palsy result in esotropia in primary position. L104 Disruption of the medial longitudinal fasciculus (MLF), which carries proje ctions of interneurons from the contralateral sixth nerve nucleus to the ipsilateral medial rectus subnucleus, r esults in ipsilateral absence or slowing of adduction and contralateral abduction nystagmus. This combination of findings is termed internuclear ophthalmoplegia (INO). Vertical nystagmus and skew deviations are frequently fou nd in association with INO but are not universal. L105 Bilateral internuclear ophthalmoplegia (INO) is more frequent in demyelinat ing disease than in cerebrovascular disease. This is because the brainstem blood supply is lateralized right and left circulations are usually discrete and end at the midline. Demyelinization does not respect the midline. L106 Exotropia in primary position can occasionally occur in association with a bilateral internuclear ophthalmoplegia (INO), resulting in a syndrome called wall-eyed bilateral INO. L107 Miller-Fisher syndrome is generally considered a variant of Guillain-Barré sy ndrome that results in ophthalmoplegia, ataxia, and areflexia. Serum immunoglobulin G (IgG) autoantibod ies and elevated cerebrospinal fluid (CSF) protein may be present. Complete recovery is common. L108 The presence of optic nerve dysfunction, manifested by decreased vision, an afferent pupillary defect, and/or dyschromatopsia, distinguishes an orbital apex syndrome from a cavernous sinus s yndrome because the optic nerve passes through the optic canal and does not enter the cavernous sinus. L109 Tolosa-Hunt syndrome.

a. Idiopathic, sterile inflammation that primarily affects the cavernous sinus. b. Pain is almost always present. c. Painful ophthalmoplegia. d. Sensory deficit of the trigeminal nerve (most commonly ophthalmic branch). e. Neuroimaging may show a mass, which often is enhanced with contrast. f. Patients with Tolosa-Hunt syndrome respond dramatically to corticosteroids. g. Normal MRI is unlikely but not impossible. h. It is DX of exclusion. L110 Iris and posterior segment neovascularization, as well as rapidly progressi ve cataract, may all be seen as complications of the ischemic oculopathy that these fistulae generate. Corneal e xposure caused by proptosis is another potential complication of carotid-cavernous fistula. L111 Both high-flow and low-flow carotid-cavernous fistulae may be associated wi th a history of head trauma. Low flow carotid cavernous fistula may also be associated with minor head trauma. L112 59-year-old man presents to the emergency room with a complaint of sudden-o nset oscillopsia and diplopia. Examination reveals an alcohol odor to his breath, normal acuity, bilateral abdu ction deficits, and coarse binocular nystagmus. Appropriate intervention should include intravenous thiamine. Acute t hiamine deficiency (Wernicke's encephalopathy) can result in central scotomas as well as ophthalmoplegia, prima rily affecting cranial nerves III and VI. It can be precipitated in nutritionally depleted alcoholics given intravenou s glucose alone because of sudden consumption of systemic thiamine stores. L113 Giant cell arteritis (GCA) is common in northern European climates. On the basis of autopsy studies, the prevalence has been estimated to be 1.1% of the Scandinavian population. L114 Clinical characteristic of chiasmal compression includes: a. Field abnormalities most notably centrally with fainter test object. b. Postfixation blindness. c. Temporal color desaturation. d. Hemifield slip result in diplopia. L115 The examiner attempts to confirm diagnostic suspicions by eliciting the Pul frich phenomenon. To do this, the examiner: asks the patient to watch the pendulum on the grandfather clock across the room and report any three-

dimensional movement. The Pulfrich phenomenon probably reflects delayed conducti on in the demyelinated nerve. Oscillating objects perceived by the affected eye appear to be behind the image seen with the healthy eye, simulating three-dimensional movement where there is only movement within one pl ane. The delayed implicit time is the electrophysiologic correlate of the bizarre perception known as the Pulfr ich phenomenon. L116 LHON, telangiectatic peripapillary vessels not leak on FA. L117 Sumatriptan is a serotonin antagonist available orally, by injection, or as a nasal spray. The -triptan antimigraine drugs are used for symptomatic relief of migraines but are contraindicated in pa tients with basilar artery migraine. They can produce myocardial infarction (MI) and should be used with caution in p atients with severe hypertension or coronary artery disease. L118 The best initial choice for prophylaxis of acute, severe migraine headache is propranolol. Methysergide is an ergot alkaloid with numerous potential side effects. The beta-blockers are effective a nd safer. L119 The Ischemic Optic Neuropathy Decompression Trial of IONDT for NAION reveal ed no significant benefit for treatment (improvement in visual acuity by at least three Snellen lines: 32.6% f or treated patients vs. 42.7% for controls}. Moreover, the treatment group showed a statistically significantly gr eater risks of acuity that worsened by three Snellen lines or more (23.9% treated vs. 12.4% control). IONDT is not curr ently recommended for the treatment of NAION. It has been suggested that levodopa has a beneficial effect, but this remains unconfirmed. L120 Pseudotumor cerebri typically (not universally) presents with visual loss a nd headaches. Neurologic abnormalities are usually absent except for occasional abducens paralysis. Although the openin g pressure is elevated, cerebrospinal fluid (CSF) protein levels are either normal or low. Many cases ar e probably based on decreased reabsorption of CSF. L121 Binding antibodies to acetylcholine receptors are found in 90% of patients with generalized myasthenia gravis (MG). L122 Relative to generalized myasthenia, ocular myasthenia is completely localiz ed to ocular or bulbar involvement. L123 Drug induced MG is D-penicillamine, with onset average 6 months after initi ation of therapy. Approximately 80% of patients will remit completely within 6-8 month of cessation. L124 The Purkinje effect is a benign condition. It is a real, subjective test an d can rarely be used to determine whether a patient's retina is attached. L125 Binocular diplopia is relieved by covering one eye. Monocular diplopia is r elieved by covering the affected eye. It can be caused by keratoconus, lenticonus, high astigmatism, retinal pathologies. L126 The following results in anisocoria in dim light. a. Horner's syndrome. b. Posterior synechiae. c. Physiologic anisocoria. d. Pharmacologic anisocoria. L127 There are a host of systemic conditions that can become exacerbated during pregnancy. Pseudotumor cerebri is

not one of them. L128 AAO. PTC is associated with endocrine or metabolic dysfunction, pregnancy, or the use of exogenous substances such as vitamin A (> 100,000 U/day), tetracycline, nalidixic acid, cyclosporine, and oral contraceptives, as well as the use of or withdrawal from corticosteroids. L129 Duane. Idiopathic intracranial hypertension (IIH) is a syndrome of increase d intracranial pressure characterized by normal brain imaging, normal cerebrospinal fluid (CSF) composition, and elevated CSF pressure. IIH occurs at the same rate in pregnant and nonpregnant women, but one study showed worsening of s ymptoms among nine of 11 patients with pre-existing IIH who became pregnant. L130 Sj.gren syndrome associated with CRAO. L131 Craniopharyngioma is the most common cause of see-saw nystagmus. Other para sellar tumour can cause the condition as well. L132 Wyburn-Mason syndrome is a sporadic condition, characterized by a retinal r acemose angioma with an intracranial arteriovenous malformation (AVM). Patients with Wyburn-Mason syndrome can have o rbital AVMs as well (and consequent ocular bruits). As of 2004, there have been no reports of photodynami c therapy (PDT) to treat WyburnMason syndrome (and it is unlikely that PDT would be effective in patients with this condition). L133 Binasal hemianopia is almost never due to chiasmal compression and is usual ly due to glaucoma. B. . OCULOPLASTY OCULOPLASTY L1 Prominent pain in orbital disorder is present in orbital pseudotumour and mal ignant mixed tumour of the lacrimal gland. If pain is present, Graves' disease is usually not the cause of orbitopathy. L2 Exorbitism refers to an angle between the lateral orbital walls that is great er than 90°, which is usually associated with shallow orbital depth. This condition contrasts with telorbitism (hypertelorism) , which refers to a wider-than normal separation of the medial orbital walls. L3 The weakest orbital wall is the anterior portion of the medial wall is also k nown as the lamina papyracea, or paper-thin layer. L4 Sinus system aerates first is ethmoid. L5 The upper limit of normal exophthalmometry in white man is 22 mm, while in bl ack is 25 mm.

L6 Children's orbits are more plastic, so disorders with a short duration can ca use orbital enlargement. This is not the case in adults which needs chronic process. L7 In pediatric autoimmune hyperthyroidism, the systemic findings are more promi nent and the orbital findings less common and less severe. Furthermore, for similar degrees of exophthalmos, childr en seem to develop exposure problems less often. L8 Ocular ultrasound is usually performed in the range of 8 to 15 MHz. Higher fr equencies give better resolution. Lower frequencies give better penetration. L9 Computed tomography CT uses less radiation than conventional tomography. L10 Orbital mass. a. Sinus mucocele will have round, extraconal, smooth regular border, and very l ow internal reflectivity. b. Lymphangioma will have irregular border. c. Neurofibroma will have high internal reflectivity. d. Metastatic cancer and pseudotumour will have irregular border and high reflec tivity. e. Cavernous haemangioma has densely packed vascular channel leads to high inter nal reflectivity. L11 The magnetic field strength used to generate MRI is 2000-15000 times that of the earth's magnetic field. L12 If fat brighter it is T1 MRI, if vitreous is brighter it is T2. L13 A bluish bulge above the medial canthus is typically meningocele, whereas on e below is typically a dacryocele. L14 In infant, increases risk of spread of preseptal cellulitis to orbital cellu litis, so that; infant with preseptal cellulitis should be treated immediately with IV antibiotic. Inappropriate or delayed treat ment increases the risk. L15 The agent most likely to cause a severe preseptal cellulitis leading to seco ndary orbital cellulitis and central nervous system (CNS) infection in infants and toddlers is Haemophilus influenzae. Intrav enous antibiotics are indicated for Haemophilus influenzae cellulitis in children. The Hib vaccine has significantly reduced the incidence of preseptal and orbital cellulitis secondary to H. influenzae. L16 Staphylococcus aureus is the most common cause of preseptal cellulitis secon dary to trauma. Streptococcus organisms are a close second. L17 90% of orbital cellulitis is secondary to paranasal sinus extension. L18 Complication of orbital cellulitis is cavernous sinus thrombosis characteris ed by sudden worsening of orbital cellulitis with virtually frozen orbit, despite inapparent increase in proptosis, corneal s ensation is likewise diminished, but VA remain grossly intact. L19 Tests that are important for distinguishing infectious orbital cellulitis fr om inflammatory orbital pseudotumor include a. Oral temperature. b. Orbital computed tomography (CT) scan. c. Complete blood count (CBC) with differential. L20 Patients with orbital cellulitis are typically febrile. For pseudotumor, thi s is uncommon except in children. A leukocytosis with left-shift is also more common in cellulitis. The sedimentation rate may be elevated in either condition, and in both, computed tomography (CT) scan may show a nonspecific ret robulbar infiltrate but may have signs specific to one of the two disorders (e.g., muscle enlargement, subperiost

eal abscess). L21 Orbital cellulitis: failure to dramatically improve after 48-72 hour of anti biotic can indicate subperiosteal abscess. Relapse after switching to oral antibiotic also may be a sign. Inappropriate cho ice of antibiotic should not be associated with modest improvement. L22 Dermoid cyst may induce bony erosion on radiography. The variety that is sil ent until adulthood is generally intraorbital (retroseptal). L23 Teratomas. a. Teratomas are rare tumors that arise from two or more germinal layers (but al l three need not to be), including ectoderm and endoderm or mesoderm (or both). b. These tumors are usually cystic and can cause dramatic proptosis at birth. c. Exenteration is sometimes performed because of the fear of malignancy. Howeve r, in some cases, cystic teratomas can be removed without an exenteration, and ocular function may be pre served. d. Orbital teratomas may simulate malignancy but rarely, if ever, metastasize. L24 Potentially important complications of capillary hemangiomas in childhood ar e occlusion amblyopia, significant astigmatism. The third is significant cosmetic deformity. L25 More than 90% of periocular capillary haemangioma manifest by 6-8 months of age. Greater than 50% by 1-2 month of age. Reach their peak at approximately 6-12 month of age. L26 . Nevus flammeus is part of Sturge-Weber syndrome and Klippel-Trenaunay synd rome Nevus flammeus is part of Sturge-Weber syndrome and Klippel-Trenaunay syndrome L27 Capillary haemangioma will blanch with pressure whereas the naevus flammeus does not. L28 Neither capillary nor cavernous hemangiomas have high blood flow, and neithe r metastasize. Cavernous hemangiomas are rarely seen in youth, whereas, as the name of each implies, the blood-filled spaces are tiny in a capillary hemangioma and large in a cavernous hemangioma. L29 Thrombocytopenia in association with visceral capillary haemangioma is known as Kasabach-Merritt syndrome.

L30 lack fibrous capsule so that its growth is infiltrative. Lymphoid tissue, ev en follicles may be found Lymphangioma Lymphangiomawithin the substance of the tumour. Unlike capillary haemangioma the cystic spaces are large. L31 The origin of orbital lymphangioma remains unclear, although vascular malfor mations with lymphatic and venous components may play a role. Furthermore, noncontiguous vascular malformations ha ve been reported in one study to occur in up to 25% of patients. Surgical strategies are frequently limited to deb ulking by any safe means possible, because complete excision is usually not possible. L32 Tram-tracking of the optic nerve is considered classic for optic nerve sheat h meningioma. L33 A radiographic feature is considered pathognomonic for optic nerve glioma is kinking of the optic nerve. L34 Optic nerve glioma may induce a secondary meningothelial hyperplastic respon se misinterpreted as meningioma. L35 The management of optic nerve glioma in children is a subject of much discus sion and honest disagreement among the experts. Any of the options may be correct in a given situation, and the ult imate choice must be individualized to the patient's circumstances (e.g., vision, size of tumor, extent of centripetal spre ad, age). L36 arise from undifferentiated pluripotential mesenchymal elements in the orbit al soft tissues Rhabdomyosarcomas Rhabdomyosarcomasand not from the extraocular muscles. They may be grouped into four categories: a. . This is by far the most common type found in the orbits of infants and chil dren, accounting for Embryonal Embryonalover 80% of cases. The embryonal form has a predilection for the supero nasal quadrant of the orbit. Embryonal rhabdomyosarcomas are associated with a good (94%) survival rate. b. . This form has a predilection for the inferior orbit and accounts for 9% of orbital Alveolar Alveolarrhabdomyosarcomas. This is the most malignant form of rhabdomyosarcoma. c. . Pleomorphic rhabdomyosarcoma is the least common and the most differentiate d form. The Pleomorphic Pleomorphicpleomorphic variety tends to occur in older persons and has the best prognosis (97% survival rate). d. . This rare variant of embryonal rhabdomyosarcoma appears grapelike. It is no t found in the orbit Botryoid Botryoidas a primary tumor; rather, the botryoid variant occurs as a secondary i nvader from the paranasal sinuses or from the conjunctiva. e. Metastatic workup includes lumbar puncture and bone marrow biopsy, best done under anesthesia. L37 Duane. Although no single regimen is appropriate for every child with orbita l rhabdomyosarcoma, a sample protocol might include multiple 3-week cycles of chemotherapy, each beginning with intrav enous vincristine, actinomycin-D, and cyclophosphamide, with vincristine repeated on the 8th and 15th days of each cyc le. The regimen might include external radiation to a total dosage of 5,040 cGy. For poor prognosis cases (e.g ., metastatic alveolar rhabdomyosarcoma), newer agents under investigation include ifosfamide, etoposid e, irinotecan, topotecan, and tirapazamine. Having made the diagnosis and contributed to local staging at the time of presentation, the orbital

surgeon continues to follow the patient along with the pediatric oncology team. In cases of treatment failure, salvage surgery may take the form of orbital exenteration or excision of residual tumor combined with brachytherapy. Rhabdomyosarcoma underscores the importance of clinical suspicion when dealing w ith acute proptosis in childhood. Prompt referral to a tertiary center after appropriate imaging is the responsibi lity of the primary ophthalmologist, family practitioner, or pediatrician who first encounters the patient. L38 Most periocular metastases in children are orbital, unlike in adults. The mo st common is neuroblastoma. L39 Fungal species likely leads to a necrotizing orbital cellulitis are Mucor, R hizopus, and Aspergillus. L40 is generally seen only in the immunocompromised (chemotherapy, posttransplan t, diabetes Mucormycosis Mucormycosismellitus). Aspergillosis is probably more common in this population also, but it may be seen in otherwise healthy individuals, and it also may be seen as a result of allergic aspergillosis sinus itis. L41 Orbital phycomycosis generally results from invasion by necrotizing fungal s inusitis. Black eschar in the nasal cavity is virtually diagnostic but is a late finding. (Its absence does not exclude the diagnosis.) L42 Sebaceous cell carcinoma is more common in women, while fibrous histiocytoma are more common in men. L43 Thyroid-related orbitopathy can occur despite a persistently euthyroid state (as indicated by clinical and laboratory findings). L44 The indications of immediate surgical intervention in patients with Graves's orbitopathy are acute corneal decompensation and evidence of optic nerve compression. L45 Sensitive immunometric assay. This is the most sensitive way to thyroid-stimu lating hormone (TSH) levels thyroid-stimulating hormone (TSH) levelsdetect hyperthyroidism; feedback regulat ion can lead to a reliably detectable decrease in thyroid-stimulating hormone (TSH), even when fluctuating levels of thyroxine (T4) and 3, 5, 3'-triiodothyron ine (T3) may not be indicative.

L46 Surgical procedures that may be indicated during the active, inflammatory ph ase of Graves' ophthalmopathy include: a. Lateral tarsorrhaphy. b. Orbital decompression. L47 The histopathologic changes evident in extraocular muscle (EOM) specimen fro m patients with Graves' ophthalmopathy also may be seen in the lacrimal glands. Fibrosis tends to be les s severe, leading to partial loss of the normal glandular tissue.

L48 Typical manifestations of idiopathic orbital inflammation (pseudotumor) incl ude all of the following: a. Dacryoadenitis. b. Extraocular myositis. c. Periscleritis. d. Optic perineuritis. e. Changes in corneal sensation may occur, but frank ulceration especially periphe rally is unusual L49 Clinical findings that are more likely in pediatric orbital pseudotumor than in the adult variety of the disease include all of the following: a. Bilateral involvement. b. Systemic symptoms and signs (malaise, fever, vomiting). c. Peripheral eosinophilia. d. Uveitis. L50 Orbital pseudotumor is generally quite painful for both children and adults. L51 Some patients do not experience the typical pain. Some may have minimal infl ammatory signs and may present with a totally fibrotic scarred lesion (idiopathic sclerosing inflammation of the orb it). Such lesions may require biopsy for diagnosis because it is misdiagnosis for malignancy. There is little response to steroid because it is fibrotic. L52 Orbital pseudotumor. Although a dosage of 60 mg of prednisone is at the lowe r end of the usual beginning ranges for steroid therapy, one should consider an orbital biopsy because of the possib ility of other inflammatory diseases and other orbital pathology. After the diagnosis is confirmed, additional attempts s hould be made to bring about a response with stronger steroid therapy, orbital radiation, methotrexate, or cyclophospham ide. L53 Unlike Graves' ophthalmopathy, pseudotumor can and commonly does involve the muscle tendons as well as their insertions, along with posterior Tenon fascia. L54 Sarcoidosis generally not associated with pain and spares orbital soft tissu e. L55 The following disorders may be associated with a clinical presentation indis tinguishable from typical inflammatory orbital pseudotumor. a. Systemic lupus erythematosus (SLE). b. Polyarteritis nodosa. c. Wegener's granulomatosis. d. Churg-Strauss syndrome. L56 Bilateral painless enlargement of the lacrimal glands is typical presentatio n of sarcoidosis and benign lymphoepithelial lesion. L57 Hemangiopericytomas are more likely to limit ocular motility than cavernous hemangiomas. L58 Hemangiopericytomas (rapidly expansile and aggressive tumour) may restrict o cular motility and cause conjunctival prolapse and engorgement of vessels in the conjunctival cul-de-sac. Hemangiopericytomas resemble cavernous hemangiomas on contrast-enhanced CT and may look blue in surgery. Thes e lesions must be completely excised because they may recur, undergo malignant degeneration, and metastasize. Histopathologically, these lesions are unique in that microscopically "benign" lesions may recur and metast asize, whereas microscopically "malignant" lesions may remain localized.

L59 Most orbital meningiomas arise outside the orbit and invade secondarily. L60 In NF, the growth pattern of meningioma tends to be paraxial (diffusely alon g the nerve sheath) rather than focal. L61 Meningioma may be associated with bony changes either osteolytic or osteobla stic, so that CT is a modality of choice in the initial evaluation. Angiography generally reveals a highly vascula rized tumour (tumour blush). MRI is particularly useful for the suspected intracranial extension. L62 Optic nerve sheath meningioma. Excisional biopsy of optic nerve biopsy gener ally result in visual loss, because fragile pial blood vessel surrounding tumour. Surgeries are usually indicated if there is intracranial extension, severe proptosis and sever visual loss. Also surgery is usually indicated if tumour gro wth is becoming a potentially a life threatening. Paediatric meningioma is more aggressive tumour than adult variety and is removed earlier. L63 Ophthalmologist specialist practice 10% of lacrimal gland lesion will be inf lammatory or lymphoid. In contrast, to orbit specialist practice probably see 50% epithelial neoplasm and 50% inflammatory/ly mphoid lesions. L64 The critical features of lacrimal disease: a. Pain is more typical of acute inflammatory, infectious, or malignant lesions. b. Duration of symptoms is helpful in separating slowly progressive benign lesio n, such as benign mixed tumour from more acute conditions such as malignant neoplasm or inflammation. c. Radiological findings osteolysis is strongly favor malignant lesion. L65 Pain in adenoid cystic carcinoma is more commonly due to perineural extensio n and bone destruction. L66 Soft-tissue contour analysis can help differentiate between lymphoid tumors (elongated smooth masses pancakes ) from parenchymal tumors (globular masses). Adenoid cystic carcinomas usu ally produce bony destruction. Benign mixed cell tumors often cause concave pressure changes in the adjacent bo ne. Lymphoid and inflammatory lesions generally do not cause bony changes. L67 Benign mixed cell tumor, the male-female ratio is 3:2. The tumor should be a pproached through a lateral orbitotomy with careful excision to avoid rupture of the tumor's pseudocapsule. Incisional or incomplete biopsy techniques can lead to infiltrative tumor recurrence (in 32% of cases) and, occasionally, malignant transformation.

L68 Histiocytic disorders is most likely to involve orbital bone is eosinophilic granuloma. L69 Fibrous histiocytoma is one of the most common mesenchymal tumours of the or bit. 10% have metastatic potential. Another 16% are termed locally aggressive but not frankly malignant. Most are benign (>90%), and have a storiform, or matlike, pattern on histopathology. It is usually very firm and ca n displace other orbital structures. L70 Fibrous dysplasia may be monostotic (affect only one bone) or polyostotic. T he polyostotic variety may present with precocious puberty and dermal hyperpigmented macules. This disorder, Albright's syndrome, rarely involves the orbit. Orbital disease is nearly always monostotic and rarely associated with precociou s puberty, regardless of age. Surgical curettage or excision is usually undertaken. Only woven bone, not cancellous, is found microscopically. L71 It is impossible to differentiate clinically or radiographically between ben ign reactive lymphoid hyperplasia It is impossible to differentiate clinically or radiographically between benign reactive lymphoid hyperplasia and orbital lymphoma. Biopsy with light microscopy, immunochemical staining, and electron microscopy is and orbital lymphoma. Biopsy with light microscopy, immunochemical staining, and electron microscopy is . necessary to distinguish between them necessary to distinguish between them L72 The central nervous system (CNS) is not routinely surveyed in patients with orbital lymphoma. This is in The central nervous system (CNS) is not routinely surveyed in patients with orbi tal lymphoma. This is in contrast to patients with intraocular lymphoma. When CNS involvement is suspecte d, computed tomography contrast to patients with intraocular lymphoma. When CNS involvement is suspecte d, computed tomography . (CT) scan or magnetic resonance imaging (MRI) is the starting point (CT) scan or magnetic resonance imaging (MRI) is the starting point L73 The key features separating pseudotumour from lymphoproliferative lesion are : a. Prominent fibrovascular stroma. b. Hypocellularity. L74 The three orbital deposits that are hyperintense on T1 weighted MRI are bloo d, melanin, and mucus. L75 Exceptions to the rule of thumb that all orbital tumors are dark on T1-weigh ted magnetic resonance images (MRI) include: a. Retrobulbar hemorrhage of at least 24 hours duration. b. Melanoma. c. Mucocele. L76 The nose and paranasal sinuses are more frequently the focus of tumors that secondarily invade the orbit. L77 The most common sinus lesion that invades the orbit is the mucocele. The mos t common sinus neoplasm to invade the orbit is squamous cell carcinoma. L78 Classic features of a tripod fracture include all of the following: a. Downward displacement of the lateral canthus. b. Infraorbital hypesthesia. c. Trismus. d. Temporal subconjunctival hemorrhage. L79 The tripod complex is produced by three distinct fractures along suture line

s zygomaticofrontal, zygomaticomaxillary, and the zygomatic arch. Ocular motility may or may not be n ormal in pure tripod fractures, but upgaze should be spared. Repair is indicated when there is marked cosmetic defor mity or potential mandibular instability. L80 Telecanthus and rounding of the medial canthus is a characteristic finding i n direct naso-orbital-ethmoidal fractures. L81 The qualifier indirect implies that forces other than direct contact with a bl unt object led to orbital wall fracture (for instance, suddenly increasing intraorbital pressure). This type is rarely associ ated with orbital rim fracture, which is seen more. L82 The following findings in orbital floor fractures is/are exacerbated by the presence of a coincident medial orbital wall fracture are enophthalmos, subcutaneous emphysema. L83 With severe muscle contusion, oedema, or haemorrhage, forced duction test ma y be falsely positive or impossible to interpret. They are more fruitful if performed 5-10 days after injury. L84 Most diplopia associated with orbital contusion disappears 7-14 days followi ng injury. Most diplopia result as reversible muscle contusion. L85 The following are considered indications for surgical repair of orbital floo r fractures: a. Disabling diplopia present 7 to 10 days after the original injury. b. Large fracture on acute computed tomography (CT) scans. c. Enophthalmos >2 mm. d. Inferior rectus entrapment in a pediatric patient present 1 day after injury. L86 Computed tomography (CT) scan of the orbit with direct coronal, axial, and s agittal views gives better definition of the soft tissue and bony structures than plain films. Magnetic resonance imaging (MRI) has a very limited role, because bone is dark. L87 Large, complex anterior fractures lead to marked inferior orbital herniation without entrapment. This causes Large, complex anterior fractures lead to marked inferior orbital herniation wit hout entrapment. This causes hypoglobus and enophthalmos. Small posterior fracture can cause significant entr apment as a crowded hypoglobus and enophthalmos. Small posterior fracture can cause significant entr apment as a crowded . muscle belly is forced or pinched into the defect. In these cases, enophthalmo s is minimal muscle belly is forced or pinched into the defect. In these cases, enophthalmos is minimal L88 : Indication of removal of intraorbital FB Indication of removal of intraorbital FB a. Vegetable FB.

b. Anterior, easily approachable FB. c. When there is evidence of optic nerve compromise as decreased VA, APD, and dy schromatopsia. L89 The most common postoperative complication of enucleation is superior sulcus deformity. L90 When ocular or optic disc perfusion is severely compromised by an orbital co mpartment syndrome, immediate canthotomy with cantholysis should be performed to decompress the orbit. L91 In direct optic nerve trauma, something physically impinges on the optic ner ve, but it need not be a foreign body (e.g., bone fragments). In indirect trauma, nothing can be found to have directl y struck the nerve. L92 Treatment of indirect optic nerve injury is most likely with emergent high-d ose intravenous methylprednisolone to reduce swelling associated with the injury. Optic canal decompression may also b e considered. L93 Risk factors that increases the chance of extrusion of orbital implants: a. Implant too large for the orbit. b. Conformer too large for the fornix. c. Exposed synthetic integrated implant. d. Orbital tissue infection. L94 The procedure of choice for severe socket contracture following enucleation involves the use buccal mucosa grafts. L95 Whitnall's ligament should not be confused with the horns of the levator apo neurosis, which are structure to be cut during ptosis surgery. L96 The levator palpebrae superioris, the deeper or posterior portion inserts on to the anterior surface of the lower half of the tarsus, not to its superior border. Müller's muscle inserts into the superior border of the tarsus. L97 Following trauma to the medial canthus, special attention must be directed t o reattaching the tendon to its more posterior attachment, the posterior lacrimal crest. L98 The normal mongoloid slant of the palpebral fissure occurs because of the high er insertion of the lateral canthal tendon (3 mm higher than the medial canthal tendon [MCT]). Antimongoloid slant occ urs when the lateral tendon inserts lower than the medial and can occur with trauma (e.g., tripod fracture) or congenital abnormalities (e.g., Treacher Collins' syndrome). L99 Lower eyelid ectropion is a variable feature not a defining feature of bleph arophimosis syndrome. L100 Blepharophimosis is also known as Kohn-Ramono syndrome. L101 Primary amenorrhea is the most common condition associated with blepharophi mosis syndrome. L102 Congenital coloboma of the upper eyelid is generally isolated whereas that of the lower eyelid is more commonly associated with other facial abnormalities. L103 The most likely outcome following inadvertent suturing of the orbital septu m into subcutaneous tissue when repairing a partial thickness eyelid laceration is lid retraction in downgaze. L104 Hairless skin of similar pigmentation must be chosen for free skin graft in the setting of lid laceration. L105 Factors important to evaluate before planning therapy for involutional ectr opion are: a. Position of the inferior punctum. b. Stability of the lower limb of the MCT. c. Stability of the lateral canthal tendon. d. Presence or absence of the contracture of the orbicularis muscle.

L106 Entropion of any aetiology is more common in the lower lid. L107 Digital pressure along the inferior border of the inferior tarsus will temp orarily correct involutional entropion Digital pressure along the inferior border of the inferior tarsus will temporari ly correct involutional entropion but not cicatricial entropion. but not cicatricial entropion. In cicatricial entropion, vertical foreshortening is present, rather than the In cicatricial entropion, vertical foreshortening is present, rather than the . redundancy of involutional entropion redundancy of involutional entropion L108 To allow maximal spontaneous return of function before surgical repair, it is generally wise to observe traumatic ptosis in an adult for 6 months. Ptosis repair before the possible spontaneous r eturn of function may lead to lagophthalmos. L109 Lid notching, scarring, hypopigmentation, and edema may be seen after lash cryoepilation. L110 Critical components in the evaluation of corneal protection in patient with ptosis surgery: a. Assessment of lagophthalmos. b. Assessment of Bell phenomenon. c. Schirmer testing. d. Assessment of corneal sensation. L111 The primary abnormality seen in ptosis after cataract surgery is in the lev ator aponeurosis. Aponeurotic dehiscence has been blamed on anesthetic injection, lid specula, and bridle sutures. L112 Ptosis of MG is often resistant to anticholinesterase or steroid. Surgery i s usually performed after medical therapy has been optimized. L113 In Graves ' disease lateral lid retraction is greater than medial, in conve rse to the collier sign. L114 Recession of the superior rectus muscle will induce lid retraction, while r esection will induce ptosis. L115 Lower lid retraction is more challenging than the upper lid repair in patie nt with Graves ' disease, because it needs spacer graft.

L116 Acanthosis nigricans: suddenly appearing multiple seborrheic keratoses know n as Leser-Trelat sign is Acanthosis nigricans: suddenly appearing multiple seborrheic keratoses known as Leser-Trelat sign is . usually associated with GIT malignancy usually associated with GIT malignancy L117 Eyelid lesions are typically slightly elevated with a central ulcerated are a or crater are: a. Molluscum lesion. b. Basal cell carcinoma. c. Keratoacanthoma. L118 Cryotherapy selectively destroys melanocytes but is insufficient for cutane ous melanoma and should be considered a palliative treatment. L119 Blepharochalasis is a rare idiopathic disorder leading to inflammatory edem a of the eyelids. It is familial, and younger patients, especially women are affected. The repeated episodes of edema may caus e ptosis and herniation of the orbital lobe of the lacrimal gland. Dermatochalasis is redundant preseptal skin caused by aging. True ptosis (involutional) may be present in either disorder. L120 Hemifacial spasm is rarely bilateral and is usually caused by vascular comp ression of the seventh cranial nerve at the brainstem and can result in synchronous contractions of the entire side of t he face. Along with partial complex seizures and myoclonic epilepsy, essential blepharospasm is effaced by sleep. L121 Any treatment for blepharospasm is designed to decrease eyelid closure. Thu s, dry eye will be aggravated. Facial nerve ablation suffers from recurrence rates as high as 30% and is associated wi th complications such as hemifacial paralysis. Consequently, its use has significantly decreased. Orbicularis myecto my is usually preferred to facial nerve avulsion. Botulinum injection is the initial treatment of choice. L122 Retrospective studies show that the mortality rate from ocular adnexal basa l cell carcinoma is 3%. The vast majority of patients who died from basal cell carcinoma had disease that started in the c anthal areas, had undergone prior radiation therapy, or had clinically neglected tumors. L123 Removal of the orbital lacrimal gland removes the efferent input and interf eres with reflex tearing, whereas removal of the palpebral lobe damages the ducts from the orbital portion, which run thro ugh the palpebral lobe. This impairs reflex tearing as well. L124 Ganglia serves as the home for cell bodies providing postganglionic efferen t innervation to the lacrimal gland is the sphenopalatine ganglion which receives parasympathetic fibers from the greater s uperficial petrosal nerve (a division of CN VII). L125 In congenital NLD obstruction, the blockage is at the valve of Hasner. In a cquired cases, the blockage is within Intraosseous NLD. Involutional stenosis is one of the most common causes of acqu ired NLD obstruction. L126 Jones 1 test, one third of normal patient will have false positive result w ith this test. L127 Single functioning punctum and canaliculus is usually sufficient. L128 Probing of acquired obstructions rarely yields permanent patency. L129 Congenital canalicular obstruction is rare in infants. L130 Acquired causes of canalicular obstruction. a. Trauma.

b. Phospholine iodide. c. Actinomyces infection. d. Idoxuridine use. e. Contraceptive pill. f. Steven-Johnson syndrome. L131 Congenital obstruction of the nasolacrimal system, general anesthesia is ge nerally necessary after the age of 6 months. L132 Mortality rate of sebaceous gland carcinoma is approximately 20%, although variable studies have published mortality rate from 3% to 41%. L133 Acute, lancinating pain in the medial canthal region with minimal noninflam ed enlargement of the lacrimal sac is most subjective of impacted dacryolith. L134 The most common malignant tumor of the lacrimal sac is squamous cell carcin oma. Treatment may consist of a dacryocystectomy and a rhinotomy. L135 The following constitute indications for dacryocystorhinostomy (DCR). a. Recurrent acute dacryocystitis. b. Chronic discharge or symptomatic epiphora with a positive secondary dye test (organic obstruction). c. Persistent epiphora in a child after probing and Silastic intubation of a con genitally impatent system. L136 In probing the nasolacrimal system of an infant with congenital stenosis, i t is better to start with the superior canaliculus. The superior canaliculus possesses more maneuverable angles and hen ce is easier to probe. Furthermore, there is less functional significance if a false passage is created. L137 The average onset of action of botulinum toxin is 48 to 72 hours. Peak botu linum effect usually occurs 2 to 3 weeks after injection. L138 Any patient with sclerokeratitis and coexisting sinus disease should be sus pected for Wegener's granulomatosis. Ocular disease occurs in 60% of patients with the disease. Eighty percent of pat ients are positive for antineutrophilic

cytoplasmic antibodies, or cytoplasmic pattern (c-ANCA). The use of immunosuppre ssive drugs (especially cyclophosphamide) has significantly improved the mortality rates of patients wit h Wegener's granulomatosis. L139 Lymphangioma is not encapsulated, while schwannoma, cavernous haemangioma, and hemangiopericytoma are all encapsulated. L140 Schwannoma, an encapsulated tumour displaying the classic Antoni-A spindle cells and nuclear palisading (Verocay bodies). Schwannomas rarely undergo malignant transformation. They can recur aft er incomplete excision. L141 The neurilemoma (also called schwannoma) arises from Schwann cells. Slow gr owing and encapsulated, this yellowish tumor may show cysts and areas of hemorrhagic necrosis. It may be soli tary or associated with neurofibromatosis. Two histologic patterns appear microscopically: Antoni-A spin dle cells are arranged in interlacing cords, whorls, or palisades that may form Verocay bodies, or collections of fibr ils resembling sensory corpuscles. Antoni-B tissue is made up of stellate cells with a mucoid stroma. Vessels are u sually prominent and thick walled, and no axons are present. L142 Schwannomas, sometimes known as neurilemomas, are proliferations of Schwann cells that are encapsulated by perineurium. These tumors have a characteristic biphasic pattern of solid areas with nuclear palisading (Antoni A pattern) and myxoid areas (Antoni B pattern). Hypercellular schwannomas sometime s recur even after what is thought to be complete removal, but they seldom undergo malignant transformation. These tumors are usually well encapsulated and can be excised with relative ease. L143 Ocular stinging/burning is the most common side effect of topical cyclospor ine occurs in 15% of cases. L144 Topical cyclosporine is contraindicated in patient with active infection as active blepharitis. L145 The treatment of choice for patient with localized orbital lymphoproliferat ive lesion is radiation. Neither surgical excision nor systemic steroid is recommended. L146 Lesions may be associated with proptosis with the Valsalva maneuver or cryi ng. a. Orbital lymphangioma. b. Capillary hemangioma. c. Orbital varix. d. Encephalocele (outside lamkin). L147 Lymphangiomas. a. Lymphangiomas are primarily a disorder of the pediatric age range. b. Superficial lesions may have a bluish or violaceous hue. c. Classic presenting symptoms include spontaneous ecchymosis, and proptosis wit h crying or following upper respiratory infections. C. . UVEITIS UVEITIS L1 Passive immunization (parenteral administration of antibody) confers immediat e but short lived protection, whereas active immunization (vaccination with active or inactivated immunogens) confers protection that lasts months to years. L2 Typically, lipids and nucleic acids are not antigenic but may become so if co upled with proteins or polysaccharides. L3 IgM or IgM-like immunoglobulins tend to be the only type present in organisms

with the most rudimentary immune systems (relative to mammals). IgM immunoglobulin class is probably the oldest p hylogenetically. L4 The cyclooxygenase pathway leads to the production of prostaglandins, thrombo xane, and prostacyclins, whereas the lipoxygenase pathway leads to the production of leukotrienes. L5 Up to 90% of patients with ankylosing spondylitis are positive for HLA-B27. H LA-B27 is also associated with several other diseases, although the chance that an HLA-B27-positive individual will hav e a seronegative spondyloarthropathy or eye disease is approximately 25%. Note that 85% to 95% of patients with Reite r's syndrome are HLA-B27-positive as well. Fifty percent of patients with acute iritis may be HLA-B27-positive. L6 Dalen-Fuchs nodules are focal accumulations of epithelioid like cells between Bruch's membrane and the retinal pigment epithelium (RPE). They may include depigmented RPE cells. They are class ically associated with sympathetic ophthalmia (SO) and Vogt-Koyanagi-Harada (VKH) syndrome. They also may be found in tuberculous choroiditis and sarcoidosis. L7 Idiopathic iridocyclitis is the most common cause of anterior uveitis, making up at least 10% of all uveitis cases. HLAB27 iridocyclitis is the second most common cause, and juvenile rheumatoid arthr itis (JRA) and herpes (simplex or zoster) follow in incidence. L8 Toxoplasmosis is the most common cause of posterior uveitis, accounting for u p to 7.0% of total uveitis cases. Other causes of posterior uveitis include retinal vasculitis, necrotizing herpetic ret inopathy, and idiopathic causes. L9 The differential diagnosis of diffusely distributed keratic precipitates incl udes Fuchs' heterochromic iridocyclitis, and rarely sarcoidosis, syphilis, and toxoplasmosis. The diffuse distribution, a long with a gelatinous, stellate appearance, makes the keratic precipitates (KPs) of Fuchs' iridocyclitis distinc tive. L10 HLA-B27 is present in 1.4% to 6% of the general population. L11 Ritonavir is a protease inhibitor which can increase CD8+ lymphocyte counts. L12 The following conditions are associated with the HLA-B27 genotype. a. Inflammatory bowel disease. b. Reiter's syndrome. c. Ankylosing spondylitis.

d. Psoriatic arthritis. L13 Up to 25% of individuals with HLA-B27 develop sacroiliac disease. Symptoms o f sacroiliac disease may be subtle. Personal or family history of back problems in patients with iritis should promp t the physician to obtain sacroiliac radiographs. L14 Asymptomatic sacroiliac disease can be seen in patients with HLA-B27 spondyl itis, particularly in young men. Because irreversible damage may occur before the onset of significant symptoms a nd simple physical therapy is effective in limiting disability, physical therapy, consisting of back flexibili ty and stretching exercises, is recommended in young men who are found to be HLA-B27-positive. L15 Although it is associated with both forms of inflammatory bowel disease, iri tis occurs more commonly in patients with ulcerative colitis. Less than 15% of patients with ulcerative colitis and <5% of patients with Crohn's disease develop acute anterior uveitis. UC > Crohn's. L16 Acute anterior uveitis is associated with psoriatic arthritis, but usually n ot with psoriasis without arthritis. L17 Behçet's disease is much more common among Japanese and individuals from easte rn Mediterranean countries. Posterior uveitis, which can include retinal vasculitis, retinal hemorrhages, an d retinal necrosis, is more common than anterior uveitis in Behçet's disease. L18 Mild anterior uveitis with nonspecific symptoms is common in glaucomatocycli tic crisis (Posner-Schlossman syndrome). Although the symptoms may be mild, intraocular pressure (IOP) may be markedly elevated during the recurrent episodes, which may last several days. L19 Herpetic uveitis is more commonly associated with elevation of intraocular p ressure (IOP) than other types of uveitis. The differential diagnosis of uveitic glaucoma also includes sarcoidosis, zoster , Fuchs iridocyclitis, and rarely toxoplasma, syphilis, and sympathetic ophthalmia (SO). L20 Aspirin is the drug of choice for Kawasaki's disease. Although most patients recover without complication, approximately 3% of children with Kawasaki's disease develop acute coronary arte ritis, which may lead to myocardial infarction (MI) and death. Corticosteroids are contraindicated in Kawasaki's bec ause of the increased risk of coronary aneurysm formation. L21 Inflammation of the uveal tract lasting >6 weeks is defined as chronic uveit is. L22 The onset of anterior uveitis in the third decade of life of a patient with a remote history of pediatric arthritis does not preclude the diagnosis of JRA. L23 Parotid gland infiltration compresses the facial nerve as an innocent bystan der (remember that the terminal branches of the facial nerve arborize within the substance of the parotid gland). The Hee rfordt-Waldenstr.m syndrome describes fever, parotid enlargement, anterior uveitis, and facial nerve palsy secondary t o sarcoidosis. L24 Interstitial keratitis (IK) usually produces intense pain and photophobia. T he immune response in IK is felt to be an immune response to treponemal antigens (and not live organisms). Standard regime ns for neurosyphilis are sufficient to treat luetic IK. Although results of the rapid plasma reagin (RPR) and Venereal Disease Research Laboratory (VDRL)

tests may be negative in congenital syphilis, those of the fluorescent treponema l antibody-absorption (FTA-ABS) are usually positive. L25 Lyme immunofluorescent antibody titers and enzyme-linked immunosorbent assay (ELISA) for IgM and IgG are positive in only 40% to 60% of cases. The earliest eye finding is typically a fo llicular conjunctivitis. The most common eye finding is a chronic iridocyclitis with vitreous cells. L26 Tuberculosis is an uncommon but increasingly frequent cause of uveitis in th e United States. Tuberculous bacilli may be found histopathologically in eyes with tuberculous uveitis. Tuberculous uveit is may be present even with a normal purified protein derivative (PPD) and normal chest x-ray. For these cases, a sec ond strength (250 tuberculin units) skin test may be positive. Systemic corticosteroids may cause a dangerous flare-up in otherwise quiescent tuberculosis. L27 80% of cases of peripheral or intermediate uveitis are bilateral. Vitrectomy m ay be helpful in clearing media opacities and alleviating vitreous traction, but chronic cystoid macular edema ( CME) is often vision limiting. Hypotony caused by chronic ciliary body inflammation aggravates this. L28 Three common signs of endophthalmitis are decreased vision, hypopyon, and vi tritis. Pain is variable; if endophthalmitis is painful, the resolution of pain may indicate improvement of t he endophthalmitis. L29 Acute bleb-associated endophthalmitis Acute bleb-associated endophthalmitiscan occur at any time following successful filtration surgery. Pneumococcus (Streptococcus pneumoniae) and Haemophilus influenzae are the most frequent pathogens. L30 Serratia species produce severe acute postoperative endophthalmitis. Certain organisms are clearly implicated in chronic postoperative endophthalmitis and are associated with typical time cours es: a. Staphylococcus epidermidis, within 6 weeks. b. Candida species, 1 to 3 months. c. Propionibacterium acnes, 2 months to 2 years. L31 Most of the fungal endogenous endophthalmitis occurs without evidence of fun gemia. L32 One helpful finding distinguishing true infectious endophthalmitis from exag gerated inflammation following trauma or intraocular surgery is vitritis that is out of proportion to anterior chamber re action in the former. Considerable inflammation can occur postoperatively, but vitritis out of proportion to anteri or chamber reaction should provoke suspicion of infectious endophthalmitis.

L33 Gradually decreased vision, floaters, and scotomata are common symptoms of p osterior uveitis. Ciliary flush and spasm with brow-ache are more typical of anterior uveitis. L34 Granulomatous inflammation of the anterior segment can occur in toxoplasmosi s. Perivasculitis near active retinal lesions is common (kyrieleis arteriolitis). The classic lesion of toxoplasmosis is exudative focal retinitis. The definitive host for Toxoplasma gondii is the cat, where it is found as an intestinal parasi te. (The gondii is a small South American rodent, which is an important intermediate host in that region of the world). En zyme-linked immunosorbent assay (ELISA) testing for antitoxoplasma antibodies is important in the diagnosis of a typical lesions. L35 Potential adverse effects of the pharmacologic management of toxoplasmosis i nclude: a. Clindamycin is clearly associated with pseudomembranous colitis. b. Sulfa drugs can cause Stevens-Johnson syndrome, as well as either hemolytic o r aplastic anemia. c. Pyrimethamine can cause aplastic anemia (hence, the concurrent use of folinic acid). d. Steroid therapy can aggravate diabetes. L36 Onchocerca volvulus is transmitted by the bite of the Simulium black fly. Mi crofilariae, released by adult worms, penetrate the eye by both direct invasion and hematogenous spread. Microfilariae may be seen swimming in the anterior chamber and may induce a severe anterior uveitis with glaucoma and cata ract. Chorioretinal and optic atrophy are common in advanced disease. The larvae of Onchocerca volvulus form subcutane ous nodules when they develop into mature worms. This is one manifestation of onchodermatitis. L37 Like onchocerciasis, the pathology of cysticercosis is often dramatically wo rsened by death of the organism. Death of the larvae produces a severe inflammatory reaction, with granulomatous inflammat ion seen around the necrotic organism. L38 Iatrogenic immunosuppression, intravenous drug abuse, and in dwelling intrav enous catheters for hyperalimentation are risk factors for candidal infections. Candida endophthalmitis is less common in acquired immunodeficiency syndrome (AIDS) (mucocutaneous candidiasis is common). L39 Ocular infection by cytomegalovirus (CMV) may cause exudative or rhegmatogen ous retinal detachments, with holes in the area of retinal necrosis. Most common site of break near the area of necr osis. L40 Vision and electrophysiologic testing are usually normal after rubella retin itis. L41 Congenital measles can cause a retinitis with blindness 6 to 12 days after t he measles rash appears. Infants usually recover fully, but some patients progress to secondary pigmentary degeneration w ith poor prognosis. Similarly, although congenital syphilitic retinopathy can be associated with normal vision, it can a lso cause extensive retinopathy leading to visual loss. L42 Toxocara canis is an intestinal parasite of dogs and cats. Dogs are more com monly implicated in human infections. After ingestion of ova, larvae are spawned that will penetrate the intestinal wa ll and take up residence in the liver and lungs. From there, larvae can disseminate to any organ, including the eye. Eye i nvolvement is usually unilateral.

L43 Peripheral histo spots begin to appear around adolescence. The maculopathy u sually does not appear until the 20s. The early stage of the disease is thought to be a choroiditis. Vitreous cells ar e not seen in presumed ocular histoplasmosis syndrome (POHS). Visual complaints are caused by the maculopathy. L44 Although vitiligo and poliosis (known as the Sugiura sign) are classic for V ogt-Koyanagi-Harada (VKH) syndrome, they also have been reported in sympathetic ophthalmia (SO). The same is true of sensorineural hearing loss and cerebrospinal fluid (CSF) pleocytosis. VKH syndrome also may feature other centr al nervous system signs, such as nuchal rigidity, fever, coma, and seizures. In VKH syndrome, the chronic diffuse granulomatous uveitis involves the choriocapillaris, whereas in SO this layer is spared. Obviously, the two disease s overlap considerably. SO must be considered in patients with a history of previous eye surgery. L45 If inflammation has been well-controlled for at least 3 months before surger y, extracapsular cataract extraction with posterior chamber intraocular lens (IOL) implantation may be successful in many types of uveitic cataract. Young patients with cataract secondary to chronic uveitis associated with juvenile rhe umatoid arthritis (JRA) should not undergo IOL implantation after cataract extraction. Persistent postoperative uve itis aggravated by an IOL can be devastating. L46 Patients with acquired immunodeficiency syndrome (AIDS) exhibit absolute lym phocytopenia, elevated immunoglobulin (especially IgA and IgG), and increased suppressor T-cell counts, particularly relative to helper T cells. Although there is an absolute lymphocytopenia, there may be no leukocytopenia or granulocytopenia. L47 Over 65% of patients with acquired immunodeficiency syndrome (AIDS) develop some ocular abnormality. Human immunodeficiency virus (HIV) retinopathy which occurs in >50% of cases is the mo st common ocular finding in patients with AIDS. One series reported that up to 92% of patients with AIDS will develop cotton-wool spots. L48 Usually, cases of ocular toxoplasmosis in patients with acquired immunodefic iency syndrome (AIDS) rarely show the typical fundus scar of a previous infection. Primary ocular toxoplasmosis, assoc iated with central nervous system (CNS) toxoplasmosis, is more common in patients with AIDS. Thus, patients with AIDS di agnosed with ocular toxoplasmosis should undergo brain magnetic resonance imaging (MRI) to rule out further CNS in volvement. L49 The occurrence of cytomegalovirus (CMV) retinitis in a patient with acquired immunodeficiency syndrome (AIDS) is thought to confer a poor prognosis because many patients die within months after onset of retinitis. Yet, the mean survival of patients with AIDS and newly diagnosed CMV retinitis has increased a s a result of newer therapies (e.g., highly active antiretroviral therapy [HAART], protease inhibitors). CMV retiniti s usually occurs with CD4 counts <50 cells/mm3.

L50 The development of herpes zoster ophthalmicus (HZO) in a young and otherwise healthy patient should raise the suspicion of immunocompromise (leukemia, chemotherapy, acquired immunodeficiency syndrome [AIDS]). Sexually transmitted diseases tend to occur together. This is particularly true of syphil is and human immunodeficiency virus (HIV). If a clinician obtains serologic studies for one disorder (lues or AIDS), he or she should strongly consider testing for the other as well (joint fluorescent treponemal antibody-absorption [FTA-ABS ] and anti-HIV titers). L51 Kaposi's sarcoma may be noted on the eyelid skin or conjunctiva. Skin lesion s usually appear as nontender, elevated, purple nodules. Conjunctival involvement is manifested by red subconju nctival masses. L52 Drug of choice for prophylaxis of Pneumocystis carinii pneumonia (PCP) in hu man immunodeficiency virus (HIV)positive patients. Bactrim is effective and has a relatively low cost. Unlike ae rosolized pentamidine, it is a systemic treatment and may also be effective for treating extrapulmonary pneumocystis (e. g., choroidal). L53 The percentage of patients with documented candidemia will develop Candida e ndophthalmitis, defined as at least some degree of vitreous inflammation is > 10%. (Earlier series documented rates approaching 40%, but these studies defined virtually every fundus finding as endophthalmitis, including nonspecific retinal vascular changes such as Roth's spots and nerve-fiber layer infarcts. The distinction is important because chori oretinal involvement without vitreous infection will respond to system antifungals, obviating the need for vitrectomy and/or intravitreal antibiotics. The reason for the lack of vitreous involvement is presumably the response to systemic anti fungal agents). L54 In a recent study, the risk of future multiple sclerosis (MS) following a di agnosis of pars planitis exceeds 16%. The risk was significantly greater among patients presenting with retinal vascular s heathing. L55 Clinical features distinguishing the progressive outer retinal necrosis (POR N) syndrome from the acute retinal necrosis (ARN) syndrome include: a. Initial involvement of the outer retina. b. Relative lack of intraocular inflammation. c. Relative lack of vasculitis. d. Earlier involvement of the macula. L56 Both progressive outer retinal necrosis (PORN) and acute retinal necrosis (A RN) are manifestations of severe posterior segment infection by the herpes family of viruses. ARN, particularly i n immunocompetent hosts, responds to intravenous acyclovir more reliably than PORN. Up to two thirds of PORN eyes wil l end up with no light perception (NLP), despite treatment. L57 One potential complication of cryotherapy for pars planitis is rhegmatogenou s retinal detachment. Laser photocoagulation may have the same salutary effect on resistant pars planitis as cryotherapy, but without the risk of retinal detachment (RD). The following are therapeutic options for pars planitis . (Treatment is usually started if cystoid macular edema (CME) is evident, or if visual acuity is <20/40). a. Subtenon steroid. b. Oral steroids.

c. Peripheral retinal cryoablation. d. Peripheral scatter laser treatment. L58 Identifiable causes of pars planitis include syphilis, tuberculosis, Lyme di sease (rare), sarcoidosis, toxocariasis, and intermediate uveitis associated with multiple sclerosis. However, most cases are idiopathic. L59 Although human immunodeficiency virus (HIV) has been detected in corneal epi thelium, as of 2004 there has never been a reported case of HIV transmission from corneal transplantation. L60 Up to 25% of patients with primary central nervous system lymphoma (PCNSL) w ill have ocular involvement. L61 Systemic corticosteroids and immunosuppressive agents initially reduce the a ppearance intraocular inflammation in patients with ocular primary central nervous system lymphoma (PCNSL). Although s ystemic corticosteroids and immunosuppressive agents can reduce intraocular inflammation initially, they wil l eventually fail. Unfortunately, the use of these agents can sometimes cause vitrectomy specimens to be nondiagnostic. L62 Retinal metastases are rare. Retinal metastases are most commonly from a pri mary cutaneous melanoma. D. . GLAUCOMA, LENS, AND ANTERIOR SEGMENT GLAUCOMA, LENS, AND ANTERIOR SEGMENT L1 Approximately 1% of the anterior chamber volume turns over every minute. The anterior chamber volume is approximately 200 to 250 microliters, and the rate of aqueous formation is appro ximately 2 to 3 .L/min. L2 Increasing age, ocular injury or inflammation, and ocular surgery can all dec rease the rate of aqueous humor formation. L3 Uveoscleral outflow is pressure independent range as low as 10% and as high a s 50% is some cases. L4 Normal outflow facility in is 0.22-0.28 .l/min/mmHg. L5 Normal ranges of episcleral venous pressure 8-12 mmHg. L6 Acute and not chronic elevation of EVP leads to elevation of equal magnitude in IOP. L7 The range of IOP in general population is not valid (not have gasserian distr ibution), it skewed toward higher IOP. L8 Typical range of diurnal fluctuation is 2-6 mmHg (it differs from person to p erson); more than 10 mmHg (some time exceeding 20 mmHg) is suggestive of glaucoma. L9 IOP in patient with glaucoma is not always similar in both eyes; hence, the u se of monocular topical pharmacotherapy trial may not necessarily prove or disprove efficacy of a particular agent. L10 Applanation tonometry unlike Schi.tz tonometry display very small amount of aqueous from the eye and doesn t increase IOP. For this reason it is independent on ocular rigidity.

L11 Perkin and tonopen tonometer both can be used in supine and erect position. L12 Plateau iris is an unusual type of PAC caused by anteriorly positioned cilia ry processes that critically narrow the anterior chamber recess by pushing the peripheral iris forward. L13 Patients who have plateau iris syndrome (e.g., resistant to laser peripheral iridotomy) should be treated with iridoplasty. L14 Gonioscopy is required to visualize the chamber angle because under normal c onditions light reflected from the angle structures undergoes total internal reflection at the tear-air interface ( not at the aqueous-corneal endothelium interface). At the tear air interface, the critical angle (approximately 46°) is r eached and light is totally reflected back into the corneal stroma. This prevents direct visualization of the angle structures. All gonioscopy lenses eliminate the tear-air interface by placing a plastic or glass surface adjacent to the front surface of the eye. L15 Koeppe, Barkan, Wurst, Swan-Jacob, and Richardson are direct gonioscopy. L16 Koeppe is considered best for evaluating a patient with potential angle rece ssion because this system allows easier comparison of one eye with other and one portion of the angle with other. Koeppe also it is least likely to distort AC angle anatomy. L17 The Koeppe lens is least likely to distort the anterior chamber anatomy. Pos terior pressure on the Goldmann lens may falsely narrow the angle by indenting the sclera. Posterior pressure with th e Zeiss lens will push the aqueous from the center to the periphery of the anterior chamber (AC). L18 Because of their smaller diameter Zeiss and Sussman may be used in indentati on gonioscopy. This maneuver distinguishes appositional angle closure from synechial angle closure by artific ially deepening AC with digital pressure on the lens. Goldmann lens are too large for this. L19 The inferior angle (viewed through the superior mirror on a Zeiss gonioprism lens) is wider and is thought to be the easiest portion for distinguishing landmarks. L20 Blood may reflux into Schlemm's canal and be visible on gonioscopy; Patholog ic causes include hypotony and elevated episcleral venous pressure, as in carotid cavernous fistula or Sturge-W eber syndrome. Also in severe thyroid eye diseases and excessive digital pressure on the globe by Goldmann lens. L21 Two hypotheses have emerged to explain the development of glaucomatous optic neuropathy, the mechanical and ischemic theories. a. The mechanical theory stresses the importance of direct compression of the ax onal fibers and support structures of the anterior optic nerve, with distortion of the lamina cribrosa p lates and interruption of axoplasmic flow resulting in the death of the RGCs. b. The ischemic theory focuses on the potential development of intraneural ische mia resulting from decreased optic nerve perfusion. This perfusion may result from the stress of IOP on the b lood supply to the nerve or from processes intrinsic to the optic nerve. L22 If glaucomatous visual field type defects corresponding optic nerve abnormal ities should exist. Otherwise alternative etiologies should be considered. L23 Baring of the blind spot and generalized constriction are not very specific and can be produced by miosis,

uncorrected refractive error, aging, and cataract. L24 Areas of retina and/or optic nerve damaged by glaucoma are believed to be mo re vulnerable to ongoing damage at lower intraocular pressures (IOP), thereby; field defects tend to become more se vere with time. New defects also may appear, of course, but generally accompany progression of previous defects. L25 Split fixation is the presence of visual field defect that comes close to fi xation. The eye at greatest risk to loss of fixation is with the split-fixation in the horizontal meridian. L26 A typical pattern of progression is loss of near fixation (paracentral scoto ma) to. split fixation to. loss of fixation. L27 A cluster of 2 or more points depressed .5 dB compared with surrounding poin ts is suspicious. A single point depressed >10 dB is very unusual but is of less value on a single visual field t han a cluster, because cluster points confirm one another. Corresponding points above and below the horizontal midline should not vary markedly; normally the superior field is depressed 1-2 dB compared with the inferior field. L28 The following should raise the doubts about the diagnosis of glaucoma: a. An optic disc that is less cupped than would be expected for observed field l oss. b. Pallor of the disc that is more impressive than the cupping. c. Marked asymmetric dyschromatopsia. d. Visual field defect uncharacteristic for glaucoma (respecting vertical meridi an). L29 RAPD can occur with glaucoma. L30 POAG occurs more frequently in African Americans than in whites and is the l eading cause of blindness in African Americans. L31 Condition misdiagnosed as NTG: a. POAG. b. Chiasmal compression. c. AAION. d. Toxic optic neuropathy (such as methanol toxicity). e. Hypotension (shock optic neuropathy).

L32 Therapy for normal-tension glaucoma generally is more aggressive, with a low er target intraocular pressure (IOP) than therapy for primary open-angle glaucoma (POAG). L33 Features that distinguish pseudoexfoliation glaucoma from primary open-angle glaucoma (POAG) include: a. Greater sensitivity to laser therapy. b. Greater degree of interocular asymmetry. c. Degree of trabecular meshwork (TM) pigmentation. L34 In pseudoexfoliation, fibrillar material is deposited in the anterior segmen t of the eye. Patients with this glaucoma are often resistant to medical therapy, but laser trabeculoplasty is often very effe ctive. Pseudoexfoliation with glaucoma also differs from primary open-angle glaucoma (POAG) in that it is often monocular or asymmetric and has greater pigmentation of the trabecular meshwork (TM), as well as pigment deposited anter ior to Schwalbe line (Sampaolesi line). L35 There is considerable overlap in the age range of patients affected by POAG and PEX glaucoma. L36 The sine qua non of the pigmentary glaucoma is radial defects in the iris pi gment epithelium. L37 Phacolytic glaucoma is the least likely glaucoma to respond to medical thera py alone. L38 Fuchs' heterochromatic iridocyclitis is considered secondary open-angle glau coma. The glaucoma can be difficult to control and does not parallel the degree of inflammation. The rubeosis in thi s condition is odd the vessels are particularly prone to bleed, but do not induce synechialization. L39 Haemolytic glaucoma may occur within days of haemorrhage, whereas ghost cell glaucoma is seen weeks to months later. L40 Patient with glaucoma is more likely to have diabetes and patient with diabe tes are more likely to have glaucoma. L41 Angle closure glaucoma generally occurs after full dilatation of the pupil, as the iris shrink to mid position. This is the region of maximal iris lens contact. L42 The increased intraocular pressure (IOP) that occurs during an attack of ang le closure can cause ischemia of the iris and may produce stromal atrophy. Small anterior subcapsular lens opacities, or g laukomflecken, also may develop as a direct result of pressure-induced lens epithelial death. Optic disc cupping may be seen if the attack is prolonged or severe, but this is not specific. L43 Both mydriatics and miotics can precipitate angle-closure in eyes with shall ow anterior chambers. This is true for both topical medications and systemic drugs that affect the pupil. Examples incl ude antihistamines, which can have anticholinergic activity. L44 The prone dark room test is the most predictive test in evaluation patient w ith susceptible narrow angle. IOP is measured before and after 30-60 minute of total dark adaptation and the patient prone. PRONE implies a position with the front of the body turned toward the supporting surface. L45 The glaucoma associated with iridocorneal endothelial (ICE) syndrome is ofte n worse than predicted by the extent of synechiae, likely because of clinically undetectable endothelialization of the a ngle. L46 The patient with very narrow angles and elevated pressure may have mixed mech anism glaucoma with partial

angle closure caused by pupillary block superimposed on open-angle glaucoma. To determine if an angle-closure component is present, the effect of minimizing pupillary block on intraocular pr essure (IOP) must be determined. Cholinergic miotics (pilocarpine) will cause miosis and lessen pupillary block a nd will also exert traction on the trabecular meshwork (TM) and lower IOP by this unrelated mechanism. Thymoxamine, a selective alpha-adrenergic antagonist, causes miosis and lessens pupillary block, without affecting outflow facility. A decrease in pressure after thymoxamine (lessened pupillary block) implies partial angle closure, and iridot omy is indicated. No change in IOP after thymoxamine-induced miosis implies that an iridotomy may not be helpful. L47 Trabeculectomy performed at the inferior limbus has been associated with a h igher risk of bleb-related endophthalmitis compared with trabeculectomy at the superior limbus due to pooli ng of infectious agents in the tear film inferiorly. L48 Initial treatment in patient with malignant treatment includes cycloplegic. Vitrectomy in phakic eye is used to alter the anterior vitreous face. In aphakic or pseudophakic, the anterior vitreous can be disrupted by laser treatment. L49 In PACG, the central chamber is deeper than the peripheral angle caused by p upillary block, in malignant glaucoma vitreous enlargement by aqueous humour causes the entire chamber to become shall ow. L50 Increase suspicious of plateau iris in patient with angle closure are deep a nterior chamber centrally, a young patient with myopia and a flat iris plane. L51 The pigmented lesion in the Cogan Reese syndrome represents a nodular collec tion of stromal melanocytes and not actually naevus cells. L52 The pigmented lesions in the Cogan-Reese syndrome represent actually nodular collections of stromal melanocytes, not nevus cells. L53 Yanoff. The predominant cell type in most iris nevi is the spindle nevus cel l. In contrast, the predominant cell type in most choroidal and ciliary body nevi is the magnocellular nevus cell. Many uveal nevi consist of an admixture of more than one cell type. Balloon nevus cells typically occur in the yellow-orange hal o that surrounds some choroidal nevi. Dendritic melanocytes, which occur in some uveal nevi, in Lisch nodules of neuro fibromatosis type 1, and in the iris nodules of the Cogan-Reese syndrome and iridocorneal endothelial syndrome, appea r to be normal uveal melanocytes and not nevus cells. L54 Epithelial downgrowth is generally more aggressive than fibrous downgrowth.

L55 Scleral buckling operations, especially encircling bands: a. Can produce shallowing of the anterior chamber angle and frank angle-closure glaucoma, often accompanied by choroidal effusion and anterior rotation of the ciliary body, cau sing a flattening of the peripheral iris with a relatively deep central anterior chamber. b. Usually, the anterior chamber deepens with the opening of the anterior chambe r angle over days to weeks with therapy of cycloplegics, anti-inflammatory agents, beta-adrenergic antagoni sts, carbonic anhydrase inhibitors, and hyperosmotic agents. c. If medical management is unsuccessful, argon laser iridoplasty, drainage of s uprachoroidal fluid, or adjustment of the scleral buckle may be required. d. Iridectomy is usually of little benefit in this condition. e. The scleral buckle can impede venous drainage by compressing a vortex vein, i ncreasing episcleral venous pressure and IOP. f. Only by shifting the scleral buckle or releasing the band can this elevation in IOP be treated permanently. g. Another mechanism of angle closure following buckling surgery is neovascular glaucoma caused by buckle induced ocular ischemia. L56 Following panretinal photocoagulation, IOP may become elevated by an angle c losure mechanism. The ciliary body is thickened and rotated anteriorly, and often an anterior annular choroidal det achment occurs. Generally, this secondary glaucoma is self-limited, and therapy is directed at temporary medical management with cycloplegic agents, topical corticosteroids, and aqueous suppressants. L57 Central retinal vein occlusion (CRVO) sometimes causes early shallowing of t he chamber angle, presumably because of swelling of the choroid and ciliary body. In CRVO there may be transu dation of serum into the vitreous, driven by elevated intravascular pressure. This hydration causes vitreous swelli ng with subsequent secondary angle closure. L58 Primary congenital glaucoma, which accounts for approximately 50%-70% of the congenital glaucomas (no associated ocular or systemic conditions), occurs much less frequently than prim ary adult glaucoma, and primary infantile glaucoma is believed to be rare (1 in 10,000 births). Of pediatric gla ucoma cases, 60% are diagnosed by the age of 6 months and 80% within the first year of life. Approximately 65% of pati ents are male, and involvement is bilateral in 70% of all cases. L59 In primary childhood glaucoma, the anterior chamber is characteristically de ep with normal iris structure. Findings include a high and flat iris insertion, absence of angle recess, peripheral iris hypoplasia, tenting of the peripheral iris pigment epithelium, and thickened uveal trabecular meshwork. The angle is typica lly open, with a high insertion of the iris root that forms a scalloped line as a result of abnormal tissue with a shag reened, glistening appearance. This tissue holds the peripheral iris anteriorly. The angle is usually avascular, but loops of vessels from the major arterial circle may be seen above the iris root. L60 Infants with congenital glaucoma may also have an impermeable TM. L61 Haab striae and enlarged corneal diameter only occur in infantile glaucoma.

L62 The proper pediatric dose of CAI is 15 mg/kg/day in three or four divided do ses. For small children it leads to weight loss, lethargy, and metabolic acidosis.kotobarabia L63 The most common side effect of CAI is paraesthesia. L64 There is a danger of precipitating angle closure in individuals with very na rrow angles through the use of any medication with mydriatic action. In the case of plateau iris with narrow angles , dilation causes the peripheral roll of iris to bunch up and obstruct the trabecular meshwork (TM). With narrow angles, mydrias is may increase pupillary block, most commonly as the agent is wearing off. L65 The agent most likely to cause topical sensitization and medicamentosa is ep inephrine. Epinephrine has a wellestablished tendency to provoke irritation and allergic responses. More than one fifth of patients will eventually experience an adverse local reaction with prolonged use. L66 Beta antagonists, in general, can cause fatigue, dizziness, and depression a t therapeutic doses. Carbonic anhydrase inhibitors share an ability to cause lethargy, malaise, and depression. Either o f these groups should be used with caution in patients with a history of significant depression. L67 Mannitol is distributed only in the blood compartment, whereas urea moves fr eely in total body water. As a result, mannitol can generate a greater osmotic gradient than urea because its intravasc ular concentration remains greater. The prevalence of intravenous urea as a hyperosmotic agent to control intraocula r pressure (IOP) is decreasing in part because of its risk for tissue necrosis with extravasation. L68 Argon iridotomies are more likely to close spontaneously than neodynium supp orted by yttrium-aluminum-garnet (Nd: YAG) iridotomies. Argon laser is often useful for pretreating dark irides b efore Nd: YAG iridotomy because Nd: YAG iridotomy on heavily pigmented irides is difficult. The most common complica tion of both Nd: YAG and argon laser iridotomies is acute glaucoma. Malignant glaucoma has been reported after a vari ety of seemingly benign ocular laser procedures. L69 The success rate after ALT during the first year of life is 80%. L70 Spherical shape (Microspherophakia) increases relative pupillary block by in creasing the degree of iris-lens contact. Miotics further exacerbate this by shifting the iris-lens diaphragm for ward and further increasing iris-lens

contact (as the lens becomes even more spherical). Cycloplegia moves the diaphra gm backward and flattens the lens. It is the medical treatment of choice. L71 Simple ectopia lentis is inherited as AD bases. L72 Anterior polar cataract are typically bilateral usually don t impair vision. N uclear cataract is usually bilateral and often causes severe visual impairment. L73 Capsular cataract is usually the least likely congenital cataract to affect vision. L74 Cataracts may occur both in galactosemia (galactose-1-phosphate uridyl trans ferase deficiency) and galactokinase deficiency. Galactosemia, however, also can feature hepatosplenomegaly, mental r etardation, and other systemic manifestations. L75 The following syndromes featuring congenital or infantile cataract may be as sociated with facial abnormalities. a. Craniosynostosis. b. Hallermann-Streiff syndrome. c. Stickler's syndrome. L76 A younger patient's lens epithelial cells are more actively growing than an older patient's and are thus more susceptible to radiation damage. L77 Glare can be disabling to patients with posterior subcapsular cataract (PSC) . Thus, standard dark room visual acuity may overestimate their functional acuity. This is the basis of the brightness ac uity test (BAT), which quantifies visual acuity under low and high ambient illumination. L78 High intraocular pressure (IOP) secondary to angle closure may be causing se vere eye pain and decreased visual acuity. Hypotony (wound leak, cyclodialysis) could precipitate painful choroidal hemorrhage. L79 In PEX syndrome cataract surgery may be more hazardous than in other patient s because of incomplete pupillary dilatation, abnormal weak lens zonule, and a thickened or tough anterior capsule . L80 Failure to relieve postoperative angle closure with iridotomy suggest malign ant glaucoma, which often respond to topical potent cycloplegic. L81 Vitreous prolapse into the AC may lead to localized or diffuse corneal oedem a, open angle glaucoma, CME and RD. L82 A recurrent or delayed hyphaema after uncomplicated cataract surgery is usua lly caused by vascularization of the wound or an implant that is rubbing against the iris. Gonioscopy is useful in id entifying these vessels. Laser photocoagulation can then be used to ablate the offending vessels. L83 CME. a. Angiographically detectable CME is present 10-20% of patients following ECCE. Visually significant CME following ECCE is present in 1-2%. b. Following ICCE is 40-60% and 2-10%, respectively. L84 Postoperative endophthalmitis is most commonly 2-5 days post cataract and mo st commonly caused by staph epidermidis. L85 Anterior chamber intraocular lenses with closed support loops are associated with a higher incidence of complications. They are no longer used because of this reason. L86 Posterior capsule opacification, glaucoma (angle closure caused by excessive inflammation), and retinal detachment (caused by vitreous loss) are more common postoperative complications in childre

n undergoing cataract extraction. The incidence of cystoid macular edema (CME) is lower in children than in adults. L87 The plasticity of the visual system in children allows them to adapt to the various distortions inherent in aphakic spectacles more readily than adults. L88 Electrical injury primarily cause cataract, typically involve changes in the anterior subcapsular cortex. L89 Corneal blood staining is a complication of hyphema that may take years to c lear. The blood staining clears in a centripetal pattern (starting at the periphery). African Americans with hyphema should be checked for sickle cell disease or trait because the sickled cells may become trapped in the trabecular meshwork (TM), increasing the intraocular pressure (IOP). Furthermore, the optic nerve is more susceptible to atrophy in p atients with sickle cell disease, even at relatively mild elevations of IOP. L90 Indications of intervention (medical or surgical) in lens subluxation or dis location are polyopia, pupillary block, and corneal lenticular touch. L91 Features of ocular siderosis include iris heterochromia, brunescent cataract , mydriasis, loss of the b-wave on ERG. L92 Antimetabolite glaucoma therapy. Typically, the surgeon should fastidiously avoid contact between the Mitomycin-C (MMC)-containing sponge and the wound edges to reduce the risk of postoperative wound leaks and hypotony. This is one of the main reasons why most MMC filters are performed with a limbus-based f lap. MMC is associated with a lower incidence of hypotony and corneal-surface disorders than 5-fluorouracil (5-FU). L93 Tonopen appear to have more accurate pressure readings after LASIK. Goldmann tonometry can be artificially low after LASIK. L94 Dexamethasone 0.1% causes highest rise in IOP compared to other steroid incl uding prednisolone 1%. L95 Lamellar cataracts are the most common type of congenital cataracts and may occur as a result of toxic influence to the fetus or may be inherited as an autosomal dominant trait. L96 Selective laser trabeculoplasty (SLT). a. Selective laser trabeculoplasty (SLT) is a relatively new procedure with effe cts for lowering intraocular pressure (IOP) comparable to argon laser trabeculoplasty (ALT).

b. It has less disruptive effects on intraocular tissue than ALT. c. Unlike argon laser trabeculoplasty (ALT), the procedure can be repeated for s ignificant pressure lowering effects SLT. d. Targets the melanin-containing cells within the trabecular meshwork (TM). e. Patients can receive up to two SLT treatments every year. L97 Generally uncomplicated cataract surgery (phaco and PCIOL) lower IOP by 1-2 mmHg. L98 Relative contraindications for glaucoma filtering surgery can be ocular or s ystemic. A blind eye should not be considered for incisional surgery. Ciliary body ablation is a better alternative for lowering IOP in such eyes if necessary for pain control, although even this procedure is not without risk. The risk of sympathetic ophthalmia should always be kept in mind when any procedure on a blind eye or an eye with poor visual potent ial is considered. L99 The common side effect of topical prostaglandin analogs. a. Eyelash hypertrichosis. b. Conjunctival hyperemia. c. Darkening of periocular skin. d. Cystoid macular edema (CME). L100 Spontaneous hyphema DDX includes JXG, child abuse, bleeding diatheses, leuk emia, and retinoblastoma. E. . CORNEA, EXTERNAL DISEASE AND REFRACTIVE SURGERY CORNEA, EXTERNAL DISEASE AND REFRACTIVE SURGERY L1 Conjunctival papillae <1mm is nonspecific of aetiology, when it becomes giant >1mm it becomes more specific. Chemosis is subepithelial oedema and it is nonspecific. L2 Giant papillae in VKC are more solid appearing, with sharper margin and great er elevation. In AKC are less well defined, less elevated, and creamier in appearance. L3 The papillae in contact lens related giant papillary conjunctivitis may last for several months or years after the symptoms are controlled. L4 A unilateral or bilateral conjunctivitis that persists for 4 or more weeks is considered chronic. When conjunctivitis accompanies blepharitis, as it frequently does, the condition is known as staphy lococcal blepharoconjunctivitis. This association is marked by a chronic papillary reaction of the tarsal conjunctiva, particularly the inferior tarsal conjunctiva near the eyelid margin, as well as injection of the bulbar and tarsal conjunctiv a. Chronic conjunctivitis tends to be associated with mild conjunctival injection and scant mucopurulent discharge. L5 The differential diagnosis for acute follicular conjunctivitis includes: a. Epidemic keratoconjunctivitis (EKC). b. Herpes simplex keratoconjunctivitis. c. Medicamentosa. d. Adult inclusion conjunctivitis. L6 The differential diagnosis for chronic follicular conjunctivitis includes all of the following except: a. Medicamentosa. b. Parinaud's oculoglandular syndrome. c. Benign folliculosis. d. Trachoma. L7 Atopic keratoconjunctivitis rarely comes with a pseudomembrane whereas vernal keratoconjunctivitis may. This is a feature that may help in differentiating between the two types 1 hypersensitivit y causes of giant papillary conjunctivitis (GPC).

L8 Two pathophysiologic factors are felt to be necessary for the development of corneal epithelial filaments increased mucus production along with increased, deranged epithelial turnover. The filamen ts themselves are composed of mucus and desquamated epithelial cells. L9 Entities to consider in the DDX of filamentary keratitis are ptosis, neurotro phic keratopathy, recurrent erosion syndrome, and medicamentosa. L10 OCP not associated with filamentary keratitis because OCP is associated with decreased mucous production. L11 Granulomatous KPs may have characteristic greasy, mutton-fat appearance, wit hout actually being granulomatous themselves. Macrophages are responsible for the clinical appearance. L12 Endophthalmitis rates are higher with laser-assisted in situ keratomileusis (LASIK) than with photorefractive keratectomy (PRK). L13 AAO. Mycobacteria keratitis treated by Amikacin 20 mg/mL topically and 20 mg in 0.5 mL subconjunctivally. L14 Yanoff. Nontuberculous mycobacterial keratitis has been successfully treated with fluoroquinolone antibiotics, and some infections respond well to topically applied amikacin. Combination therapy of amikacin and a fluoroquinolone should be considered. Prolonged therapy of these recalcitrant infections is ofte n necessary and, in certain cases, lamellar or penetrating keratoplasty might be necessary L15 The standard antimicrobial agents for mycobacterium tuberculosis are isoniaz id, ethambutol, and rifampin. Aminoglycosides are also effective, particularly against some of atypical mycoba cteria. L16 Syphilis. a. Treponema pallidum causes venereal syphilis. b. By dark-field illumination, T. pallidum appear fine and corkscrew shaped, wit h rigid, uniform spirals.

c. Aside from direct identification of T pallidum by dark-field techniques, the diagnosis of syphilis is based on specific serologic tests. d. The serologic microhemagglutination assay for T. pallidum (MHA-TP) and fluore scent treponemal antibody absorption (FTA-ABS) test are specific, but not reliable for testing cerebrospin al fluid. e. The rapid plasma reagin (RPR) and Venereal Disease Research Laboratory (VDRL) assays are nonspecific but are useful for assay of cerebrospinal fluid in patients with suspected neuro syphilis. f. The MHA-TP and FTA-ABS tests remain reactive for life, while the VDRL and RPR titers drop with antibiotic therapy and thus can be used to assess the response to therapy. g. In clinical practice, MHA-TP or FTA-ABS reactivity is used to confirm the cli nical suspicion of syphilis, and the quantitative RPR (or VDRL) test is performed to determine disease activity a nd to compare with subsequent testing following treatment. L17 The organism responsible for Lyme disease, Borrelia burgdorferi, is notoriou sly difficult to culture. Serology remains superior to culture techniques. L18 The most common ocular manifestation of cryptococcal infection is endogenous endophthalmitis. L19 Mold is the most common fungus encountered in fungal keratitis, second are y east, dimorphic as Histoplasma are rare. L20 Most periocular herpes infections are type 1. Antibody titers may be helpful in cases of primary disease, but not secondary (reactivation) cases. L21 Multinucleated giant cells seen in epithelial HSV infection are best seen wi th Giemsa stain. This known as the Tzanck prep, (remember Tzanck God, I don t have herpes). L22 In laser-assisted subepithelial keratomileusis (LASEK), laser ablation is pe rformed under an epithelial flap. The epithelium is trephined and peeled back after focal placement of 20% ethanol ont o the trephination site through a well. This procedure may offer both decreased risk because no stromal flap is created and increased comfort because the corneal epithelium is preserved. L23 Like Lyme disease, culture for chlamydial disorders is difficult. Detection of antigen in tissue scrapings using immunofluorescent techniques is the diagnostic standard. L24 . Adult chlamydial conjunctivitis Adult chlamydial conjunctivitis a. Onset of conjunctivitis is typically 1-2 weeks after ocular inoculation and i s not as acute as adenoviral keratoconjunctivitis. Often patients may complain of mild symptoms for weeks to months. b. Signs of adult inclusion conjunctivitis include a follicular conjunctival res ponse that is most prominent in the lower palpebral conjunctiva and fornix, scant mucopurulent discharge, and palpab le preauricular adenopathy. c. Inflammatory conjunctival membranes do not develop in chlamydial keratoconjun ctivitis. d. Corneal involvement may consist of fine or coarse epithelial infiltrates, occ asionally associated with subepithelial infiltrates. The keratitis is more likely to be found in the super

ior cornea but may also occur centrally and resemble adenoviral keratitis. A micropannus, usually extending le ss than 3 mm from the superior cornea, may develop. L25 The noncontact holmium- YAG laser can heat and shrink corneal stroma (laser thermokeratoplasty), thus producing corneal steepening and correction of hyperopia or hyperopic astigmatism. L26 The contact holmium-YAG laser was withdrawn from the market because of regre ssion and unpredictability of effect. Clinical investigations are continuing into the use of diathermy for correction of hyperopia. L27 Acyclovir and trifluridine has a mechanism of action specific for herpes sim plex viruses with scarcely any effect on the host cells. L28 Topical antiviral toxicity presents most commonly as diffuse punctate cornea l epithelial erosions with conjunctival injection. Neurotrophic keratopathy may develop in those patients w ith reduced corneal sensation secondary to past herpetic infection. Punctate epithelial erosions, sometimes wi th a vortex pattern of punctate fluorescein staining, chronic epithelial regeneration lines, and frank neurotrop hic ulcers characterize neurotrophic keratopathy. These ulcers can be distinguished from herpetic epithelial keratiti s by a relative absence of rose-bengal staining. Topical antiviral toxicity also causes preauricular lymphadenopathy. L29 Idoxuridine and vidarabine inhibit DNA synthesis by acting as false nucleic acids (analogs). Trifluridine inhibits thymidylate synthetase, a herpes-specific enzyme. Acyclovir, a guanosine analog, is activated only by thymidine kinase, another virus-specific enzyme. Both thymidylate synthetase and thymidine kinase are critical for herpetic DNA synthesis. L30 The following antivirals have a mechanism of action specific for herpes viru ses, with scarcely any effect on host cells. a. Trifluridine. Trifluridine (Viroptic) inhibits another virus-specific enzyme, thymidylate synthetase. At high concentrations, it may act as a false nucleoside and affect human DNA synthesis as well. b. Acyclovir. Acyclovir is activated by viral thymidine kinase and is relatively nontoxic to mammalian cells. L31 Polyene class of antifungal agents. a. They are classified by the number of double bonds present. b. Amphotericin has less predictable activity against filamentous species than n atamycin. c. The corneal epithelium is a significant barrier to penetration of natamycin.

d. Acceptable intraocular concentrations of polyene antifungals can only be achi eved with direct intravitreal injection. e. Topical amphotericin must be stored in dark glass or opaque containers. L32 Flucytosine commonly induces acquired fungal resistance. L33 The drug of choice for Aspergillus keratitis is clotrimazole. L34 . Polyenes are insoluble in water and have systemic The target of toxicity f rom the polyene class of antifungal agents The target of toxicity from the polyene class of antifungal agentstoxicity by bi nding to renal tubular cells and erythrocytes. L35 is hepatic; therefore liver enzymes should be monitored during The target of the imidazole class of antifungal agents The target of the imidazole class of antifungal agents long-term ketoconazole therapy. L36 are currently being used for the treatment of low myopia. They are also bein g studied as a Intracorneal rings (ICR) Intracorneal rings (ICR) potential treatment of keratoconus. The cornea's positive asphericity is maintai ned with the procedure. More than 90% of patients have <1D of change in refraction at 1-year postoperative follow-up. Corneal perforation is a potential complication of the procedure but occurs less frequently than decreased corneal sensation. L37 Yanoff. Intracorneal rings. The adverse event rate was 1.1% including infect ious keratitis (0.2%), shallow placement (0.2%), loss of 2 lines of best-corrected visual acuity (BSCVA) (0.2%) , and anterior chamber perforation during initial and exchange procedures (0.4%). Nine per cent of patients had a r educed central corneal sensation = 20mm 6months after surgery, and 5.5% showed a reduction in corneal sensitivity a t 12months after surgery. At 12months, 4.4% of patients reported difficulty with night vision, 2.9% reported blurry vision, 1.6% diplopia, 1.3 % glare, and 1.3% halos. L38 Medrysone has weak antiinflammatory activity but the lowest risk of intraocu lar pressure (IOP) elevation. L39 Local steroids do not reactivate latent virus, but caution should be exercis ed because intermittent shedding of the virus from the trigeminal ganglion (unrelated to steroid use) may coincide with steroid use and exacerbate subsequent disease. L40 Pseudodendritic epithelial keratitis may refer to causes of dendritiform ker atitis other than herpes simplex virus (HSV) (e.g., zoster). L41 Pedunculated conjunctival papillomas are generally associated with human pap illoma virus (HPV) types 6 and 8, whereas sessile conjunctival papillomas are associated with HPV-16 and HPV-18. L42 When using standard intraocular lens (IOL) calculations in a postoperative p atient who underwent laser-assisted in situ keratomileusis (LASIK), hyperopia after cataract surgery is common, because the standard calculations lead to overestimation of central corneal power. After LASIK, the cornea becomes aspheri c, and consequently the standard calculations for IOL power are no longer valid. L43 An anaphylactoid reaction is rapid in onset with itching, conjunctival eryth ema, and chemosis and is usually caused by penicillin, bacitracin, sulfacetamide, or anesthetics. L44 Toxic follicular conjunctivitis may take weeks, months, or even years to dev elop.

L45 The incidence of giant papillary conjunctivitis (GPC) is greatest with soft contact lenses (SCL), followed by rigid gaspermeable (RGP) lenses, and then polymethylmethacrylate (PMMA) lenses. L46 Contact lens-induced papillary conjunctivitis is felt to be secondary to imm une system interactions with mucus or materials absorbed on the lens. Increased frequency of enzyme treatments may hel p. Heat sterilization bakes the irritating deposits onto the lens, so chemical methods (avoiding thimerosal) are preferred. L47 Collarettes indicate Staphylococcus; scurf indicates seborrhea. Sleeving of the lashes is a sign of Demodex infestation. L48 Staphylococcal blepharoconjunctivitis may be asymptomatic, or may present wi th red eyes and symptoms of ocular inflammation such as superficial punctate keratitis and subsequent epiphora and photophobia. Steroids are usually used only in acute exacerbations and only with concurrent antibiotic therapy. Du ring treatment, episodic intraocular pressure (IOP) measurement and slit-lamp examination (for cataracts) are essenti al. L49 The most common cause of phlyctenulosis is Staphylococcus, followed by activ e or latent tuberculosis. L50 Bacterial conjunctivitides in a healthy host are self-limited with the excep tions of Neisseria gonorrhoeae, Haemophilus species, and Neisseria meningitidis. Neisseria species can invade in tact corneal epithelium, causing keratitis and endophthalmitis. Haemophilus influenzae can cause orbital cellulit is or septicemia in children. L51 The following would be an effective treatment of culture-proven gonococcal c onjunctivitis ceftriaxone 1g intramuscularly daily for 5 days and doxycycline orally twice daily for 3 weeks. Doxycycline (or tetracycline) is added to treat potential chlamydial infection (sexually transmitted diseases tend to run together). L52 is a possible sequela of adenoviral infection. Symblepharon formation Symblepharon formation L53 The key feature distinguishing acute hemorrhagic conjunctivitis from epidemi c keratoconjunctivitis (EKC). Epidemic keratoconjunctivitis (EKC) may produce a hemorrhagic or membranous conjunctiviti s. Keratitis is generally not seen in acute hemorrhagic conjunctivitis caused by picornavirus. L54 AAO. Acute hemorrhagic conjunctivitis (AHC), caused by enterovirus type 70 a nd coxsackievirus A24 variant, and less commonly adenovirus type 11, is one of the most dramatic ocular viral syndr omes. Sudden onset of follicular conjunctivitis associated with multiple petechial hemorrhages of bulbar and tars al conjunctiva characterizes AHC. The hemorrhages may become confluent and appear posttraumatic. Eyelid edema, preauri cular adenopathy, chemosis, and

punctate epithelial keratitis may be associated with infection. AHC is highly co ntagious and occurs in large and rapidly spreading epidemics. In approximately 1 out of 10,000 cases due to enter ovirus type 70, a polio-like paralysis follows; neurologic deficits are permanent in up to one third of affected indivi duals. L55 The complications of trachoma are due to scarring and cicatrization of the o cular surface. Treatment with antibiotics should be prolonged, and may require retreatment with multiple agents because of resistance. L56 Although neonatal inclusion conjunctivitis usually responds to topical thera py, systemic erythromycin is recommended because of associated chlamydial infections, such as otitis media an d pneumonitis. L57 Important differences between neonatal inclusion conjunctivitis and adult in clusion conjunctivitis include all of the following except: a. More discharge in neonates. b. Pseudomembranes or membranes in neonates. c. More prominent cytoplasmic inclusion bodies in neonates. d. Better response to topical therapy in neonates. e. The follicular response of inclusion conjunctivitis is rarely seen in the neo natal form. L58 Clinical features distinguishing seasonal allergic conjunctivitis from verna l keratoconjunctivitis include: a. Response to topical antihistamine. b. The presence of giant papillae. L59 Topical antihistamines do not work well in atopic keratoconjunctivitis; syst emic antihistamines are critical for its control. L60 Long-term corticosteroid use should be avoided in patients with limbal verna l keratoconjunctivitis because of possible side effects. Topical cromolyn sodium is a more useful adjunct for long-term man agement. Palpebral vernal is more commonly associated with shield ulcers. L61 : Features distinguishing atopic keratoconjunctivitis from vernal keratoconj unctivitis include Features distinguishing atopic keratoconjunctivitis from vernal keratoconjunctiv itis include a. Age range of typically affected patients. b. Seasonal variations of incidence. c. Presence of extensive conjunctival and corneal scarring. L62 Although posterior uveitis may occur in Reiter's syndrome, including cystoid macular edema, a severe retinal vasculitis is not a feature of this syndrome. L63 Type 2 herpes usually resides latently in spinal ganglia. Latent type 1 herp es simplex virus (HSV) (responsible for recurrent orofacial infection) generally resides in the gasserian ganglion (The gasserian (or trigeminal) ganglion, located in Meckel's cave, contains the cell bodies of the trigeminal nerve). L64 : Clinical features differentiating primary from recurrent herpes simplex vi rus (HSV) infection include Clinical features differentiating primary from recurrent herpes simplex virus (H SV) infection include a. Prominent follicular membranous conjunctivitis. b. Duration and size of corneal dendrites. Dendrites are fleeting in primary inf ection. c. Preauricular lymphadenopathy. Lymphadenopathy is rare in secondary disease. d. Vesicular blepharitis may occur in either primary or recurrent herpes simplex

virus (HSV) infection. L65 Decreased corneal sensation is sometimes difficult to detect and certainly n ot specific for herpetic infection. L66 Topical antiviral medication may require up to 21 days to work. Topical ster oids are contraindicated in active epithelial herpetic disease. L67 : Potential outcomes of overtreatment with topical antivirals for herpes sim plex virus (HSV) keratitis include Potential outcomes of overtreatment with topical antivirals for herpes simplex v irus (HSV) keratitis include a. Sterile corneal ulceration. b. Pseudodendrites. c. Punctate keratitis with photophobia. L68 Decreased corneal sensation is sometimes difficult to detect and certainly n ot specific for herpetic infection. L69 Corneal epithelial healing for herpes simplex virus (HSV) epithelial keratit is is impaired by decreased sensation, stromal inflammation, and antiviral toxicity. L70 Peripheral anterior synechiae occur commonly following anterior uveitis due to herpes simplex virus (HSV) but are not usually seen with disciform keratitis. L71 The differential diagnosis of the Wessely's ring. a. Herpes simplex virus (HSV). b. Acanthamoeba keratitis. c. Anesthetic keratopathy. d. Behçet's disease. L72 Indications of topical steroids in the management of cases of disciform kera titis. Herpetologists feel that either (i) the visual axis must be involved or (ii) neovascularization must be progress ing before steroids are indicated. L73 Herpetic interstitial keratitis (IK) is generally more aggressive than disci form keratitis. Stromal necrosis can be devastating in interstitial keratitis (IK). L74 The following are justifications for the use of oral acyclovir in the manage ment of herpes simplex ocular infection. a. Primary infection. b. Concomitant chronic oral steroid therapy for a nonrelated disorder. c. Recalcitrant epithelial or stromal keratitis.

d. Systemic immune deficiency. L75 Medications useful in the control of postherpetic neuralgia include: a. Oral tricyclic antidepressant. b. Prednisone orally, started 7 to 10 days after eruption of active dermatitis. Prednisone is generally delayed until virus replication has ceased. L76 All of the following features of corneal dendrites favor the diagnosis of he rpes zoster ophthalmicus (HZO): a. Herpes zoster dendrites (pseudodendrites) are typically smaller and less branch ing than their simplex counterparts. b. A dendrite with overlying plaque of epithelial cells. c. A dendrite with no terminal bulb. d. Coarse, ropy dendrites with blunt ends. e. A dendrite with dull fluorescein and no rose-bengal staining. L77 Yanoff and Kanski. HZO Epithelial keratitis. The earliest corneal finding is punctate epithelial keratitis. Slit-lamp examination findings include multiple, fine, swollen, raised intraepithelial les ions located paracentrally or at the limbus, which stain only mildly with fluorescein but intensely with rose bengal dye. The se lesions likely contain live virus and may resolve or coalesce into elevated, dendriform epithelial lesions, referr ed to as pseudodendrites. Pseudodendrites are typically smaller than dendrites, and lack terminal end-bulb formations. They are transient and usually resolve within 2 weeks after the cutaneous eruption. Corneal sensation m ay be profoundly decreased or normal, and steroids have little to no effect on these lesions. L78 The following are systemic risk factors for the development of bacterial ker atitis. a. Drug abuse. b. Aging. c. Vitamin deficiency. d. Diabetes mellitus. e. Drug abuse, particularly crack cocaine smoking, is associated with contaminat ion and damage of corneal epithelium. Diabetes mellitus is associated with defective epithelial adherence, as is aging (which also comes with relative hypesthesia). L79 Local factors that may increase the risk of bacterial keratitis include: a. Dry eye syndrome. b. Impaired corneal sensation. c. Recurrent erosions. d. Chronic topical steroid therapy. L80 Compared with Gram-negative organisms, Gram-positive organisms tend to produ ce focal infiltrates. L81 Compared to Gram-positive organisms, Gram-negative organisms are more likely to produce fulminant liquefactive corneal necrosis. L82 In Acanthamoeba keratitis. The organism classically causes a radial keratone uritis, with pain that is out of proportion to the findings. L83 Compared with congenital syphilis, tuberculous interstitial keratitis (IK) i s more frequently unilateral and sectoral. L84 Cogan's syndrome is frequently associated with polyarteritis nodosa. L85 Thygeson's superficial punctate keratitis. Topical steroids have been used f or symptomatic relief, but may prolong the natural history of the disorder. L86 Most experts now agree that steroids are not effective therapy for superior

limbic keratoconjunctivitis (SLK). L87 About one half of patients with superior limbic keratoconjunctivitis (SLK) h ave some form of thyroid disease. Treatment of the thyroid disorder, however, has little effect on the SLK. A rece nt study suggests that orbital decompression for thyroid optic neuropathy is required more frequently in patien ts with thyroid associated SLK. L88 Discontinuation of contact lenses, at least temporarily, is therapeutic for contact lens related superior limbic keratoconjunctivitis (SLK). L89 The primary goal in intermediate therapy for chemical burns is reepitheliali zation of the corneal surface. Epithelial continuity is essential for the prevention of infections, inflammation, and scar ring. Unfortunately, severe chemical injuries retard healing. L90 Following chemical injury, if corticosteroids are to be used, they should be restricted to the first 5 to 10 days. They are useful in reducing corneal and intraocular inflammation and helpful in comba ting the formation of symblepharon. However, corticosteroids may enhance collagenase induced corneal melting, which often begins 1 to 2 weeks after the injury. L91 Generally, episcleritis causes minimal pain, whereas scleritis is moderately to severely painful. Episcleritis is usually of rapid onset as opposed to scleritis, which is usually gradual, over days. In scleritis, the scleral (deep episcleral) plexus is immobile and bluish-red in color; episcleritis appears salmon pink. Sc leral vessels do not blanch with phenylephrine, as do those in episcleritis. Lastly, scleritis is frequently seen with systemic connective tissue disorders; this is not true of episcleritis. L92 Diffuse anterior scleritis is the most benign form of scleritis and is assoc iated with the least severe systemic conditions. Necrotizing scleritis with inflammation is the most destructive form of scleritis. Sixty percent of affected

patients develop complications (in addition to scleral thinning) and 40% suffer visual loss. These patients are at risk for mortality, secondary to associated autoimmune disease. L93 Posterior uveitis occurs in virtually all patients with posterior scleritis and is not uncommon in patients with anterior scleritis. Anterior uveitis occurs in about one third of all patients with scler itis. L94 Sclerokeratitis occurs when corneal changes develop in conjunction with scle ritis. There are three forms: acute stromal keratitis, sclerosing keratitis, and marginal keratolysis (associated wi th collagenase production, seen in patients with autoimmune connective tissue disease). L95 Subtenon's injections of corticosteroids are relatively contraindicated in p atients with autoinflammatory sclerokeratitis because these drugs may result in scleral thinning and increased potential for p erforation. L96 Acute bacterial dacryoadenitis (The most common cause of acute, painful enla rgement of the lacrimal gland) is most often caused by ascending staphylococcal infection, often in patients who h ave dehydration and is associated with a purulent conjunctival discharge. Viral dacryoadenitis is frequently painless. L97 Silent dacryocystitis is usually produced by Streptococcus pneumoniae and ma y present with no clinical symptoms other than occasional epiphora. L98 A lacrimal sac abscess has formed and is not responsive to oral antibiotics. It must be drained. L99 If probing of a congenitally impatent nasolacrimal system is unsuccessful, s ilicone tube intubation should be considered and is effective in 85% of the cases. Dacryocystorhinostomy should be delayed until 3 years of age for longstanding success. L100 . Wavefront analysis and wavefront aberrations Wavefront analysis and wavefront aberrations a. Eyes without any astigmatism will not have any wavefront aberration. b. Wavefront analysis is used to analyze irregular astigmatism. c. An eight-leaf clover will be produced with a penetrating keratoplasty (PKP) u sing eight interrupted sutures (and a four-leaf clover will be produced using four interrupted sutures). d. Spherical aberration occurs when peripheral rays focus in front of central ra ys, leading to night myopia in postoperative patients who underwent laser-assisted in situ keratomileusis (LASI K) surgery. L101 Features differentiating epibulbar epithelium from epidermal epithelium inc lude: a. Lack of rete ridges. b. Lack of a granular layer. c. Presence of goblet cells. d. Mucosal epithelia differ from epidermis in that they do not normally keratini ze (absence of granular and keratin layer) and they do not have rete ridges. Mucosal epithelia also contain goblet cells. L102 A melanophage is a macrophage that has engulfed melanin, not a melanocyte. L103 Keratoacanthoma is a reactive tumor that develops rapidly over 4 to 8 weeks . It is a large, elevated, and round cutaneous tumor that contains a central core of keratin. Although acanthosis, hy perkeratosis, and dyskeratosis may be extreme, dysplasia is often absent. Inflammation is prominent because keratoacan thoma is a type of pseudoepitheliomatous hyperplasia. Keratoacanthoma is likely a low-grade squamou

s cell carcinoma. Spontaneous regression is typical. However, it is relatively easy to excise thes e lesions without recurrence. L104 The following statements about seborrheic keratosis. a. It is a lesion most commonly seen in elderly people. b. There may be an inherited tendency for its development. c. Seborrheic keratoses are usually verrucoid and greasy looking. d. Histopathologically, there is prominent dyskeratosis and hyperpigmentation in a papillary growth pattern. e. The lesion must be carefully distinguished from actinic keratosis. L105 Features serving to differentiate between actinic keratosis and seborrheic keratosis include: a. 2. Elastosis. b. 4. Epithelial atypia. L106 Squamous cell carcinoma of the eyelid, the upper eyelid is more frequently involved than the lower eyelid; these lesions tend to be slow-growing. (Kanski lower eyelid is more commonly affected) . L107 Although squamous cell carcinoma may occur in the eyelids, it is at least 1 0 and perhaps up to 40 times less common than basal cell carcinoma. Because the majority of squamous cell carcinom as arise in solar-damaged skin (actinic keratoses), the lower eyelid is more frequently involved than the upper . (AAO4). L108 Peripheral palisading of tumor cell nuclei. This is a feature of basal cell , not sebaceous cell, carcinomas. L109 Adnexal tumors of hair follicle origin include all of the following: a. Trichoepithelioma. b. Trichilemmoma. c. Pilomatrixoma. d. Trichofolliculoma. L110 An alternate name for pilomatrixoma is the calcifying epithelioma of Malherb e. L111 Frozen section techniques allow for preservation of tissue lipid, staining for which (Oil-red O stain) plays a role in diagnosing sebaceous cell carcinoma.

L112 The difference between ephelis and lentigo is an increased number of melano cytes in the latter. The hyperpigmentation of ephelis is due to increased melanin content of normal (or d ecreased) numbers of melanocytes. L113 The difference between nevi and lentigo is modification of the melanocyte p opulation in nevi. Melanocytes in nevi are referred to as nevus cells to reflect this difference. Nevus cells are rounder, wi th distinct margins and more abundant, eosinophilic cytoplasm. L114 Sebaceous cell carcinoma is not related to chronic sun exposure. L115 . Kaposi's sarcoma Kaposi's sarcoma a. It may be considered the malignant counterpart of a pyogenic granuloma. b. The disorder is endemic in Central Africa. c. In the setting of normal immune regulation, the disease typically affects the lower extremities of older men. d. The disorder is generally more aggressive and lethal in the immunocompromised individual. e. Radiation therapy is a very important and effective modality for treating the se lesions. L116 A stye or external hordeolum must arise from: a. Glands of Zeis. b. Eccrine sweat glands. c. hair follicles. L117 Blocked glands of Zeis, in particular, can produce an external hordeolum th at is purely inflammatory in nature. (Sebum is very irritating to tissues). L118 Xanthelasma tend to be flat, plaquelike lesions, as opposed to nodules. L119 : Epibulbar dermoids are solid choristomas. The key difference between epib ulbar and adnexal dermoid tumors is The key difference between epibulbar and adnexal dermoid tumors isAdnexal dermoi ds are generally cystic. L120 The most common location of origin for corneal intraepithelial neoplasia is the limbus. The same is true for conjunctival neoplasms. L121 By definition, the neoplasia of the corneal epithelium is not invasive unti l it has begun to penetrate Bowman's layer. L122 . Oculodermal melanocytosis Oculodermal melanocytosis a. Oculodermal melanocytosis is more common in African Americans and Asians, but malignant transformation is more common in whites. b. Ocular melanocytosis is a congenital blue nevus of the episclera and is most common in whites. c. When ocular melanocytosis occurs in combination with periocular cutaneous mel anosis (nevus of Ota), it is termed oculodermal melanocytosis. This is more common in African Americans and Asi ans and is nearly always unilateral. Malignant transformation is rare, and it seems to occur almos t exclusively in whites. L123 Biopsies of conjunctival melanomas are not thought to increase the risk of metastases. Therefore, suspicious melanotic epibulbar lesions should be examined by biopsy, followed by complete e xcision if found to be malignant. L124 The most important adjunctive therapy to excisional biopsy of conjunctival melanoma is Cryotherapy. L125 It is difficult to distinguish conjunctival lymphomas from benign reactive lymphoid hyperplasia, but a prominent vascular stroma, numerous follicles, and cellular heterogeneity {heterogeneous c

ell population (neutrophils, eosinophils, plasma cells, and numerous lymphocytes)} are all indicative of reactive (nonneop lastic) lesions. L126 The Schwalbe line is the termination of the corneal endothelium. L127 Congenital glaucoma can be distinguished from birth trauma by the presence of increased intraocular pressure (IOP) and horizontal (as opposed to vertical) breaks in Descemet's membrane. In cornea l birth trauma, residual hypertrophic ridges in Descemet's membrane are often visible even after the corneal edema cle ars. L128 Microscopically, lymphomas are characterized by sheets of relatively monoto nous cells. Unlike most other malignancies, atypia is not a feature of lymphoma. In fact, mitotic figures are rare in lymphoma and common in reactive lesions. L129 An anterior cataract or dislocated lens may be present but is not necessary for the diagnosis of Peters' anomaly. L130 Pingueculae, the lesions may calcify or become chronically inflamed. L131 A conjunctival deposit of amyloid is examined by biopsy and stained with Co ngo red. As a polarizing filter between the illuminating light and the specimen is rotated 90 degrees, the amyloid depos its seem to change from cherry red to apple green. This phenomenon is known as dichroism. Note that only one polarizin g filter is required to elicit dichroism. L132 Primary localized amyloidosis (The most common form of conjunctival amyloid osis) consists of conjunctival amyloid plaques, which occur without systemic involvement and without a local cause. L133 Primary systemic amyloidosis produces ecchymotic, waxy eyelid papules is th e most common type of eyelid amyloidosis. L134 The lipid deposition in arcus senilis tends to occur at the superior and in ferior poles (where the local temperature is highest) and then spreads into the palpebral fissure. In the setting of high-gra de carotid stenosis, the ipsilateral eye is protected from lipid deposition. L135 The guttae in Fuchs' dystrophy as well as Hassall-Henle bodies result from dysfunctional production of basement membrane by endothelial cells. L136 Hassall-Henle bodies are normal aging changes found in the peripheral corne a. Similar changes in the central cornea are known as corneal guttae, and these may be forerunners of corneal edema.

L137 is a condition that is characterized by asymptomatic tiny deep stromal opac ities, which are best Cornea farinata Cornea farinataviewed by retroillumination. L138 Band keratopathy starts at the horizontal periphery. It may spread centrall y to form a horizontal band. Occasionally, it starts paracentrally. It is always in Bowman's zone. L139 Salzmann's nodular degeneration, spheroidal degeneration, and Coats' white ring are corneal degenerations that typically do not involve neovascularization. (However, if located peripherally, each of the three may have surface neovascularization.). L140 The variety of Mooren's ulcer that occurs in young African American men is usually more aggressive and responds poorly to medical or surgical management. Perforation is more common in this gro up. It is felt that some patients in this group may have developed the corneal ulceration as a result of antigen-antibody reactions to helminthic toxins. The toxins may get deposited during the blood-borne phase of certain parasitic infec tions. L141 : Laser-assisted in situ keratomileusis (LASIK) is contraindicated in Laser-assisted in situ keratomileusis (LASIK) is contraindicated in a. Patients with unreasonable expectations (e.g. an expectation of 20/8 vision p ostoperatively). b. Patients with potentially impaired healing (e.g., those on long-term topical or oral corticosteroids, or those who are immunocompromised). c. Patients with chronic ocular conditions (including keratoconus). d. A history of gas-permeable contact lens use is not a contraindication for LAS IK per se, but patients should discontinue their lenses for at least 8 to 10 weeks and then obtain serial topog raphic keratometry to ensure stability before surgery. L142 have been used to treat patients who are unable to undergo refractive surge ry Phakic intraocular lenses (IOLs) Phakic intraocular lenses (IOLs) because of high refractive error (either high hyperopia or high myopia). Their l ongterm safety and efficacy is still being investigated. L143 Dots seen in anterior membrane dystrophy are clumps of degenerated epitheli al cells and basement membrane material within the epithelium. L144 Epithelial cysts are seen in both map-dot-fingerprint and Meesmann's dystro phies. The cysts in the former are translucent or opaque and represent degenerated epithelial cells and basement me mbrane. Those of Meesmann's dystrophy are transparent and are filled with periodic acid-Schiff (PAS)-positiv e material known as peculiar substance". L145 Recurrent erosions and anterior corneal scarring lead to visual loss in Rei s-Bückler dystrophy sooner and to a greater degree than in the others. Pre-Descemet's dystrophy is usually clinicall y silent, like central cloudy dystrophy. L146 Macular dystrophy may have focal granular like deposits but differs from gr anular dystrophy in several ways. a. First, the intervening stroma is cloudy. b. Second, the peripheral cornea is involved much earlier in macular dystrophy. c. Third, macular dystrophy is inherited recessively, so that parents or offspri ng are unlikely to be involved, whereas siblings are likely to be involved. d. Patients with macular dystrophy have a deficiency in enzymatic synthesis of k

eratan sulfate, and serum levels are typically depressed. The mnemonic for stromal dystrophies (dystrophy, deposit, stain) is: Marilyn Monroe Always Gets Her Man in Los Angeles County. Marilyn Monroe Always (Macular/Mucopolysaccharide/Alcian blue) Gets Her Man (Granular/Hyaline/Masson's trichrome) in Los Angeles County (Lattice/Amyloid/Congo red). L147 . Lattice dystrophy Lattice dystrophy a. May have granular deposits but also features linear, branching (lattice) depo sits. b. Both types of deposits are amyloid and will demonstrate birefringence and dic hroism when stained with Congo red. c. In type I, as in this patient, the deposits are randomly distributed, greater centrally. This form is localized. d. In type II, the deposits are along the course of the corneal nerves, so they are denser peripherally. e. Type II is seen only in familial amyloidotic polyneuropathy type IV (Meretoja 's syndrome) and is associated with cranial nerve palsies and dry, redundant skin. f. Like granular dystrophy, lattice dystrophies are inherited dominantly. g. This is the stromal dystrophy most likely to be associated with recurrent ero sions. L148 . Fleck dystrophy Fleck dystrophy a. The stromal dystrophies may be associated with keratoconus, atopy, or pseudox anthoma elasticum. b. This disorder may be highly asymmetric or unilateral and associated with a wi de range of ocular or systemic disorders. c. Tiny flecks are visible in corneal stroma. d. Vision is not affected. L149 Central cloudy dystrophy of Francois Central cloudy dystrophy of Francoisis nearly always clinically silent (least li kely to be associated with poor vision). L150 Congenital hereditary endothelial dystrophy Congenital hereditary endothelial dystrophypresents like congenital glaucoma exc ept that the corneal diameters and intraocular pressure (IOP) are normal.

L151 Acute hydrops reflects sudden stromal edema associated with an acute break in the Descemet's membrane. Pain may be severe, but frequently the only symptom is blurry vision. Once the break in the Descemet's membrane spontaneously seals (in 6 to 10 weeks), the stromal edema clears. Residual scarr ing may or may not require penetrating keratoplasty (PKP). L152 Rigid gas-permeable (RGP) contact lenses are the optical correction of choi ce in keratoconus because these lenses are the most successful at eradicating irregular astigmatism. L153 Conductive keratoplasty Conductive keratoplastyhas been approved for the treatment of low hyperopia (<3. 0D) in patients with mild astigmatism (0.75D or less). L154 A satisfactory optical outcome in surgery for keratoglobus is more difficul t to obtain than for keratoconus. L155 Higher corneal protrusion above the area of corneal thinning makes fitting very difficult in pellucid degeneration. Scleral contact lenses have been tried with some success. L156 , also known as Sands of the Sahara, typically manifests itself within the fi rst week after Diffuse lamellar keratitis Diffuse lamellar keratitislaser-assisted in situ keratomileusis (LASIK) surgery. If it is diagnosed early, topical steroid treatment usually provides for an excellent prognosis. However, later stages (clumping of white blood cells [WBCs] in the central visual axis) usually require flap lifting, irrigation, intense topical steroids, and occasion ally systemic steroids. L157 Typically, tear volume is deficient in dry eye syndrome (inadequate aqueous phase), but a large subset of patients have problems with surface wetting due to deficiencies of the other phases (mucu s problems in keratinizing disorders, lipid problems in blepharitis). Schirmer testing in these cases may be normal or supranormal despite a history and examination suggestive of dry eye. Tear breakup time is frequently the important clue. L158 Blepharoconjunctivitis may produce a syndrome similar to dry eye syndrome e xcept that the symptoms are typically worse in the morning. In dry eye syndrome, symptoms are worse in the evening. Ch alazia are associated with chronic blepharoconjunctivitis, not dry eye syndrome. L159 The definition of secondary Sj.gren's syndrome is sicca complex plus coexis tent autoimmune disease. Autoantibodies may be more prevalent and at higher concentrations in primary dis ease (no systemic disorder), but this is not a reliable differentiating feature. Dry mouth is a feature of both types. Although human leukocyte antigen (HLA)Dw3 may be seen more commonly in primary disease, this is also not foolproof. L160 Patients with primary Sj.gren's syndrome are at increased risk for subseque nt development of: a. Autoimmune thyroiditis. b. Waldenstr.m's macroglobulinemia. c. Lymphoma. L161 In ocular cicatricial pemphigoid (OCP), a deficient mucus phase leads to te ar dysfunction. In Bell's palsy, there is exaggerated drying due to exposure and lagophthalmos. Oral contraceptives have b een associated with decreased aqueous tear production. Herpes zoster renders a cornea hypesthetic with impaire d reflex tearing and drying. L162 The distribution of keratopathy seen in long-standing Bell's palsy is simil

ar to that seen in primary Sj.gren's syndrome, in both disorders, the interpalpebral cornea is the area affected earl iest and most severely. Like exposure keratopathy, neurotrophic keratopathy affects the interpalpebral zone preferenti ally because this is the area of greatest exposure and drying. L163 The combination of corneal hypesthesia and exposure/lagophthalmos is nearly always disastrous for the corneal surface. Lateral tarsorrhaphy is probably advisable before irreversible, destruc tive changes can commence. Remember: Five (V) + Seven (VII) =T. That is, Vth nerve (hypesthesia) plus VIIth nerve (ex posure) dysfunction equals tarsorrhaphy. L164 Comedones are not a feature of rosacea dermatitis. This differentiates acne rosacea from acne vulgaris. L165 The following statement about the pathology of acne rosacea. a. Demodex infestation may play an inciting or aggravating role. b. Biopsy of involved tissues frequently reveals granulomatous inflammation. c. There may be irreversible obliteration of meibomian glands. d. Type IV hypersensitivity may play a role in the pathogenesis. L166 Potent steroids may precipitate melting in eyes with rosacea keratitis. Any steroid preparation should probably be avoided in the setting of frank corneal ulceration. Once epithelial continuity h as been restored and secondary infection adequately treated (hygiene, antibiotic ointments, and oral tetracycline), mild steroid preparations are probably safe but must be used with adequate surveillance. L167 The following statements about erythema multiforme. a. The distinction between minor and major variants is the involvement of mucous membranes in the latter. b. The acute phase of the major type lasts longer than the minor type. c. Inflammation is generally confined to the dermis or submucosal stroma. d. Erythema multiforme minor is strictly a skin disease. The systemic variety (e rythema multiforme major) has mucous membrane ulceration, as well as dermatitis, and is life threatening (20% systemic mortality from secondary infection). e. The inflammation is primarily angiocentric. L168 . Erythema multiforme Erythema multiforme a. Mucous membrane lesions are bullous. b. Skin lesions are macular and consist of a target lesion red center, inner pale an nulus, outer red annulus.

c. Some experts feel that lysis of symblepharons may lead to secondary infection and/or worsened scarring. In addition, despite vigorous attempts, subconjunctival scarring, may still occurs. d. Mortality approaches 20% in severe cases. e. The most frequent cause of death is secondary infection. f. Systemic corticosteroids have substantially diminished mortality. g. The long-term ocular prognosis in toxic epidermal necrolysis is somewhat bett er than that of systemic erythema multiforme. L169 , Oral and conjunctival epithelia are the most frequently involved, with a slight Cicatricial pemphigoid Cicatricial pemphigoidpreponderance of the latter. L170 Erythema multiforme is a vasculitic process. Ocular cicatricial pemphigoid (OCP) features deposition of immunoglobulin and complement in the basement membranes of skin and mucous membr anes, with subsequent bulla formation. L171 The inferior fornix is generally the most rewarding area of the eye to exam ine in ocular cicatricial pemphigoid (OCP). L172 The classic histologic finding in ocular cicatricial pemphigoid (OCP) is co mplement and immunoglobulin bound to epithelial basement membrane. This is in distinction to pemphigus, in which epit helial acantholysis and intraepithelial bullae are characteristic features. L173 Dapsone is an acceptable first choice in mild to moderate cases of OCP in p atients with no evidence of glucose-6phosphate dehydrogenase deficiency (in whom dapsone can cause a fatal hemolytic anemia). L174 Hypertonic saline may be useful in maintaining epithelial deturgescence and enhancing adherence but must be used for several (4 to 6) months if it is to have long-term effect. L175 Ligneous deposits nearly always recur after excision if the disease process remains active. L176 The most reliable prognostic indicator for long-term outcome in ocular cica tricial pemphigoid (OCP) is the severity and rapidity of active inflammation. Therefore, severe cases should be treated w ith more potent immunosuppressive agents from the outset, such as cyclophosphamide. L177 In tyrosinemia, elevated serum tyrosine levels lead to lysosomal instabilit y with dermal and ocular inflammation, as well as mental retardation. Nonstaining pseudodendrites may recur and be misdiag nosed as herpes simplex virus (HSV). L178 The following statements about vortex keratopathy are. a. Drugs associated with the finding include amiodarone, indomethacin, chloroqui ne, and chlorpromazine. b. The findings in drug-induced vortex keratopathy are identical to those of Fab ry's disease. c. The pathophysiology of the deposits in drug-induced vortex keratopathy is ide ntical to that of Fabry's disease. d. Of the agents known to cause vortex keratopathy, only chlorpromazine is assoc iated with pigmentary retinopathy. The two are generally independent. (Chloroquine can cause a bull'seye maculopathy as well.). e. Cessation of drug therapy will usually lead to resolution of vortex keratopat hy. L179 : A Kayser-Fleischer ring may be seen in the following disorders

A Kayser-Fleischer ring may be seen in the following disorders a. Primary biliary cirrhosis. b. Chronic active hepatitis. c. Wilson's disease. d. Chalcosis. L180 The most accurate characterization of donor endothelial cell counts followi ng penetrating keratoplasty (PKP) is rapid loss of endothelial cells over the first postoperative year, slow loss of endoth elial cells over the next 10 to 15 years, with stable cell counts after 15 years. L181 The following considered favorable prognostic factors for penetrating kerat oplasty (PKP) are lack of neovascularization, lack of inflammation, and no history of previous graft failu re. L182 : The following are considered unfavorable prognostic factors for penetrati ng keratoplasty (PKP) The following are considered unfavorable prognostic factors for penetrating kera toplasty (PKP) a. Considerable stromal vascularization. b. Tear deficiency state. c. Active intraocular inflammation. d. Corneal hypesthesia. e. Other unfavorable factors include youth, larger grafts, and glaucoma. L183 Most intraocular lens (IOLs) capable of causing corneal decompensation also can cause chronic intraocular inflammation and secondary cystoid macular edema. L184 The primary cause of poor vision following penetrating keratoplasty (PKP) f or aphakic bullous keratopathy is cystoid macular edema. L185 Preoperative steps that are helpful in penetrating keratoplasty (PKP) inclu de: a. Pilocarpine 1% preoperatively. b. Honan balloon placement before surgery. c. Intravenous mannitol preoperatively. d. Indirect miotics such as phospholine are generally stopped several weeks befo re surgery to stabilize the blood-aqueous barrier and allow repletion of serum cholinesterase. Hypotensive m aneuvers such as the

Honan balloon and mannitol lower the incidence of vitreous loss. Pilocarpine pre operatively protects the lens from incidental damage. L186 Scleral support ring (Flieringa's ring) Scleral support ring (Flieringa's ring)during penetrating keratoplasty (PKP) is most helpful in eyes that are aphakic (or will become so during the procedure). Aphakic globes have lower rigi dity and structural integrity, leading to collapse and vitreous presentation. A support ring helps prevent this. L187 The main advantage of smaller donor size is decreased risk of rejection, wh ereas the main disadvantage is increased central astigmatism. For larger grafts, the opposite is true. L188 The main advantage of oversizing the donor button (0.5 mm or more, larger t han the host bed) is to decrease the incidence of peripheral anterior synechiae and subsequent elevated intraocular p ressure (IOP). L189 Smaller-than-host-bed donor buttons tend to be flatter and thus more hypero pic than the larger-than-hosted donor buttons. This effect helps to neutralize part of the high myopia seen in eyes wi th keratoconus. L190 In the setting of corneal edema and cataract, the triple procedure is proba bly advisable. Guttae alone are not a sufficient indication for this procedure. There should be stromal or epithelial edema before it is undertaken. L191 The most common postoperative complication seen after penetrating keratopla sty (PKP) is high astigmatism. This complication can frequently be controlled with selective suture removal, but in some cases refractive surgery is necessary. L192 In the setting of corneal edema and cataract, the triple procedure is proba bly advisable. Guttae alone are not a sufficient indication for this procedure. There should be stromal or epithelial edema before it is undertaken. L193 Ocular surgery may precipitate increased disease activity in ocular cicatri cial pemphigoid (OCP). L194 Recurrence of the original disease process has been reported following pene trating keratoplasty (PKP) for each of the following conditions: a. Lattice dystrophy. b. Herpes simplex keratitis. c. Reis-Bückler dystrophy. d. Macular dystrophy. L195 . Infectious crystalline keratopathy Infectious crystalline keratopathy a. Frequently, prolonged topical and systemic antibiotic therapy is necessary, w ith or without surgical therapy (lamellar or penetrating excision). b. This infectious crystalline keratopathy is closely associated with keratoplas ty and topical steroid use. L196 In first-time grafts, rejection rarely, if ever, presents within the first 10 days postoperatively. It is possible, however, with subsequent grafts in sensitized patients. Graft rejection should spare the host bed, unlike glaucoma. A linear array of keratic precipitates (KPs), the Khoudadoust line, may be seen in the earlier phases of immunologic rejection but is not universal. L197 The following statements about tissue adhesive closure of corneal defects. a. Polymerization is rapid when the adhesive contacts free anions. b. All epithelium and necrotic tissue must be cleared from around the perforatio

n to enhance adhesion. c. The bed to which the adhesive is to be attached must be dry at the time of tr eatment. d. The absence of significant heat release makes adhesives safer and more reliab le. e. Bandage contact lenses are generally needed following successful adhesive app lication. L198 A conjunctival flap will not provide an adequate seal over an underlying fi stula or perforation. The hole must be closed with a tectonic graft (full-thickness or lamellar, with sclera or cornea) before a flap may be advanced. L199 Azithromycin and clarithromycin are among the first members of a new class of bacterial ribosomal inhibitors known as azalides. They are related chemically to erythromycin but are much more rapidly distributed to tissues with better absorption, bioavailability, and higher intracellular penetration. Azithromycin' s tissue half-life is also quite long, 2 to 4 days, rendering it useful as a single-dose agent. This enhances compliance relat ive to the traditional prolonged topical treatment, and also offers the advantage of treating extraocular reservoirs of C hlamydia L200 The multiple subepithelial infiltrates seen in the EKC are most typical of the postinfectious stage of epidemic keratoconjunctivitis (EKC). The history and findings are most consistent with th is as well. EKC can be membranous in severe cases. The infiltrates themselves are felt to represent hypersensitivity reactions to viral antigens, without active microbial replication. Topical corticosteroids are quite helpful in controlling potentially disabling photophobia and pain but can lead to a medication-dependent state requiring months or years to revers e. Subepithelial infiltrates that may be indistinguishable from those of EKC may be seen in Thygeson's disease. In the la tter, there is no conjunctivitis prior to the corneal manifestations. L201 Pyogenic granuloma are neither pyogenic (copious neutrophils) nor granuloma tous; instead, they feature abundant immature vascular channels in a matrix of ground substance secreted by active fi broblasts. Intensive topical steroids are typically curative. L202 . Anterior stromal puncture Anterior stromal puncture a. A recent study has documented its utility in bullous keratopathy as well. b. It reduces bullae and lessens pain and may provide an alternative for penetra ting keratoplasty (PKP) in selected cases.

L203 The Herpetic Eye Disease Study The Herpetic Eye Disease Studyhas answered several questions regarding the utili ty of topical steroids in herpes simplex stromal keratitis. a. Topical steroids reduced persistence or progression of stromal keratitis comp ared with placebo. b. Delayed initiation of steroid therapy slowed resolution but did not affect lo ng-term (6 months) visual outcome. c. Recurrences following steroid taper responded well to a more gradual taper. d. Topical trifluridine was used in both treatment arms of the study to help low er the incidence of epithelial keratitis. e. A large proportion of patients treated with trifluridine and placebo had sign ificant progression of herpetic inflammation, implying that trifluridine alone does not exert a significant bene ficial effect for stromal keratitis. L204 The critical uncut depth of the cornea necessary to maintain normal integri ty has not been determined but is presumed to be at least 250-300 .m or 50% of the stromal depth, whichever is gre ater. L205 Tangential incisions (arcuate or linear) lead to flattening in the meridian of the incision and steepening in the meridian 90° away. The closer radial incisions approach the visual axis (i.e., the smaller the optical zone), the greater their effect. For optimum effect, the incision should be 85%-90% deep to allow a n intact posterior lamellae and maximum anterior bowing of the other lamellae. L206 Following incisional keratotomy, radial corneal wounds regain up to 50% per centage of unincised corneal tensile strength. L207 Flap folds occur after laser-assisted in situ keratomileusis (LASIK). More than 50% of flap folds occur within the first day, and >90% of flap folds occur within the first week postoperatively. Not all folds need to be repaired, but visually significant folds should be repaired within 24 hours. F. . PEDIATRIC OPHTHALMOLOGY AND STRABISMUS PEDIATRIC OPHTHALMOLOGY AND STRABISMUS L1 Healthy infant have shallower AC than adult because the lens is proportionall y the largest part of the infant eye. L2 The optic nerve is generally myelinated to the lamina cribrosa 4 to 6 months before foveal maturation. L3 Testing of VA in preverbal infant. OKN and PLT are fairly close. OKN is more cumbersome and difficult quantify accurately than PLT. L4 The lower lids are more frequently involved with congenital ectropion and ent ropion. L5 Hypertelorism implies an abnormally wide distance between medial orbital wall s. Telecanthus is an abnormally wide distance between medial canthi. Telecanthus may be isolated (primary) or seconda ry to hypertelorism. L6 The following statements about congenital toxoplasmosis. a. 70% of pregnant women are seronegative. b. Placental transfer of organism is common in newly infected mother; however, m any infected infants will not have systemic manifestation. L7 Sign and symptoms of typical toxoplasmosis include hepatosplenomegaly, seizur e and intracranial calcifications, vomiting and diarrhea, and focal chorioretinal lesions. Microphthalmos and catar

act are rare. L8 Tests from infants would support the diagnosis of congenital toxoplasma infec tion include Antitoxoplasma IgM antibody and CT scan revealing intracranial calcification. L9 Pyrimethamine blocks synthesis of folinic acid, so it must be given as well. Note that folinic acid is synthesized from dietary folic acid, so folic acid administration is not sufficient. L10 Most mothers in developed countries are seropositive (protected) to rubella, due to vaccination. L11 The most common clinical finding in infants with congenital rubella syndrome is sensorineural deafness. Many cases of oculoauditory syndrome are caused by rubella. L12 Although children with congenital rubella syndrome may develop either catara ct or glaucoma, their coincidence in an infant is rare. L13 The most common congenital infection in humans is cytomegalovirus (CMV). App roximately 2% of human infants are infected. Most cases are silent. L14 Most cases of congenital herpes declare themselves with vesicular dermatitis , keratoconjunctivitis, pneumonitis, or meningoencephalitis. Posterior inflammatory component is far more common in cong enital disease, with severe retinitis and vitritis. This is unusual in acquired disease but may be the cause of some c ases of acute retinal necrosis. L15 Hutchinson triad, considered diagnostic of congenital syphilis, includes peg -shaped teeth, 8th nerve deafness, and IK. Rhagades, linear scar around the mouth, are characteristic of congenital lues, b ut they are not a part of Hutchinson triad. L16 . Conclusion of the Multicenter Trial of Cryotherapy for ROP Conclusion of the Multicenter Trial of Cryotherapy for ROP a. In the multicenter trial of cryotherapy for ROP (CRYO-ROP Study), 37% of infa nts weighing less than 750 g developed severe (stage 3) ROP, whereas only 21.9% of those weighing 750-999 g a nd 8.5% of those weighing 1000-1250 g did so. b. The results of the Multicenter Trial of Cryotherapy for Retinopathy of Premat urity Cooperative Group indicated that treatment is associated with a 41% decrease in the occurrence of posterior retinal traction folds or detachments and a 19% to 24% decrease in the incidence of blindness whe n evaluated 5 years later.

c. Treatment benefit is independent of birth weight, race, and number of sectors of stage 3 involvement. d. Treatment of threshold disease increases the proportion of normal appearing p osterior pole at the age of 5 years as compared to no treatment. e. Treatment clearly reduces the likelihood of a no light perception outcome, bu t the long term Snellen acuity is disappointing. L17 The following considered common etiologic agents for conjunctivitis in child ren are Staphylococcus aureus, Streptococcus pneumoniae, and Haemophilus influenzae are far more common. L18 The agent most commonly responsible for preseptal cellulitis in children is Staphylococcus aureus, most common cause is minor skin injury. L19 The agent most likely associated with orbital cellulitis following bacterial conjunctivitis is H. influenzae. L20 Posttraumatic preseptal cellulitis occurs following puncture, laceration, or abrasion of the eyelid skin. In these cases, organisms found in the skin, such as Staphylococcus aureus or Streptococcus pyog enes, are the most common bacteria responsible for infection. L21 The third mechanism for preseptal cellulitis tends to occur in young childre n and is secondary to upper respiratory or sinus infection or to unknown cause. Until the 1990s, Haemophilus influenzae was the most common causative agent. It could be recognized by a typical deep violaceous hue of the infected lids. Haemo philus preseptal cellulitis may still be seen in medically underserved regions, but the incidence has dropped markedly si nce widespread use of Hib vaccine began in the early 1990s. Now, S pneumoniae and other streptococcal infections a nd S aureus are the most common causes. L22 Sudden deterioration in ocular motility without dramatic increase in proptos is in patient with orbital cellulitis suggests cavernous sinus thrombosis. L23 Subconjunctival scarring is common in trachoma whereas it is rare in vernal conjunctivitis. Both conditions may have prominent limbal nodules (limbal papillae and Horner-Trantas dots in vernal, lim bal follicles in trachoma). L24 Horner-Trantas dots are clumps of degenerated eosinophils, whereas Herbert's pits are depressed necrotic follicles at the limbus. L25 Vernal disease rarely if ever leads to vascularization of the cornea. Superf icial vascularization is far more common with trachoma, but it has been reported with severe vernal KC as well. Boys are more frequently involved than girls, and the upper tarsal conjunctiva is more commonly and more severely involved than the lo wer. Deep corneal vascularization is unlikely in vernal disease. So-called shield ulcers large oval, central epithelial defects are classic corneal manifestations of the disease. L26 Corneal manifestations of vernal disease may include: a. Superficial punctate keratitis. b. Superior corneal pannus. c. Transverse oval sterile ulceration in the superior cornea. L27 Some factors other than mechanical irritation are responsible for sterile co rneal ulceration in vernal disease. Some experts have suggested that inflammatory factors released by diseased conjunctiv

a may be responsible. L28 . Kawasaki Syndrome Kawasaki Syndrome a. Kawasaki syndrome, also known as mucocutaneous lymph node syndrome, is a febr ile illness primarily affecting children younger than 5 years. The cause is unknown. b. The diagnostic criteria are an unexplainable fever lasting 5 or more days and at least four of the following: i. Bilateral conjunctival injection. ii. Mucous membrane changes of injected or fissured lips, injected pharynx, or " strawberry tongue" iii. Extremity changes involving erythema of the palms or soles, edema of the ha nds or feet, or generalized or periungual desquamation. iv. Rash. v. Cervical lymphadenopathy. c. The most significant complication of Kawasaki syndrome is coronary artery ane urysm. d. Coronary artery evaluation by two-dimensional echocardiography is therefore i ndicated. e. Anterior uveitis during the acute phase of the illness is common but generall y self-limited. f. Conjunctival scarring can occur, and bilateral retinal ischemia has been obse rved histopathologically. g. Treatment is mainly supportive, and aspirin is considered the drug of choice. h. Corticosteroid therapy is contraindicated because of its association with an increased rate of coronary artery aneurysm formation. L29 The medial portion of the eyelid is more susceptible to tearing injuries bec ause of absence of the tarsal plate. L30 . A dacryocele A dacryocele a. Sometimes can be decompressed by digital massage, which in combination with t opical antibiotic administration may lead to resolution without complications. b. Lacrimal probing should be performed no later than 1 month after birth in per sistent cases. c. Once dacryocystitis develops, systemic antibiotics are indicated, and prompt surgical decompression of the sac is usually necessary. This can be accomplished initially bypassing a probe o r lacrimal cannula through

a canaliculus into the distended sac, but permanent and complete relief requires elimination of associated NLD obstruction. d. Any associated nasal mucocele can be marsupialized to facilitate drainage int o the nose. e. Probing the lower system can be difficult because of the considerable anatomi cal distortion that frequently accompanies dacryocele. f. Sometimes, intranasal surgery to marsupialize the bulging lower end of the du ct is required. g. Incision and drainage of an infected dacryocele through the skin should be av oided because of the danger of creating a persistent fistulous tract. L31 Initial probing with irrigation of an impatent nasolacrimal system is succes sful 90% of the time. Failure is generally followed by a repeat probing and irrigation. If this fails a second time, then s ilastic intubation is tried. If this also fails, then dacryocystorhinostomy is the next step. L32 NLD obstruction, recurrent conjunctivitis is far more common than dacryocyst itis. L33 Keratoconus is typically associated with Marfan syndrome. Keratoglobus is ty pically associated with EhlersDanlos syndrome. L34 Congenital hereditary endothelial dystrophy (CHED) occurs secondary to a def ect of the corneal endothelium and Descemet's membrane. Although the corneal edema may resemble that seen in congen ital glaucoma, there is no elevated intraocular pressure (IOP) or increased corneal diameter. L35 In congenital hereditary stromal dystrophy, the cloudy stroma is of normal t hickness, and the epithelium is normal. In congenital hereditary endothelial dystrophy (CHED), there is epithelial edema an d a thickened stroma. In both conditions, intraocular pressure (IOP) is usually normal. L36 . CHED CHED a. CHED is an uncommon corneal dystrophy with onset at birth or shortly thereaft er. b. The cornea is diffusely and uniformly edematous secondary to a defect of the corneal endothelium and Descemet's membrane. c. The edema involves both the stroma and epithelium. d. The hallmark of CHED is increased corneal thickness. e. The appearance of the cornea is similar to that in congenital glaucoma but wi thout increased corneal diameter and elevated intraocular pressure. f. CHED can be inherited in an autosomal dominant or autosomal recessive manner. L37 Congenital hereditary stromal dystrophy Congenital hereditary stromal dystrophyis a very rare congenital stationary opac ification of the cornea transmitted in an autosomal dominant manner. Flaky or feathery clouding of the stroma, which is of normal thickness, is covered by a smooth, normal epithelium. These features are in contrast with CHED, which has a thickened stroma and epithelial edema. L38 Infectious mononucleosis is caused by Epstein-Barr virus. The disease usuall y occurs between ages 15 and 30 years and is benign and self-limited. Findings include fever, widespread lymphadenopat hy, pharyngitis, hepatic involvement,

and the presence of atypical lymphocytes and heterophil antibodies in the circul ating blood. Conjunctivitis occurs in a very high percentage of cases. Treatment is supportive, including bed rest, anti pyretics, analgesics, and cool compresses to the eyes. Lacrimal gland involvement can often occur in Epstein-Ba rr virus infection. L39 Lacrimal gland involvement can often occur in acute Epstein-Barr virus infec tion. The authors suggest it may be far more common than the 20% reported previously. Neither streptococcal nor diphther itic infections tend to cause dacryoadenitis. Sarcoidosis never causes acute pharyngitis, and the dacryoadenit is is typically painless. L40 . Cystinosis Cystinosis a. A metabolic disease characterized by elevated levels of cystine within the ce ll is rare. b. French Canada has the highest incidence in the world. c. Cystine crystals are deposited in various places throughout the body. d. In the infantile form of the disease, the major presenting symptoms are failu re to thrive, rickets, and progressive renal failure, collectively called Fanconi syndrome. The ocular find ings are pathognomonic. Iridescent elongated corneal crystals appear at approximately 1 y ear of age, first in the peripheral part of the cornea and the anterior part of the stroma. These crystal s are also present in the uvea and can be seen with the slit lamp on the surface of the iris. Severe photophobi a can make a slit-lamp examination almost impossible without anesthesia. e. Cystine can be found in conjunctival biopsy specimens. f. Oral cysteamine has been shown to help the systemic problems but not the corn eal crystal deposition. g. Topical cysteamine drops must be applied every 1-2 hours, have an unpleasant odor, and are difficult to obtain but can markedly reduce crystal deposition in the cornea. L41 Corneal ulceration seen in patient with Riley-Day syndrome Riley-Day syndromeis due to impaired corneal sensation and decreased tearing. L42 . Congenital, or infantile, glaucoma Congenital, or infantile, glaucoma a. Primary congenital glaucoma occurs in about 1 out of 10,000 births and result s in blindness in 2%-15% of cases.

b. Visual acuity is worse than 20/50 in at least 50% of cases. c. This condition is bilateral in about two thirds of patients and occurs more f requently in males (65%) than in females (35%). d. No specific racial or geographic prevalence has been identified. e. Most cases of primary congenital glaucoma occur sporadically, but an autosoma l recessive form with variable penetrance has been identified. f. When no family history of congenital glaucoma exists, the chance of an affect ed parent having an affected child is approximately 5%. g. If the first child is affected, the risk of a second child being affected is still approximately 5%, rising to approximately 25% per subsequent offspring if two siblings are affected. h. Patient with infantile glaucoma has approximately a 5% chance of having a chi ld similarly affected. i. Primary congenital glaucoma does not appear to be associated with adult prima ry open-angle glaucoma; the incidence of steroid-induced intraocular pressure (IOP) elevation is no high er in parents of affected children than in controls. j. Corneal edema results from elevated IOP and may be gradual or sudden in onset . k. Corneal edema is often the presenting sign in infants younger than 3 months, because younger infants spend most of the day with their eye closed. Microcystic edema initially involve s the corneal epithelium but later extends also to the stroma, often accompanied by one or more curvilinear b reaks in Descemet's membrane (Haab striae). l. Although edema may resolve with IOP reduction, a scar will remain permanently at the site of Haab striae. m. Photophobia, epiphora, and blepharospasm result from the glare and epithelial abnormalities associated with corneal edema and opacification. n. Corneal enlargement occurs with gradual stretching of the cornea as a result of elevated IOP and often appears in slightly older infants up to about 2-3 years of age. o. The normal newborn has a horizontal corneal diameter of 9.5-10.5 mm; a diamet er of greater than 11.5 mm is suggestive of glaucoma. By age 1 year, normal corneal diameter is 10-11.5 mm; a diameter greater than 12.5 mm suggests abnormality. Glaucoma should be suspected in any child with a c orneal diameter greater than 13 mm. L43 Corneal clouding or tearing is more likely to be the presenting symptom in p atients with glaucoma in whom the onset is before the age of 3 months, whereas corneal enlargement will probably be the pre senting finding in older infants. This is probably true because younger infants spend most of the day with their eyes clos ed, making corneal enlargement difficult to detect. L44 Poor prognostic factors are present of an APD, corneal enlargement >14mm and onset <3month. L45 Ketamine and succinylcholine elevate intraocular pressure. Ketamine only doe s so at high doses. Succinylcholine does so by triggering undirected extraocular muscle contractions (fasciculations). L46 Simple ectopia lentis is usually bilateral and symmetrical with upward and t emporal lens displacement. Autosomal

dominant inheritance is most common. All inherited variety is bilateral, but rec essive variety is asymmetric. Dominant variety may not be present until the third, fourth, fifth or sixth decade. The r ecessive variety may be accompanied by a displaced and abnormally shaped pupil. L47 Ocular abnormalities occur in over 80% of patients with Marfan syndrome, wit h lens dislocation being the most common. In approximately 75% of cases, the lens is upwardly dislocated. L48 Ocular findings consist mainly of dislocated lenses (90% of patient with hom ocystinuria) (normally downward), a condition that typically occurs between the ages of 3 and 10 years. The lenses m ay dislocate into the anterior chamber, a finding suggestive of homocystinuria. L49 Homocystinuria is an elastosis and leads to hypercoagulability with thrombot ic vascular occlusions. The elastosis also causes defective zonular function and ectopia lentis in 90% of patients. It is inherited on an autosomal-recessive basis. L50 Ectopia lentis associated with mental retardation may be seen in which of th e following disorders: a. Hyperlysinemia. b. Weill-Marchesani syndrome. L51 Ectopia lentis is unusual in patient with Down syndrome (only after trauma). L52 Bilaterality makes a systemic condition more likely. Monocular cataracts rar ely herald significant systemic derangements. Anterior lenticonus may be part of Lowe's syndrome, but posterior lenticonus is usually isolated. Pigmentary retinopathy raises the possibility of congenital infections. L53 Congenital cataract formation secondary to a maternal HSV infection would be extremely unusual because the infection is typically acquired at birth. L54 A complete systemic evaluation of a patient with bilateral congenital catara cts should rule out congenital rubella infection, hypocalcemia, Lowe's syndrome, and trisomies. L55 Nystagmus is prominent in Leber's congenital amaurosis, but photophobia woul d be extremely unusual. Photophobia is prominent in cystinosis, but unless there are other complicating features, nysta gmus would not be expected. L56 The following statements about juvenile rheumatoid arthritis (JRA). a. It is the most common etiology of anterior uveitis in the pediatric populatio n.

b. Although it may have onset at any age, it is extraordinarily rare in children younger than 2 years. c. To conclude that uveitis is associated with JRA, there must be an antecedent history of joint symptoms. d. The uveitis of JRA is nearly always entirely anterior. e. Pauciarticular, early-onset patients are most likely to develop uveitis, but they are generally antinuclear antibody (ANA) positive, rheumatoid factor negative. Uveitis is rare in Still's disease, which is more common in boys. L57 There is a pediatric uveitis identical to juvenile rheumatoid arthritis (JRA ) with no previous history of joint disease, but this is generally considered a separate disorder. L58 . Uveitis in children Uveitis in children a. A specific cause is found in 50%-65% of pediatric cases. b. Traditionally, posterior uveitis has been thought to account for 40%-50% of u veitis cases in children, anterior uveitis 30%-40%, and intermediate uveitis about 20%, and panuveitis les s than 10%. c. Toxoplasmosis is the most commonly identified infectious cause, and juvenile rheumatoid arthritis (JRA) is the most commonly identified cause overall. In a recently studied population, JR A accounted for over 40% of childhood uveitis cases. d. The uveitis associated with JRA. i. Usually asymptomatic and chronic. ii. Such uveitis is characterized by anterior chamber cells and flare. iii. Prolonged inflammation may lead to posterior synechiae, band keratopathy, c ataract, hypotony, and glaucoma. iv. Vitritis and macular edema occur infrequently. v. Visual loss in JRA may be associated with multiple factors, including amblyop ia in susceptible patients. e. Surgery for cataracts and severe band keratopathy may be necessary for childr en with severe uveitis. Cataract surgery should be performed only in eyes that have been quiet for sever al months. Aggressive postoperative anti-inflammatory therapy is required. Severe band keratopathy may require chelation therapy with ethylenediaminetetraacetic acid (EDTA) after debridement of the epithelium. Chelation may be performed for decreased vision, for pain associated with recurrent erosions, or to improve the appearance of the eye. f. IOL is absolutely contraindicated. L59 Paediatric uveitis in HZV. Reactivation disease may be accompanied by a kera titis, either epithelial or stromal. In immunocompromised patients, a high risk of systemic spread with severe complicat ions exists and may be more likely if systemic steroids are used. Topical steroids may be used in immunocompromised pa tients. L60 Although keratitis is not universal in patients with herpes simplex virus (H SV) uveitis, it is usually present in affected children. L61 Up to 50% of pediatric patients with posterior uveitis have ocular toxoplasm osis. L62 Manifestation of acquired systemic toxoplasmosis includes rash, meningoencep

halitis, influenzae like syndrome, and retinitis. L63 Recurrent toxoplasmosis frequently presents as a granulomatous anterior uvei tis. Recurrent uveitis in toxoplasmosis may occur as anterior flare up without obvious reactivation of retinal disease. L64 Ocular Histoplasmosis. The presence of cells in the vitreous dictates recons ideration of the diagnosis. Although skin testing may support the DX, it may lead to worsening of macular disease. L65 Intermediate uveitis has a gradual onset, and the course may be mild in as m any as 50% of patients. About 75% of patients have bilateral involvement. Initially, a peripheral perivasculitis usua lly occurs along with small exudates adjacent to the inflamed peripheral retinal vessels. These exudates enter the vi treous, giving rise to floaters in the field of vision. In addition, the patient may note blurring of distance vision and dif ficulties with accommodation. The diagnosis is made clinically. Because of the high incidence of bilaterality in intermediat e uveitis, unilateral inflammation should cause the clinician to consider sarcoidosis, toxocariasis, peripheral toxoplasmo sis, or a retained intraocular foreign body. L66 The number of aqueous cells in the anterior chamber, and not the amount of f lare, should be the basis for treatment with topical steroids. Topical steroids may worsen the cataract and will not rev erse band keratopathy. L67 Studies of the natural history of retinopathy of prematurity (ROP) show that 65% of premature infants with birth weight <1,250 g will develop some stage of the disease. L68 The risk of retinopathy of prematurity (ROP) does not correlate with the pos tdelivery age but with gestational age (postconception age). African Americans may tend to develop less severe forms of the disorder. L69 Administration of supplemental oxygen to the newborn has been implicated as the cause of ROP; however, extremely premature infants vary highly in severity of and susceptibility to ROP, suggesti ng that factors other than hyperoxia play a role in the etiology of this disease. Currently, birth weight is a greater pre dictor of ROP than is oxygen exposure. ROP, or a disease process similar to it, has also been reported in infants who did no t receive supplemental oxygen. Oxygen monitoring is considered the standard of care in the intensive care nursery. Oxy gen deprivation can cause loss of life or of neurologic function, limiting how significantly oxygen administration can be cut back in these very ill infants.

L70 Zone I (posterior pole)-Circle with radius of 30°, twice disc-macula distance. L71 Sequelae of retinopathy of prematurity (ROP) may include: a. Angle-closure glaucoma. b. Cataract. c. Pseudoexotropia. d. Pseudoesotropia. L72 ROP. Pseudoexotropia is more common than pseudoesotropia but both can occur. L73 The following statements about screening protocols for premature infants. a. Only infants of birth weight <1,500 g require screening. b. Initial examination should take place 4 to 6 weeks after birth or at 30 weeks ' gestational age, whichever is later. c. In patients with mild or no retinopathy of prematurity (ROP), examinations ma y be conducted every 2 weeks. Examinations should be conducted every 2 weeks unless stage 3 prethreshol d ROP is found, in which case examinations should be conducted at least weekly. d. In patients with threshold ROP, cryotherapy should be delivered within 72 hou rs. L74 The incidence of juvenile cataract appears to be higher in patients with poo rly controlled diabetes. L75 Aside from retinal hemorrhage, orbital infiltration (causing proptosis, eyel id swelling, and ecchymosis) is leukemia's most frequent clinically evident ocular manifestation, occurring in 1%-2% of pat ients. Infiltration of the optic nerve by leukemic cells may cause optic disc edema and loss of vision, requiring prompt t reatment with low-dose radiation. The most common eye findings are retinal hemorrhages, especially flame-shaped lesion s in the nerve fiber layer. Whitecentered hemorrhages (Roth's spots), cotton-wool spots, optic disc swelling, and perivascular infiltration also may be seen but are less common. Choroidal infiltration is usually not ophthalmoscopica lly apparent. L76 Optic nerve involvement is visible if the disc is infiltrated by leukemic ce lls. Translucent swelling of the disc obscures the normal landmarks; with florid involvement, only a white mass is visible in t he region of the disc. If the nerve is involved centrally, a papilledema-like fundus picture ensues; such retrolaminar optic nerve involvement results in the loss of central vision. Therefore, a leukemic child with papilledema and a loss of central vision should be considered to have this complication. Early optic nerve involvement in leukemia constitutes a medical emergency because permanent loss of central vision is imminent. Such patients should undergo radia tion therapy as soon as possible (within 24-48 hour). However, patients receiving cancer chemotherapy ma y be abnormally sensitive to radiation therapy; blinding optic nerve atrophy has been reported in these patie nts. Radiation dosage may need to be adjusted accordingly. L77 Leukemia. Glaucoma may occur because of posterior synechiae formation with i ncreased relative pupillary block or pupillary seclusion, from tumor cells clogging the trabecular meshwork or from s pontaneous hyphema. Cataract may occur secondary to inflammation or radiation therapy. L78 . Gangliosidoses Gangliosidoses

a. Most common is type 1 Tay-Sachs disease. b. Inheritance is AR. c. Prominent cherry red spot typically in Tay-Sachs and Sandhoff's disease. d. Patient generally succumbs to neurological deterioration in their first decad e of life. L79 . Oculorenal syndromes include Lowe (oculocerebral) syndrome Oculorenal syndromes include Lowe (oculocerebral) syndrome a. This X-linked recessive disorder with renal defects presents in the first yea r of life, producing aminoaciduria, metabolic acidosis, proteinuria, hematuria, granular casts in the urine, and ric kets. b. Affected children are mentally retarded, hypotonic, and areflexic. c. The biochemical defect is in a phosphatase important in Golgi complex vesicul ar transport. d. No specific treatment exists. e. The most common eye defect is cataract. The lenses are small, thick, and opaq ue and may demonstrate posterior lenticonus. f. Congenital glaucoma may be present. L80 Ocular findings in patients with renal failure includes are exudative RD, ca lcium crystals in the conjunctivae, macular oedema. Diffuse arteriolar attenuation is not a characteristic finding in renal failure, although it may occur if there is severe associated hypertension. L81 Reliable methods distinguishing juvenile retinoschisis from Goldmann-Favre i nclude EOG (abnormal in GoldmannFavre, and normal in juvenile retinoschisis), and careful family history (AR in Goldmann-Favre and X-linked in juvenile retinoschisis). Both diseases are characterized by foveal and peripheral retinos chisis, as well as retinal pigment epithelial disturbances, although the latter are more severe in Goldmann-Favre. L82 Fundus finding that may be seen in association with tapetoretinal degenerati on includes CMO, vitreous cells, PSC and subretinal degeneration (mimic Coat's disease). L83 Although electrophysiologic testing may show diffuse retinal involvement in sector retinitis pigmentosa, the condition rarely progresses, and the prognosis is good. This is reflected in normal b-wave implicit times. L84 The most common underlying lesion causing bull's eye maculopathy is cone dys trophy. L85 . Leber Congenital Amaurosis Leber Congenital Amaurosis

a. Hyperopic refraction and sluggish pupillary responses are characteristic find ings, although some patients are myopic. b. 5-10% of patients with Leber Congenital Amaurosis have associated hearing los s. L86 Central vision is usually preserved in patients with pattern dystrophies of the RPE. L87 Most cases of morning glory disc have poor vision. The condition is usually unilateral and may be associated with cranial vault abnormalities. It is more common in girls. L88 Nonrhegmatogenous retinal detachments occur with optic nerve pits, primarily involving the macula. L89 Myelinated (medullated) retinal nerve fibers. a. Normal myelination starts at the lateral geniculate ganglion and stops at the lamina cribrosa. b. The most common location is along the disc margin. c. More common in boys than girls. d. 20% bilateral. L90 is far more common in bilateral optic nerve hypoplasia. Most De Morsier's sy ndrome (septooptic dysplasia) De Morsier's syndrome (septooptic dysplasia) authorities feel that neuroimaging should be conducted in both unilateral and bi lateral optic nerve hypoplasia. L91 Optic disc pit is more common in inferotemporal and central disc. L92 Although most patients with drusen can be identified by simple funduscopic e valuation, occasionally it is difficult to tell for certain that drusen are responsible for the swollen disc appearance. B-scan ultrasonography can be helpful in this situation. This technique permits detection of bright objects in the end of the optic nerve. CT can also detect these particles. Red free photograph is no value, however, disc drusen do autofluoresc ence on FA, to elicit this. Fluorescein barrier filters must be used with no injection. L93 Devic's disease, or neuromyelitis optica, is considered by most experts as a n entity separate from multiple sclerosis (MS). It is characterized by acute optic neuritis (usually bilateral) and transv erse myelitis. Visual prognosis is much poorer for the optic neuritis of Devic's disease than for typical optic neuritis . L94 Options for the treatment of the histiocytoses include: a. Curettage. b. Intralesional steroid injections. c. Low-dose radiation. d. Systemic steroid therapy. L95 . Histiocytosis X (Langerhans cell histiocytosis) Histiocytosis X (Langerhans cell histiocytosis) a. Treatment consists of observation of isolated asymptomatic lesions, excision of painful and easily accessible lesions, systemic corticosteroid administration, or low-dose radiatio n; all modalities have a high rate of success. b. Prognosis is unfavorable for children with onset younger than 2 years and hep atic and bone marrow involvement. L96 Tumors of bony origin. a. Fibrous dysplasia has a slow progression that ceases when skeletal maturation is complete. b. The most serious complication is visual loss caused by optic nerve compressio n, which may occur acutely.

c. Periodic assessment of vision, pupil function, and optic disc appearance is i ndicated. d. Surgical treatment is indicated for functional deterioration or significant d isfigurement. e. Fibrous dysplasia and ossifying fibroma are similar disorders characterized b y destruction of normal bone and its replacement by fibroosseous tissue. In both conditions, orbital radiogra phy and CT show varying degrees of lucency and sclerosis. Histopathologically, ossifying fibroma is dist inguished by the presence of osteoblasts. f. Ossifying fibroma tends to be a more locally invasive lesion than fibrous dys plasia; some authorities recommend early excision. g. Polyostotic variety of fibrous dysplasia may be accompanied with sexual preco city and hyperpigmented skin macules. L97 Phlebolith are characteristic of cavernous not capillary haemangioma and occ ur due blood stagnation. L98 (Rhabdomyosarcoma, neuroblastoma, Wilms tumor) are three most common solid p ediatric malignancies. Leukemia is the most common of all pediatric malignancies. L99 Any case of acquired ptosis should have neuroimaging to rule out rhabdomyosa rcoma. L100 Well-differentiated pleomorphic rhabdomyosarcoma least common but the best prognosis. L101 Exenteration is reserved for failure or recurrences in the treatment of rha bdomyosarcoma. L102 . Neurilemoma Neurilemoma a. 1.5% of patient with NF1 will have schwannoma. b. The lesion can be tender or painful. c. Malignant degeneration is rare. d. The two classic histologic patterns are Antoni A (regular arrangement of eosi nophilic spindle cells with palisading nuclei) and Antoni B (haphazardly arranged stellate cells in a myxoma tous matrix). e. Schwannomas grow in close relation to peripheral nerves. f. Perineural spread and compression accounts for pain.

L103 Unlike schwannomas, neurofibromas grow independent of peripheral nerve. In addition they are generally osteolytic. L104 Features might be used to differentiate between neurofibroma and neurilemom a are: a. The presence of axon and perineural cells in the neurofibroma. b. The presence of true capsule around neurilemoma. c. Schwann's cells (neural crest derivative) may be seen in both. d. Positive staining for S-100 is characteristic of any cell derived from neural crest. Thus this will be seen in both lesions. L105 Optic gliomas are usually benign in children. If the tumor is located withi n the orbit, observation may be appropriate. On the other hand, if the tumor is posterior (i.e., involving the chiasm and opt ic tract), radiation therapy may be appropriate to avoid hydrocephalus and damage to contiguous central nervous syst em structures. L106 Radiographically meningiomas are more likely to produce hyperplastic bony c hanges (although bone destruction can be seen). L107 . Neuroblastoma Neuroblastoma a. In some pediatric series, the incidence of neuroblastoma is greater than rhab domyosarcoma. b. It usually originates in either the adrenal gland or the sympathetic ganglion chain in the retroperitoneum or mediastinum. c. Second most common site of origin is retroperitoneal sympathetic chain. d. The site of origin is the adrenal glands in at last half of all cases. e. The tumor presents with metastases in about one third of patients. f. The mean age at diagnosis of patients with orbital neuroblastoma metastasis i s about 2 years. g. Histopathologies of metastatic neuroblastoma are sheets of indistinct round c ells with scanty cytoplasm, copious mitotic figures, area of tumour necrosis, and bony invasion. h. Homer-Wright rosettes are usually not seen in orbital metastasis, being limit ed to the better differentiated primary tumour. i. Ominous prognostic factors are age <1 year (AAO and Wright better prognosis c onsiderably better in infants under age 1 year than in older children) and bone metastasis. Even with metastasis to the liver, bone marrow, and spleen, survival may be as high as 84% of cases. j. Opsoclonus is the most common paraneoplastic syndrome associated with neurobl astoma. L108 . Ewing sarcoma Ewing sarcoma a. Tumor composed of small round cells that usually originate in the long bones of the extremities or the axial skeleton. b. Ewing sarcoma is the second most frequent solid tumor source of orbital metas tasis. Usually present with proptosis, hemorrhage, and inflammation from tumour necrosis. c. It occurs in individual aged 10-25 years. d. Usually, there is no globe involvement, and treatment is principally chemothe rapy, although radiation to remote sites has been used with some success. L109 The histopathology starry sky (histiocytes scattered amidst a monotonous ba ckground of lymphocytes) is the classic appearance of Burkitt's lymphoma.

L110 Generally dermoid cyst does not enlarge after the first year of life. L111 Idiopathic orbital inflammatory disease (orbital pseudotumor). a. The typical pediatric presentation is acute and painful, resembling orbital c ellulitis more than tumor or Graves ophthalmopathy. b. Bilaterality (one third) and episodic recurrence are common, as are associate d systemic manifestations such as headache, fever, nausea and vomiting, and lethargy. c. Uveitis is common and occasionally constitutes the dominant manifestation. d. Peripheral blood and local tissue eosinophilia are more common in children. L112 One study in which 302 epibulbar lesions were removed from children younger than 15 years reported the following frequencies: a. Nevus: 29%. b. Dermoid: 19%. c. Epithelial inclusion cyst: 11%. d. Dermolipoma: 10%. e. Pyogenic granuloma: 6%. L113 Approximately 30% of epibulbar dermoid are associated with other ocular and systemic abnormalities. L114 . Retinoblastoma Retinoblastoma a. Onset later than age 5 is rare, but primary retinoblastoma can present in adu lthood. b. Both of the patient's parents and all siblings should also be examined. c. In about 1% of cases, a parent may be found to have an unsuspected fundus les ion that represents either spontaneously regressed retinoblastoma or a benign growth known as retinoma or r etinocytoma that can be an expression of the retinoblastoma gene. d. The retinoblastoma gene maps to a locus within the q14 band of chromosome 13. (13q14)

e. If a couple has a child with bilateral retinoblastoma, the chance that this c hild will have an affected brother or sister is approximately 6%. f. If two or more siblings are affected, this implies a chromosomal defect in on e parent. Therefore each additional child has a 40% chance of inheritance the tumour. g. The probability of a patient who survives bilateral retinoblastoma giving bir th to a child with retinoblastoma is 40%. h. Post radiation malignancies are osteogenic sarcoma of bone, malignant melanom a of the eyelid, and leiomyosarcoma of the eye and orbit. i. Blood vessels may absorb released nucleic acids from the necrotic cells, and consequently take on appearance of basophilia (given an acidic nature). j. Light microscopic characteristic of retinoblastoma is zonal necrosis surround ing the blood vessels, over grow exceeds the blood supply. k. The most common site of retinoblastoma spread outside the eye is CNS, second through the lymph node. l. In trilateral retinoblastoma, pinealoblastoma represent primary malignancy of the pineal gland, not as a metastasis. m. CT and US are standard because they can demonstrate otherwise undetectable ca lcification. MRI can determine the tumour spread. Serum level of carcinoembryonic antigen may be elev ated in large tumours. Paracentesis is not a typical of part of the workup but may be used in difficult cases to show elevated level of lactate dehydrogenase in aqueous humour relative to serum level. n. Metastatic workups include bone marrow biopsy, bone scan, and lumbar puncture . o. Enucleation is usually employed in patients with large unilateral tumors, esp ecially when the involved eye is judged to have no visual potential. Enucleation has also traditionally been perf ormed in the eye with more advanced disease in patients with bilateral retinoblastoma. The surgeon should o btain as long a section of optic nerve as is possible. This is best achieved via a temporal approach with s lightly curved enucleation scissors. p. Retinoblastoma cells are highly radiosensitive. External beam radiotherapy is a method of delivering wholeeye irradiation to treat advanced retinoblastoma, particularly with diffuse vitr eous seeding. Side effects include cataract, radiation retinopathy, optic neuropathy, chronic dry eye, and atrophy of muscles and subcutaneous tissue. The risk of secondary tumors is increased in patients who r eceive external-beam radiotherapy. This risk is most pronounced in children who are younger than 12 m onths at the time of irradiation. q. Cryotherapy and laser photoablation Small tumors may respond to local therapy . Cryotherapy can be used in small anterior tumors that are 3.5 mm or less in diameter and 2.0 mm or less in thickness. Laser photocoagulation is useful in tumors 4.5 mm or less in base diameter and 2.5 mm or less in thickness. Complications include retinal tears and detachments, preretinal fibrosis, and va

scular occlusion. Recurrences can be a problem. r. Chemotherapy traditionally, chemotherapy had been reserved for patients with metastatic disease. Chemotherapy has recently been introduced as a primary treatment to reduce tumor bulk in an attempt to avoid external-beam radiation (chemoreduction). Cytotoxic drugs are given initia lly for 2-3 months to decrease tumor size. This method is effective even in cases of total retinal det achment. Treated retinoblastoma sometimes disappears altogether, but more often it persists as a calcified mass (type 1, or cottage cheese, pattern) or a translucent grayish lesion (type 2, or fish flesh, pattern, which is difficult to distinguish from untreated tumor). L115 Although the presenting symptoms and signs depend on the extent of the tumo r, the frequency of some of the presenting signs and symptoms are summarized below, with leukocoria and strabism us presenting in most of the instances: a. Leukocoria: 60%. b. Strabismus: 22%. c. Decreased vision: 5%. d. Incidental finding: 4%. e. Orbital cellulitis: 2%. L116 Ocular disorders associated with optic nerve hypoplasia are albinism, aniri dia, coloboma, and Duane syndrome. L117 Ocular disorders associated with optic nerve hypoplasia include: a. Albinism. b. Aniridia. c. Coloboma. d. Duane's syndrome. L118 The typical color of JXG is orange, the combination of high lipid content ( yellow) and high vascularity (red). They may also yellow-brown especially when located on the iris. L119 . Juvenile xanthogranuloma Juvenile xanthogranuloma a. Nonneoplastic histiocytic proliferation that develops in infants younger than 2 years.

b. It is characterized by the presence of Touton giant cells. c. Skin involvement is typically but not invariably present in the form of one o r smaller round papules, orange, or tan in color. d. Iris lesions are relatively rare and virtually always unilateral. The fleshy yellow-brown mass may be small and localized or diffusely infiltrative of the entire iris, with resulting heter ochromia. e. Spontaneous bleeding with hyphema is a characteristic clinical presentation. f. Secondary glaucoma may cause acute pain and photophobia and ultimately signif icant visual loss. g. Juvenile xanthogranuloma is a self-limited condition that usually regresses s pontaneously by age 5 years, but treatment is indicated for ocular involvement to avoid complications. h. Topical corticosteroids and agents that lower intraocular pressure, given as necessary, are generally sufficient to control the problem; surgical excision or radiation should be cons idered if intractable glaucoma is present. L120 . Medulloepithelioma Medulloepithelioma a. Originates from the nonpigmented epithelium of the ciliary body and most ofte n presents as an iris mass during the first decade of life. b. Secondary glaucoma, hyphema, and ectopia lentis are less frequent initial man ifestations. Leukocoria may be the presented finding. c. This rare lesion shows a spectrum of clinical and pathological characteristic s, ranging from benign to malignant. d. Although metastasis is rare, local invasiveness can lead to death. The only m edulloepithelioma that associated with metastatic potential are those that extend into the orbit. e. Teratoid elements are often present. f. Enucleation is usually required and is curative in a large majority of cases. L121 If indicated, prophylactic irradiation following the DX of acute leukemia i n children should involve both eyes. The highly vascular uveal tissues are common sites for leukemic infiltration. L122 Café-au-lait spots, Lisch nodule, and neurofibroma are found in >90% of patie nts with NF, whereas other CNS tumour are present in less than 10% of cases. L123 Cutaneous melanoma and choroidal osteoma are rare in patient with NF. L124 Light microscope appearance of Lisch nodules is indistinguishable from thos e of iris nevi. Both are composed of groups of benign, nevus type melanocytes. L125 Glioma associated with NF are more hamartomatous and less neoplastic than i solated glioma. L126 ERG is not affected in patient with optic nerve hypoplasia because photorec eptor is not affected. L127 Adenoma sebaceum is common in tuberous sclerosis and may be confused with a cne vulgaris. L128 The complete Sturge-Weber syndrome includes facial hemangioma, ipsilateral glaucoma, and epilepsy. The seizures are contralateral (secondary to the presence of leptomeningeal hemangioma overly ing the ipsilateral cerebral convexity). L129 . Mechanism of glaucoma is due to elevated EVP, angle anomaly, and rubeosis

of the Sturge-Weber syndrome Sturge-Weber syndromeangle. L130 Hemangioblastomas of the retina associated with central nervous system, mos t often the cerebellum in 20% of cases. L131 Like Sturge-Weber syndrome, iris neovascularization may complicate the cour se of retinal angiomatosis. In SturgeWeber syndrome, the retina overlying a diffuse hemangioma may develop serous det achment or ischemia and provoke neovascularization. In retinal angiomatosis, the cause tends to be serous detach ment of the retina secondary to leakage at the capillary hemangioma. L132 . Ocular manifestations are unilateral and congenital but may progress some what during Wyburn-Mason syndrome Wyburn-Mason syndromechildhood. Rare bilateral cases have been reported. L133 Ataxia-telangiectasia (AT) Ataxia-telangiectasia (AT)usually associated with hypoplasia of the thymus and d ecreased levels of circulating immunoglobulin especially immunoglobulin A (IgA) deficiency. Recurrent respirato ry tract infections are a serious problem, frequently causing death in adolescence or young adulthood even with op timal antimicrobial and supportive treatment. The increased susceptibility to various malignancies, particularly ly mphomas and leukemias, contributes to early mortality in one third to one half of cases. L134 Potential ocular manifestations of the craniosynostoses include: a. Papilledema. b. Exposure keratitis. c. Tortuous retinal vasculature. d. Optic atrophy. L135 A patient with strabismus associated with craniosynostosis. Abnormalities i n the development and origins of the extraocular muscles make correction of the strabismus very difficult; furthermor e, should reconstructive surgery for the orbits be necessary, as is often the case, any good results obtained from muscle surgery would almost certainly be ruined.

L136 The Pierre Robin sequence The Pierre Robin sequenceis characterized by micrognathia, glossoptosis, and cle ft palate. L137 . Fetal Alcohol Syndrome Fetal Alcohol Syndrome a. A pattern of malformations has been observed in children born to women with a history of heavy alcohol use during pregnancy. b. Alcohol and other teratogens can produce a wide range of effects on the devel oping fetus, depending on consumption or dose, timing of intake, genetic background, and other factors. c. Ocular involvement can include hypoplasia of the optic nerve head and increas ed tortuosity of retinal vasculature. Optic disc anomalies are the most common malformations of the fundu s (up to 48%). Frequently, the disc is small with sharp and often irregular margins; the condit ion may be unilateral or bilateral. A combination of anomalies of the optic disc and the retinal vessels is a typical finding in the fundus of a child with fetal alcohol syndrome. d. Telecanthus but not hypertelorism. e. Anterior segment dysgenesis. f. Blepharophimosis like picture. L138 The pupillary light reflex can be shown to develop at approximately what ag e 30 weeks' gestation. L139 Signs of poor vision in a 1-month-old infant might include: a. Paradoxic pupillary response to light. b. Oculodigital sign. c. Fixation only on extremely bright lights. L140 The superior oblique has its anatomic origin at the orbital apex. Its mecha nical origin is at the trochlea because this is where its effective force is generated. L141 Shortest length active muscle belly is the superior oblique muscle (32 mm); others are (37-40 mm). L142 All six extraocular muscles that attach to the globe must penetrate Tenon's capsule. The levator muscle does not. L143 Extraconal orbital fat may be encountered as far as 10 mm anterior from the limbus. L144 Two muscles may nearly always be safely removed. Removal of four muscles is certain to cause ischemia. Removal of three muscles is likely to lead to some degree of ischemia. L145 Fusional convergence is always accompanied with stimulation of accommodatio n, while divergence is not always accompanied with relaxation of accommodation. L146 For fusion to exist, the two retinal images must be similar in size and sha pe, otherwise confusion occur. L147 Monocular clues Monocular cluescontribute to depth perception. These monocular clues include obj ect overlap, relative object size, highlights and shadows, motion parallax, and perspective. Monocular clues are probably more important than stereopsis for depth perception. At distances farther than 20 feet, we rely almo st entirely on monocular clues for depth perception. L148 The critical period for proper development of the visual pathways is longer i n humans than in most experimental animals. L149 The incidence of amblyopia in the general population is approximately 2% to 3%.

L150 Microtropia Microtropiais most likely after successful strabismus surgery, as well as in the setting of anisometropia. L151 Most of the normal adult population have small phoria mostly exophoria. L152 Optokinetic nystagmus has been used to quantify acuity, but the procedure i s more cumbersome and difficult to score accurately than preferential looking. L153 The Krimsky test uses prisms over the fixating eye to center the light refl ex over the pupil in the deviating eye. Foveal fixation is not required. L154 . Simultaneous prism cover test Simultaneous prism cover test a. Whereas the alternate cover test measures the total deviation (phoria and tro pia), the simultaneous prism and cover test is helpful in determining the actual heterotropia when both eyes are uncovered (tropia alone). b. The test is performed by covering the fixating eye at the same time the prism is placed in front of the deviating eye. c. The test is repeated using increasing prism powers until the deviated eye no longer shifts. Again, the power of the prism is the measure of the deviation. L155 Patient with new-onset excyclotropia when tested with a Maddox rod held ove r the affected eye with its cylinders running horizontally he will see an oblique line running superonasal to inferote mporal. In excyclotropia, the superonasal retinal quadrant is rotated vertically toward the 12-o'clock position, and the i nferotemporal quadrant is rotated toward the 6-o'clock position. The vertical line from the Maddox rod will run superonas al to inferotemporal on the retina and be perceived the same way. L156 The Lancaster red-green test is used primarily in patients with diplopia ca used by incomitant strabismus and requires that the patient have normal retinal correspondence. In right esotropia with red infront the right, the red light will be seen to the left of green light. Lamkin 328 L157 Alternate-cover tests, Maddox rod testing, and cover-uncover testing utiliz e actual fixation behavior and will not fall victim to large angle kappa or facial anomalies. The Krimsky and Hirschberg meth ods use apparent eccentricity of pupillary reflexes and may be misguided by these factors.

L158 A patient whose distance esotropia increases by >10. at near is said to hav e clinically high AC/A ratio. L159 Factors known to be associated with decompensated latent strabismus include alcohol, fatigue, and illness. Decreased VA (e.g. cataract) also may lead to fusional breakdown, with resultant manifest strabismus. L160 Suppression with the distance worth four-dot test but fusion on near worth four-dot testing indicates peripheral fusion with no central fusion. L161 Use of the Worth four-dot test at distance illuminates a smaller, more cent ral portion of the retina; testing at closer distances illuminates a progressively larger, more peripheral portion. Distance testing can reveal small suppression scotomata that may be inapparent on testing at near. Results of testing should b e reported as fusion or suppression of one eye at distance and at near. Many cases of small-angle strabismus (eg, that associated with mono fixation syndrome) may combine fusion at near with suppression of one eye at distance. L162 Atropine is relatively contraindicated as a cycloplegic agent in: a. Albinos. b. Neonates. c. Patients with Down's syndrome. L163 Exaggerated sensitivity to cholinergic blockade has been reported in albino s and patients with Down's syndrome. Infants are also particularly sensitive. Atropine is a treatment for heart block (acutely) and should not cause arrhythmias in patients with this disorder. L164 The prism adaptation test The prism adaptation testinvolves the preoperative use of prisms to neutralize a deviation with prisms for a given period of time, followed by surgery for the amount of strabismus fully neutraliz ed by the prisms. Thus, the prism neutralization may be used to predict the outcome of surgery for a given deviati on, to determine the maximum deviation, and to estimate fusion potential at that deviation. In addition, some patients e xhibit a different deviation with the prism adaptation test than with cover testing. It remains to be shown whether the form er deviation provides better surgical results than the latter. In a controlled, randomized study of patients who had a cquired esotropia, 60% underwent prism adaptation and 40% did not; of those who responded to prisms with motor stabilit y and sensory fusion, half underwent conventional surgery and half underwent augmented surgery based on the prism-ada pted deviation. Success rates were highest (89%) in the prism adaptation responders who underwent augmented su rgery, 79% in those who underwent traditional amounts of surgery and lowest (72%) in those who did not u ndergo prism adaptation. L165 Nystagmus blockage syndrome Nystagmus blockage syndromedevelops purposefully as the patient seeks his or her null point. L166 High AC/A ratio are more likely to develop stress induced decompensation th an accommodative esotropia with fully correction due to high hyperopia. L167 Patient with intermittent esotropia should not be corrected fully, because full hyperopic correction will weaken the patient's fusional divergence, which is the force keeping accommodative esotropi a intermittent at its outset. Then the esotropia may become constant without crutch of the spectacle.

L168 Cyclic esotropia. The cycle is typically 48 hours long: 1 day of esotropia, and 1 day of orthotropia. Many experts believe that this is a variety of accommodative esotropia, with many features si milar to it. The angle of deviation is generally moderate. The time of onset is similar to accommodative esotropia. Ful l hyperopic correction may correct the esotropia. Amblyopia is possible but relatively uncommon. L169 Divergence Insufficiency is indistinguishable from sixth nerve palsy except it is typically comitant. Sixth nerve palsy is more likely to have esotropia at near as well. L170 Congenital sixth nerve paralysis is thought to be caused by the increased i ntracranial pressure associated with the birth process and usually resolves spontaneously. Sixth nerve palsy occurs much more frequently in childhood than in infancy. A careful history should be taken to define antecedent infections, head trauma, or other possible inciting factors for sixth nerve palsy. Neurologic evaluation and computed tomography or magnetic resonance imaging are indicated when neurologic signs or symptoms are present. L171 Abducens palsy in a child is a common postviral syndrome and typically reso lves uneventfully. Second most common cause is increased ICP. L172 Sensory deprivation in a child younger than 6 years is more likely to lead to esotropia, whereas that occurring in adults is more likely to lead to exotropia. L173 Some cases of exotropia may resolve entirely (Intermittent exotropia usuall y fades away as the child progresses through early adolescence), but this is less common than with accommodative esot ropia. Most cases remain intermittent, become manifest with fatigue, stress, illness, or alcohol consumpt ion. Some cases become constant and require surgery. L174 DVD become manifest only when visual input to the affected eye is interrupt ed (occlusion, amblyopia, and other organic disease). The deviation is highly variable and difficult to measure, mak ing surgery difficult to quantify accurately. L175 There are two potential explanations for the vertical deviation in lateral gaze. The first is overaction of the inferior obliques. In this condition, on lateral gaze, there is hypotropia of the abducte d, nonfixing eye. In dissociated vertical deviation (DVD), in lateral gaze there is no associated hypotropia, in violation of Hering's law. In fact, if the DVD is bilateral, there may be hyperdeviation of the abducted, nonfixing eye. The disti nction between the two is important for surgical planning. L176 Unlike inferior oblique muscle overaction, superior oblique muscle overacti on is usually not divided into primary and secondary forms because paresis of the inferior rectus and inferior oblique musc les are uncommon.

L177 The procedure of choice in acquired bilateral superior oblique pareses with symptomatic diplopia is the bilateral superior oblique tuck. L178 In unilateral superior oblique palsies, lateral gaze toward the involved si de generally relieves the diplopia. Aggravation of diplopia in both directions argues for bilateral involvement, as do aggravation with head tilt in either direction. L179 Indications for surgical treatment of superior oblique paresis are signific ant head tilt, large hyperdeviation in primary position, and symptomatic diplopia. L180 Each millimeter of recession of vertical muscle will correct 3 prism diopte rs of vertical strabismus. L181 The procedure of choice for superior oblique paresis with excyclotorsion on ly (no vertical diplopia) is the lateral transposition of the superior oblique tendon (Harada Ito procedure). L182 In brown syndrome duction and version testing mimics paresis of the ipsilat eral inferior oblique. Acquired cases should be observed for a considerable period because many spontaneously improve. L183 For an A deviation to be significant, the difference in measurements betwee n upgaze and downgaze must exceed 10 prism diopters. For V patterns, the difference must measure 15 prism diopters. A pproximately 15% of horizontal strabismus cases have a significant A or V component. L184 In V-pattern esotropia with IO overaction, IO myectomy will cause 15-25 pri sm diopters of eso-shift in upgaze. This only makes deviation more comitant. Thus the deviation in primary gaze must be a ddressed with appropriate medial rectus recession and lateral rectus resection. L185 Adjustable Suture is also useful for superior oblique tenotomy, because qua ntifying effect for superior oblique tenotomy is difficult. It is also useful for DVD, strabismus associated with Gra ves' disease, and muscle transposition surgery in A- or V-pattern horizontal strabismus. L186 The medial rectus muscle nearly always undergoes progressive contracture in the setting of non-resolving sixth nerve palsy. This virtually never occurs in Duane's syndrome. Forced ductions ma y help differentiate between the two in puzzling cases of a long-standing abduction deficit. L187 Indications for surgical intervention in Duane's syndrome include: a. Significant head turn. b. Significant deviation in primary position. L188 Amblyopia in Duane's syndrome is generally caused by anisometropia rather t han strabismus. In any case, it must be treated before attempting surgical realignment. Globe retraction may be lesse ned by lateral rectus recessions but is rarely if ever a solitary indication for surgery. L189 The etiologies of pediatric third nerve palsy in descending frequency are c ongenital, traumatic, inflammatory, migrainous, and neoplastic. In adult are microvascular, aneurysmal, traumatic, a nd neoplastic. L190 The deviation in Graves' disease is classically noncomitant and not easily addressed with prisms. Resection technique is generally avoided because Graves' disease generally causes consider able restriction. L191 Duane's syndrome is more common in girls, whereas congenital apraxia is mor e common in boys. L192 Motor nystagmus is virtually always of the jerk type. In fact, some nystagmo

logists are reluctant to term any pendular variety as true nystagmus, maintaining that biphasic velocity is part o f the definition. Many pendular cases of motor nystagmus are actually jerk, with very similar velocities requiring electr ooculography (EOG) for differentiation. L193 The hallmarks of congenital motor nystagmus include dampening with converge nce and aggravation by fixation. Near work may be associated with convergence and dampening with subsequent impro vement in acuity. L194 Spasmus nutans generally has its onset within the first year of life and re solves within 2 years. L195 The following procedures might a two-stage adjustable suture technique be a dvisable. a. Disassociated vertical divergence (DVD). b. Strabismus associated with Graves' disease. c. Muscle transposition surgery in A- or V-pattern horizontal strabismus. L196 Transposition procedures involve moving the extraocular muscles out of thei r original planes of action. These procedures are generally reserved for treatment of paralytic strabismus, small v ertical and horizontal deviations, and Aand V-patterns. The usual indications for transposition procedures include paral ysis of cranial nerves III and VI as well as monocular elevation deficiency. L197 Bifocal might be added later if a significant residual esodeviation remains at near with distance correction. L198 It is better to overcorrect than to undercorrect exodeviation. L199 The maximum advisable recession of the medial rectus muscle in the initial surgical management is 6 mm. If this result is undercorrection, then additional surgery is necessary. L200 The maximum advisable resection of the lateral rectus muscle in the initial surgical management is 9 mm. L201 The maximum advisable recession of the lateral rectus muscle in the initial surgical management is 10 mm. L202 The maximum advisable resection of the medial rectus muscle in the initial surgical management is 10 mm. L203 In A-pattern exotropia, superior oblique tenotomy will cause esodeviation i n downgaze is 40., little in primary gaze and non in upgaze. L204 Surgery on inferior rectus muscle is more likely to lead to lid malposition postoperatively than surgery on superior rectus, because Surgery on inferior rectus muscle always involve manipulation of the lid retractors. L205 . Malignant Hyperthermia Malignant Hyperthermia

a. MH is often triggered by many inhalational anesthetics, by the muscle relaxan t succinylcholine, and by local anesthetics of the amide type (lidocaine). Wright (The incidence of MH is betwee n 1:6000 and 1:30000, but it is thought to be higher in children. The mortality rate used to be as high as 70 %, but it is less than 10% now. MH is triggered by succinylcholine, halothane, enflurane, and isoflurane. All lo cal anesthetics may be used safely. Creatine phosphokinase is elevated in up to two-thirds of MH patients, b ut normal results have no predictive value and should not be relied upon. Frequently, the earliest sign is tachycardia or elevated end-tidal carbon dioxide. Other early signs include unstable blood pressure, tachypnea, sw eating, muscle rigidity, cyanosis, and dark urine. A rise in temperature is a later sign.). b. AAO. Malignant hyperthermia (MH) is an acute metabolic disorder that can be f atal if diagnosis and treatment are delayed. In its fully developed form, MH is characterized by extreme heat pr oduction. MH is often triggered by many inhalational anesthetics, by the muscle relaxant succinylcholine, and by local anesthetics of the amide type (lidocaine, Bupivacaine and Mepivacaine). c. MH can occur as an isolated case or as a dominantly inherited disorder with i ncomplete penetrance. d. Frequently, the earliest sign of MH is tachycardia that is greater than expec ted for the patient's anesthetic and surgical status. Other arrhythmias may also occur, as can unstable blood pressur e. Other early signs include tachypnea, sweating, muscle rigidity, blotchy discoloration of skin, cyanosis, a nd dark urine. Onset may be manifested during the induction of anesthesia by trismus caused by masseter musc le spasm, although the significance of masseter spasm is controversial. e. A later sign is a rise in temperature, which may reach extremely high levels. Other later signs include respiratory and metabolic acidosis, hyperkalemia, hypercalcemia, myoglobinuria, and renal failure, skeletal muscle swelling, heart failure, disseminated intravascular coagulation, and card iac arrest. f. Ideally, MH should be diagnosed and treated before the temperature rises sign ificantly. Survival is greatly improved when treatment begins early. g. Once the condition is recognized, anesthetic agents should be discontinued, h yperventilation with oxygen started, and treatment with intravenous dantrolene begun. works to prevent relea se of calcium Dantrolene Dantrolenefrom the sarcoplasmic reticulum, preventing the excessive contractile response of muscle. h. Surgery should be terminated as soon as possible, even if incomplete. i. Temperature monitoring should be established along with an intra-arterial cat heter for the monitoring of blood pressure and arterial blood gases. Electrolytes, electrocardiograms, urine outpu t, prothrombin time, partial thromboplastin time, fibrinogen, and pulse oximetry should also be monitored. Ce ntral venous or pulmonary artery pressure may be monitored if indicated by the patient's condition. L206 Botulinum toxin Botulinum toxininhibits release of acetylcholine from presynaptic nerve terminal s. Most common complications are

ptosis, second, is secondary vertical strabismus. L207 Risk factors for esotropia are white race, maternal cigarette smoking, incr easing maternal age, and low birth weight. L208 Risk factors for exotropia are maternal cigarette smoking, and low birth we ight. G. . RETINA AND VITREOUS RETINA AND VITREOUS L1 Effects of acute or chronic systemic hypertension on the retinal vascular sys tem include all of the following: a. Focal or generalized vasoconstriction. b. Breakdown of the blood-retinal barrier with subsequent hemorrhage and exudate . c. Thickening of arteriolar walls with secondary nicking of arterioles. d. Histopathologic evidence of endothelial hyperplasia. e. Development of micro- and macroaneurysms. L2 The following statements about blood pressure-induced choroidal disease. a. Crucial pathophysiologic events lead to occlusion of the choriocapillaris. b. Elschnig's spots are characteristic. c. Exudative retinal detachment (RD) may develop as a secondary manifestation. d. Hypertensive choroidopathy may be associated with acute elevations in intraoc ular pressure (IOP). e. Acute hypertensive episodes (rather than chronic hypertension) can lead to fi brinoid necrosis of choroidal arterioles. L3 The prevalence of endophthalmitis is 1.4% after intravitreal triamcinolone in jection. However, this also includes cases of pseudoendophthalmitis. Excluding pseudoendophthalmitis, the prevalence is approx imately 0.6%. L4 Evidence of breakdown in the blood-retinal barrier in patients with diabetes may be detected before the onset of clinical retinopathy. Vitreous fluorophotometry may reveal leakage of fluorescein into th e vitreous from abnormally permeable retinal vessels before any clinically detectable changes have occurred. L5 In diabetes, the visual prognosis for diffuse macular edema is worse than tha t for focal macular edema. The diffuse pattern is associated with more widespread vascular insult, often ischemic, and is more difficult to treat than most cases of focal edema. L6 In untreated eyes with preproliferative disease, the probability of progressi on to proliferative retinopathy over 2 years is approximately 50%.

L7 The factor that is most strongly correlated with the development of choroidal effusion following panretinal photocoagulation is short axial length (<23 mm). Shorter eyes have greater resis tance to fluid efflux from the suprachoroidal space. The percentage of retinal surface area treated is a very i mportant factor and represents the combined effect of total treatment area and the size of the eye. L8 The following statements are valid conclusions of the Diabetes Control and Co mplications Trial (DCCT). a. Among patients with type 1 diabetes with no retinopathy, intensive treatment can lower the incidence of progressive retinopathy by a factor of five compared to conventional treatment. b. The early worsening seen in patients initiating intensive control has no long -term effect on the severity of retinopathy. c. The study only tested patients with type 1 diabetes and the conclusions reach ed may not necessarily apply to patients with type 2 diabetes. d. The benefit of intensive control is not seen for the first 3 to 5 years of tr eatment. e. The early worsening seen in the Diabetes Control and Complications Trial (DCC T) was temporary and had no long-term implications regarding the progression of retinopathy. Patients who had early worsening were more likely to recover, and actually improve, than patients in the conventional treatment group. f. The benefit of intensive control was seen in all subgroups of patients strati fied by severity of retinopathy at the study's outset. g. The benefit of intensive treatment took up to 3 years to be realized for pati ents with proliferative retinopathy and 5 years for patients with macular edema. Apparently, diabetic retinopathy bu ilds momentum that requires years of intensive control to arrest. L9 The favorable clinical prognostic features for visual stabilization following laser treatment of diabetic macular edema is focal leakage and thickening. L10 It is not accurate to conclude that laser treatment is likely to improve vis ion in diabetic macular edema. L11 The Diabetic Retinopathy Study (DRS) showed that panretinal photocoagulation could reduce the incidence of severe visual loss in certain patients by >50%. A 60% reduction in progression t o severe visual loss in patients with socalled high-risk characteristics was reported. L12 Disc neovascularization (NVD) greater than one fourth to one third of a disc area is considered high risk, whether or not it is associated with vitreous hemorrhage. This is in distinction to neovasc ularization elsewhere (NVE), which must be associated with bleeding to qualify as high risk. Furthermore, to meet the Di abetic Retinopathy Study (DRS) criteria for high-risk disease, NVE also must comprise at least half the disc area. Any N VD associated with hemorrhage is high risk. L13 Adverse effects of PRP include: a. Decreased night vision results from destruction of extramacular rods. b. Angle-closure glaucoma may occur after particularly heavy treatment associate d with choroidal effusions. c. Regression of neovascular fronds may be associated with contracture and secon dary rhegmatogenous

retinal detachment (RRD) or traction retinal detachment (TRD). d. Laser can cause temporary or permanent dysfunction of the long ciliary nerves passing through the outer choroid and cause difficulties with corneal sensation and accommodation. The lat ter probably occurs far more often than suspected. e. PRP also can aggravate macular edema, further compromising near acuity. L14 With sickle cell (SC) disease, obstruction of the central macular blood supp ly may occur due to thrombosis of a cilioretinal or macular branch artery. This complication is very unusual in othe r forms of sickle hemoglobinopathies. Conversely, neovascularization (NV) is a relatively uncommon feature of SC retin opathy. If choroidal neovascularization (CNV) occurs, it is usually related to angioid streaks. As in diabetes, contract ure of neovascular fronds may lead to retinal detachment (RD). L15 Although rare, cases of retinopathy have been reported in association with s ickle cell trait (Hb AS) only. In addition, sickle cell SC disease can be associated with angioid streaks and comma-shaped c onjunctival capillaries. The incidence of proliferative retinopathy is as follows: sickle cell SC disease (Hb SC), 33%; sickle cell thalassemia (Hb SThal), 14%; sickle cell SS disease (Hb SS), 3%. Preretinal hemorrhage may rarel y be seen in nonproliferative disease, but it typically heralds neovascularization (NV). L16 A number of systemic disorders are associated with angioid streaks, includin g pseudoxanthoma elasticum ( P ), Ehlers-Danlos syndrome ( E ), Paget's disease of bone ( P ), sickle cell (SC) disease an d SC trait ( S ), and idiopathic ( I ) causes remember these by the acronym PEPSI. Approximately 50% of cases of angioid streaks are idiopathic. Photodynamic therapy may be of some benefit in the treatment of choroidal neovas cularization (CNV) secondary to angioid streaks. L17 Modalities useful in the treatment of proliferative sickle cell (SC) retinop athy include: a. Pars plana vitrectomy with endolaser. b. Scatter photocoagulation in the region of a neovascular frond. c. Feeder-vessel photocoagulation. d. Photocoagulation to close the feeder vessels of neovascular fronds can cause regression of the neovascularization (NV) but is associated with a relatively high rate of complic ations (hemorrhage). Scatter

photocoagulation to the region involved with the NV can effectively control it, with fewer complications. With dense vitreous hemorrhage, vitrectomy may be necessary to permit laser treatment . L18 Ranibizumab (Lucentis) is a potent antiangiogenic agent administered through intravitreal injection and is derived from bevacizumab (Avastin), a monoclonal antibody to VEGF. Phase III data from G enentech's MA-RINA study demonstrated visual acuity improvement in approximately one third of patients. I t is derived from a monoclonal antibody to vascular endothelial growth factor (VEGF). L19 The development of retinopathy of prematurity (ROP). a. It is hypothesized that immature vascular precursor tissue is susceptible to oxygen-induced cytotoxicity. b. Environmental oxygen may be sufficient to induce retinopathy of prematurity ( ROP) in some cases. L20 Several retrospective studies have suggested that laser therapy is at least as effective as cryotherapy, with fewer complications and less stress on the child. Laser treatment causes significantly less soft-tissue inflammation than cryotherapy. L21 Complications of branch retinal vein occlusion (BRVO) can be divided into ac ute and chronic categories. Macular edema, macular nonperfusion, and hemorrhage may occur acutely. Macular edema, su bretinal fibrosis, and posterior neovascularization (NV) can be delayed causes of visual loss following BRVO. Rub eosis iridis occurs in <1% of cases with BRVO. L22 The Branch Vein Occlusion Study (BVOS) documented the recovery of a final vi sual acuity of 20/40 or better in nearly twice as many patients treated with argon macular grid laser (compared wi th those who were untreated). The Branch Vein Occlusion Study (BVOS) showed that eyes treated for macular edema wi th photocoagulation had a 60% chance of recovering 20/40 or better vision, compared with 34% for controls. L23 Other conclusions of the Branch Vein Occlusion Study (BVOS) include: a. Quadrantic scatter photocoagulation reduces the risk of vitreous hemorrhage i n the eyes with established neovascularization (NV). b. Quadrantic scatter photocoagulation reduces the risk of developing NV if the area of retina affected by the vein occlusion is at least 5 disc areas in size. c. Large areas of nonperfusion were a significant risk factor for the developmen t of NV. d. Treatment is generally not undertaken until signs of neovascularization (NV) are evident, because the longterm incidence of severe visual loss in patients with extensive nonperfusion is low (only a subset of these actually develop NV). This risk increases once NV occurs. Greater than 5 disc di ameters of ischemia was found to be associated with a 31% risk of developing NV. L24 According to one study, approximately 48% of nonischemic central retinal vei n occlusion (CRVO) will resolve completely without treatment. L25 The most common systemic disease associations for central retinal vein occlu sion (CRVO) include atherosclerotic heart disease, hypertension, and diabetes. Less common associations include para proteinemias, syphilis, sarcoidosis, increased intraorbital pressure, and hypersensitivity and other vasculitides.

L26 Extremely high intraocular pressure (IOP) may occlude a healthy central reti nal vein. In central retinal vein occlusion (CRVO), the vitreous swells as transudate and seeps out of the congested retinal veins. This may cause narrowing of the angle and precipitate angle closure. L27 Important markers of ischemic central retinal vein occlusion (CRVO), compare d to nonischemic CRVO, include all of the following: a. Numerous cotton-wool spots. b. Extreme venous dilation. c. Extensive nonperfusion on fluorescein angiography (FA). d. b-wave/a-wave ratio of <1.0 on dark-adapted electroretinography (ERG). e. Any central retinal vein occlusion (CRVO) (ischemic and non-ischemic) will ha ve hemorrhage in all quadrants, although asymmetrically. L28 The retinopathy of carotid artery occlusive disease may feature retinal hemo rrhages and venous dilation such as central retinal vein occlusion (CRVO), but the central retinal artery pressure i s low. In CRVO, the retinal arteriolar pressure should be normal or elevated. Cotton-wool spots should not be seen in s imple carotid occlusion. L29 Venous stasis retinopathy (carotid occlusive retinopathy) and central retina l vein occlusion (CRVO) routinely share the following features: a. Venous dilation. b. Midperipheral blot retinal hemorrhages. c. Cotton-wool spots should not be seen in simple carotid occlusion. L30 The visual prognosis for ischemic central retinal vein occlusion (CRVO) is p oor, with only 10% of affected eyes obtaining better than 20/400 vision. Central retinal arterial occlusion (CRAO) h as a better visual prognosis than ischemic CRVO, although it is still devastating. In one study, 66% of eyes with CRAO had final vision of <20/400, whereas 18% of eyes with CRVO recovered vision of at least 20/40. L31 Type I is probably a forme fruste of Coats' disease, but types II and III ar e bilateral and occur in both sexes. Microscopically, the structural abnormalities are similar to diabetic microangio pathy, rather than a true telangiectasis. Unlike diabetes, there is no stimulus for retinal neovascularization (NV). Howev er, exudation in type I may respond to laser photocoagulation. Choroidal neovascularization (CNV) is another cause of v isual loss.

L32 Eales' disease is an idiopathic retinal vasculitis in young boys or men (mos t commonly from India) and is generally bilateral. The original syndrome was defined as retinal vasculitis in a young ma n with associated epistaxis, constipation, and positive reaction to dermal purified protein derivative. A potentially letha l cerebral vasculitis also has been recognized as an occasional finding. Neovascularization (NV) can be treated with scatter photocoagulation and visual prognosis is generally good with prompt appropriate therapy. L33 Retinal vasculitis associated with the following conditions. a. Systemic lupus erythematosus (SLE). b. Multiple sclerosis (MS). c. Polyarteritis nodosa. d. Intravenous drug abuse. e. Additional causes of retinal vasculitis include Behçet's disease, temporal arte ritis, Wegener's granulomatosis, sarcoidosis, herpes zoster, syphilis, and toxoplasmosis. L34 Potential causes of visual loss in macroaneurysm include subinternal limitin g membrane hemorrhage, vitreous hemorrhage, subretinal hemorrhage, and macular edema. Photocoagulation around th e macroaneurysm may reduce edema and improve acuity. Direct focal treatment of the aneurysm is not recommen ded because hemorrhage is likely. L35 Intraocular lens (IOL) implantation at the time of extracapsular cataract ex traction does not affect the incidence of cystoid macular edema (CME). L36 Retinal telangiectasia (Coats' disease, Leber's miliary aneurysms) is define d by the presence of an exudative retinal detachment (RD) with associated vascular anomalies. This condition is no t hereditary and is not associated with systemic vascular abnormalities. Usually, only one eye is involved, and men are predominantly affected (85%). L37 An ophthalmologist treating a retinal hemangioma may expect temporary worsen ing of exudation following successful treatment. In fact, treatment may be associated with a total exudativ e detachment (which typically resolves). L38 Both congenital retinal arteriovenous malformations and retinal capillary he mangiomas (von Hippel-Lindau disease) are associated with subretinal fluid and exudate. Cavernous hemangiomas bleed bu t usually do not leak. L39 The retinal pigment epithelium (RPE) continually ingests membranes shed by t he outer segments of the photoreceptor cells. RPE phagosomes containing ingested outer segment debris are discharged into Bruch's membrane after being processed. This is seen most prominently in the macula, the most metabolically active portion of the retina. L40 Hypertrophy of the retinal pigment epithelium (RPE) generally leads to a fla t, jet-black subretinal lesion, whereas hyperplasia most frequently leads to intraretinal bone spicule pigment depositio n. Metaplasia of the retinal pigment epithelium (RPE) can result in both preretinal and subretinal membranes. L41 The following statements about the characteristics of hyperfluorescence patt erns on fluorescein angiography (FA). a. Staining generally refers to the uptake of fluorescein by solid collagenous t issue. b. Transmitted fluorescence, or a window defect, generally implies a focal defec t in the retinal pigment epithelium (RPE).

c. Pooling implies collections of fluorescein within fluid-filled spaces. d. Leakage appears as an area of early hyperfluorescence that gradually increase s in size and intensity throughout the angiogram. L42 The following features are considered necessary for the development of retin al neovascularization (NV). a. Production of a local angiogenic factor. b. Some source of endothelial cells capable of replication and migration. c. A structural scaffold for vascular growth, presumably posterior cortical vitr eous. d. Focal defect in the internal limiting membrane is not necessary for retinal n eovascularization (NV) to progress. L43 Central serous retinopathy (CSR) presents in men most commonly in their 30s. Women tend to present with CSR later, in their 40s. The classic smokestack pattern is uncommon (10% of cases). It is much more common to see a small focal hyperfluorescent leak from the retinal pigment epithelium (RPE) that appears early in the angiogram and increases in size and intensity with time (inkblot). Laser treatment shortens th e course of each attack, but final visual acuity and rate of recurrence are not affected. L44 Eighty percent to 90% of the cases of central serous retinopathy (CSR) will spontaneously resolve, but 40% to 50% will recur. L45 The types of functional visual prostheses have been successfully implanted i n humans. a. Optic nerve stimulators. b. Epiretinal implants. c. Passive subretinal implants. d. Occipital cortex implants. L46 The diagnosis of age-related macular degeneration (AMD) is made when eyes wi th drusen or their associated complications develop a decrease in visual acuity not attributable to other ocul ar conditions. L47 Possible risk factors for progressive visual loss in age-related macular deg eneration (AMD) include a positive family history, hyperopia, cigarette smoking, and light iris color. Hypertension has no t been directly related (hypertension is a risk factors for AMD in Kanski and Yanoff). L48 The following statements about the classification of age-related macular deg eneration (AMD).

a. Dry AMD accounts for 90% of all patients affected by this disorder. b. Wet AMD accounts for 90% of all patients with severe visual loss (worse than 20 /200) who are affected by this disorder. c. Patients with either pigment epithelial detachment or choroidal neovasculariz ation (CNV) should be considered as part of those affected by the wet variety of AMD. d. Findings in the nonexudative form of age-related macular degeneration (AMD) i nclude pigmentary changes, drusen, and areas of geographic atrophy. Patients with central geographic atroph y generally have a guarded visual prognosis. L49 Pigment epithelial detachment in a patient younger than 50 years generally h as a better prognosis than in a patient older than 50 years. A pigment epithelial detachment in a patient younger than 5 0 years is generally thought to be a variant of central serous retinopathy (CSR) and has a better prognosis. Visual a cuity often returns to 20/30. L50 Argon laser photocoagulation for exudative age-related macular degeneration (AMD) has been shown to be effective in reducing the rate of severe visual loss or preserving visual function among w hich categories of the disease. a. Extrafoveal (>200 microns from the center of the foveal avascular zone) choro idal neovascularization (CNV). b. Juxtafoveal CNV (1 to 199 microns from the center of the foveal avascular zon e). c. Subfoveal CNV. d. The Macular Photocoagulation Study. i. Documented that argon laser photocoagulation is effective in reducing the rat e of severe visual loss for extrafoveal choroidal neovascularization (CNV) (>200 microns from the center of the foveal avascular zone) and juxtafoveal CNV (1 to 199 microns). ii. Contrast sensitivity was better preserved over both the short term and the l ong term among treated patients, relative to untreated patients. This is in distinction to acuity and r eading speed, both of which were worse among the treated patients, over the short term only. iii. Subgroup analysis of outcomes by the Macular Photocoagulation Study evaluat ed outcome based on lesion size and presenting acuity. Age and gender were not analyzed. The ideal c andidate has poor acuity (<20/125 for small lesions and <20/200 for medium lesions) and a sma ll (<1 disc area) or a medium (1 to 2 disc areas) lesion. L51 The following factor(s) support(s) an etiologic connection between ocular Hi stoplasma infection and the presumed ocular histoplasmosis syndrome (POHS). a. More than 90% of patients with POHS will have a positive histoplasmin skin re action. b. Histoplasma organisms have been recovered from the human choroid. c. Endemic areas include Ohio and the states of the Mississippi River valleys. d. Systemic antifungal treatment does not lead to resolution of the ocular findi ngs. L52 The best predictor of future contralateral visual loss in a patient with a d isciform macular scar from presumed ocular histoplasmosis syndrome (POHS) is the presence or absence of a focal macular sca r in the better eye. The likelihood of

contralateral choroidal neovascularization (CNV) in a patient with a disciform m acular scar from presumed ocular histoplasmosis syndrome (POHS) is increased if there are focal macular scars in the better eye. L53 Angioid streaks Angioid streaksrepresent discontinuities in abnormally thickened and calcified B ruch's membrane. They do not always extend in continuity from the optic nerve head and appear to radiate from the optic nerve head, rather than forming concentric circles around it. Because the overlying retinal pigment epit helium (RPE) is often atrophic, angioid streaks may appear as window defects on fluorescein angiography (FA). L54 Up to 50% of patients with angioid streaks have no identifiable systemic ill ness. L55 High myopia is not associated with angioid streaks but may feature lacquer c racks, which are similar histopathologically. L56 Idiopathic epiretinal membrane is 20% bilateral. Approximately 75% of eyes w ith idiopathic epiretinal membranes maintain a visual acuity of 20/50 or better. L57 The first study of surgery for macular holes evaluated patients with the ear liest stage, stage 1. The natural history of the disorder, with up to 50% enjoying spontaneous improvement, is at least as fa vorable as, if not superior to, surgical intervention. L58 The prevalence of bilaterality of idiopathic macular holes, including earlie r stages before a full-thickness dehiscence develops, will be higher, approaching 20% to 25%. Because a large proportion of early holes spontaneously resolve, only a small fraction (5%) develop bilateral full-thickness defects. L59 The lesion that is felt to be the immediate precursor to a full-thickness ma cular hole is a sensory retinal detachment (RD) involving the fovea. Idiopathic macular holes are believed to ar ise because of both tangential and anteroposterior vitreous traction. The earliest change is loss of the normal fov eal depression due to elevation of the fovea itself off the retinal pigment epithelium (RPE), thereby constituting a ti ny sensory retinal detachment (RD). This appears clinically as a yellow dot or ring (stages IA and IB, respectively). L60 Although hydroxychloroquine is thought to be less toxic, both hydroxychloroq uine and chloroquine have a significant risk of retinal toxicity. Color vision testing and threshold central visual fiel d testing are thought to be important in evaluation subclinical retinopathy. The electrooculogram (EOG) is not the most s ensitive parameter in detecting

chloroquine retinopathy, although it does reflect pathophysiology of retinal pig ment epithelium (RPE) damage when abnormal. L61 Because of their slow excretion, toxic effects of chloroquine and hydroxychl oroquine may progress despite cessation of the drug. Any abnormalities caused by these medications are probably permanen t, although mild deficits may be reversible. L62 Both the phenothiazines and the antimalarials (chloroquine and hydroxychloro quine) may cause electroretinographic abnormalities and peripheral pigmentary retinopathy. Thioridazine is probably th e most likely phenothiazine to cause retinal toxicity. L63 Both thioridazine and chlorpromazine may lead to abnormal pigment deposition in eyelids, cornea, lens, and retina. L64 Type II (Hunter's) is indeed X-linked recessive. However, the rest of the mu copolysaccharidoses (MPS) are autosomal recessive (not autosomal dominant). L65 Corneal clouding is common in mucopolysaccharidoses (MPS) types I-H (Hurler' s), I-S (Scheie's), IV (Morquio's), and VI (Maroteaux-Lamy). It may rarely be seen in type II (Hunter's) but is neve r seen in type III (Sanfilippo's). L66 Retinal pigmentary degeneration resembling typical retinitis pigmentosa (RP) is seen in types I-H (Hurler's), I-S (Scheie's), II (Hunter's), and III (Sanfilippo's). Not seen in Type IV (Morquio' s syndrome). L67 Optic atrophy may be seen in which of the following mucopolysaccharidoses (M PS). a. Type II (Hunter's syndrome). b. Type III (Sanfilippo's syndrome). c. Type IV (Morquio's syndrome). d. Not seen in type VII (Sly's syndrome). e. Optic atrophy can be found in all of the types in the Macular Photocoagulatio n Study except type VI (mild phenotype) and type VII. L68 Gangliosides are sphingolipids found only in the central nervous system (CNS ), whereas cerebrosides are sphingolipids found throughout the body. Gangliosides are found in cerebral gray matter, whereas cerebrosides are found in cell membranes throughout the body. L69 In the sphingolipidoses, a cherry-red spot is caused by accumulation of sphi ngolipids in the retinal ganglion cell layer. The orange red color of the retinal pigment epithelium (RPE) and choroid stands out in the central fovea, where the ganglion cell layer is very thin or absent. There is no gross accumulation o f lipid in Fabry's or Krabbe's disease; therefore, no cherry-red spot is present. L70 Ocular manifestations of Fabry's disease include: a. Cornea verticillata. b. Spoke-like posterior subcapsular cataract. c. Conjunctival and retinal telangiectases. d. Pigmentary retinal changes are not characteristic of Fabry's disease. L71 Deposition of cystine crystals in the conjunctiva and cornea with resultant photophobia occurs in all three types of cystinosis (nephropathic, late onset, and benign). A pigmentary retinopathy with a salt-and-pepper appearance occurs only in the nephropathic type. However, no significant visual disturbance occurs . In addition to its benefits as a topical medication, systemic cysteamine can also be used to help prevent renal failure.

Cysteamine, administered topically, may reduce the corneal crystal load, and associated photophobia. L72 The following are statements about solar retinopathy. a. It is generally associated with sun gazing and, less commonly, arc welding. b. The lesion is a photochemical and photothermal insult to the retinal pigment epithelium (RPE). c. Visual acuity in solar retinopathy is generally not reduced below 20/200, and is frequently only minimally reduced. Recovery is good. d. Most patients recover near normal acuity over several months. e. The lesion appears as a small yellowish-white spot in the center of the fovea , which fades over time, often leaving permanent focal RPE changes. L73 The ratio of rods to cones ranges between 12:1 and 15:1. Some investigators claim a ratio as high as 20:1. Rod density is maximal in a ring that is 20 to 40 degrees eccentric to the fovea, wh ereas cone density is greatest in the fovea. Although the cone density is greater in the macula than in the peripheral retina, the number of rods and cones in the macula is roughly equal, and nearly half (about 40%) of all cones lie outsid e the macula. L74 Giant retinal tear is a circumferential retinal break of 90 degrees or great er (3 clock hours or more). L75 The scotopic electroretinogram (ERG) represents the ERG with rod input only. It is generated with a dim white or blue flash below the cone threshold in a dark-adapted state. L76 Some rod function is represented even in a light-adapted electroretinogram ( ERG) especially when stimulated with a blue flash of light. Rod input can be minimized or eliminated by using only the longest wavelengths (orange-red), or more practically, by using a flicker stimulus. L77 Rods can respond to flickering stimuli of rates up to 20 Hz. Only cones can respond at higher frequencies, with a maximum of approximately 70 Hz. L78 Although the b-wave amplitude can be decreased in focal or stationary retina l disease, the b-wave implicit time is increased only in diffuse, progressive retinal disease.

L79 With increasing stimulus intensity, the b-wave amplitude increases and the i mplicit time (the time from stimulus to bwave peak) decreases. L80 Posterior vitreous detachments are generally absent in eyes with retinal dia lyses. The converse is true in eyes with giant retinal tears. L81 The following statements about ocular antiangiogenesis. a. Ocular antiangiogenesis therapy may be beneficial in the treatment of age-rel ated macular degeneration (AMD) and diabetic retinopathy. b. Pegaptanib sodium () is an mRNA aptamer (not a monoclonal antibody) that has been approved Macugen Macugenby the U.S. Food and Drug Administration (FDA) for treatment of certain f orms of wet AMD. c. Ranibizumab () is a monoclonal antibody fragment that binds to vascular endot helial growth factor Lucentis Lucentis(VEGF), preventing VEGF from binding to VEGF receptors. d. Both Ranibizumab and Pegaptanib sodium are administered by intravitreal injec tion. e. VEGF is a potent cell mitogen that induces angiogenesis and increases vascula r permeability, and it is elevated in various retinal disorders involving hypoxic conditions and ischemia. L82 In electrooculogram (EOG), amplitudes generally increase with light adaptati on and diminish with dark adaptation. The EOG is considered abnormal if the light-peak to dark-trough ratio (the Arden ratio) is <1.75. L83 An abnormally low light-peak to dark-trough ratio on electrooculogram (EOG) has been found to occur in retinal toxicity from hydroxychloroquine and chloroquine. L84 Rods are approximately 1,000 times more sensitive than cones during dark ada ptation. L85 Anaphylaxis, although extremely rare, is a potential complication of intravi treal injection. L86 The Purkinje shift refers to a shift in peak spectral sensitivity, from 555 nm to 505 nm, with dark adaptation. Dark adaptation does increase light sensitivity by a factor of 1,000, but this is not the Purkinje shift. L87 Refsum's disease. a. Characterized by ataxia, pigmentary retinopathy, deafness, and cardiac myopat hy. b. Both infantile and adult forms exist. c. Night blindness can be an early symptom in patients. d. Phytanic acid levels are typically elevated. e. Dietary restriction may slow down the disease. L88 Deuteranomalous trichromats are the most common. They are X-linked recessive and found in 5% of the male population. L89 Rod monochromats typically have nystagmus and acuities in the 20/200 range. Blue cone monochromats have variable nystagmus and acuities in the 20/40 to 20/200 range. L90 The following help to distinguish between rod monochromats and blue cone mon ochromats. a. Specific vision color testing. b. Careful family history. Rod monochromatism is an autosomal-recessive disorder , whereas blue cone monochromatism is X-linked recessive. c. Decreased acuity and nystagmus are seen in both disorders. The electroretinog rams (ERGs) in both

disorders are similar in that they show cone dysfunction with relatively normal rod function. L91 The following color vision tests detect both red-green and blue-yellow defec ts. a. Hardy-Rand-Rittler plates. b. Farnsworth-Munsell 100 test. c. Farnsworth panel D-15 test. L92 Ishihara plates detect only red-green defects. L93 Up to one third of patients with intracranial hemorrhage will have intraocul ar hemorrhage (the presence of both simultaneously is called Terson's syndrome ). L94 AAO. Terson syndrome is recognized as vitreous and sub-ILM or subhyaloid hem orrhage caused by an abrupt intracranial hemorrhage. Although the exact mechanism is not known, it is suspec ted that the acute intracranial hemorrhage causes an acute rise in the intraocular venous pressure, resulting in a rupture of peripapillary and retinal vessels. Approximately one third of patients with subarachnoid or subdural hemor rhage have associated intraocular hemorrhage, which may include intraretinal and subretinal bleeding. Terson syndr ome occurs mostly in individuals between 30 and 50 years old, but it can occur at any age. In most cases, visual function is unaffected once the hemorrhage clears. Spontaneous improvement generally occurs, although vitrectomy is occasionally required to clear the ocular media. L95 The following conditions are generally associated with Purtscher's or Purtsc her like retinopathy. a. Systemic lupus erythematosus (SLE). b. Thrombotic thrombocytopenic purpura (TTP). c. Long-bone fractures. d. Chronic renal failure. e. Hepatic encephalopathy is generally not associated with Purtscher's. L96 The vast majority of severe hearing loss associated with retinitis pigmentos a (RP) is congenital.

L97 Usher's syndrome only refers to the association of pigmentary retinopathy an d congenital deafness. Types I and III feature vestibular dysfunction. Patients with profound deafness are more likely to have cerebellar atrophy. L98 Preservation of central acuity past the age of 45 in a patient with a retina l degeneration and an X-linked inheritance pattern suggests the diagnosis of choroideremia. With choroideremia, central acu ity is spared until later in life relative to X-linked retinitis pigmentosa (RP). The retinal findings in choroideremia includ e scalloped retinal pigment epithelium (RPE) atrophy with no or scanty hyperpigmentation. L99 Gyrate atrophy is an autosomal-recessive deficiency in ornithine aminotransf erase activity. This deficiency causes an increase in serum ornithine levels and a decrease in serum lysine levels. Life s pan is normal in this disorder. L100 The following statements about fundus flavimaculatus. a. The pisciform lesions seen in the posterior fundus represent lipofuscin depos its within hypertrophied retinal pigment epithelium (RPE) cells.QT29KQH4 b. The most common mode of inheritance is autosomal recessive. c. Visual acuity loss may be mild or severe depending on the extent of macular i nvolvement. d. The degree of electroretinographic (ERG) abnormality parallels the amount of fundus involvement. L101 The following statements about . familial drusen (Doyne's honeycomb dystrop hy) familial drusen (Doyne's honeycomb dystrophy) a. Familial drusen are an autosomal-dominant disorder that begins with asymptoma tic retinal changes in the third decade of life and does not become symptomatic until the fourth or fifth d ecade of life. b. The electroretinogram (ERG) is usually normal or mildly decreased. c. There are both atrophic (dry) and exudative (wet) forms of the disease. L102 (Normalization of scotopic ERG after 4 to 8 hours of dark adaptation) This scotopic electroretinographic (ERG) pattern helps to distinguish fundus albipunctatus from retinitis punctata albesc ens and other forms of congenital stationary night blindness. Normalization of the scotopic ERG after dark adaptat ion has been shown in two pedigrees. L103 In Best's disease, the macular appearance is considerably worse than the vi sual acuity. Vision may be 20/30 to 20/60 in the early stages of the disease, despite the prominent central egg yolk l esion. On the contrary, patients with Stargardt's disease may have poor vision with barely detectable macular changes. L104 The Mizuo-Nakamura effect occurs in Oguchi's disease and X-linked cone dyst rophy. The eponym refers to a relative lightening of the retinal pigment epithelium (RPE) and fundus after 3 t o 4 hours of dark adaptation. Mizuo phenomenon, there is an accompanying change in fundus colour from golden-brown i n the light-adapted state to a normal colour in the dark-adapted state. L105 Symptoms of cone degeneration include the following: a. Loss of visual acuity. b. Photophobia. c. Progressive dyschromatopsia. d. Difficulty driving at night. Although difficulty in driving at night would se em to be more likely with rod degenerations, urban night driving is generally performed with background illumi

nation at low photopic intensities. Therefore, patients with cone degeneration may complain of this. L106 The differential diagnosis of peripheral pigmentary retinopathy includes: a. Syphilitic chorioretinitis. b. Prior exudative retinal detachment (RD). c. Phenothiazine toxicity. d. Ophthalmic artery occlusion. e. Harada's disease and toxemia of pregnancy are exudative disorders that may re solve and lead to pseudoretinitis pigmentosa (RP). L107 Foveal hypoplasia may be associated with the following disorders: a. Albinism. b. Aniridia. c. Persistent hyperplastic primary vitreous (PHPV) also has been associated with foveal hypoplasia. L108 The following statements about Stargardt's disease. a. Stargardt's disease is generally inherited on an autosomal-recessive basis, b ut dominant pedigrees also have been described. b. It may be associated with peripheral flecks indistinguishable from fundus fla vimaculatus. c. In its early stages, the maculopathy has a beaten metal appearance, similar t o cone-rod dystrophy. d. In fundus flavimaculatus, the fluorescein angiogram (FA) may reveal a dark cho roid. e. The terminal stages of the maculopathy resemble central areolar choroidal dys trophy. L109 The following about Best's vitelliform dystrophy. a. The general mode of inheritance is autosomal dominant. b. Histopathology reveals abnormal accumulation of lipofuscin within the retinal pigment epithelium (RPE) and choroid. c. Despite the striking fundus appearance of the egg yolk lesion, visual acuity is usually quite good (20/30 to 20/50) at this stage. d. The amount of lipofuscin cannot be correlated with the amount of electrooculo gram (EOG) depression.

L110 The adult type of vitelliform macular dystrophy features egg yolk lesions tha t is generally smaller than those of classic Best's disease. L111 The following statements about pattern dystrophies of the retinal pigment e pithelium (RPE). a. Visual acuity is generally well preserved until late in life. b. The electroretinogram (ERG) is generally normal. c. Pattern dystrophies of the retinal pigment epithelium (RPE) are better seen o n fluorescein angiography (FA) than with ophthalmoscopy. d. As in Best's disease, the electrooculogram (EOG) is usually abnormal despite a normal electroretinogram (ERG). L112 Albinoidism differs from albinism in that the former does not have severe v isual consequences. Both share certain clinical features, including photophobia, iris transillumination defects, and fu ndus hypopigmentation. L113 In general, albinoidism is inherited as an autosomal-dominant trait with in complete penetrance, whereas true albinism is inherited as either an autosomal-recessive or X-linked-recessive tra it. L114 Nettleship-Falls ocular albinism is inherited in an X-linked-recessive patt ern. L115 Xenon arc emits polychromatic white light (unfocused white light emission) that cannot be as precisely focused as monochromatic light. In addition, blue light (harmful to the retina and lens) em ission is significant. Other disadvantages include more pain and a greater rate of breaks in Bruch's membrane (leading to c horoidal effusions and/or choroidal neovascularization [CNV]) after xenon prethreshold retinopathy of prematurity (R OP) compared to argon laser photocoagulation. L116 Laser that is least absorbed by hemoglobin or red blood cells is krypton re d. Krypton red is useful for laser therapy in the presence of vitreous hemorrhage. In addition, because it is poorly absorbed by xanthophyll, it is useful for macular laser treatment. Red light penetrates cataract and vitreous hemorrhage better th an lights of other wavelengths. L117 Decreasing the spot size of an argon laser burn increases the energy delive red per unit area. This is in contrast to the xenon arc, in which decreasing the spot size decreases the total energy deli vered. L118 Points of firm vitreoretinal attachment include: a. The vitreous base. b. The edge of retinal scars. c. The edge of lattice retinal degeneration. d. Other areas of firm vitreoretinal attachment include the optic nerve and majo r blood vessels. The vortex veins represent a firm point of attachment between the choroid and sclera. L119 The proportion of patients with an acute symptomatic posterior vitreous det achment have a retinal tear is 10%. L120 The proportion of patients with acute symptomatic posterior vitreous detach ment and associated vitreous hemorrhage have a retinal tear is 75%. Hemorrhage or vitreous cells ( tobacco dust ) are suggestive of retinal breaks. L121 Spontaneous and familial dialyses occurs inferotemporally. Traumatic dialys es also may occurs superonasally; this location is probably reflective of contrecoup injury to the ora from blunt force s delivered inferotemporally, where the

globe is most exposed. L122 The prevalence of lattice retinal degeneration in the adult population is a pproximately 8% to 10%. The prevalence of lattice retinal degeneration in patients with rhegmatogenous retinal detachment (RRD) is approximately range from 20% to 40%. L123 Lattice degeneration and meridional complexes (redundant retinal folds in t he same meridian as a ciliary process, usually occurring superonasally) increase the risk of rhegmatogenous retinal det achment (RRD). Cobblestone degeneration has occasionally been observed to limit the spread of retinal detac hment (RD) and does not predispose to RRD. It also may be the site of secondary retinal breaks as the advancing subret inal fluid reaches the edge of a cobblestone, where chorioretinal adherence is enhanced. L124 Approximately 22% of patients older than 20 years have cobblestone degenera tion (RD), usually in the inferior quadrants. L125 In 97% of cases of rhegmatogenous retinal detachment (RRD), a definite brea k can be found. In the other 3%, one break is presumed to be present. This most frequently occurs in the setting of a phakic or pseudophakic retinal detachment, in which the breaks are frequently tiny. L126 Signs suggestive of rhegmatogenous retinal detachment (RRD) include (i) low er intraocular pressure (IOP) compared with the other eye, (ii) tobacco dust (small clumps of retinal pigment ep ithelium [RPE] cells floating in the vitreous), and (iii) corrugated appearance of the retina that undulates with eye movements. A smooth, domed appearance and shifting fluid are more suggestive of exudative detachment. L127 Proliferative vitreoretinopathy (PVR) is the most common cause of redetachm ent after successful repair. Retinal pigment epithelium (RPE) and glial cells proliferate and subsequently contract, causing fixed folds, traction, and/or detachment. L128 Approximately 100% reattachment can be achieved with rhegmatogenous retinal detachment (RRD) secondary to dialysis or small round holes, detachments with demarcation lines (chronic), or those with minimal subretinal fluid. Giant tears and combined retinal-choroidal detachments have a poorer prognosis, wherea s aphakic detachments have intermediate prognosis. L129 The key prognostic factor in predicting postoperative visual acuity followi ng surgical repair of rhegmatogenous retinal detachment (RRD) is the presence and duration of macular detachment. Degeneratio n of photoreceptors limits recovery

of vision. Seventy-five percent of patients with macular detachment for a durati on <1 week will recover vision >20/70. In contrast, those patients with macular detachment for duration of 8 days or more have only a 50% chance of regaining acuity of at least 20/70. L130 In virtually all cases of typical degenerative retinoschisis, peripheral cy stoid degeneration can be found, usually adjacent to the schisis cavity. L131 Greater than 70% of patients with retinoschisis are hyperopic, and this con dition is bilateral in 50% to 80% of cases. However, in typical retinoschisis, complications such as hole formation and reti nal detachment (RD) are rare. With retinoschisis posterior to the equator, the scotoma, although absolute and readi ly identifiable on perimetry, is rarely noted by the patient. L132 Demarcation lines with retinoschisis indicate current or previous full-thic kness retinal detachment (RD). L133 A hole in the outer wall alone is sufficient for formation of RD. Indeed, h oles in both inner and outer walls may lead the cavity to collapse, making the origin of detachment difficult to locate. L134 The surgical treatment of proliferative vitreoretinopathy (PVR). a. Silicone oil and perfluoropropane (C3F8) gas are equally effective tamponade agents. b. When silicone oil is used as tamponade in PVR, there is no significant differ ence in outcomes between eyes undergoing primary vitrectomy and previously vitrectomized eyes. c. When C3F8 is used as tamponade in PVR, there is no significant difference in outcomes between eyes undergoing primary vitrectomy and previously vitrectomized eyes. d. Postoperative macular pucker is equally common following tamponade with gas o r oil. e. The Silicone Oil Study Group found that reattachment rates and visual outcome s were both worse when using SF6 as the tamponade agent, compared with either perfluoropropane (C3F8) o r oil. L135 Persistent fetal vasculature or persistent hyperplastic primary vitreous (P HPV) should be included in the differential diagnosis of leukocoria and differentiated from retinoblastoma. PHPV is unilater al in >90% of cases and is associated with microphthalmia. Retinoblastoma is often bilateral and has no significant as sociations with microphthalmia or cataracts. L136 Classification of hereditary hyaloideoretinopathies into two categories has been suggested: those with ocular manifestations only (Wagner's disease) and those with systemic symptoms (Stickle r's syndrome). Wagner's disease consists of high myopia, posterior subcapsular cataract, and an optically empty vitreous. Stickler's syndrome is Wagner's disease plus increased incidence of (i) retinal detachment (RD), (ii) f acial anomalies, and (iii) musculoskeletal anomalies. L137 Familial exudative vitreoretinopathy is autosomal dominant and occurs in fu ll-term infants with normal respiratory status. L138 The following statements about asteroid hyalosis. a. It is more common with aging. b. Asteroid hyalosis is monocular in 75% of cases. c. It is generally associated with no decrease in visual acuity. d. The vitreous is otherwise normal.

e. The particulate matter seen clinically consists of calcium soaps. L139 Asteroid hyalosis usually does not settle, suggesting some collagenous supp ort of asteroid bodies within formed vitreous. Synchysis scintillans, the crystalline lipid breakdown products of pre vious hemorrhage, usually settle inferiorly in liquefied vitreous (or in an eye that has undergone vitrectomy). L140 Estimates of the proportion of vitreous hemorrhage caused by diabetic retin opathy have ranged from 39% to 54%. L141 Condition mandate(s) repair with pars plana vitrectomy rather than conventi onal scleral buckling +/- pneumatic retinopexy. a. Retinal detachment (RD) with accompanying vitreous hemorrhage following penet rating trauma. b. RD associated with marked proliferative vitreoretinopathy (PVR). c. Aphakic retinal detachment (RD) and retinal dialysis may be repaired with scl eral buckling +/- pneumatic retinopexy alone. However, traumatic RD with vitreous hemorrhage or with severe proliferative vitreoretinopathy (PVR) may require vitrectomy to remove vitreous scaffolds that can promote future neovascularization (NV) and contraction. L142 Advantages of early vitrectomy surgery in a patient who is diabetic and has vitreous hemorrhage include: a. An opportunity to remove the vitreous scaffold that fosters neovascularizatio n (NV). b. An opportunity to treat retinal ischemia intensively with endolaser. L143 Severe visual loss after vitrectomy seems more common in patients who are d iabetic. Lens removal may increase the risk of rubeosis, although a concomitant retinal detachment (RD) may be the key factor prompting lensectomy. L144 Indications for emergent pars plana vitrectomy surgery on an eye with an ac ute (i.e., unrepaired) rupture or laceration include: a. Vitreous hemorrhage and associated retinal detachment (RD). b. Intraocular foreign body. c. Traumatic endophthalmitis.

d. NB. Indications for emergent pars plana vitrectomy in the setting of globe pe netration include retinal detachment (RD), intraocular foreign body, and endophthalmitis. Vitreous hemorrh age without RD may be observed or addressed 7 to 10 days after the initial repair of globe rupture. L145 Traumatic macular hole and subfoveal hemorrhage may lead to a permanent dec rease in visual acuity. Commotio retinae and Valsalva retinopathy generally have a good visual prognosis. L146 Among men, each of the following constitutes a significant risk factor for the development of a central retinal vein occlusion (CRVO). a. Systemic hypertension. b. Diabetes mellitus. c. Open-angle glaucoma. d. Decreased physical activity. L147 According to the Eye Disease Case-Control Study, elevated erythrocyte sedim entation rate is a risk for central retinal vein occlusion (CRVO) only among women. L148 Moderate alcohol consumption reduces the risk of central retinal vein occlu sion (CRVO) for both women and men. Exogenous estrogen use also reduces risk. L149 Among patients with initially non-perfused (ischemic) central retinal vein occlusion (CRVO), the following is a risk factor for the development of at least 2 hours of iris or angle neovascularizati on (NV). a. >30 disc areas of non-perfusion. b. Male gender. c. Duration <1 month. d. Severe intraretinal hemorrhage. L150 The percentage of patients diagnosed with a non-ischemic central retinal ve in occlusion (CRVO) of <1 year's duration will develop iris or angle neovascularization (NV) within 4 months 5% t o 6%. L151 Findings of the Central Vein Occlusion Study (CVOS) include each of the fol lowing: a. Clinical and angiographic edema did respond to grid laser treatment. There wa s no significant effect on long-term visual acuity, so the treatment is no longer recommended. b. Grid pattern photocoagulation does not significantly alter visual outcome in patients with CME associated with CRVO. c. Roughly 25% to 30% of patients with at least 10 disc areas of nonperfusion wi ll develop iris or angle neovascularization (NV) within 3 years. d. Prophylactic scatter laser showed a trend toward reducing the incidence of ir is or angle NV, but baseline differences in the treatment versus no treatment groups eliminated any statistic al significance. e. Prophylactic scatter laser appears to reduce the efficacy of additional scatt er laser treatment of iris or angle NV that develops subsequently. L152 The approximate prevalence of idiopathic macular hole in a population of he althy Americans older than 55 years is 3 in 1,000. (Macular holes are rare conditions, estimated to occur in the vicinity of 1 in 5000 patients. Yanoff). L153 The approximate prevalence of age-related macular degeneration (AMD) in a p opulation of healthy Americans older than 55 years is 1 in 10,000. ([AAO] The risk of AMD increases with age. In the Framingham Eye Study, 6.4% of

patients aged 65-74 years old and 19.7% of patients older than 75 years had sign s of AMD. Other risk factors for AMD include positive family history, cigarette smoking, hyperopia, light iris color, hypertension, hypercholesterolemia, female gender, and cardiovascular disease). L154 The following statements about optical coherence tomography (OCT). a. Optical coherence tomography (OCT) only uses light waves to create images. b. Images created with OCT have much better resolution than standard 10-MHz B-sc an ultrasound. c. OCT image quality will decrease as the amount of vitreous hemorrhage increase s. d. Ultra-high resolution OCT can create images with resolutions of 2 to 3 micron s. e. OCT also provides quantitative information with regard to macular thickness a nd retinal nerve fiber layer thickness. f. Ultra-high resolution OCT uses a different laser to create images with much g reater resolution. L155 The single most important physical property of perfluoropropane (C3F8) gas rendering it useful as a vitreous substitute in vitreoretinal surgery is its high interfacial surface tension. The surface tension at a gas-aqueous interface is very high and responsible for the ability of a gas-fluid exchange to flatten a d etached retina. The interfacial tension prevents the aqueous phase from re-entering the subretinal space, unless the ret inal breaks are very large or under severe traction. These factors can outweigh the beneficial effects of surface te nsion. L156 The single most important physical property of silicone oil rendering it us eful as a vitreous substitute in vitreoretinal surgery is its: high viscosity. Interfacial surface tension between silicone oil and aqueous is very low. The high viscosity of oil makes it more difficult for the aqueous phase remaining in an eye to gain access to retinal breaks. The oil is also less likely to enter the subretinal space, once again because of its viscosity. Once aqueous reaches a break, however, the oil cannot prevent it from entering the subretinal space because there is no significant interfacial surface tension. Silicone oil also has lower solubility for various inflammatory and vasogenic me diators, which may beneficially compartmentalize the eye following complicated retinal surgery.

L157 The single most important physical property of perfluorocarbon liquids rend ering them useful as vitreous substitutes in vitreoretinal surgery is their high specific gravity. The high sp ecific gravity of perfluorocarbon liquids renders them heavier than water and makes them a revolutionary addition to the v itreoretinal armamentarium. They can steamroll giant retinal tears flat, act as third hands in complicated surgery, and e ffect drainage of subretinal fluid through breaks in the far periphery, obviating the need for drainage retinotomie s. They also have been used for repositioning dislocated lens implants and draining choroidal hemorrhage. As tim e passes, their utility is bound to increase. They are not tolerated by the retina for >24 to 48 hours (histopatholo gy shows compression of the retina). They are not tolerated by anterior chamber structures at all. Liquids with highe r vapor pressures (e.g., perfluoro-noctane) may be safer to use because they will readily evaporate in the gas-fille d eye. L158 Several case reports have documented significant deterioration in patients with central serous retinopathy (CSR) started or maintained on oral corticosteroids. The most common finding is dramat ically increased exudation with the development of bilateral bullous retinal detachments (RDs). L159 The only factor independently predictive of bilateral choroidal neovascular ization (CNV) is the presence of histo spots in the contralateral eye at the time of diagnosis of unilateral CNV. The a nnual incidence of development of choroidal neovascularization (CNV) in the fellow eyes of patients with ocular hi stoplasmosis syndrome and initially unilateral CNV is approximately: 2%. L160 ESR and CRP (when both are abnormal) have 97% specificity for AAION. L161 Macular epiretinal membrane. a. The chance of this condition being bilateral is approximately 15% to 20%. b. Most patients have visual acuities that are 20/50 or better, although they of ten complain of metamorphopsia. c. Intravitreal perfluoropropane (C3F8) will not help improve visual acuity. d. Instead, pars plana membranectomy is required to improve visual acuity. L162 Serpiginous chorioretinopathy. a. A bilateral condition that usually spreads outward from the optic nerve and/o r macula in a serpentine fashion. b. Serpiginous chorioretinopathy is chronic and recurrent with poor visual progn osis and scotomata affecting the areas of involvement. c. Treatment with potent immunosuppressives may slow down the disease in some ca ses, but generally the visual prognosis is poor, especially with macular involvement. L163 Asteroid hyalosis has a clear association of this condition in patients wit h diabetes. The prevalence of this condition is increased in patients with diabetes. It usually occurs in patients older than 50 years, and vitrectomy may sometimes be used to help visualize other retinal conditions (e.g., diabetic retinopathy), which are difficult to assess because of the asteroid hyalosis. L164 Posterior neovascularization (NV) is not common with Coats' disease. L165 The Diabetic Retinopathy Vitrectomy Study (DRVS) demonstrated the benefit o f early vitrectomy (within 1 to 6 months after vitreous hemorrhage) in patients with type 1 diabetes or those pati

ents with vitreous hemorrhage and severe proliferative diabetic retinopathy. L166 Fundus albipunctatus. It can be differentiated from retinitis punctata albe scens because of the normal vasculature (typically attenuated in retinitis punctata albescens, a retinitis pigmentosa [R P] variant). L167 Bietti crystalline corneoretinal dystrophy, which is characterized by limba l corneal opacities and tapetoretinal dystrophy. L168 The patient has Irvine-Gass syndrome (e.g., cystoid macular edema [CME] dev eloping after cataract surgery). The incidence of Irvine-Gass is higher after intraoperative complications, but most cases spontaneously resolve. Peak incidence is at 6-10 weeks postoperatively. L169 Further analyses supported retinal ablative therapy for eyes with type 1 RO P, defined as zone 1, any stage ROP with plus disease; zone 1, stage 3 ROP without plus disease; or zone II, sta ge 2 or 3 ROP with plus disease. L170 Analyses supported a wait-and-watch approach to type 2 ROP, defined as zone I, stage 1 or 2 ROP without plus disease or zone II, stage 3 ROP without plus disease; these eyes should be considered for treatment only if they progress to type 1 or threshold ROP. H. . INTERNATIONAL OPHTHALMOLOGY INTERNATIONAL OPHTHALMOLOGY L1 Severe visual loss is defined as a vision of 20/200 or worse. L2 Profound visual loss is defined as visual acuity worse than 20/400. L3 The World Health Organization (WHO) defines blindness as vision worse than 20 /400. L4 The World Health Organization (WHO) defines low vision as vision worse than 2 0/60. L5 Eyes with severe visual loss (20/200 or worse) are considered legally blind i n the United States. L6 Average visual acuity is significantly better than 20/20 in individuals young er than 60 years. L7 Worldwide, more adults than children are blind. L8 Pediatric blindness most commonly occurs from corneal scarring and retinal di seases. L9 Africa has the most cases of pediatric blindness, affecting approximately hal f a million children.

L10 If the mother has a history of lower socioeconomic status or a prior history of sexually transmitted diseases, then the risk of developing ophthalmia neonatorum is higher for the fetus. Studies have s hown that ethnicity does not affect the incidence of ophthalmia neonatorum, but socioeconomic status certainly does. L11 Most retinopathy of prematurity (ROP) is observed in zone II. ROP is classif ied into three zones (I, II, and III). Note that this is not necessarily true in developing countries where ROP is often dia gnosed at more advanced stages. L12 Children with xerophthalmia can be treated with high-dose oral vitamin A cap sules, which are inexpensive (<$0.10 per dose). L13 Night blindness is the earliest manifestation of xerophthalmia. L14 The catabolic state associated with measles infection can lead to severe cor neal xerophthalmia. L15 Lamellar lacerations are usually closed-globe injuries. A perforating injury involves two full-thickness lacerations. A contusion occurs with blunt trauma when the globe has not ruptured. L16 Refractive error not cataract is the most common cause of vision impairment in the world. L17 Macular degeneration is the most common cause of blindness in US seniors. L18 Cataracts accounted for >50% of the etiology for blindness in one survey in Middle East. L19 Nuclear cataracts are the most common form in white individuals. L20 The effect of body size as a risk factor for primary open angle glaucoma (PO AG) is unclear. In fact, the Barbados Eye Study found that lean body mass may be a risk factor for POAG in the African American population. L21 Children aged 3 to 6 years have the highest prevalence of trachoma. L22 Clinical (e.g., slit-lamp) examination is the most common method to diagnose trachoma. Polymerase chain reaction (PCR) and culturing are time consuming and not cost effective. L23 Streptococcus pneumoniae is one of the most common causes of microbial kerat itis worldwide. L24 There is currently no effective way to prevent retinopathy of prematurity (R OP). Regular screening is useful to provide treatment, which reduces the incidence of ROP progression. Unfortunately , a significant number of infants in developing countries are only diagnosed when their ROP is very advanced. L25 Sushruta, the first known surgeon, performed cataract surgery in India by co uching. Couching was performed in India as early as 800 BC. 2) AAO. MCQ.ii A. NEURO-OPHTHALMOLOGY. A1 Horizontal gaze palsy typically indicates damage to the ipsilateral pontine g aze center. A2 4th nerve palsy is the most common cause of diplopia posttrauma. A3 Tangent screen is most useful in testing those with nonorganic visual loss. A4 Change in pattern of headache indicate structural lesion. A5 Sildenafil may cause perception of a blue tinges (an illusion). Formed and un formed visual hallucination may occur with bilateral visual loss (Charles Bonnet syndrome). CAR (cancer-associated ret inopathy) frequently causes flashing lights (unformed visual hallucination), and parietal lobe lesions may p roduce either formed or unformed hallucination. A6 Simultanagnosia (the inability to appreciate more than one aspect of a visual stimulus at a time-the patient does not see the whole picture as the sum of its parts).

A7 Optic ataxia (inability to accurately direct the eyes to the desired visual t arget), and disturbances of visual attention. A8 Balint Syndrome is a rare phenomenon from bilateral occipitoparietal lesions. It consist of a triad of simultanagnosia (a failure to integrate multiple elements of a sense to form the total picture), optic ataxia (inability to accurately direct the eyes to the desired visual target), and acquired oculomoto r apraxia. A9 Intramuscular interferon beta-1a at the first episode of optic neuritis is be neficial in reducing the development of clinical MS over the following 3 years in patients at high risk based on the pre sence of subclinical brain lesions on MR. A10 Retrobulbar optic neuritis. vision usually improves over 3 months. (Kanski. Course. Vision worsens over several days to 2 weeks, and then begins to recover within 2-4 weeks. Initial recovery i s fairly rapid and then levels off but continues over 6-12 months). A11 The most common cause of monocular or binocular transient visual loss lastin g 30 minutes in a third decade woman is migraine. Typical migraine aura is a binocular positive visual phenomen on lasting between 15 and 30 minutes. It is more common in patients younger than 50 years of age and female m ore frequently affected. Transient visual loss associated with thromboemboli rarely lasts more than 10 minutes and is almost always unilateral. Papilloedema due to pseudotumour cerebri can cause transient bilateral visual gr ay outs that last only seconds. Giant cell arteritis is not a diagnostic consideration in a patient's 29 years o f age. A12 Myasthenia gravis (neuromuscular junction) would not cause clinically obviou s pupillary involvement. A13 MRI with gadolinium is the imaging modality of choice for cavernous sinus le sions. A14 Abducens nerve palsy combined with Horner syndrome is known as Parkinson's s ign. A15 For acute hemorrhage, CT (without contrast) is the imaging modality of choic e. A16 Disc elevation can be seen with drusen and papilledema. Capillary dilation a nd early hyperfluorescence are findings consistent with papilledema. A normal nerve fiber layer is not consiste nt with papilledema because axoplasmic stasis (nerve fiber layer opacification from edema) is the earliest s ign of papilledema.

A17 NAION is the most common optic neuropathy in people over age 50 years. Appro ximately 40% of cases of NAION become bilateral. There is no proven treatment. A18 Tobacco-alcohol amblyopia, like most toxic optic neuropathies, should be bil ateral, simultaneous, and symmetric. A19 Gaze-evoked amaurosis is pathognomonic of an orbital mass. The other entitie s (Carotid artery disease, Papilledema, Optic disc drusen) can cause transient visual obscurations/loss, bu t not gaze evoked visual obscurations/loss. A20 Nystagmus secondary to medication is usually bilateral. A21 Third ventricle tumors may be associated with see-saw nystagmus. The other e ntities (Lithium, Arnold-Chiari malformation, Cerebellar atrophy) have all been associated with downbeat nystagm us. A22 Convergence-retraction nystagmus, part of the dorsal midbrain syndrome, is b est demonstrated on attempted upward saccade. The downward rotation causes repeated upward refixation saccades . A23 Congenital ocular motor apraxia. The saccades are normal in amplitude, an ex ception to the general rule that patients with slow saccades have hypometric saccades. A24 Aberrant regeneration is never seen on a microvascular basis. It occurs with traumatic and compressive lesions. This situation necessitates imaging. A25 A 53-year-old female gives a 20-year history of flashing lights in only her left visual field; the lights last 20-30 minutes and occur several times a year. There are no other neurologic symptoms? i. The time course and description are consistent with migrainous visual phenome na. However, because the lights occur in the same visual field every time, a fixed lesion (eg, arterioven ous malformation) must be ruled out. A26 The ONTT found that in patients with abnormal MRI results who had been rando mized to IV corticosteroids, rates of second neurologic events were lower over the next 2 years. However, there was no difference in visual outcome, which is generally very good, among any of the treatment groups (placeb o, high-dose IV corticosteroids/oral taper, oral drugs alone), although 1 mg/kg oral corticoster oid dosing alone caused a higher rate of second optic neuritis attacks and is contraindicated. A27 Certain antibiotics (notably tetracycline and the rarely used nalidixic acid ) are well-known causes of pseudotumor cerebri. Oral contraceptives have been linked to pseudotumor cerebri. Exogenous sources of vitamin A, such as dietary supplements or exuberant consumption of liver (several pounds per week), which has large amounts of vitamin A ("liver lover's lament"), also cause pseudotumor cerebri. Weight gain, not weight loss, is a risk factor for pseudotumor cerebri. A28 Hypertension, diabetes mellitus, and a small cup-to-disc ratio (the so-calle d crowded disc or disc at risk) are the three principal risk factors for NAION. Other risk factors include smoking, migr aine, and hyperlipidemia. Carotid occlusive disease is not a significant risk factor for NAION. A29 An RAPD occasionally occurs in significant macular disease or amblyopia. A p atient with optic neuropathy may not have a relative afferent pupillary defect if a contralateral afferent pupillary

defect balances the defect in the first eye. For example, someone with no light perception in both eyes may have bilateral ab solute afferent pupillary defects but no relative defect. An asymmetric chiasmal lesion can produce an RAPD. On the ot her hand, symmetric damage may not cause a relative defect. A30 Demyelinating optic neuritis. a. Frequently leads to Uhthoff's phenomenon: the patient notices that whenever t he body temperature is elevated from fever, hot tub soaks, exercise), vision decreases until the body i s cooled. b. The 5-year risk of clinically definite MS is 16% if MRI results are negative and 51% if MRI reveals three or more lesions. c. Pain on ocular rotation is the rule, occurring in about 90% of cases of optic neuritis. d. Optic neuritis is remarkably common in the course of MS: clinical evidence is found in 75% of cases, and consistent pathologic changes are seen in 90%. A31 In young persons, the acute onset of internuclear ophthalmoplegia is often a presentation of demyelinating disease. In the older vasculopathic age group, this condition is often the sign of a stroke. In many internuclear ophthalmoplegias, the adduction deficit is always present, whereas the abducting nystagmus may be absent. Although paraneoplastic eye movement disorders do exist (eg, Lambert-Eaton syndr ome), they are rare. A32 Both the sleep test and the ice pack test (an improvement of 2 mm in ptosis following a 2 minutes ice application to the eyelid) are useful when the Tensilon test is equivocal or contraindicated. A cetylcholine receptor antibodies are abnormal in only about 60% of cases of MG. Thymoma, although a critical finding is present in a minority (about 10%) of patients with MG. The see-saw sign (manual elevation of the ptotic lid a ccompanied by ptosis of the contralateral lid), although a useful and suggestive (although not specific) dia gnostic sign is an example of Hering's law of yoked innervation, not Sherrington's law of reciprocal innervation. Conco mitant thyroid eye disease is present in 5%. As many as 85% of patient who present with ocular myasthenia go on to dev elop systemic myasthenia. If the patient has ocular myasthenia for only 2 years, the conversion rate is about 10% . A33 AAO. Acetylcholine receptor antibodies Binding antibodies are usually reques ted, because they are detected in approximately 90% of patients with generalized MG and 50% of patients with ocula r MG. A34 Miller Fisher syndrome, a variant Of Guillain-Barre syndrome, may cause bila teral facial nerve paresis.

A35 If the isolated pupil-sparing complete third cranial nerve palsy becomes chr onic (i.e., no recovery after 3-6 months)even if the patient is diabetic the clinician must rule out a structural lesion; MRI of the brain is best. A36 Vertebrobasilar insufficiency causes a transient homonymous hemianopic defic it. A37 Acute painful Horner syndrome is an emergency and should be considered a car otid dissection until prove otherwise. The reason for such urgency is high risk of embolization to the eye o r brain within 2 weeks following the dissection. Early anticoagulation is generally recommended. A38 One of the key findings of the Optic Neuritis Treatment Trial (ONTT) was tha t the risk of MS can be predicted by the number of plaques visible with MRI of the brain performed during the episode of optic neuritis. Furthermore, the risk of MS over the next 2-3 years, which is higher in patients with at least tw o typical plaques, can be significantly reduced by a 3-day course of IV corticosteroids (methylprednisolone) followed by 11 days of oral prednisone. A39 A lesion in the left temporal lobe may produce a right homonymous upper quad rantanopia because of damage to visual fibers in Meyer's loop. This would affect the upper nasal field in the le ft eye and the upper temporal field in the right eye. A40 Stroke causes 90% of isolated homonymous hemianopias. A41 Formed visual hallucinations may be seen with temporal lobe lesions. Percept ual problems, including agnosia, apraxia, and acalculia, occur with parietal lobe lesions. A42 Endarterectomy is indicated for symptomatic severe stenosis (70-90%) but not for milder stenosis (50-69%). Aspirin may reduce the incidence of stroke. A43 A diffusion weighted imaging (DWI) sequence of MRI demonstrates infarcts eve n in the hyperacute stages, whereas CT and other MRI modalities are often still unremarkable. A44 Occipital lobe lesions otherwise produce congruous homonymous defects, don t a ffect VA unless bilateral and are not associated with optokinetic defects unless the adjacent parietal lobe is involved. A45 Adie's pupils tends to becomes small over time but still retain light near d issociation and sector palsies of the iris sphincter muscle. A46 The Pulfrich phenomenon, metamorphopsia, and micropsia are illusion; an obje ct is seen in a distorted fashion. A47 Bell's palsy occurs unilaterally and affects the muscles of both the upper a nd lower face. Bell's palsy most commonly affects patients over the age of 50 years. Recovery of function typical ly occurs within the first 3-6 weeks after the onset of the deficit. A48 Simultaneous bilateral facial palsy need prompt additional evaluation. A49 In static perimetry, higher decibel values indicate higher visual sensitivit y. A50 Stroke of the optic chiasm is rare. A51 Spasm of the near reflex associated with pupillary constriction on attempted lateral gaze, dynamic retinoscopy on attempted lateral gaze with associated myopic shift would further confirm the co rrect diagnosis. A52 Lesions of the dominant parietal lobe cause Gerstmann syndrome, a combinatio n of acalculia, agraphia, finger agnosia, and left-right confusion.

A53 Optic atrophy usually requires 4-8 weeks to develop after optic injury. A54 Sarcoidosis can cause iritis, facial nerve palsy, and optic neuropathy with moderate optic disc swelling. A55 With horizontal gaze palsy MRI is indicated urgently. A56 Bilateral papilledema requires neuroimaging and, if results are negative, a spinal tap. B. . PEDIATRIC AND STRABISMUS PEDIATRIC AND STRABISMUS A1 Uncorrected myopia often worsens control of exotropia by blurring distance vi sion and by decreasing the accommodative requirement for intermediate and near fixation. In response, overm inused glasses were prescribed to promote increased accommodative demand at all fixation distances. Although ov erminused glasses may be helpful and necessary, they are not indicated as the initial treatment. Antisupp ression patching (either of the dominant eye or alternating) can also improve control of exotropia but is unlike ly to have much effect in the face of uncorrected myopia. Therefore, the best initial treatment of this child is to co rrect her myopia. A2 A neurologically normal child nearing 6 months of age with a large-angle esot ropia, who has established crossfixation, is very unlikely to experience spontaneous improvement. Depending on t he personality of the child, it may be several more months before alternate cover testing of any accuracy can be per formed, and the measurements are unlikely to be more than 5 different from the results of a good Krimsky test . In order to maximize the likelihood of some level of binocular function, the child's eye should be surgically straighte ned as soon as possible. There is no reason for delay. Subsequent eye misalignments can be dealt with by subsequent s urgeries, and the parents should be informed that more than one operation maybe needed during childhood. A3 A small subset of children with accommodative esotropia present in infancy ra ther than the more typical range of 1836 months. If the condition is caught early enough, these patients respond well to spectacles containing the full cycloplegic refraction. Prescribing less than the full cycloplegic refraction ma y work if the child is highly hyperopic (eg, >+6 sphere OU), but in this case to eliminate a near angle of 30, success i s most likely with full refractive correction. A4 Glaucoma with neurofibromatosis is seen in association with plexiform neurofi bromas of the eyelids (a secondary form of glaucoma); in the absence of a lid lesion, there is no reason to suspect that this child has elevated IOP and no reason to sedate her to measure IOP.

A5 Lisch nodules eventually develop in over 90% of patients with neurofibromatos is type I, but they are rarely present in early childhood. Lisch nodules are therefore a sensitive, but not a specific, fi nding for neurofibromatosis at this age (2 years). A6 A CT scan with coronal views, rather than MRI, is the best way to image a sub periosteal abscess. This is fortunate because a 5-year-old child usually requires deep sedation or general anesthesia for MRI but not for CT. A7 Congenital glaucoma can be mistaken by pediatricians for nasolacrimal duct ob struction, particularly if obvious corneal edema or buphthalmos are absent when the parents first complain of teari ng. A8 Upper lid lesions can cause amblyopia due to pupil occlusion. Usually, the li d must cover half or more of the pupil at all times to be significantly amblyopic. Even infants adopt head turns or chin-u p postures to avoid pupil occlusion. A9 Tucking too much or too close to the trochlea can hinder the movement of the tendon through that structure; hence, tucking generally is performed in the portion of the tendon adjacent to the nasa l border of the superior rectus muscle or more distally. A10 Absence of hyperdeviation in the lower field, where there is no occasion for the short or otherwise restricted superior oblique tendon to generate a limited rotation, is characteristic of all but the most severe cases of Brown syndrome. Divergence in straight upgaze is an important sign helping to distingu ish Brown syndrome from inferior oblique muscle palsy; it was considered by Brown to be a defining feature of thi s condition and is not associated with the opposite horizontal deviation in downgaze. A11 Exotropia in upgaze was described by Brown as an essential feature of this s yndrome, although it is not prominent in every case. Hypotropia in primary position is present in monocular elevation deficiency, but the most severe cases of Brown syndrome show this as well. A12 Possible causes of papilledema and a sixth nerve palsy are dural AVM, crania l venous thrombosis, pseudotumour and sleep apnea. Diabetes is not. A13 The restrictive changes that occur in the extraocular muscles in Graves' dis ease are preceded and accompanied by an inflammatory phase that adds a variable weakening effect to th eir action. A14 Masses presenting above the medial canthal ligament in infancy are usually d ermoid cysts or encephaloceles. It is important to rule out this last condition by imaging, as it would be a ser ious error to attempt excision of a dacryocele in this area without knowing that it does not communicate intracrania lly. Systemic antibiotics without drainage do not affect lacrimal system blockage definitively. Percutaneous incis ion and drainage too often results in fistula formation and should be avoided unless the skin is already seriously com promised. Bony stenosis of the nasolacrimal canal typically gives signs of a chronic partial obstruction rather than of an acute total closed-end obstruction. A15 Enucleation is still frequently used to treat large tumors in eyes with poor visual potential or when other methods of treatment have failed. Cryotherapy and laser therapy are usually rese

rved for small tumors, less than 2.0-2.5 mm in thickness. Plaque radiotherapy can be used as a secondary treatmen t for retinoblastoma but may be associated with a higher incidence of radiation retinopathy and optic neuropathy when used in children who have already undergone chemotherapy. A16 Steroid administration is more likely to result in cataracts or glaucoma in children than in adults. A17 The critical period for the development of deprivation begins earlier and la sts longer than that for strabismic or anisometropic amblyopia. Furthermore, a shorter period of time is necessary for visual deprivation to cause amblyopia than is the case for strabismic or anisometropic amblyopia. A18 It would be controversial to substitute surgery for glasses in a patient wit h fully accommodative esotropia. Surgery for the accommodative excess portion of the deviation at near to enable an older child to wear single-vision glasses or contact lenses is less controversial. Surgery for any nonaccommodativ e portion of an esodeviation, as long as it has been confirmed that the patient is wearing the full hyperopic cor rection, is commonly performed. A19 In the gradient method of determining the AC/A ratio, plus or minus lenses c an be used to evaluate the effect of accommodation on the deviation at a given distance. A20 In-office probing can be performed in children younger than 6 months but bec omes increasingly difficult as the child gets older. By 9-12 months of age, the child is usually too big to hold do wn and forcibly probe. A21 The success rate of probe and irrigation alone drops to 33% if not done by 2 years of age. A22 Obstruction of drainage below the lacrimal sac occurs in about 5% of full-te rm newborns. Usually, a thin mucosal membrane at the lower end of the NLD is the cause. Symptoms become manif est by age 1 month in 80%90% of cases. A23 The classic presentation for congenital glaucoma is epiphora, light sensitiv ity, and a hazy cornea. IOP should be measured at the beginning of the examination under anesthesia because anesthe sia has a rapid lowering effect on IOP. Refraction might show a myopic anisometropia, which helps confirm the diagnosis and may require treatment to prevent amblyopia. A24 Bilateral congenital cataracts are either inherited (due to a metabolic or i nfectious cause) or developmental. The genetics of cataracts are not well-known, and although cataracts can be asso ciated with a chromosomal defect, this is rare and analysis is usually not indicated. The one treatable metabolic disorder is galactosemia, which can be detected by looking at the red blood cell enzymes.

A25 Wyburn-Mason and Sturge-Weber syndromes are the only two phakomatoses that h ave no genetic inheritance pattern. Tuberous sclerosis, von Hippel-Lindau disease, and neurofibromatosis ar e typically autosomal dominant. Ataxia-telangiectasia has autosomal recessive transmission, and incontinentia pi gmenti is X-linked dominant. A26 Retinal angiomas. a. Retinal angiomas are often located in the peripheral fundus and may be asympt omatic when small. b. As the lesion grows, its capillaries may become more incompetent and allow fo r transudation of fluid. c. If the retinal edema and exudates are extensive enough to involve the macula, vision becomes compromised. d. The peak incidence of retinal angioma occurs a decade before its cerebellar c ounterpart. e. The tumors are bilateral in up to 50% of cases. f. Treatment with cryotherapy or laser photocoagulation may be especially effica cious with smaller lesions. A27 Brachycephaly BrachycephalyPremature bilateral coronal suture closure, a characteristic featur e of Crouzon syndrome. denotes the asymmetry caused by premature closure of a unilateral coronal suture . PlagiocephalyScaphocephaly PlagiocephalyScaphocephalyis the abnormal skull shape caused by premature sagitt al suture closure. A28 Undercorrections are much more common than overcorrections. Overcorrections can occur after suture adjustment. Slippage of a resected muscle causes an undercorrection. A29 Retinal detachments (especially in children) and endophthalmitis are rare ev en in cases with known scleral perforation. Although the incidence was as high as 10% in the 1970s, spatulated needles have made the complication much less common recently. Most surgeons recommend cryopexy or lase r therapy, although some do not. A30 Localized conjunctival injection and chemosis noted several days postoperati vely at the site of eye muscle surgery may be caused by a. Suture allergy. b. Poor closure of the conjunctival wound. c. Conjunctival inclusion cyst. A31 Transposition procedures and other surgeries involving three or more rectus muscles pose a special risk of anterior segment ischemia. Such ischemia can occur even following two-muscle sur gery, although typically in elderly patients with poor circulation or blood dyscrasias. Iritis and sector iris atrop hy are characteristic of anterior segment ischemia. A32 Botulinum toxin is particularly useful for small residual angles following s trabismus surgery. A33 Hospitalization, patching, bed rest, cycloplegia, and steroids are all contr oversial in the treatment of hyphemas, which should be individualized according to the risk of complications. Hyphemas are not more common in sickle cell disease or trait, but they are more dangerous because of impeded resorption and risk of retinal vascular occlusion. Because of the risk of corneal blood staining and the difficulty obtaining accur ate pressure measurements, total

hyphemas in young children should be evacuated in this time frame. A34 The cornea flattens during the first year of life, resulting in a decrease i n corneal power. A35 Changes of the eye parameters. a. The horizontal palpebral fissure at birth is almost as long as in childhood, yet a newborn's vertical fissure is only half the size of an adult's. b. The diameter of the eye at birth is about 66% of that in adulthood. The eye g rows rapidly during the first 2 years of life; growth then slows until puberty. c. Infants often have astigmatism during their first months of life, and most yo ung children are hyperopic. Absence of hyperopia during early childhood is usually a harbinger of myopia, wh ich increases as growth progresses. d. As the axial length of the eye increases (4 mm during the first 6 month of li fe), the lens flattens. At birth, the anteroposterior length of the eye is about 17 mm, increasing to approximately 24 mm in adulthood. e. The anterior chamber depth at birth is 2.3-2.7 mm, which is shallower than in adults because of the steep anterior lens surface in children. f. The average corneal horizontal diameter is 9.5-10.5 mm in newborns and 12 mm in adults. g. The radius of corneal curvature is 6.6-7.4 mm in newborns and 7.4-8.4 mm in a dults. Keratometry values change markedly in the first year of life, starting at approximately 52 D at bir th and flattening to 42-44 D in adulthood. Thereafter, keratometry remains relatively stable. h. The infant sclera is about half of its adult thickness and strength. A36 In Duane syndrome exotropia in gaze away from the affected eye of a unilater al case can sometime be seen. This finding is a feature differentiating type 1 Duane syndrome from sixth cranial nerve palsy. A37 Many cases of congenital motor nystagmus have a sensory visual defect. A38 Acquired unilateral cataracts in full size eyes are due to almost exclusivel y to posterior lenticonus or trauma. Both PFV and congenital nuclear cataracts are associated with microcornea. Lamel lar (zonal) cataracts are always bilateral. A39 In general posterior lenticonus behaves as an acquired cataract, and therefo re the visual potential is often good.

A40 Following strabismus surgery, Diplopia is more common with overcorrections. Perforation of the sclera usually causes a chorioretinal scar with no visual sequela. A41 IOL implantation in children with JIA remains controversial. A42 A cranial venous thrombosis can mimic an orbital pseudotumour. A43 The craniosynostoses are often associated with excyclorotation of the orbits and abnormal insertions of the horizontal rectus muscles. This may result in a V-pattern exotropia, in which th e exotropia increases in upgaze and is minimized in downgaze. It is also associated with secondary apparent IOOA. A44 The main difference between the phenotypes of Crouzon vs other craniofacial syndromes especially Apert is that Crouzon don t have hand or foot abnormalities. A45 All of the following are characteristic of the shaken baby syndrome. a. The child is younger than 12 months. b. Parenchymal brain damage and intracranial hemorrhage are common. c. Retinal hemorrhage is typical, especially in the posterior pole. d. Adnexae and anterior segments are typically uninvolved. C. . ORBIT, EYELID AND LACRIMAL SYSTEM ORBIT, EYELID AND LACRIMAL SYSTEM A1 The optic canal is located immediately superior and lateral to the sphenoid s inus wall. A2 The five major branches of the facial nerve include the temporal, buccal, mar ginal mandibular, cervical, and Zygomatic. A3 AAO neuro-ophthalmology. The Extracranial trunk of the nerve passes between t he superficial and deep lobes of the parotid gland, where it divides into two trunks, the temporofacial superiorly, a nd the smaller cervicofacial inferiorly. These further variably divide into five major branches: the temporal, zygomatic, infraorbital, buccal, and mandibular. The temporal and zygomatic branches laterally innervate the orbicularis oculi mu scles. The infraorbital and buccal branches may also variably contribute to the inferior orbicularis. A4 The parotidomasseteric fascia is generally a thin, wispy structure that overl ies the facial nerve branches, which overlie the masseter muscle, which is therefore the deepest of the structures li sted. A5 A 65-year-old woman presents with a progressively enlarging mass in the right inferior orbit. Distraction of the lower eyelid reveals a "salmon patch" appearance to the fornix. Approximately 90% of o rbital lymphoproliferations will prove monoclonal, suggesting a diagnosis of lymphoma; while 10% are polyclonal, suggesting reactive lymphoid hyperplasia. A6 Eyelid retraction is the most common clinical feature of Graves's ophthalmopa thy (and Graves's ophthalmopathy is the most common cause of eyelid retraction). A7 Orbital abscesses in adults are more likely to arise from chronic sinusitis. These infections are more likely to be mixed polymicrobial infections of aerobic and anaerobic bacteria. A8 Subperiosteal abscess of the orbit in children is more likely than in adult t o respond to single antibiotic therapy. In patient younger than 9 years, most of these infections are due to single organis ms affecting the ethmoid sinuses, and they may drain spontaneously if vision is not threatened. A9 Although Graves ophthalmopathy occurs most commonly in association with hyper thyroidism (and treatment of hyperthyroidism is important in the overall care of the Graves patient), the cou

rse of the ophthalmopathy does not necessarily parallel the activity of the thyroid gland or the treatment of thyro id abnormalities. A10 The most common location for orbital lymphoma is lacrimal fossa. Up to 50% o f orbital lymphoproliferative lesions arise in the lacrimal fossa. A11 Although systemic steroids are useful in idiopathic orbital inflammatory syn drome (orbital pseudotumor), they are not recommended in the treatment of lymphoproliferative lesions. Radiotherap y is the treatment of choice for patients with localized ocular lymphoproliferative disease. A surgical cure usua lly cannot be attained because of the infiltrative nature of lymphoid tumors. Patients must be monitored indefinitely for development of additional lymphoproliferative lesions. A12 Treatment recommendations vary for isolated MALT lesions. Simple excision ma y be diagnostic and therapeutic, with some authors recommending observation for isolated stable lesi ons. A13 Surgery should be avoided for patient with orbital varices except when pain or visual loss necessitates treatment. A Valsalva maneuver during CT scan may be required for diagnosis. Eno phthalmos on the involved side is common. A14 At 4 days after injury with exophthalmos and restricted motility, additional observation is necessary to allow traumatic edema to subside. This subsidence may be enhanced by the administratio n of oral prednisone, 1 mg/kg per day for 1 week. Urgent exploration of orbital blowout fractures is necessary only if there is radiographic evidence of gross extraocular muscle entrapment beneath the fracture fragments. It is gen erally preferable to allow 10-14 days for swelling to resolve and motility to be reevaluated before proceeding with su rgical repair. A15 the following features is most likely to be found on an orbital CT scan of a patient with Graves ophthalmopathy is an increased amount of orbital fat in the presence of normal-sized extraocula r muscles. A16 Capillary hemangioma is the most common benign primary orbital tumor among c hildren. Orbital cellulitis is the most common cause of unilateral exophthalmos in children. Metastatic neuroblasto ma is the most common

metastatic cancer of the orbit in children. Neurofibromas are rarely malignant a nd are uncommon orbital tumors in children. Graves's ophthalmopathy is very rare in children. A17 Basal cell carcinomas occur approximately 40 times more often than either se baceous carcinomas or squamous cell carcinomas. Even though both sebaceous cell carcinomas and squamou s cell carcinomas occur more often in the upper eyelid than they do in the lower and basal cell carcinom a occurs more often in the lower lid than in the upper, the far greater frequency of basal cell carcinoma results in its still being the most common malignant neoplasm of the upper eyelid. (The tumour has a predilection for the l ower eyelid and the lid margin. Kanski). A18 The majority of histologically malignant lymphoid lesions of the orbit are r elatively indolent or low-grade lymphomas. The majority of orbital lymphomas are well differentiated. A19 Sudden pain associated with tense, ecchymotic eyelids following blepharoplas ty is indicative of a postoperative orbital hematoma. If there is no decreased vision, altered pupillary response, o r other indication of decreased optic nerve function, the patient may be managed conservatively with close observation . If there is any evidence of optic nerve or ocular compromise, the wounds should be opened immediately and drains i nserted to decompress a possible orbital hemorrhage. A lateral canthotomy and inferior cantholysis shoul d be performed next if the above is not effective. A20 Posttraumatic ptosis, immediate treatment. If ptosis is present, the wound s hould be explored. If orbital fat (preaponeurotic fat pad) is visible, exploration of the deeper orbital structure s should be undertaken. Repair of the levator aponeurosis is indicated as a primary procedure. A21 Medial canthal trauma usually results from tangential (lateral) forceful tra ction on the lower eyelid causing an avulsion type of injury at the weakest point of the eyelid, the medial canthal t endon. This frequently involves the canalicular system. Repair can be easily delayed 12-24 hours. Delayed lacrimal r epair (>6 months) will require a CJDCR with Jones tube. A22 Indications for repair of orbital blowout fracture include all of the follow ing: a. Cosmetically unacceptable enophthalmos (2 mm or more of enophthalmos). b. Fractures involving more than half of the orbital floor. c. Pain and oculocardiac reflex on upgaze. d. Significant inferior rectus entrapment. A23 Early implant extrusion is typically associated with either poor wound closu re or an implant that is too large. Early implant extrusion is typically associated with either poor wound closure o r an implant that is too large. Late implant extrusion can be associated with tumor recurrence. Late implant ext rusion can be associated with a Late implant extrusion can be associated with tumor recurrence. Late implant ext rusion can be associated with a . conjunctival cyst conjunctival cyst A24 The most important step in repair of a full thickness eyelid laceration is c areful approximation of the tarsus. The tarsus is the backbone of the eyelid and must be appropriately approximated so t

hat the repair is stable. A25 The most important determinant in selecting a corrective procedure for any t ype of ptosis is amount of levator function. A26 Invasive aspergillus infections of the orbit. a. In addition to acute fulminant fungal sinusitis with orbital invasion, asperg illosis can cause chronic indolent infection resulting in slow destruction of the sinuses and adjacent structures ( bone). b. Corticosteroids may produce an initial clinical improvement. c. Septate branching hyphae of uniform width are seen histologically. D. . EXTERNAL DISEASE AND CORNEA EXTERNAL DISEASE AND CORNEA A1 In general, visual acuity is reduced earlier when there is corneal surface ir regularity as opposed to diffuse clouding with a smooth epithelial surface. Therefore, dystrophies such as primary familia l amyloidosis, where amyloid occurs as subepithelial deposits; Reis-Bucklers dystrophy, which affects Bowman's layer ; and lattice dystrophy, where amyloid is deposited most heavily in the anterior stroma and subepithelial area, would all cause some vision difficulty by the second decade of life. Central cloudy dystrophy of Francois, on the other hand, appears as deep, polygonal, gray areas with an unaffected anterior stroma and smooth epithelium. Vision is t ypically not reduced by this condition. A2 If corneal surface irregularity from severe anterior basement membrane dystro phy is the main cause of decreased visual acuity, overrefraction with a rigid gas-permeable contact lens will mask the surface irregularity and improve visual acuity, so we can differentiate that VA is reduced due to anterior surfac e irregularity not due to cataract. Glare testing is of no benefit since both corneal pathology and cataract can manifest decreased vision with glare testing. A3 Although fluorescein has no antiviral properties, rose-bengal, a halide deriv ative of fluorescein, has antiviral and other toxic effects. A4 Although the mucin layer is the innermost of the three layers and is secreted by conjunctival goblet cells, it is a hydrophilic layer that overlies the hydrophobic ocular surface. A5 In lacrimal gland dysfunction states, the normal production of proteins by th e lacrimal gland is diminished. A decreased tear lysozyme or lactoferrin level is highly suggestive of dry eye. Te ar film osmolarity has been shown to be increased in patients with aqueous tear deficiency as well as those with m eibomian gland dysfunction. The tear meniscus is normally 1.0 mm in height and convex. A tear meniscus 0.3 m m or less is considered abnormal.

A6 Sarcoidosis is a multisystem disorder of uncertain etiology. After pulmonary involvement, ocular involvement is the most common manifestation, seen in up to 50% of affected patients. The most comm on corneal manifestation is calcific band keratopathy, often associated with chronic uveitis or elevated ser um calcium levels. Nummular keratitis, Thickening of Descemet's membrane, and deep stromal vascularization can occur wi th sarcoidosis. A7 The application of dilute (1%) bleach to tonometer tips for at least 10 minut es is recommended to prevent transmission of adenoviruses. A8 The Acanthamoeba life cycle includes the motile trophozoite and the dormant c yst forms. Free-living amoebae are found in both forms, but only the trophozoite is infectious. Both forms are foun d in infected human tissues. Acanthamoeba can be grown on blood agar and on buffered charcoal yeast extract a gar. Acanthamoeba can be visualized with calcofluor white, acridine orange, or the Giemsa stain. Acantham oeba keratitis can appear as a pseudodendrite early in its course. A9 EKC is the only adenoviral syndrome with significant corneal involvement. The infection is bilateral in a majority of patients and may be preceded by an upper respiratory infection. Photophobia and reduced vision from adenoviral subepithelial infiltrates may persist for months to years. It is not associated with enlarged corneal nerves, which are seen in such conditions as MEN syndrome and leprosy. It may be associated with l arge central geographic corneal erosions. A10 Herpes zoster ophthalmicus (HZO). The current recommendation for HZO is oral famciclovir 500 mg three times per day, valacyclovir 1000 mg three times per day, or acyclovir 800 mg fiv e times per day for 7-10 days, best if started within 72 hours of the onset of skin lesions. Topical antiviral medicati ons are not effective. Intravenous acyclovir therapy is indicated in patients at risk for disseminated zoster due t o immunosuppression. A11 Graft-versus-host disease Graft-versus-host disease . . a. A relatively common complication of allogenic bone marrow transplantation per formed most commonly for hematopoietic malignancies. b. Conjunctival inflammation in GVHD can be severe and even associated with limb al stem-cell deficiency and secondary corneal scarring, but the latter is a rare complication. c. Typical presentations include conjunctival and corneal epithelial erosions, a queous tear deficiency, and cicatricial lagophthalmos. d. Aggressive lubrication and punctal occlusion are the mainstays of local thera py. Severe filamentary keratopathy can be additionally treated with mucolytic agents (10% acetylcystein e) as needed. Severe ocular surface disease in GVHD may be associated with active nonocular (often sk in) GVHD and may require increased systemic immunosuppression by cyclosporine or tacrolimus (FK50 6). A12 Although congenital syphilis was the first infection to be linked with inter stitial keratitis, many other agents can

also cause IK as (Herpes simplex virus, Herpes zoster virus, and Chlamydia trach omatis). A13 Several disorders can be associated with limbal stem-cell dysfunction, inclu ding aniridia, contact lens-induced keratopathy, atopic keratoconjunctivitis, ocular cicatricial pemphigoid, Stevens -Johnson syndrome, chemical injuries, and keratitis associated with multiple endocrine neoplasia (MEN IIb or III). A14 Surgical treatment of squamous cell carcinoma should include all of the foll owing: a. Excision of conjunctiva 4 mm beyond the clinically apparent margins of the tu mor. b. Resection of a thin lamellar scleral flap beneath the tumor. c. Treatment of the remaining sclera with absolute alcohol. d. Cryotherapy applied to the conjunctival margins. A15 The combined ocular and cutaneous pigmentations seen in patient are referred to as oculodermal melanocytosis. a. Approximately 5% of cases are bilateral. b. Malignant transformation is possible but rare and seems to occur only in pati ents with a fair complexion. c. Malignant melanoma can develop in the skin, conjunctiva, uvea, or orbit. d. The lifetime risk of uveal melanoma in a patient with ocular melanocytosis is about 1 in 400, significantly greater than the risk of 1 in 13,000 of the general population. A16 Malignant transformation of primary acquired melanosis should be suspected w hen a lesion shows nodularity, enlargement, or increased vascularity. Small typical areas of primary acquired m elanosis can be observed and biopsy delayed until progression is observed. A17 Corneal clouding at birth has many different etiologies, including congenita l glaucoma, birth trauma, sclerocornea, Peter's anomaly, or CHED2. A18 Two forms of CHED are recognized. The more common autosomal recessive type ( CHED2) presents at birth, remains stationary, and is accompanied by nystagmus. The bluish white cornea may be two to three times' normal thickness and have a ground-glass appearance, but this finding is not associated with tearing or photophobia. The dominant form (CHED1) presents in the first or second year of life, although exp ressivity is variable. It is slowly progressive and accompanied by pain, photophobia, and tearing, but nystagmus is not present. The cornea exhibits a diffuse blue gray ground-glass appearance. A19 The most distinctive microscopic finding in posterior polymorphous dystrophy is the appearance of abnormal, multilayered endothelial cells that look and behave like epithelial cells or fib roblasts. These cells

a. Show microvilli. b. Stain positive for keratin. c. Show rapid and easy growth in cell culture. d. Have intercellular desmosomes. e. Manifest proliferative tendencies. f. Both angle closure and open-angle glaucoma can occur, and 14% of patients hav e elevated IOP. A20 The frequency of regraft is highest in lattice dystrophy, followed by granul ar dystrophy and then macular dystrophy. Fuchs dystrophy does not typically recur in the graft. LGMF. A21 Schnyder crystalline corneal dystrophy: features of this rare, slowly progre ssive autosomal dystrophy include central subepithelial crystals (seen only in 50% of patients), central corneal o pacification, dense corneal arcus lipoides, mid peripheral corneal opacification, and decreased corneal sensation. A22 Calcific band keratopathy: this calcific degeneration of the superficial cor nea involves mainly Bowman's layer. There are six main causes: chronic ocular disease; hypercalcemia caused by hyper parathyroidism, vitamin D toxicity, milk-alkali syndrome, sarcoidosis, and other systemic disorders; hered itary transmission; elevated serum phosphorus with normal serum calcium; chronic exposure to mercurial vapors or to mercurial preservatives in ophthalmic medications; and silicone oil instillation in an aphakic eye. A23 An unusual syndrome, diffuse lamellar keratitis (DLK, sands of the Sahara, S OS) with diffuse interface haze, may appear on postoperative day 1-3 with possible decrease in vision. The condit ion appears to be a nonspecific inflammatory reaction in response to a variety of toxic insults. Central or peri pheral WBCs with granular appearance usually respond to frequent topical steroids; however, when dense clumps appear in the visual axis, lifting the flap and irrigating, followed by intensive topical steroid treatment is usually requi red. For severe cases, systemic steroids may be used adjunctively. A24 Bilateral herpes simplex occurs in about 3% of the cases and is unrelated to treatment or other herpetic viral infections. It may occur with HIV infections, but ocular atopy is much more freq uent. A25 Eyes with posterior choroidal melanoma may be considered acceptable; however , most medical eye bank directors decline their usage. All other conditions exclude the donor cornea (Le ukemia, Previous PRK for correction of 2 D of myopia, Active septicemia). A26 Dry eye can be detected with corneal topography which shows as irregular ast igmatism from disruption of the tear film. A27 A small wound leak (with no anterior chamber shallowing) seen on the first p ostoperative visit after penetrating keratoplasty. The wound is always checked carefully for leakage on postoperative visits. Small wound leaks that do not cause anterior chamber shallowing frequently close spontaneously. Patching, therapeutic contact lenses, and use of inhibitors of aqueous production may hasten wound closure. Resuturing is advised for leaks lasting longer than 3 days. A28 Corneal transplant rejection takes three clinical forms-epithelial, subepith elial, and stromal-which may occur

either singly or in combination. In epithelial rejection, lymphocytes cause an e levated, linear epithelial ridge that advances centripetally. In subepithelial rejection, subepithelial infiltrates ar e seen, sometimes accompanied by a cellular anterior chamber reaction. Isolated stromal rejection is not common but can be seen as stromal infiltrates and neovascularization. In very aggressive severe or prolonged bouts of graft re jection, the stroma can become necrotic. Endothelial rejection is the most common and serious form of corneal t ransplant rejection. Inflammatory precipitates are seen on the endothelial surface in fine precipitates, in random clumps, or in linear form (Khodadoust line). Inflammatory cells are usually seen in the anterior chamber as well. E. . INTRAOCULAR INFLAMMATION AND UVEITIS INTRAOCULAR INFLAMMATION AND UVEITIS A1 Significant inflammation is usually seen in sympathetic ophthalmia and phacoa ntigenic endophthalmitis. Milder inflammation is usually seen in Propionibacterium acnes endophthalmitis. In phac olytic glaucoma, macrophages engulf leaking lens protein and there is usually little or no intraocular inflam mation. A2 Patients with rheumatoid arthritis, Behçet syndrome, and Reiter syndrome are mo re likely to have a nongranulomatous uveitis and patients with sarcoidosis, a granulomatous uveitis. A3 Prednisolone acetate 1% (Pred Forte) and prednisolone phosphate 1% (Inflammas e Forte) are more effective than loteprednol (Lotemax), fluorometholone 0.25% (FML Forte), and dexamethasone 0.1% (Decadron) in treating intraocular inflammation in uveitis. A4 Kawasaki syndrome is usually seen in childhood. 85% of patients are less than 5 years old. A5 Rigid, closed-loop anterior chamber intraocular lenses are most associated wi th recurrent uveitis. Iris plane intraocular lenses occasionally cause inflammation, and, less commonly, a sulcus -placed posterior chamber intraocular lens causes inflammation from iris chafing or uveitis-glaucoma-hyphe ma (UGH) syndrome. Silicone intraocular lenses have been associated with iritis in the past. A6 In one series of JIA-associated uveitis cases, cataract was seen in 84%, band keratopathy in 70%, macular edema in 42%, and glaucoma in 26% of patients. A7 Periocular corticosteroid injections may cause proliferation of the organism (toxoplasmosis) and should be avoided. A8 The IgM antibody titer is the best laboratory test for a newly acquired toxop lasmosis infection. The IgM antibody titer will be elevated early after the infection but will not be detectable 2-6 months after the initial infection.

A9 Ophthalmologic signs of systemic lupus erythematosus include cotton-wool spot s, retinal arteriolitis and vascular occlusion, vitreous hemorrhage, choroidal infarction, choroiditis, and subretina l exudation. Anterior uveitis is not usually seen with systemic lupus erythematosus. A10 A salt-and-pepper fundus may be seen in congenital syphilis, and uveitis may be seen in secondary syphilis and in other stages. A lumbar puncture should be performed in patients with uvei tis and syphilis. Syphilic uveitis can be cured with proper treatment, even in patients with AIDS. A11 In tertiary syphilis, serum FTA-ABS and MHA-TP, and CSF VDRL are positive. H owever, serum VDRL may be negative in a patient with tertiary syphilis. A12 Blunt trauma is not likely to incite sympathetic ophthalmia. A13 In an older patient with a nonresponsive uveitis, vitritis, subretinal infil trates, and neurological signs, a largecell, non-Hodgkin lymphoma should be suspected. A CT scan or MRI of the head wou ld be the first important step in this patient's management. CSF fluid analysis or vitreous biopsy can confirm the diagnosis. Sarcoidosis, tuberculosis, or giant cell arteritis are possible; however, it is important to rule out the possibility of a large-cell lymphoma. A14 The Bacillus cereus organism is frequently (26%-46%) found in traumatic endo phthalmitis and may cause a fulminant endophthalmitis. A15 Bleb-related endophthalmitis may present months or years after glaucoma filt ering surgery. The visual prognosis is poor. Haemophilus influenzae and a Streptococcus species are common causative agents of blebrelated endophthalmitis. In eyes with endophthalmitis after cataract surgery or secondary intraocular lens implantation, a preoperative visual acuity of better than light perception argue s for a "tap and inject" rather than vitrectomy Endophthalmitis Vitrectomy Study). However, the EVS conclusion is not applicable to patients with blebrelated endophthalmitis, and a vitrectomy is usually necessary in severe bleb-re lated endophthalmitis. A16 In one study, perinatal transmission was the least frequent (1%) mode of tra nsmission of HIV. Sexual intercourse (70), intravenous drug abuse (27), and blood transfusions (2%-3%) we re more common causes of HIV transmission. A17 Anterior uveitis is the most common morphologic form of uveitis worldwide. M en and women appear to be equally affected. The paediatric age group has the lowest incidence and prevalen ce and the age over 65 years of age has the highest prevalence and incidence. A18 Cataract may occur after intravitreal injection in phakic patient, especiall y if multiple injections are given. Ocular hypertension may occur in 25% of patients requiring topical therapy, but up to 10% (same book 1%) may need surgical treatment of their glaucoma. Patients with CME may benefit from in travitreal corticosteroid injection, but the effect rarely last more than 6 months. A19 Yanoff. Intravitreal triamcinolone. Approximately a quarter of patients had a rise in intraocular pressure requiring therapy, including 1% of patients who required glaucoma surgery. Patie nts receiving intravitreal

triamcinolone have approximately a 40% risk of developing increased intraocular pressure and 100% chance of developing a cataract. A20 Late-onset, pauciarticular, JRA/JIA-associated iridocyclitis is usually acut e, recurrent, and symptomatic. It has somewhat better long term visual prognosis than ANA-positive, early onset pa uciarticular group. 75% of patients are HLA-B27 positive. Many go on to develop seronegative spondyloarthropathy lat er in life. Older boys are most often affected. A21 Behçet, necrotizing scleritis, SO, chronic recurrent VKH are considered absolu te indications for early institution of immunomodulatory therapy. A22 APMPPE may be associated with cerebral angiitis which may be fatal if untrea ted with IV corticosteroids. A23 A patient with bilateral granulomatous panuveitis whose workup revealed a po sitive PPD skin test would require treatment with the following medications, isoniazid, rifampicin, pyrazin amide, and corticosteroid. A24 Sarcoidosis, MS, birdshot RC, and Eales disease are associated with retinal vasculitis affecting venules. SLE, PAN, and necrotizing herpetic retinitis are associated with retinal vasculitis p rimarily affecting arterioles. A25 The most common cause of hypopyon AU in USA is HLA-B27 associated disease. I diopathic AU will not typically result in hypopyon. A26 Fuchs heterochromic iridocyclitis. It is chronic and associated with catarac t in 50% and glaucoma in 60% of patients. A27 Yanoff. Cataract is seen in over 80% and elevated IOP in 25 60% of patients, a rising either from the inflammation or from corticosteroid therapy. Posterior subcapsular cataracts are most common, and patients tend to tolerate cataract extraction without much difficulty. Glaucoma develops in 6.3 59% of patients, and 25 60% of those individuals require a filtering procedure during the course of treatment. Most i mportantly, the complications from chronic inflammation in FHI do not usually include posterior synechiae or cystoi d macular edema, unlike other anterior uveitis entities. Fine, stellate keratic precipitates can be seen scatt ered over the entire endothelial surface; but all types of precipitates, including the granulomatous mutton-fat type, have b een described. Bilateral disease may be discerned by subtle signs, such as loss of detail on the iris surface, an d by the more specific sign of diffuse stellate keratic precipitates A28 p-ANCA is associated with PAN, relapsing poly chondritis and renal vasculiti s.

A29 MEWDS. Recurrences are uncommon, occurring in 10-15% of patients, and bilate ral disease is rare. The prognosis is excellent, with vision recovering completely in 2-10 weeks without treatment. A30 Granular vs fulminant appearance in CMV retinitis is a function of retinal t hickness. Peripheral CMV retinitis appears granular. A31 Extremely low CD4 counts, fewer than 50 cells/mm3, are associated with CMV r etinitis. A32 A PPD skin test is imperative prior to therapy with infliximab because milit ary or disseminated TB may occur in patient treated with infliximab who have latent undetected TB. F. . GLAUCOMA GLAUCOMA A1 Average central corneal thickness ranges between 536 and 544 .m, depending on the type of pachymeter used.

A2 In AGIS, the Advanced Glaucoma Intervention Study, patients had significantly better outcomes if their IOP was controlled below 18 mm Hg at all visits. A3 In the CNTG, Collaborative Normal-Tension Glaucoma Treatment Study, progressi on was reduced by nearly threefold by a reduction in IOP of 30% (The target pressure set in CNTG was 30%) . The Collaborative NormalTension Glaucoma Study (CNTGS) found that reducing IOP by greater than 30% reduc ed the rate of visual field progression from 35% to 12%, confirming a clear role of IOP in this disease. A4 In the GLT, the Glaucoma Laser Trial, the group treated with laser, compared to the group treated with medical therapy, showed nearly equivalent outcomes (The GLT found that both arms of the study did equally well). A5 A feature associated with exfoliation syndrome are: a. Spontaneous lens dislocation. b. Earlier cataract formation. c. Higher incidence of vitreous loss during cataract surgery. d. Volatile IOPs. e. Narrow anterior chamber angles. A6 In pigmentary dispersion syndrome with elevated IOP. a. Patients with pigmentary dispersion syndrome are usually myopic with increase d pigmentation of the trabecular meshwork. As a result, they require less energy with laser trabeculop lasty and have a higher incidence of hypotony maculopathy. b. Laser peripheral iridectomy may flatten the peripheral iris contour but will not deepen the anterior chamber in this condition. A7 Pooled data from large epidemiologic studies indicate that the mean IOP is ap proximately 16 mm Hg, with a standard deviation of 3 mm Hg. IOP, however, has a non-gaussian distribution wit h a skew toward higher pressures, especially in individuals over age 40. For the population as a whole, no clear l ine exists between safe and unsafe IOP. Screening for glaucoma based solely on IOP >21 mm Hg may miss up to half of the people with glaucoma in the screened population. A8 Medications have limited long-term value for congenital and infantile glaucom

a in most cases, and the preferred therapy is surgical. The initial procedures of choice are goniotomy or trabeculo tomy if the cornea is clear, and trabeculotomy ab externo if the cornea is hazy. Brimonidine should not be used i n infants and topical beta blockers should be used cautiously. A9 Use of brimonidine in infants and young children should be avoided due to an increased risk of somnolence, hypotension, seizures, apnea, and serious derangements of neurotransmitters in t he CNS, presumably due to increased CNS penetration of the drug. A10 Automated perimetry uses a variety of "staircase" strategies to estimate the threshold sensitivity. The strategy chosen will affect the speed and reproducibility of the visual field produced. A11 The prevalence of glaucoma in the black population is estimated to be 3 to 6 times higher than in the white population. A12 The 6 described POAG loci all appear to be inherited in an autosomal dominan t pattern. A13 It is surprising that the 3 genes identified for primary congenital glaucoma are estimated to account for 75% of all known forms of the disease. A14 The data are the least compelling for diabetes being a risk factor for highpressure POAG. A15 Yanoff. Population-based studies fail to consistently show an association be tween diabetes and POAG, although two studies on Hispanics and Whites in the United States showed a weak association between the two conditions A16 Although initial success rates are high after laser trabeculoplasty, the suc cess rate declines over time to approximately 50% after 3-5 years and 30% after 10 years. A17 Many patients require more than one glaucoma medication to control their dis ease. Adverse reactions may occur with any glaucoma medication. These reactions resolve when the medication is discontinued. Usually an alternative medication or laser trabeculoplasty can be used to successfully trea t the patient. A18 Use of mitomycin C during filtration surgery has been associated with persis tent ocular hypotony, bleb leaks, and infections. The blebs are often less vascular than the surrounding tissues t hat are not treated with mitomycin C.

A19 The usual primary glaucoma surgery is trabeculectomy. If patients have faile d prior trabeculectomy, have inadequate conjunctiva (eg, due to extensive prior ocular surgery), or have a po or prognosis for successful trabeculectomy (eg, active uveitis, neovascular glaucoma, ICE syndrome), they ma y be candidates for drainage implant surgery. A20 Cyclophotocoagulation may be associated with visual loss, hypotony, pain, in flammation, cystoid macular edema, hemorrhage, and even phthisis bulbi. Sympathetic ophthalmia is a rare but serious complication. A21 The IOP in eyes with low scleral rigidity may be underestimated with applana tion tonometry, although this effect is more pronounced when techniques of indentation tonometry are used. A22 ICE syndrome has an abnormal corneal endothelium that allows for the PAS to extend anterior to Schwalbe's line. Neovascular glaucoma and Fuchs heterochromic iridocyclitis have a normal c orneal endothelium. In AxenfeldRieger syndrome, Schwalbe's line is displaced anteriorly; however, the PAS are l imited to this anterior displacement. A23 Elevated episcleral venous pressure may be associated with: a. Sturge- Weber syndrome. b. Facial cutaneous angiomas, such as nevus flammeus. c. Thyroid ophthalmopathy. d. Dilation of episcleral vessels. e. Foreshortening of the conjunctival fornices not associated with elevated EVP. A24 The 6 described POAG loci all appear to be inherited in an autosomal dominan t pattern. A25 It is surprising that the 3 genes identified for primary congenital glaucoma are estimated to account for 75% of all known forms of the disease. A26 OHTS. Although all patients who were treated initially receive 1 medication and the target IOP lowering 20%, approximately 50% require at least 2 medications to maintain the target at 5 yea rs. The average IOP lowering was 22.5% and the treated group had significantly lower conversion rates to glaucoma . G. . LENS LENS A1 Major epidemiologic studies confirm an increased prevalence of cataract in wo men. A2 Current smoking, white race, and lower education are all risk factors for nuc lear opacification. A3 A small capsulorrhexis leaves more anterior capsule, which leads to greater r esistance in nuclear rotation for quadrant removal techniques, as well as increased difficulty in chopping techniq ues. The zonular laxity allows the larger anterior capsule remaining to contract to a much smaller opening. YAG ant erior capsular relaxing incisions can be made in the early postoperative period to reduce the anterior capsular ph imosis, which can further reduce the zonular integrity. A4 With mature lenses, laser interferometry is incapable of passing through the lens to allow accurate responses. However, patients with mature lenses and normal macular function may be able to discriminate entoptic phenomena, color, and even the placement of 2 lights separated from one another. A5 The lens may be dislocated into the posterior chamber without loss of visual

function. Couching of the lens was an accepted method of restoring vision before lens removal was attempted first by D aviel in the 17th century. If the posterior dislocation of the lens is accompanied by inflammation or disruption o f the lens capsule, removal may be indicated by a pars plana route. A6 Recent studies have shown a higher incidence of endophthalmitis with clear co rneal incisional though the mechanism has not been demonstrated. Both poorly constructed wounds and wound bu rns can lead to a leaky wound, which many suspect may be the cause of increased infection rates. Recent experimental studies have demonstrated that transient reduction in postoperative IOP may also result in po or wound apposition in clear corneal incisions, with potential for fluid flow across the cornea and into the anterior chamber, with the attendant risk for endophthalmitis. A7 Postoperative ptosis don t eliminated with topical anesthesia because still may occur due to trauma from lid speculum or postoperative ocular inflammation which weaken levator muscle. A8 Loss of vitreous is not a problem for the eye; vitreous traction is. When rem oving vitreous, the goal is to prevent any possibility of traction by removing sufficient vitreous to keep it away from oth er intraocular structures such as the IOL or away from the wound. Therefore, vitrectomy is not complete until all vitreous is removed anterior to the posterior capsule, thus ensuring less risk of traction. This is the best way to decrease t he chances of postoperative ME. A9 Early detection of capsular rupture is critical to the satisfactory resolutio n of this unexpected occurrence. As soon as the surgeon notes a rupture or suspects one, the first thing to do is stop worki ng and freezes the action. This prevents further trauma to the capsule and allows for a calm assessment of the s ituation before proceeding any further. Without removing the phaco handpiece, viscoelastic can be instilled thr ough the paracentesis port to freeze the action; the phaco is then removed. The next step is determined by the extent of the rupture, how much nucleus is left to be removed, and the presence or absence of vitreous. A10 Early morning Pachymetry is better predictor for postoperative endothelial f unction than specular microscope. Since the endothelium is under greater stress in the early morning. Normal corneal pachymetry measurements obtained in the early morning suggest that the cornea will probably remain clear following cataract surgery.

A11 Occlusion of the phaco tip reduces or interrupts fluid evacuation through th e phaco handpiece. This results in an increased buildup of heat within the handpiece and a transfer of thermal ener gy to the incision. Use of lower ultrasound power reduces heat buildup. Aspiration of viscoelastic, use of more e asily aspirated cohesive viscoelastics, higher aspiration flow rates and vacuum levels, and a loose fit b etween the phaco handpiece and the incision all contribute to a more efficient fluid flow through the handpiece and /or the incision, reducing the transfer of thermal energy. A12 Posterior infusion syndrome causes shallowing of the anterior chamber during cataract surgery, not in the postoperative period. This rare complication typically occurs during hydrodissec tion, when fluid may be misdirected into the vitreous cavity, resulting in forward displacement of the lens. Wound l eakage and suprachoroidal hemorrhage may result in a flat chamber during or following surgery. Suprachoroi dal effusion, pupillary block, and ciliary block with aqueous misdirection occur in the postoperative period. Supra choroidal effusion is often associated with hypotony and may be associated with a wound leak. Pupillary and ciliary blo ck, as well as suprachoroidal hemorrhage, is often associated with normal or elevated IOP. A13 If the capsulorrhexis tear starts to extend too far peripherally, the flap c an sometimes be salvaged and the tear brought more centrally. First, the surgeon should check for positive vitreous pr essure associated with forward displacement of the lens. This may be caused by the capsulotomy instrument, the surgeon's fingers, or the lid speculum pressing against the globe; and it can be corrected. Refilling the ante rior chamber with viscoelastic, and/or inserting a second instrument (such as an iris spatula) through the paracentesis to press posteriorly on the lens may help reduce forward displacement of the lens and allow for redirection of the ca psular tear. Using the bent cystotome needle to redirect the tear centrally may also be helpful, as this instrument ca uses minimal wound distortion when inserted in the eye, and it can create sharp changes in the direction of the tea r over very short distances. Often, several of these just-described maneuvers need to be employed in order to redire ct and salvage the capsulorrhexis. A14 The surgeon must distinguish carefully between an actual cataract and appare nt lens opacity due to fibrin coating on the anterior lens capsule of an otherwise clear lens. Cataracts shoul d never be extracted through a corneal laceration; this procedure would cause additional injury to an already t raumatized corneal endothelium. Traumatic cataracts can develop long after the actual ocular injury. The use of phacoemulsification through a limbal incision would not be preferred when the view through the cornea is inadequate. In this situation, cataract surgery should either be postponed or, if necessary, combined with penetrating keratopla sty. When the lens is subluxated as a result of zonular dehiscence, a pars plana approach is preferred. Insertion of an IOL as part of a combined cataract extraction and corneal laceration repair procedure is controversial. Preoperativ e biometry is usually impossible and/or inaccurate, which can lead to significant anisometropia. Zonular status,

as well as capsular integrity, may not be certain. A15 Systemic .1A antagonists (for treatment of benign prostatic hypertrophy) may cause intraoperative floppy iris syndrome (IFIS) by competitively bind to the postsynaptic nerve endings of the i ris dilator muscle for long periods. Atrophy of the iris dilator results in intraoperative billowing of the iris and progressive miosis. H. . RETINA AND VITREOUS RETINA AND VITREOUS A1 The Central Vein Occlusion Study (CVOS) showed a benefit from panretinal phot ocoagulation when neovascularization occurred (not immediately) but no benefit from gird photocoag ulation for macular edema in older patients. In younger patients, there was a trend toward benefit from grid photoc oagulation, but this was not statistically significant. A2 The Central Vein Occlusion Study Group (CVOS). Thus, even though grid laser t reatment in the macula reduced angiographic evidence of macular edema, it yielded no benefit in improving visua l acuity. However, there was a trend in the CVOS that grid laser treatment might be beneficial in improving visual ac uity for eyes with macular edema in younger patients. A3 Although the Early Treatment Diabetic Retinopathy Study (ETDRS) reported that focal laser photocoagulation should be applied in patients with clinically significant macular edema even if the vis ion is 20/20, this was not mandated, and patients can be observed closely, especially if most of the edema is in the fove al avascular zone. Macular edema in the ETDRS was defined by clinical examination, not imaging methods like fluoresc ein angiography and optical coherence tomography (OCT). The DCCT findings, although correct, apply only to p atients with type I diabetes. The Diabetic Retinopathy Study (DRS) reported that immediate panretinal photocoagula tion should be applied with highrisk proliferative diabetic retinopathy. A4 The patient suffered a branch vein occlusion. The Branch Vein Occlusion Study (BVOS) reported that grid photocoagulation should be applied if macular edema is present for more than 3 m onths and no retinal hemorrhages would prevent laser treatment. PRP should be applied when neovascularization occ urs-not if ischemia (>5 disc diameters of capillary nonperfusion) is present. Because patients with macular n on-perfusion were excluded from the BVOS, the study findings do not apply. Embolic workups are not required in branc h vein occlusion. A5 The Studies of Ocular Complications of AIDS (SOCA) trials reported that patie nts treated with ganciclovir had a 79% higher mortality than those assigned to foscarnet, with both drugs equal in cont rolling CMV retinitis and with tolerable side effects. Retinitis progression was best controlled with combination treatme nt. Color photographs are the gold standard to follow for disease progression.

A6 Fundus albipunctatus is a form of congenital stationary night blindness chara cterized by striking yellow-white dots in the posterior pole. Patients have normal visual acuity and color vision. The rod ERG is minimal but normalizes after patients spend several hours in a dark environment. It is non-progressive and sh ould be differentiated from retinitis punctata albescens, which is a variant of retinitis pigmentosa. Nyctalopia is a feature of Fundus albipunctatus. A7 Reduction and delay of cone (or rod) b-waves signifies damage to cells diffus ely throughout the retina. This can occur in dystrophic disease, such as retinitis pigmentosa, in widespread ischemi c disorders such as central vein occlusion, and in diffuse infections or inflammations such as syphilis. Diffuse cone dysfunction is diagnostic of cone dystrophy. Diseases such as sector retinitis pigmentosa, which destroys only foc al regions of retina, may reduce bwave amplitude, but the shape and timing of the waveforms (being generated by th e remaining healthy areas of retina) is usually normal. A8 A loss of red-sensitive pigment results in a red-green color confusion defect and also makes the longer wavelength portion of the spectrum appear darker than normal. Because cone photo receptors are not actually missing, acuity and photosensitivity are normal. A9 A subnormal EOG in the setting of a normal ERG is a consistent, classic findi ng in Best disease. It can also be seen occasionally with the various forms of adult-onset pattern dystrophies. In retin itis pigmentosa, both the ERG and EOG are subnormal. In rubella retinopathy, the RPE is diffusely affected but the EOG is normal. A10 The vast majority of cases of Stargardt disease are autosomal recessive. Bes t vitelliform dystrophy, familial drusen, pattern macular dystrophies, Sorsby macular dystrophy are typically inhe rited in autosomal dominant fashion. A11 A constant diagnostic feature of congenital X-linked retinoschisis is foveal schisis. There is 100% penetrance for foveal schisis in this disorder, even in young children. The a-wave is typic ally normal whereas the b-wave is reduced, reflecting the Muller cell dysfunction thought to play a role in pathog enesis. Fluorescein leakage is absent in foveal schisis. Peripheral retinoschisis and pigmentary changes are each pres ent in approximately half of affected patients. A12 Monitoring for hydroxychloroquine retinopathy. a. The earliest signs of toxicity include relative paracentral scotomata and sub tle paracentral macular pigment granularity. b. The "safe" daily dose is considered to be less than 6.5 mg/kg/day (based on l ean body weight). c. Baseline evaluation should include complete ophthalmologic examination, fundu s photographs, and evaluation of the central visual field with a red test object. d. In addition to higher daily and cumulative doses, other risk factors for reti nal toxicity include obesity, kidney or liver disease, older age, and possibly concomitant retinal disease. e. Although the incidence of retinal toxicity from hydroxychloroquine is very lo w in the United States, it is of serious concern because associated visual loss rarely recovers and may even prog

ress after the drug is discontinued. Annual screening examinations are recommended for everyone in high er risk categories (including all patients with more than 5 years of usage). A13 Pigmentation in retinitis pigmentosa (RP) is variable, and many patients hav e few or no bone spicules. Rhodopsin gene abnormalities account for only about 30% of dominant RP, and most recessive RP has not been genetically defined. From a clinical standpoint, the ERG is the most critical me asure because it documents the diffuse photoreceptor damage that defines the group of hereditary dystrophies th at we call RP. Most RP patients have mild tritan (blue-yellow) color deficiency. Small tubular fields are a char acteristic late finding in RP, but they are not pathognomonic, and many younger patients still have large areas of periphera l vision. A14 Fifty percent of rhegmatogenous retinal detachments associated with blunt tr auma in young eyes is found within 8 months. Young eyes rarely develop an acute rhegmatogenous retinal detac hment following blunt trauma because their vitreous has not yet undergone syneresis. Therefore, the vitreous provides an internal tamponade. Over several months, however, the vitreous over a tear may liquefy, permitting f luid to pass through the break to detach the retina. A15 The Joint Statement recommends that infants meeting any of these criteria un dergo at least 2 screening examinations for retinopathy of prematurity. a. A birth weight less than 1500 grams. b. A gestational age of 28 weeks or less. c. A birth weight between 1500 and 2000 grams and an unstable clinical course. A16 Scatter laser treatment is indicated in patients with high-risk PDR. If a ca taract is present, the ideal timing for laser application is 1-2 months pre-cataract extraction to allow the proliferati ve changes time to respond. A17 PIC PICis a bilateral condition that typically affects young, otherwise healthy, wom en who have a mild to moderate degree of myopia. Choroidal neovascularization remains a major cause of visual l oss in affected individuals. A18 DUSN, although rare, is an important disease to consider, as it is a treatab le cause of severe visual loss that often affects children. If left untreated, it will lead to widespread RPE disrup tion and is frequently mistaken for "unilateral retinitis pigmentosa." The condition has been described in almost ev ery region of the world and is not associated with any specific travel history.

A19 Pneumatic retinopexy Pneumatic retinopexyworks by tamponade of causative breaks and not on buoyant fo rces on the retina itself. Chronic subretinal fluid typically has delayed resorption, and pneumatic procedures have a poorer success rate in this setting. Chronic detachments are a relative contraindication for pn eumatic retinopexy. A20 Typically, retinal veins are dilated with both CRVO and carotid artery occlu sive disease, but often they are tortuous only in CRVO. Ophthalmodynamometry measures the retinal artery pressure , which is normal in CRVO and low in carotid artery occlusive disease. A21 is characterized by, enlargement of the physiologic Multiple evanescent whit e dot syndrome (MEWDS) Multiple evanescent white dot syndrome (MEWDS) blind spot on visual field testing, typically presents with unilateral photopsia s and loss of vision in young females with myopia, absence of cell in the anterior chamber, granular appearance of the fove a. The hyperfluorescent spots in MEWDS are actually wreathlike clusters of smaller hyperfluorescent dots and not individual spots arranged in a wreathlike configuration around the fovea. A22 In a randomized, controlled clinical trial, pneumatic retinopexy provided sl ightly better visual outcome than scleral buckle in patients with macula-involving rhegmatogenous retinal detachme nts of less than 14-day duration. Visual acuity outcome was slightly superior in patients with macula-involving rh egmatogenous retinal detachments of less than 14-day duration who underwent pneumatic retinopexy than in those patie nts who underwent scleral buckling primarily. Only patients with a causative break(s) in the superior two thirds of the retina were included in the study. Anatomic success rates were slightly greater in patients undergoing prima ry scleral buckle, but visual outcome was not affected in patients who underwent unsuccessful pneumatic retinopexy and subsequently underwent scleral buckle procedure. A23 No evidence exists that APMPPE responds to systemic steroid therapy. APMPPE, although typically bilateral, may occur in one eye or be highly asymmetric. Typically a monophasic disease, a recurrent or relentless course may occur and has sometimes been termed "ampiginous choroidopathy." A24 Retinal dialysis is usually treated in phakic patients even when asymptomati c. Atrophic holes and operculated tears are treated only in special circumstances. A25 The visual prognosis is generally good after treatment with pars plana vitre ctomy and removal of the retained lens fragments. In eyes with medium to large amounts of retained lens fragments, marked intraocular inflammation is common; secondary glaucoma is also relatively common. Retinal detachment is less common but has been reported in approximately 15% of these eyes in large published series. A26 Tobacco dust, also known as Shafer's sign, is manifested by small clumps of pigmented cells in the vitreous and is practically diagnostic of rhegmatogenous retinal detachment. Tractional r etinal detachments nearly always have a concave surface that is smooth, rather than corrugated; they almost never extend to the ora serrata. Sickle cell retinopathy is a well-known cause of tractional retinal detachment.

A27 In treating CNV associated with ocular histoplasmosis, the ophthalmologist c an decrease the risk of recurrent CNV by attaining a uniform white intensity of the area of photocoagulation at le ast as great as the minimal intensity standard published by the Macular Photocoagulation Study (MPS) and covering the entire lesion with laser treatment. Only one prospective trial compared one laser wavelength to another w hen treating CNV: the MPS subfoveal trials of CNV associated with AMD. In these trials, wavelength was not shown to affect the incidence of recurrence. Although the duration of treatment should be relatively long to crea te an intense lesion without suddenly breaking through Bruch's membrane, duration has not been shown to affect the rat e of recurrence. However, failure to cover the entire lesion or to achieve a white intensity at least as great as the minimal intensity standards published by the MPS are each factors that independently increased the likelihood of devel oping persistent CNV. A28 Laser treatment of subfoveal lesions should be undertaken only in the presen ce of some classic CNV. However, occult CNV can also be present as long as there is evidence of classic CNV. Because the laser treatment should cover the entire lesion, the boundary of the entire lesion should be well demarcated so that the treating ophthalmologist can clearly recognize the extent of the CNV. Blood can be presen t as a lesion component provided that the area of CNV is greater than any areas of blood or blocked fluorescence (not from blood) or serous detachment of the RPE. A29 Diffuse hemangiomas are associated with Sturge-Weber syndrome, whereas circu mscribed ones are not. Both types are associated with serous detachments of the retina. A30 An area of neovascularization in a patient with age-related macular degenera tion shows leakage during fluorescein angiography, additional fluorescein characteristic is needed to help define the neovascularization as "classic" is the vessels are seen early in the angiogram. The additional piece o f information needed is the ability to see the vessels early in the fluorescein angiogram. Being "well defined" is n ot a necessary characteristic. Soft drusen occur in age-related macular degeneration but are not a requisite fi nding for the designation "classic." A31 The maculopathy caused by Tamoxifen is characterized by crystalline deposits and sometimes macular edema. Moderate degrees of both functional loss and anatomic degenerative change s can occur. A crystalline maculopathy may also be seen after ingestion of high doses of canthaxanthin, a c arotenoid available in health food stores used to simulate tanning. Canthaxanthine retinopathy is generally asympto matic, and the deposits resolve when the drug is stopped. I. . REFRACTIVE SURGERY REFRACTIVE SURGERY

A1 Topographic changes in keratoconus include central and inferior corneal steep ening, hemimeridional asymmetry and a high inferior-superior (I-S) number. LASIK should be avoided in keratoconu s to prevent progressive ectasia of the cornea. A2 Optical aberrations consisting of sphere and cylinder are considered lower or der (second-order) aberrations. Other optical aberrations are called higher-order aberrations. A3 In conventional non-wavefront-guided laser treatment of myopia, the central c ornea is flattened. Defocus (a secondorder aberration) is corrected at the expense of induced spherical (high-order) aberrations. A4 The following is true after conventional laser ablations for myopia (LASIK, L ASEK, or PRK). a. Lower-order (second-order) aberrations are generally reduced. b. Higher-order aberrations often increase. c. The central corneal curvature is flattened and the corneal apical radius of c urvature is increased. d. Corneal asphericity changes often accompany myopic laser ablations. A5 For the same amount of intended dioptric correction, increasing the size of t he ablation zone results in deeper treatment. Munnerlyn's approximation of the ablation depth per diopter of correc tion = diameter²/ 3, where the diameter is in mm and the depth is in .m. Thus the ablation depth is proportiona l to the square of the diameter. The depth of tissue ablation per diopter of correction is greater for wider abla tion zones. A6 For low amounts of correction (<7 diopters), the amount of tissue to remove c entrally is estimated by Munnerlyn's formula: Ablation depth in micrometers equals diopters (D) of myopia divided by 3 times the square of the optical zone (mm). In this case, 3 D myopia, 6 mm optical zone. . 3 D / 3 multiplied by (6 mm) ² = 1 X 36 = 36 .m. A7 Contact lens wear can change the shape of the cornea and affect refractive er ror. In extreme cases, it can significantly distort the corneal curvature (a condition called corneal warpage) . Removal of contact lenses prior to the refractive surgery evaluation and surgery is important to allow the corneal curvature to return to normal. While recommendations vary, soft contact lenses are generally discontinued for at leas t 3-14 days and rigid gas-permeable contact lenses for 14-21 days. A stable refraction and a normal-appearing cornea l curvature are important prior to any refractive surgery. Repeat examinations after discontinuation of contact len ses should be performed in patients who do not have stable refractions and normal corneal curvatures. A8 Patients under age 35 are generally not interested in monovision. Success wit h monovision in the past indicates that the patient is likely to have success in the future. Ocular dominance is tested, because most patients are happiest with the nondominant eye corrected for near. However, monovision should be demon strated in trial spectacles and/or contact lenses to assess the patient's preoperative satisfaction with thi s approach. Both low and high myopes can enjoy successful monovision. A9 LASIK is generally used to correct from -10 to +4 D and PRK from -8 to +4 D. Unlike LASIK, PRK for higher levels of myopia is associated with a greater incidence of postoperative corneal haze. Con sequently, most surgeons prefer

LASIK to PRK for higher levels of myopia (above -6 D). If LASIK is contraindicat ed for reasons such as insufficient corneal thickness, some surgeons will perform PRK with "prophylactic" mitomycin to attempt to decrease the possibility of corneal haze postoperatively. A10 Thermokeratoplasty is associated with refractive regression, does not correc t astigmatism, and does not create a multifocal cornea that results in good uncorrected distance and near vision. I t is best for patients over 40 years of age. A11 The following are true about radial keratotomy (RK). a. RK is not a commonly performed refractive surgical procedure. b. Diurnal fluctuations occur after RK because the cornea is flatter upon awaken ing and steepens during waking hours. c. In the Prospective Evaluation of Radial Keratotomy (PERK) study, 43% of eyes showed a change of .1 D in the hyperopic direction between 6 months and 10 years postoperatively. d. Intersecting radial and astigmatic incisions result in irregular corneal topo graphical changes and in undesirable visual disturbances. A12 The following is true regarding arcuate keratotomy (AK) and/or limbal relaxi ng incisions (LRIs). a. AK incisions are placed in the midperipheral cornea and are helpful in treati ng Postkeratoplasty astigmatism. b. LRIs are more useful to correct astigmatism during or after phacoemulsificati on and IOL implantation. c. Coupling is a hallmark of LRIs and AK and results in concomitant steepening o f the orthogonal meridian. d. LRIs and AK correct astigmatism without inducing a substantial change of the spherical equivalent of the preoperative refraction. A13 In the previously normal cornea, all of the following are complications of i ncisional keratotomy (radial and astigmatic). a. Corneal perforation. b. Traumatic rupture of the globe through the keratotomy incision. c. Bacterial keratitis. d. Loss of best spectacle-corrected visual acuity.

e. Unlike LASIK, where corneal tissue is removed, there is no removal of corneal tissue in incisional keratotomy. While LASIK may result in corneal ectasia if excessive tissue is rem oved, corneal ectasia should not result after radial or astigmatic keratotomy. f. However, corneal incisions may cause corneal instability with diurnal fluctua tion of vision and/or continued effect of surgery over time. g. Any type of refractive surgery performed on a cornea with abnormal preoperati ve topography such as forme fruste keratoconus or pellucid marginal degeneration may be associated with unpr edictable postoperative results. A14 The mechanism for necrosis of the stroma overlying a corneal inlay made of a n impermeable substance such as glass can best be described as lack of nutrients from the underlying cor nea. A15 Intacs have been used to treat mild keratoconus and ectasia after LASIK. In addition, the ring segments have been used to correct residual myopia after LASIK where there is an insufficient residual stromal bed to perform additional excimer laser surgery. The ring segments cannot be used to treat apha kia. A16 Overcorrection in PRK has been associated with de-epithelialization using a slow and precise manual scraping. Dehydration of the stroma with any prolonged technique results in more tissue removal per pulse and therefore may cause overcorrection. A17 Compared with scanning lasers, broad-beam lasers are associated with a highe r incidence of Central islands. The dynamics of the large exposure area of broad-beam lasers are associated with central islands because the laser beam may have inhomogeneous energy. A18 The LASIK flap adheres to the cornea on the first postoperative day because of the endothelial pump. The endothelial pump creates a negative suction pressure in the corneal stroma. A19 In most cases, the surgeon can lift the original flap for at least several y ears after the original procedure. A residual stromal bed of at least 250 .m is recommended to avoid postoperative ec tasia. A20 Diffuse lamellar keratitis is a nonspecific inflammatory response to a varie ty of toxic insults and is usually noninfectious. While diffuse lamellar keratitis may be associated with primary o r enhancement LASIK procedures, it may also occur in other settings such as after corneal epithelial erosion. Diffu se lamellar keratitis is treated primarily with topical corticosteroids. Severe cases of diffuse lamellar keratitis (stage 4) are at risk for collagenase release and corneal melting. A21 Conductive keratoplasty is approved for the temporary treatment of 0.75 to 3 .00 D of hyperopia with astigmatism of 0.75 D or less. The probe heats the corneal epithelium as well as the stroma. Although a decentered corneal apex may have less than optimal results, it is not a contraindication to conductive keratoplasty. There is regression toward increasing hyperopia after conductive keratoplasty. A22 . Conductive keratoplasty Conductive keratoplasty a. CK appears to have advantages both in cost and in allowing flexible (off-labe

l) treatment patterns, since the tip can be placed anywhere on the cornea. b. CK is being evaluated for the treatment of astigmatism after LASIK or catarac t surgery. When used to correct astigmatism, the spots are applied to the flatter meridian to induce ste epening. c. More experience and long-term data will be required to determine how importan t CK will be in the refractive surgeon's armamentarium. CK is contraindicated in patients with Radial Keratotom y, keratoconus, or ectatic disorders and with significant irregular astigmatism. d. The upper limit of +3.0 D appears to be the current treatment ceiling for thi s technology and multiple applications over time or more spots do not seem to enhance or increase that lim it. Patients with a decentered apex (a significant difference between the corneal geometric apex and the corneal sighting point) may have less-optimal results with all hyperopic procedures. e. The advantage of CK over LASIK and PRK is that CK does not require making a f lap, removing corneal tissue, or treating the central visual axis. CK may be advantageous in very flat corneas, since large flap creation can be problematic, and in patients with dry eyes, because CK does not cause any significant corneal denervation. The greatest drawback of this surgery is its narrow treatme nt range. f. CK is being investigated to induce mild myopia in an otherwise emmetropic pat ient with presbyopic symptoms. g. Radiofrequency energy is used during conductive keratoplasty, and the resista nce to the current creates localized heat. A23 The following statements regarding laser thermokeratoplasty. a. The laser emits light in the infrared region. b. A high degree of treatment regression was observed. c. The laser leads to corneal shrinkage by creating a thermal gradient, in which the epithelial surface is hotter than the deeper stroma. d. Laser thermokeratoplasty is approved for the temporary treatment of 0.75 D to 2.50 D of hyperopia with less than 1.00 D of astigmatism.

A24 Potential treatment of astigmatism with conductive keratoplasty would involv e placing treatment at the flatter axis; therefore, in a patient with astigmatism of + 1.50 D at axis 180, the trea tment spots would be placed in the 90° meridian. A25 PIOLs can correct a wide range of refractive error from high myopia to high hyperopia. PIOLs are inserted over the clear lens. They have the advantage of not being dependent on corneal thickn ess or steepness. A26 The ideal candidate for a multifocal IOL has <1 D of astigmatism, has a pupi l >4.0 mm in diameter in order to fully benefit from the multifocal effect, has less concern about glare, requires bilateral surgery, and has good potential vision. A27 The following statements about clear lens extraction. a. Retinal detachment risk is 4 times higher 7 years postoperatively than 4 year s after surgery. b. Nd: YAG laser posterior capsulotomy doubles the retinal detachment risk over a 7-year period. c. The technique of clear lensectomy is significantly different in the myope tha n in the hyperope. d. The anterior chamber tends to become very deep in the myopic eye and is somew hat shallow in the hyperopic patient. e. Most retinal surgeons would prefer to have an IOL placed even when the power is near 0; the barrier of the IOL is felt to reduce the risk of retinal detachment. f. Clear lensectomy over LASIK in the very high myope is that clear lensectomy d oes not significantly alter the corneal contour. A28 A patient who is legally blind in one eye (as amblyopia with vision <20/200) should not consider refractive surgery in the better eye. Dry eyes should be effectively treated before refract ive surgery. Some patients with controlled glaucoma may be candidates for refractive surgery. A patient who has a history of retinal detachment or herpes virus may qualify for refractive surgery. A29 Ring segments are undergoing investigational studies in keratoconus. No refr active surgery procedure is approved by the FDA for keratoconus. Refractive surgery is typically contraindic ated in keratoconus because of unpredictable results and loss of vision. A30 Optimal topical IOP-lowering therapy should be determined before LASIK is co nsidered in the glaucoma patient. Because of the difficulty in interpreting IOP measurements, PRK and LAS IK should not be considered until the IOP is well controlled. Ocular hypertension is not a contraindication to LAS IK. LASIK is contraindicated in any patient with marked optic nerve cupping, visual field loss, or loss of visual ac uity. A31 The most important reason that IOL calculation is less accurate after refrac tive surgery is inability to obtain accurate central corneal power measurements. The primary difficulty in obtaining accurate IOL calculations after refractive surgery is from the inability to obtain accurate central corneal powe r measurements from our instruments, which were developed to measure curvature in "normal" eyes. A32 After laser surgery for myopia, the measured IOP is lower than the actual IO P. This can result in glaucomatous

damage despite normal IOP readings. IOP should be measured from the side of the cornea and the optic nerve should be carefully evaluated on an ongoing basis to reduce the likelihood of gl aucomatous damage going undetected. A33 Acceptable criteria for phakic PIOL includes age 21 or older, internal AC de pth at least 3.0mm, a clear lens with a low risk of cataract formation, a stable refraction, and no more than 2 D of astigmatic correction. A34 In LASIK, dry eye usually lasts 3-6 months before resolving and is most comm only manifested by fluctuating visual acuity. A35 Treatment of dry eye disease following LASIK is often multifactorial and inv olves management of meibomian gland disease, immunomodulation of the lacrimal gland, and conservation of exist ing tears. For this reason, omega-3 fatty acids such as flaxseed oil and fish oils, cyclosporine improve the quality of the tear film; and punctal occlusion preserves the existing tear film. These therapies may be given together or seque ntially to manage dry eye following LASIK. A36 Ectatic corneal disorders are the second most common reason for keratoplasty about 15%. A37 Possible risk factors for the development of ectasia following laser vision correction include: a. High myopia. b. Low preoperative corneal thickness. c. Low residual stromal thickness after ablation. d. Asymmetrical corneal steepening. A38 Large diameter microkeratome flap are associated with steep corneal curvatur e. A steep cornea protrudes higher above the suction ring and therefore more corneal surface area is exposed to the microkeratome. A39 DLK involves diffuse distribution of inflammatory cells without a focal infi ltrates associated with a minimal pain and photophobia and begins at the flap periphery. A40 There is no convincing evidence that retinal detachments are more or less fr equent after LASIK than after PRK (when the degree of preoperative myopia is controlled for). While the surgeon sh ould avoid trauma to the LASIK flap to decrease the risk of flap dehiscence and epithelial defect, retinal detachmen t surgery is no more difficult in LASIK than in PRK. A41 Although possible risk factors for post-LASIK ectasia have been identified i n literature, ectasia can develop in eyes with no currently identifiable risk factors. A computer generated diagnosis of keratoconus suspect is not

necessarily a contraindication to LASIK, as this decision should be based on the entire clinical picture. No specific test or measurement is diagnostic of a corneal ectatic disorder. A42 The combination of corneal and intraocular refractive surgery, so-called bio ptics, may ultimately allow patients at the extremes of refractive error to achieve predictable outcomes by combining the advantages of the PIOL in treating large corrections with the adjustability of a keratorefractive techniqu e. In addition, the optical quality may be improved by dividing the refractive correction between two different locations. Patients who have bioptics will still require reading glasses unless the treatment is adjusted for monovision. A43 The Prospective Evaluation of Radial Keratotomy (PERK) study. The most impor tant finding in the 10-year PERK study was the continuing longterm instability of the procedure. A hyperopic shift of 1.0 D or greater was found in 43% of eyes between 6 months and 10 years postoperatively. More worrisome was the fact that there was no evidence that the amount of hyperopic shift decreased between 5 and 10 years aft er RK 3) .iii REVIEW QUESTIONS REVIEW QUESTIONS A. . NEURO-OPHTHALMOLOGY NEURO-OPHTHALMOLOGY R1 Optic neuropathy and amblyopia may both occur in children. Several tests may help to distinguish between these entities. Brightness sense is diminished in optic neuropathy, not amblyopia. Col or vision testing helps differentiate because color vision is not affected by amblyopia but decreased with optic neuri tis. Neutral density filters would reduce visual acuity with optic neuropathy, but in amblyopia, acuity actually improves. Visual acuity tested by linear letters is reduced in both amblyopia and optic neuritis. R2 When a patient has isolated ocular myasthenia gravis for more than 2 years, t here is a less than 20% chance that he or she will develop systemic disease. R3 The levator palpebrae are innervated by a fused central nucleus. Therefore, u nilateral ptosis as a result of a nuclear lesion is not possible. Ipsilateral nuclei innervate the inferior rectus, medial rectus, inferior oblique and lateral rectus muscles. The nuclei controlling the superior rectus and superior oblique muscles have crossed projections. The superior oblique muscle is innervated by the contralateral IV nucleus. R4 Botulinum toxin produces a temporary blockade of acetylcholine release at the motor nerve terminal. This treatment has been used for nonrestrictive strabismus (not used for restrictive strabismus as thyroid disease) and for reducing muscle spasms such as in essential blepharospasm and hemifacial spasm. R5 Vertical fusion amplitudes are useful for distinguishing congenital from acqu ired fourth nerve palsies. Normal vertical fusion amplitudes are 3 to 5 PD. In congenital IV palsy, patient can develop amp litudes of 10 to 25 PD, and large vertical fusion amplitudes are an indication of a longstanding vertical deviatio n usually since childhood. If the coveruncover test shows no deviation but the alternate cover test discloses a deviati on, this means a phoria and indicates fusion amplitude. In this case the patient has a right hyperphoria of 12 PD; thi s means that the vertical fusion amplitude is at least 12 PD (much more than normal), indicating a long-standing deviation.

Although trauma is the most common cause of acquired IV palsy, it is also (often coincidentally) the trigger that a llows congenital palsies to be manifest. In addition to increased vertical fusion amplitudes, these patients may have a head tilt to the contralateral side to reduce the hypertropia. Old photographs may be helpful in demonstrating this feature. R6 The right parietal lobe causing visual fields show a left, inferior, homonymo us hemianopsia. Acutely, ipsilateral pursuit may be affected by parietal lesions. Thus, the patient could have difficulty pur suing the optokinetic drum as it rotates to the left, and poor optokinetic nystagmus would result. Unformed visual hallucina tions typically occur with occipital lobe damage, whereas formed visual hallucinations are associated with temporal lobe l esions. R7 Positive forced ductions indicate a restrictive process that may be caused by thyroid eye disease, blow-out fractures, fat adherence syndrome, and Brown's syndrome, among others. Disorders of the neuromu scular junction (myasthenia) and of the muscle itself (CPEO) do not cause restriction. R8 Eaton-Lambert syndrome is thought to be an autoimmune disease similar to myas thenia gravis. Abnormalities of conduction at the neuromuscular junction lead to muscle weakness and fatigue. Oc ular manifestations are less common with Eaton-Lambert than with myasthenia. R9 Myasthenia gravis might produce ophthalmoplegia that mimics supranuclear vert ical gaze abnormalities, but the pathophysiology is at the acetylcholine receptor. Parkinson's disease, pineal re gion tumors (dorsal midbrain syndrome), and ataxia-telangiectasia all can cause supranuclear gaze palsy. R10 Hemifacial spasm is typically caused by irritation of the facial nerve as it exits the brainstem. Rarely, compression by tumor may cause this condition. R11 The presence of a relative APD without any visual loss localizes damage to t he contralateral brainstem. R12 Sympathetic innervation plays a role in the development of pigmentation of t he iris. Congenital interruption of the sympathetics to one eye will result in the ipsilateral iris having less pigment than the fellow eye. R13 One dilated pupil does not preclude the detection of an afferent pupillary d efect. Because the pupillary efferents are equal bilaterally, the consensual light response can be used instead of the dire ct response to detect an afferent defect. With damage to the right optic nerve, both pupils will dilate when the light swi ngs from left to right. R14 Caloric Vestibular Stimulation.

Awake (Jerk nystagmus-FAST phase) Comatose (Tonic deviation-no nystagmus)

unilateral Bilateral Unilateral

Bilateral Cold water Opposite Upward Same Downward

Warm water Same downward Opposite Upward

R15 Tensilon prolongs the effect of acetylcholine in the synaptic cleft, causing a cholinergic crisis (sweating, nausea and vomiting, salivation, fever). Atropine blocks acetylcholine receptor sites o n the postsynaptic terminal. R16 The patient now has an optic neuropathy. Patients with thyroid dysfunction c an develop or experience worsening of ophthalmopathy at any time after appropriate systemic treatment. Thus, the first consideration should be to rule out optic nerve compression by enlarged extraocular muscles. CT scan with direct cor onal views would be the best test. Visual fields would also be helpful. Once the diagnosis has been established, a short course of prednisone can be used to decrease optic nerve compression until definitive treatment (orbital rad iation, orbital decompression) can be instituted. Thyroid function tests to ascertain appropriate systemic trea tment should be obtained but are not the first consideration. One should never simply observe a patient with an unexplain ed optic neuropathy. R17 Compressive optic neuropathy associated with thyroid ophthalmopathy should b e accompanied by the typical signs of proptosis, chemosis, and lid retraction. R18 Patients with thyroid orbitopathy may develop proptosis and non-fatigable op hthalmoplegia from extraocular muscle involvement. Myasthenia gravis is associated with dysthyroidism, and thus fatigable ptosis could occur. R19 Individuals with von Recklinghausen's disease (neurofibromatosis) are prone to CNS tumors, including optic nerve glioma and meningioma, chiasmal glioma, and acoustic schwannoma. Bournevil le's syndrome (tuberous sclerosis) is characterized by seizures, mental retardation, and calcified CNS l esions ( tubers ). Individuals with SturgeWeber syndrome (encephalo-trigeminal angiomatosis) typically have intracranial c alcification associated with pial angiomatosis. Louis-Bar syndrome (ataxia-telangiectasia) is not associated with any CNS abnormalities detectable with neuroimaging. R20 Pulsations are either from a) abnormal vascular flow (arteriovenous malforma tions or carotid-cavernous sinus fistulas) or b) transmission of normal intracranial pulsations (mucocele, enceph alocoele), surgical removal of bone or sphenoid abnormalities in neurofibromatosis. Additionally, pulsation without bru its may be produced by neurofibromatosis meningoencephaloceles or as a result of the surgical removal o f the orbital roof. Capillary hemangiomas consist of endothelial cells and small vascular spaces. Although pri

marily located periocular, a significant orbital component may be causing proptosis. However, the flow through these tumo rs is not high enough to cause pulsation. R21 In multiple sclerosis Bitemporal hemianopsia and skew deviation can occur if demyelination occurs in the optic chiasm or supranuclear vertical gaze pathway, respectively. Uveitis, including i ritis, pars planitis, and retinal venous sheathing, has been reported in multiple sclerosis. R22 Uhthoff's symptom occurs with optic neuritis and is a decrease in vision wit h an increase in body temperature. Exercise or hot showers may trigger this symptom. Lhermitte's sign is the electr ic shock sensation with neck flexion and is found in patients with multiple sclerosis. A bilateral medial occipitotempora l lesion causes prosopagnosia, the inability to distinguish faces. The Riddoch phenomenon occurs in patients with cortical bl indness that is able to perceive objects in motion, but cannot see stationary objects. R23 Although migraine can cause persistent visual field deficit, it is unusual. Cerebral arteriovenous malformations and tumors can mimic migraine and cause visual field loss. Therefore, neuroimaging i s essential. Oral contraceptives can exacerbate migraine. Sumatriptan (Imitrex) is an effective form of migraine trea tment in about 80% of patients. Both of these measures would apply only if the appropriate evaluation was normal. R24 Pseudotumor cerebri is a diagnosis of exclusion. Obstructive, compressive, a nd infiltrative CNS lesions must be excluded. Papilledema, although commonly present, is a result of the increased i ntracranial pressure and is not necessary for the diagnosis. The optic nerve swelling may be unilateral or asymm etric. R25 Patients with myasthenia gravis are at risk for other autoimmune diseases, i ncluding systemic lupus erythematosus, rheumatoid arthritis, and hyperthyroidism. Thymic hyperplasia and thymomas are also more common, and patients should have a chest CT to investigate this possibility. R26 The Ischemic Optic Neuropathy Decompression Trial (IONDT) was an NIH-sponsor ed randomized, single-masked, multicenter trial comparing close observation with optic nerve sheath fenestrati on for nonarteritic anterior ischemic optic neuropathy. The study was terminated early by the Data and Safety Monitoring Com mittee. Patients in the surgery group did no better when compared with the observation group regarding improved visual acuity of three or more lines at 6 months. Approximately one third of the surgery patients had improvement in acuity, whereas over 40% of the observation patients improved. Moreover, surgery was associated with a higher ri sk of loss of three or more lines of acuity (surgery: 24%, observation: 12%). The IONDT conclusively states that opti c nerve sheath decompression is not effective. R27 The supranuclear control of vertical saccades originates in the frontal eye fields or in the superior colliculus. They project to neurons in the rostral interstitial nucleus of the medial longitudina l fasciculus (riMLF) and on to the nuclei of cranial nerve III and IV. The interstitial nucleus of Cajal is involved with ver tical pursuit control. The paramedian pontine reticular formation (PPRF) controls horizontal eye movements. B. . PEDIATRICS AND STRABISMUS PEDIATRICS AND STRABISMUS

R1 . Homocystinuria Homocystinuria a. Homocystinuria is an autosomal recessive inborn error of methionine metabolis m.

b. Patients exhibit elevated serum levels of methionine and homocystine. c. Lens dislocation is bilateral, with 30% occurring in infancy and 80% occurrin g by age 15. d. Normal lens zonules have a high concentration of cysteine, and deficiency res ults in abnormal, brittle zonules. e. A diet low in methionine and high in cysteine can reduce lens dislocation. f. Patients are normal at birth and develop seizures, mental retardation, and os teoporosis. The patients are usually tall with light-colored hair. R2 Congenital lacrimal fistula shows fluorescein exiting from the fistula when t he nasolacrimal duct is irrigated. This congenital anomaly represents an accessory epithelial-lined communication betwee n the lacrimal system (usually the common canaliculus or the lacrimal sac) and the skin and is a problem of surface ectoderm not neural ectoderm. If it is associated with nasolacrimal duct obstruction, there is often discharge and refl ux through the fistula. In this situation, topical antibiotics, probing of the nasolacrimal duct and excision of the fistul as are appropriate therapy. There has been no association between this condition and a fistula to the maxillary sinus. R3 . Megalocornea Megalocornea a. By 2 years of age, the cornea is approximately adult size. b. Simple megalocornea is defined as both corneas measuring greater than 13 mm i n children older than 2 years, and greater than 12 mm in infants. c. Congenital glaucoma is associated with epiphora and increased IOP and must be ruled out of the differential diagnosis. d. The most common type of megalocornea is associated with anterior megalophthal mos, an X-linked recessive disorder. R4 Peter's anomaly results from a developmental problem of faulty migration of n eural crest cells. Neural crest cells usually migrate between the surface ectoderm of the cornea and the separating le ns. As a result of faulty separation, both lens and iris may remain adherent to the central cornea, causing a central corneal opacity. Both Descemet's membrane and layers of the posterior stroma may be absent. The peripheral cornea is characteristically clear because Descemet's membrane and the endothelium are intact. In many cases, the corneal o pacity will decrease over time. R5 Corneal/limbal dermoids are hamartomatous lesions consisting of fibrofatty ti ssue, surrounded by keratinized epithelium, often containing hair follicles, sebaceous glands, and sweat glands. They are usually located at the inferotemporal limbus, and they are associated with lipid in the corneal stroma surrounding the edge of the lesion. Large dermoids can cause astigmatism and amblyopia. Excision may be difficult because they may involve Bowman's layer and corneal stroma and, as a result, cause postoperative scarring, astigma tism, and amblyopia. Some require corneal patch grafts. Corneal dermoids can be found in association with Goldenhar's syndrome, which also includes preauricular appendages, aural fistulas, maxillary or mandibular hypopl asia, hemifacial microsomia, vertebral deformities, notching of the upper eyelid, and Duane's syndrome. R6 Waardenburg's syndrome is an autosomal dominant disorder characterized by dev

elopmental anomalies of the eyelids, nasal root, and eyebrows along with heterochromia iridis, white foreloc ks, and sensorineural deafness. R7 Albinism results in bilateral loss of iris pigmentation and therefore does no t result in heterochromia. R8 When determining the etiology of congenital cataracts, it is best to first de termine whether they are unilateral or bilateral. Bilateral cataracts are often inherited in an autosomal dominant fashion. Bilate ral disease may also indicate a metabolic or systemic disease such as diabetes mellitus, galactosemia, or Lowe's syndrome. In contrast, unilateral congenital cataracts are caused by local dysgenesis and are not inherited. PHPV, anterior p olar and posterior lenticonus are commonly unilateral. R9 The optimum time to operate on a patient with bilateral dense congenital cata racts is as soon as possible, even within the first few weeks of life. The critical period of visual development is the fi rst few months of life. During this time, the visual areas of the brain are developing rapidly. Earlier treatment can result i n better visual acuity. Bilateral visually significant cataracts can cause irreversible amblyopia and sensory nystagmus. R10 Most likely be present on ophthalmic examination of this newborn child with congenital syphilis (An infant girl is born 4 weeks prematurely and manifests jaundice, an intractable rash, persistent rhin itis, pneumonia, anemia, generalized lymphadenopathy, and bony abnormalities on radiograph) is Segmental pigmentation of the retinal periphery and chorioretinitis. Some patients may manifest active chorioretinitis, in most; the only evidence of chorioretinitis is segmental pigmentation of the retinal periphery with a salt-and-pepper appearanc e to the fundus. Interstitial keratitis represents an inflammatory response to treponemal antigens and usually presents between 7 and 17 years of age. Anterior uveitis and glaucoma may develop, but these are less common manifestati ons and are usually not found in the newborn. Hutchinson's triad for syphilis includes interstitial keratitis; widely spaced, peg-shaped teeth; and deafness. Other systemic manifestations include saddle nose, saber shins, and rhagades (li near scars often found around the mouth). R11 Causes of vitreous hemorrhage in children include conditions such as pars pl anitis, juvenile X-linked retinoschisis, and trauma. R12 . Juvenile X-linked retinoschisis Juvenile X-linked retinoschisis a. Juvenile X-linked retinoschisis is characterized by cleavage of the retina at the nerve fiber layer as opposed to senile retinoschisis where cleavage is in the outer plexiform layer.

b. Because the photoreceptors are unaffected, the a-wave on the ERG is intact, b ut both the scotopic and photopic b-waves are reduced in proportion to the amount of retinal schisis. c. The EOG and dark adaptation test are normal or abnormal depending on the stag e of disease. d. The macula is involved early, showing microcysts and radiating retinal folds, but fluorescein angiography exhibits no leakage. e. Peripheral schisis usually develops later. f. Vitreous veils and strands form. If a vessel is torn with these veils, vitreo us hemorrhage results; this often is how children present. Typically, vision is reduced to the 20/50 to 20/100 level, but expressivity is variable. R13 When an infant presents with poor vision, searching nystagmus, and an appare ntly normal examination, conditions When an infant presents with poor vision, searching nystagmus, and an apparently normal examination, conditions . Achromatopsia, or rod to consider include albinism, achromatopsia, and Leber's congenital amaurosis to consider include albinism, achromatopsia, and Leber's congenital amaurosismon ochromatism, is an autosomal recessive disorder with total lack of cones, with c olor blindness that results in 20/200 vision, photophobia, and nystagmus. The fundus is usually normal in infancy and the EOG is usually normal. In albinism, the signs in infancy are often subtle and may be missed. They would in clude iris transillumination defects, hypopigmented fundus, and foveal hypoplasia. Congenital stationary night blindne ss is marked by infantile onset of night blindness but not searching nystagmus. The ERG shows normal to near-normal photopic waveform but nearly non-recordable scotopic waveform. Other diagnoses include optic nerve hypoplasia and aniridia but they are usually diagnosed by the ocular exam. R14 An infant presents for evaluation of poor vision, the following signs is LEA ST worrisome. It is the Eye-popping reflex. R15 The eye-popping reflex The eye-popping reflexis a neonatal reflex described by Perez in 1972. It involv es a pronounced widening of the palpebral fissures after an abrupt decrease in ambient illumination primarily, o r after loud noises. It is present within the first 3 weeks of life in 75% of infants born after 28 weeks' gestation. In contr ast, eye pressing, or gouging, and light gazing are abnormal behavioral mannerisms in visually-impaired children. A parad oxical pupillary response is rare but when present, it is highly suggestive of congenital stationary night blindne ss, achromatopsia, or optic nerve hypoplasia. A paradoxic response refers to an immediate constriction durin g the first 20 seconds after room lights are turned off, followed by a slow dilation after 1 minute. R16 Toxoplasmosis is caused by a protozoan with a propensity to infect the retin a and other CNS structures. It is highly prevalent in North America. Cats shed oocysts in their feces, which may remain i nfective for up to 1 year. Ingestion of food contaminated by oocysts or undercooked meat containing tissue oocysts may r esult in human infection. Maternal infection early in pregnancy results in a greater risk of transmission to the fe tus (I think false {the chance of congenital toxoplasmosis is increased if the maternal infection is in the third trimester b ut the severity of illness is greatest if the

infection occur in the first trimester. Ferris} {Rate of congenital toxoplasmosi s increases from 10% to 15% following exposure in the first trimester, to 60% following third-trimester exposure. Cong enital disease is much more severe when acquired early in pregnancy; the low rate of congenital disease with infection i n the first trimester is thought to be due to the risk of spontaneous abortion with early infection. Yanoff.}). The organism m ay produce a retinochoroiditis, which is usually bilateral and frequently involves the macula. Inactive lesions may re-ac tivate later in life, producing whitish, elevated lesions and a severe vitritis resulting in a headlight through fog appear ance on ophthalmic examination. R17 Cone dystrophy. a. Characteristics of cone dystrophies include decreased central vision, color b lindness, and photophobia, which develop within the first or second decade of life. The mode of inheritance is usually autosomal recessive; however, most familial cases are autosomal dominant. (Kanski. Most ca ses are sporadic; the remainders are AD or XL). b. Night blindness and loss of peripheral vision are uncommon even in advanced c ases, distinguishing this disorder from retinitis pigmentosa. Subjective visual complaints typically prece de macular changes on eye examination. The most common abnormality is the bull's-eye macular lesion, followe d by the salt-andpepper appearing macula with diffuse pigment stippling. The least common form is characterized by atrophy of the choriocapillaris, choroidal vessels, pigment epithelium, and phot oreceptors. Some patients may develop a pattern of degeneration that mimics closely that of Stargardt's di sease or fundus flavimaculatus. Optic atrophy, especially temporally, may occur as well. c. ERG findings of decreased amplitude on the single-flash photopic and flicker responses along with reduced flicker fusion frequency confirm the diagnosis. Ultimately, visual acuity ranges from 20/60 to 20/400 with symmetric involvement of both eyes. R18 . Neuroblastoma Neuroblastoma a. Neuroblastoma is the most frequent source of orbital metastasis in children. b. Metastases occur from the adrenals, mediastinum, and neck. c. Approximately 20% of all neuroblastoma patients exhibit ocular involvement, w hich can be the initial manifestation of the tumor. d. The mean age of presentation in orbital neuroblastoma metastasis is about 2 y ears. e. Their prognosis is very poor in general, but prognosis is considerably better in infants under 1 year of age. f. Spontaneous regression of this tumor may be seen in rare instances. R19 Neurofibromatosis.

a. Neurofibromatosis is a progressive disorder with a wide range of clinical man ifestations. b. It occurs in 1 of 3000 births. c. Café-au-lait spots appear in over 99% of patients, and if five or more spots gr eater than 0.5 cm in diameter are observed, the diagnosis is established. d. Tumors of the CNS occur in 5% to 10% of patients. e. These patients are at increased risk for other malignancies as well, includin g neurofibrosarcomas, Wilms' tumor, rhabdomyosarcoma, pheochromocytomas, and leukemia. f. Lisch nodules (melanocytic hamartomas) on the iris appear in over 90% of pati ents over the age of 6 years and are useful for establishing or excluding the diagnosis of neurofibromatosis. They are not found in the They are not found in the . normal patient population normal patient population g. Plexiform neurofibroma involving the upper eyelid may produce ptosis and is a ssociated with glaucoma in up to 50% of patients. R20 The capillary hemangiomas are characteristically , in contrast to cavernous hemangiomas, which high flow lesions high flow lesionsare hemodynamically low flow. R21 Capillary hemangiomas can cause amblyopia in young children. Visual deprivat ion can occur if the lesion is so large that the lid obscures the visual axis. Even if the lesion does not block vision, it can cause astigmatism and a blurred image. These patients need to be observed closely with refractions to ensure tha t amblyopia does not develop. R22 A child demonstrates eccentric fixation. This indicates poor vision, usually 20/200, or worse. R23 The Bruckner test The Bruckner testis a bilateral red reflex test, and if strabismus is present th e brighter reflex is in the deviated eye. This is because the light reflects from peripheral retina in the deviated eye. B ecause there is less pigment in the peripheral retina than the macula, there is more reflection of light from the pe ripheral retina of the deviated eye. R24 A child presents with both eyes in the adducted position. To best determine if this is the result of a bilateral lateral rectus palsy, you could try each of the following: a. Doll's head movements. b. Saccadic eye movements generated by an OKN drum. c. Patching one eye and testing ductions. d. By patching one eye, a patient with cross-fixation will have to abduct to see things in the temporal field. Doll's head movements generated by gentle, but rapid, turning of the baby's head can induce abduction past midline. The presence of a good saccadic abduction movement generated by an OKN drum can also indicate a well-functioning lateral rectus. If an eye in full adduction is secondary to palsy, there most likely would not be enough lateral rectus function to generate a normal sac cade. Forced duction testing identifies restriction not paresis. R25 The indication for prescribing bifocals to treat esotropia is specific. The patient must be able to fuse in the distance and be esotropic at near while wearing the full hyperopic correction. Giving a b ifocal to a patient who has a deviation at

distance despite full hyperopic correction will have no effect on the distance d eviation. This patient needs surgery. R26 Accommodative esotropia occurs from infancy to 4 years of age with hypermetr opia ranging from 2.00 to 7.00 D. Most neonates are not yet accommodating, so the esotropia is acquired when the i nfant starts to accommodate to bring vision in focus. As a result of the hyperopia, additional accommodative effort t o bring images into focus is required, and an over-convergence response is seen. Accommodative esotropias are usually hyper opic but rarely more than + 7.00 D because if they are very hyperopic they develop bilateral amblyopia and do not e ven try to accommodate. R27 An 8-year-old girl returns 1 day after bilateral lateral rectus recessions f or intermittent exotropia. She measures 8 PD of consecutive esotropia and has diplopia. Observe the child; tell the parents y ou are satisfied because this is the desired result postoperative day 1. An immediate overcorrection of 8 to 12 PD is a desirable result. Often, over the An immediate overcorrection of 8 to 12 PD is a desirable result. Often, over the . first or second postoperative week, the effect of the surgery will lessen and the eyes will straighten first or second postoperative week, the effect of the surgery will lessen and th e eyes will straighten R28 A small esotropia of 8 to 15 PD postoperatively is desirable, with 20 PD bei ng the upper limit of normal. Postoperative diplopia associated with an initial overcorrection is normal and u sually resolves by 1 to 2 weeks. In younger children (<4 years old), part-time alternate patching helps prevent the development of suppression. Patients with residual exotropia of more than 20 PD in the first postoperative week are u nlikely to improve, and many will worsen. R29 At 3 weeks postoperative exotropia surgery, the child is still 15 PD esotrop ic. The next most appropriate step is to prescribe just enough prisms to alleviate the diplopia but leave a residual esop horia to encourage divergence. If the consecutive esotropia is present only at near, one can consider a bifocal add, m iotics, or even base out prism. If the esotropia persists after 8 weeks, consider reoperation (usually a bilateral medi al rectus recession). If the patient demonstrates lateral incomitance or significant limitation of abduction, a slipp ed muscle is a possibility. R30 The patient that demonstrates convergence insufficiency. This condition is b est treated with orthoptic convergence training exercises. Base out prism therapy and pencil push-up exercises builds u p convergence amplitudes. Reading glasses would help accommodative insufficiency but not convergence insufficiency . Very few physicians, if any, advocate surgery for this problem. R31 Electromyography in Duane's retraction syndrome type I would most likely rev eal Absence of electrical activity in the left lateral rectus muscle on abduction, with paradoxical activity on adduct ion.

R32 The incidence of amblyopia in patients with Duane's retraction syndrome is o nly approximately 10%. The lid fissure narrowing is secondary to retraction of the globe with co-contraction of the med ial and lateral rectus muscles. The strabismus is generally noncomitant. A Faden procedure (posterior fixation of th e horizontal rectus muscles near the equator) may reduce the upshoot of the affected eye on adduction, as it stops ve rtical slippage of the lateral rectus muscle. R33 In general, inferior oblique surgery is indicated for overaction of 2+ or mo re. R34 DVD is present in 60% to 80% of patients with congenital esotropia. DVD is u sually bilateral and asymmetric. The etiology is unknown but appears to be associated with an early disruption of bin ocular development. Thus, high-grade stereopsis and bifoveal fixation are not seen. During times of visual inattentio n, the nonfixating eye slowly drifts up, extorts, and abducts without a corresponding hypotropia of the fellow eye on alt ernate cover testing. This is the hallmark of this disorder it does not obey Hering's law. DVD can simulate Inferior Oblique Overaction (IOOA) in side gaze when the nose acts as an occluder. The hyperdeviation in DVD is of the same amou nt in adduction, abduction, and primary position. This is in contrast to IOOA, in which the hyperdeviation i s greatest in its field of action. R35 R36 . Botulinum toxin (Botox) Botulinum toxin (Botox) a. Botox is a purified form of botulinum toxin type A, derived from the Hall str ain of Clostridium botulinum. b. It blocks neuromuscular conduction by binding to receptor sites on motor nerv e terminals interfering with the release of ACh into the synaptic cleft. c. When injected intramuscularly, Botox produces a localized chemical denervatio n muscle paralysis. The nerve ending atrophies but will re-sprout over time. d. Paralysis onset occurs in 2 days, increases in intensity over the next week, and lasts 3 months in extraocular muscles. e. Botox is indicated for blepharospasm associated with dystonia. The efficacy o f Botox in strabismus is low and surgery remains the primary treatment for most types of strabismus. f. Multiple injections may be necessary but should not exceed 200 units in 1 mon th to decrease the incidence of antibody production. g. Reported side effects include ptosis, diplopia, and spatial disorientation. T hese are, fortunately, temporary. Perforation of the globe has been reported. h. Systemic effects of Botox are not seen because a dose over 100 times greater than the normal amount is required for toxicity. R37 Latent nystagmus is characterized by all of the following statements: a. Null point in adduction. b. Fast phase to the fixing eye. c. Increases with monocular occlusion. d. Latent nystagmus increases with monocular occlusion, when the fixing eye is i n abduction (null point in adduction) and the fast phase is toward the fixing eye. It is associated with di sruption of early binocular

visual development (congenital ET, congenital monocular cataracts), not acquired strabismus (intermittent XT or accommodative ET). R38 The classic triad of spasmus nutans includes monocular or dissociated smallamplitude nystagmus, head bobbing, and torticollis. The differential diagnosis for this disorder includes patients with chiasmatic gliomas and subacute necrotizing encephalomyopathy. Features of these two disorders may include optic atrophy, irritability, vomiting, and increased intracranial pressure. Spasmus nutans should be considered only after these two disorders are excluded. Spasmus nutans usually disappears within 2 years of onset. C. . PLASTICS PLASTICS R1 The lacrimal sac is positioned anterior to the orbital septum and, therefore, does not lie within the orbit. R2 Lockwood's ligament (Lockwood's suspensory ligament) acts as a suspensory sys tem for the globe. It is the lower eyelid retractor system with contributions from intermuscular septae and Tenon's capsule. Posteriorly, it arises from fibrous attachments to the inferior side of the inferior rectus muscle and conti nues anteriorly as the capsulopalpebral fascia (lower eyelid retractors). Medial and lateral horns extend to attach to t he retinacula. The medial retinaculum attaches to the posterior lacrimal crest, and the lateral retinaculum attaches t o the lateral orbital tubercle of Whitnall. These tissues form a suspensory hammock for the globe. R3 Orbital fat. a. Orbital fat is located posterior to the orbital septum and is commonly encoun tered during eyelid surgery. The orbital fat is divided into intraconal fat (central) that is located inside of the muscle cone, and extraconal fat (peripheral) that is located outside of the muscle cone. b. The removal of extraconal orbital fat is commonly done during upper and lower eyelid blepharoplasty. Numerous fine connective tissue septae course through the orbital fat and conden se in several areas to form compartments. c. The upper eyelid has a small medial fat pad and a larger preaponeurotic fat p ad (located between the orbital septum and levator aponeurosis), which primarily is located centrally. T he orbital lobe of the lacrimal gland is located laterally and should not be removed.

d. The color of the medial or nasal upper eyelid fat is typically whiter or pale r when compared with the yellow color of the preaponeurotic, centrally and laterally located fat. e. The upper eyelid medial fat pad often moves anteriorly with aging more than t he preaponeurotic fat, which results in a bulge that is located inferior to the trochlea area of the upper me dial orbit. f. The lower eyelid has a small lateral fat pad and a larger medial fat pad. The small fat pad is located inferiorly to the lateral canthus and is separated from the large fat pad by fib rous tissue connecting the capsulopalpebral fascia and the orbital septum. g. Posteriorly, the main fat pad of the lower eyelid is divided into two fat pad s by the inferior oblique muscle. Therefore, some references will state that the lower eyelid has three fat pads i nstead of two. h. It is important to realize that this is normal fat that occupies the orbit an d is bulging from a weakening of the orbital septum. This fat may serve a role in protection of the globe and facilit ate its movement. Removal of too much fat may result in restriction of the extraocular muscles or a cicatrici al blepharoptosis. In addition, aggressive removal of fat without careful attention to hemostasis can also resul t in an orbital hemorrhage and blindness. R4 Although Whitnall's ligament attaches to the fascia on the superior and media l surfaces of the lacrimal gland; it does not pass anteriorly to the lacrimal gland. The lacrimal gland is divided into or bital and palpebral lobes by the lateral horn of the levator aponeurosis. R5 The anterior lamella consists of the skin and the orbicularis oculi (eyelid p rotractor) muscle. R6 The deep head of the pretarsal muscle (tensor tarsi muscle of Horner) encircl es the canaliculi to facilitate tear drainage. The corrugator draws the head of the brow to the nose, and the proceru s depresses the eyebrow. R7 The inferior orbital fissure is bordered medially by the maxillary bone, ante riorly by the zygomatic bone, and laterally by the greater wing of the sphenoid and is also bounded by the palatine bone. Th e anterior and posterior ethmoidal foramina are located at the junction of the ethmoid and frontal bones. R8 The carbon dioxide laser operates at a wavelength in the infrared portion of the electromagnetic spectrum. At 10.6 .m or 10600 nm, the carbon dioxide laser is not within the visible spectrum, whi ch means that it is not visible to the human eye. The laser may be visibly guided with a helium-neon (He-Ne) laser-aimi ng beam. The carbon dioxide laser is absorbed by water. The increase in temperature ultimately causes cell destructio n. The carbon dioxide laser has a high absorption coefficient with water that is inversely proportional to the absorpti on length, which is very small. Therefore, carbon dioxide has the ability to remove tissue in small increments. The laser h as wide application for orbital tumor removal. R9 Botulinum toxin type A inhibits acetylcholine release by binding to receptors of the motor nerve terminals. The denervation that results usually lasts several months. Some patients develop ant ibodies to the medication, resulting in a decreased efficacy of the botulinum toxin. Botulinum toxin does not affect the f

unction but may actually increase the level of acetylcholinesterase. In addition, there is no direct effect of botulin um toxin type A on the striated muscle. The introduction of botulinum toxin type A in medicine has a variety of applications . Examples include benign essential blepharospasm, hemifacial spasm, paralytic strabismus, spasmodic dysphonia, and torticollis. Although botulinum toxin is a potent paralytic agent, the doses used for an adult are well below the amou nt necessary for systemic toxicity. R10 Sun exposure, fair skin, family history of skin cancer, previous patient his tory of skin cancer, and red hair are all associated with increased risk of skin cancers. Having more darkly pigmented nat ural skin pigmentation is not associated with increased risk for cutaneous cancer. R11 Features most consistent with a malignant eyelid lesion include: lash loss, central ulceration, rapid growth. Although most cancers have slow, progressive growth, rapid growth does not rule out a mal ignancy. R12 Features of a keratoacanthoma include all of the following EXCEPT: a. Spontaneous resolution. b. Loss of eyelashes. c. Ulcerated crater filled with keratin. d. Rapid growth. R13 Both squamous cell carcinomas and basal cell carcinomas can have central ulc erations. A small melanoma would be least likely to have central ulceration. R14 The Hughes flap transposes a bridge flap of tarsus and conjunctiva into the defect. In addition, a full-thickness skin graft or advancement flap is used to replace the anterior lamella. A Cutler-Bear d flap is a reconstructive technique that replaces full-thickness defects of the upper eyelid with a full-thickness tissue flap from the lower eyelid on the same side. R15 Basal cell carcinoma, metastatic cancer, sebaceous cell carcinoma, and squam ous cell carcinoma are all malignant. In the malignant category of eyelid lesions, basal cell carcinoma is the most co mmonly encountered lesion. It is followed in decreasing frequency by squamous cell carcinoma, sebaceous cell carcinoma, an d metastatic eyelid lesions. R16 No lymphatic vessels or nodes are typically present within the orbit. The co njunctiva does have lymphatic vessels. R17 Xanthelasma are typically flat, yellow skin lesions, located in the dermis. Histologically, these lesions consist of foamy histiocytes. The majority of patients do not have an associated hyperlipid emic condition (associated with systemic hyperlipidemic conditions in approximately 25% of patients). Erdheim-Ch ester disease is a multisystem disease with lipogranuloma formation in the liver, heart, kidneys, and bones. Hi stologically, these lipogranulomas

contain histiocytes, Touton giant cells, lymphocytes, and plasma cells. Ophthalm ic manifestations of this disease can include proptosis and xanthelasma-like skin lesions. R18 Ankyloblepharon filiforme adnatum is a form of ankyloblepharon in which the eyelid margins are connected by thin strands of tissue. R19 The most common form of blepharoptosis is: involutional blepharoptosis (apon eurotic ptosis). R20 In myogenic congenital ptosis, the levator complex (in the ptotic eye) is hi stologically different from normal levator complex with decreased muscle fibers and fatty infiltrates. R21 Blepharoplasty. a. Dermatochalasis is a term used to describe redundant eyelid skin of the upper or lower eyelids. Steatoblepharon describes bulging orbital fat that commonly occurs with aging an d is often removed during blepharoplasty. This fat tends to bulge forward with the aging process and may b e related to a weakening or stretching of the orbital septum. b. A retroblepharoplasty (transconjunctival blepharoplasty) or blepharoplasty vi a the transconjunctival approach is used in lower eyelid blepharoplasty, NOT upper eyelid blepharoplasty . The advantage of this approach is to avoid a visible scar on the skin of the eyelid. One disadvantage of this approach is that it does not allow for skin removal and is best used in patients with significant lo wer eyelid steatoblepharon without coexisting significant dermatochalasis. c. Blepharoplasty of the upper eyelid is typically done using an eyelid crease i ncision. When the skin is reapproximated, the suture needle should be passed into the fascia located poste rior to the orbicularis. This step accentuates the crease fixation and allows the healed incision to be hidden in the upper eyelid crease. d. The inferior oblique muscle originates at the lacrimal spine located on the m edial inferior orbital rim. The origin or muscle belly of the inferior oblique can be injured easily during lowe r eyelid blepharoplasty when the eyelid fat pad is removed. e. Lower eyelid retraction is a common complication of lower eyelid blepharoplas ty. Although the mechanism is unknown, the retraction may be related to scarring of the lower eyelid retrac tors and/or excessive removal of skin or muscle proportional to the eyelid's horizontal laxity. R22 Numerous materials have been described for use in frontalis suspension. Thes e materials include donor fascia lata, autogenous fascia lata, temporalis fascia, supramid, silicone, and Gore-Tex. An absorbable suture such as Vicryl would not be effective in suspending the eyelid for the long term. R23 The osteotomy site created at the time of a DCR or CDCR is at the level of t he middle turbinate. This site is most often within 10 mm of the cribriform plate. Typically, the lacrimal sac is divid ed and secured to the nasal mucosa. This division allows an open outlet from the common opening into the nasal cavity. Th e valve of Hasner is clinically significant more often in patients with a congenital tearing problem. This valve is located at the distal end of the nasolacrimal duct under the inferior turbinate. R24 The two most common causes of DCR failure are obstruction at the common cana

liculus and obstruction at the bony ostomy site. Recurrent obstruction can result in recurrent infection and dacryol ith formation. Unsuspected tumors may also cause recurrent obstruction but are not the major cause of DCR failure. R25 The most frequently seen primary malignant tumor of the lacrimal sac is squa mous cell carcinoma. The second most common primary lacrimal sac malignancies are lymphomas. These are treated by rad iotherapy after biopsy and systemic evaluation have been completed. Complete excision is not usually necess ary. R26 The dye disappearance test, Jones I, Jones II, and canalicular probing and i rrigation all provide information that leads to a proper diagnosis in a patient with a tearing problem. The dye disappe arance test is performed by placing fluorescein in the cul-de-sac and observing the degree of clearing after 5 minut es. A positive test is interpreted as significant retention of dye after 5 minutes. It does not localize an abnormalit y within the system but assesses the overall clinical functioning of the lacrimal system. Jones I is positive if dye can be visualized within the nasal cavity. Jones II is a method for determining if dye has entered the lacrimal sac. A posi tive Jones II indicates dye is in the nasal cavity after lacrimal irrigation. Jones III is useful when assessing the functio ning of a surgical osteotomy site. A positive Jones III means dye is found in the nasal cavity of a post DCR or CDCR patient. R27 Dacryocystograms demonstrate the nasolacrimal sac well. DCG involves forcibl y injecting radiopaque dye into the lower canaliculus, with radiographs being taken thereafter. Because force is use d, it is not a test of lacrimal function. Because the lacrimal cannula is inserted into the canaliculus, a DCG usually doe s not demonstrate the canalicular system well. A DCG can show the size and filling defects within the sac (diverti cula, fistulas, lacrimal tumors, stricture locations). In most situations, diagnosis of a nasolacrimal system obstruction c an be made without the use of a DCG. R28 Clinical signs of lacrimal sac tumors include epiphora, irreducible swelling of the lacrimal sac, bleeding on attempted probing and irrigation, and secondary dacryocystitis. R29 Studies have demonstrated that in up to 50% of patients tested, creation of a monocanalicular state resulted in symptomatic epiphora. Canalicular trauma affects a generally younger patient pop ulation. Leaving open the possibility of future trauma and disease to the remaining canaliculus, and considering prese nt surgical techniques with a high success rate of repair, many recommend attempted repair of all recent canalicula r lacerations. Additionally, acute trauma is generally easier to repair before scarring. One may wait a few days to allow acute soft tissue swelling to decrease before attempted repair. The longer the silicone stents remain in place , the better the likelihood of patency for

the system (after all surrounding scarring has occurred). Many doctors prefer si licone stent intubation with microanastomosis of the lacerated canalicular system to maximize the success of canalicular patency. R30 The ampulla has the largest diameter of the canalicular system (approximatel y 2 mm). The canaliculus has an average diameter of 1 mm. These relationships are important to keep in mind when probing the canalicular system. R31 Chronic use of the following medications has been reported to cause canalicu lar stenosis: a. Echothiophate. b. Idoxuridine. c. Epinephrine. R32 Indications for a conjunctivodacryocystorhinostomy (Jones tube procedure). a. Lacrimal canaliculi have been destroyed. b. Canalicular remnants cannot be anastomosed with the intranasal cavity. c. Paralytic or scarred eyelids with absent canalicular pumping mechanism. R33 Acute dacryocystitis is treated with systemic antibiotics and warm compresse s. Probing the system does not successfully treat the problem in adults. DCR is usually necessary to re-establi sh lacrimal outflow in adults. R34 Dacryocystitis results from tear stasis. Pneumococci (Streptococcus pneumoni ae) are the most common organism in dacryocystitis. Other organisms include streptococci, diphtheroids, Klebsiella p neumonia, Haemophilus influenzae, Pseudomonas aeruginosa, and mixed organisms. Actinomyces and fungi, such as Cand ida, are also frequently seen. R35 Infection of the lacrimal sac is usually secondary to obstruction of the nas olacrimal duct. It may be acute or chronic and is most commonly caused by staphylococci. Kanski. {Staph and streptococcus A AO.}. Yanoff. A common organism involved is Staphylococcus aureus. R36 Treatment of canaliculitis includes: a. Canalicular curettage. b. Canalicular incision and debridement. c. Canalicular irrigation. d. Topical penicillin may also be necessary. Oral antibiotics are not a method o f treatment. R37 An 11-year-old girl presents with acute, unilateral, left-sided periocular p ain, proptosis, and double vision. Twentyfour hours later (and without any treatment), the pain has resolved. Periocular ecchymosis has developed, and the double vision has stabilized. The hemorrhage and ecchymosis would be more charac teristic of lymphangioma than rhabdomyosarcoma. R38 Indications for surgical intervention in orbital cellulitis include decreasi ng vision and afferent pupillary defect, failure to respond to IV antibiotics, and progression of clinical signs such as motility changes and proptosis. R39 The hemorrhage from an orbital lymphangioma can cause compressive optic neur opathy. In a patient with visual loss from optic nerve compression, drainage of the cyst with CT guidance or open surg ery may be considered. Because the lymphangioma in this patient interdigitates with extraocular muscles and the opt ic nerve, it would be impractical to attempt total surgical excision of the lesion. Sometimes a lymphangioma will be more localized and amenable to surgical excision with preservation of normal structures. IV corticosteroids may also be beneficial.

R40 Lymphangiomas may increase in size during viral infection, presumably caused by the lymphocytic components of the tumor. Orbital cellulitis is the most common cause of proptosis in children; lymphangioma is not a malignant tumor; and capillary hemangioma (not lymphangioma) responds to intralesional corticoste roids. R41 The patient in a middle-aged woman with very gradual onset of proptosis and no other symptoms. The CT scan shows a well-circumscribed, rounded mass inferonasal and anterior to the globe t hat displaces, but does not invade, surrounding tissue. The A-scan ultrasound shows high internal reflectivity consi stent with nonhomogeneous tissue. These findings make the most likely diagnosis . Typically, the cavernous hemangi oma is a cavernous hemangioma cavernous hemangiomaslow-growing mass that becomes symptomatic with proptosis or diplopia as the mass displaces the globe. The location of this lesion is surgically approached through anterior approach through the in ferior conjunctival fornix. R42 Orbital floor fractures most commonly occur within the maxilla medial to the infraorbital canal. R43 Before 1965, the standard treatment for orbital rhabdomyosarcoma involved ex enteration, and it had a poor survival rate. Since 1965, this mutilating procedure has been abandoned as primary manage ment. Radiation and systemic chemotherapy are the mainstays of treatment based on the guidelines set forth by the Intergroup Rhabdomyosarcoma Study. The total dose of local radiation varies from 4500 to 6000 rad given over a period of 6 weeks. Chemotherapy is used to eliminate microscopic cellular metastasis. The survival rate using these modalities has improved significantly. R44 Predisposing conditions for mucormycosis include: 70% have diabetes mellitus , 5% have renal disease, 18% have other immunosuppressed states, 3% have leukemia, and only 4% have no systemic il lness. D. . UVEITIS UVEITIS R1 Histologically, the snowbank is composed of fibroglial and vascular elements. It is not an exudate, but a preretinal membrane that forms in response to an inflammatory stimulus. The memb rane also contains vascular elements that occasionally bleed and result in a vitreous hemorrhage. The periph eral retinal veins often show a perivascular cuff of lymphocytes, and the vitreous snowballs are composed of epi thelioid cells and multinucleated giant cells. R2 The patient that has a severe anterior uveitis with a hyphema. Any severe uve itis can cause damage to the iris vasculature resulting in anterior chamber bleeding. In particular, VZV iridocycl itis and HLA-B27 can result in this picture.

A traumatic injury can cause a tear at the iris root in addition to inciting an anterior uveitis. Dispersed blood and fibrin can appear similar to active uveitis. R3 The following conditions may be associated with diffusely distributed keratic precipitates over the corneal endothelium: Fuchs' heterochromic iridocyclitis, sarcoidosis, syphilis, kerato u veitis, and, rarely, toxoplasmosis. The finding of diffusely distributed keratic precipitates may be a useful diagnostic sign. R4 Fuchs' heterochromic iridocyclitis. This condition is generally characterized by heterochromia, although up to 15% of cases may have bilateral involvement without obvious heterochromia. Diffuse iris stromal atrophy with variable pigment epithelial layer atrophy accounts for the change in iris color in the involved e ye. Because of the pigment loss, brown eyes will appear less brown; blue eyes will appear bluer. This condition is also characterized by small, white, stellate keratic precipitates that are diffusely present over the endothelial surface. Sy nechiae are almost never present. The inflammatory nature of this disease is supported by the presence of plasma cells and lymphocytes on pathologic examination of ocular tissue. R5 Posttraumatic endophthalmitis after penetrating trauma typically occurs in fe wer than 10% of cases. The most common agents responsible for this infection include Staphylococcus epidermidis and Bacillus species, although many other organisms have been recovered. The prognosis for visual recovery in this s etting is generally poor, especially when Bacillus cereus is implicated. R6 Commonly used agents in the treatment of Behçet's disease include systemic cort icosteroids, cytotoxic agents (alkylating agents such as chlorambucil), colchicine, and cyclosporine. Systemic corticosteroids may be initially effective in treating the ocular inflammation in this condition; however, its use has clea rly not been found to arrest the long-term progress of this condition. Colchicine is known to inhibit leukocyte migration a nd has been found to be useful in preventing recurrences. Cyclosporine has also been found to be particularly usef ul in the treatment of Behçet's disease; however, its use must be carefully monitored because of renal toxicity. R7 Inflammation in Behçet's disease may affect the anterior and posterior segments and is generally a bilateral process. Recurrent explosive inflammatory episodes are typical with active episodes that range from 2 to 4 weeks. A chronic, lingering inflammatory stage typically does not develop. A nongranulomatous ante rior uveitis with formation of a transient hypopyon is common. Posterior inflammation is characterized by recurre nt vascular occlusive episodes with retinal hemorrhage and vitreous inflammation. The inflammation is typically conf ined to the retina and retinal vasculature. Choroidal involvement is rarely seen. The retinitis of Behçet's disea se is very suggestive of a viral retinitis and must be considered in the differential. R8 Two-thirds of patients with sarcoid uveitis have anterior uveitis. Two forms of anterior uveitis exist. One is a chronic, recurrent anterior uveitis that is difficult to treat and control with corticost eroids. The other is an acute granulomatous iridocyclitis that responds well to corticosteroid therapy. R9 Adequate hydration, which may include either oral intake of 2 to 3 liters of

fluid or IV intake of 1 to 2 liters of fluid, is important to prevent hemorrhagic cystitis with the use of cyclophosphamide. Oral cyclophosphamide is more likely to produce hemorrhagic cystitis than is IV cyclophosphamide. R10 Systemic corticosteroid therapy has been associated with numerous complicati ons. Ocular complications include cataracts and glaucoma. Systemic complications include exacerbation of hypertens ion or diabetes mellitus, osteoporosis, gastric ulceration, hirsutism, weight gain, capillary fragility, a nd cushingoid appearance. R11 Posner-Schlossman syndrome presents with unilateral ocular pain, mild anteri or uveitis, and elevated IOP. The episodes that occur are typically self-limited, but they often require the use o f topical glaucoma medications to control IOP. Mild topical corticosteroids may be used to control intraocular inflammatio n. Systemic and periocular corticosteroids are not indicated. Recurrent attacks of this syndrome may lead t o eventual optic nerve damage and lateonset glaucoma. R12 Although syphilitic uveitis may mimic any other form of uveitis, intermediat e uveitis is a distinctly uncommon presentation. Pars planitis is by far the most common intermediate uveitis entit y, a diagnosis of exclusion. Between 5% and 25% of patients with multiple sclerosis may have evidence of periphlebitis a nd intermediate uveitis. R13 Elevated serum angiotensin-converting enzyme levels may be seen in any diffu se granulomatous disease affecting the lung. Blind conjunctival biopsies have a characteristically low yield in pat ients who have only the presumptive diagnosis of sarcoidosis. If the diagnosis has been made based on elevated angio tensin-converting enzyme levels and a positive chest radiograph, conjunctival biopsies may have yields as high as 60 % to 70% for noncaseating granulomas. Pulmonary findings of sarcoid include no changes, hilar adenopathy, and diffuse interstitial lung disease that may progress to severe end-stage pulmonary fibrosis. Ocular involvement occurs i n approximately 25% of patients with systemic sarcoidosis. Uveitis may be seen in up to 60% of patients with ocu lar involvement. R14 Vogt-Koyanagi-Harada syndrome. This diagnosis is one of exclusion, and appro priate laboratory evaluation for this entity should be performed. Vogt-Koyanagi-Harada syndrome is a bilateral granulo matous panuveitis that causes disc edema and bilateral serous retinal detachments with panuveitis in the acute phas es of the disease. It is important to rule out infectious entities, including syphilis and possibly tuberculosis. An antinu clear antibody test may also be done to rule out lupus choroidopathy. B-mode echography may be useful in showing the amount o f choroidal infiltration, which is quite profound in the posterior pole. This condition is treated with systemic co rticosteroids. Fluorescein angiography characteristically reveals multiple pinpoint areas of hyperfluorescence that gra dually leak fluorescein into the subretinal space. Retinal vascular staining is uncommon. Differential diagnosis would inclu de posterior scleritis, sympathetic ophthalmia, hypotony, uveal effusion syndrome, and lupus choroidopathy.

R15 Choroidal neovascularization is common in many posterior uveitides. Choroida l inflammation may enhance production of angiogenic factors, and, when coupled with retinal pigment epithel ium Bruch's membrane disruption, choroidal neovascularization can develop. R16 Recently, rifabutin-associated acute anterior uveitis has been reported in A IDS patients. This can occur weeks to months after starting therapy with rifabutin. Culture of anterior chamber fluid has been negative. Prompt resolution of the uveitis occurs with discontinuation of rifabutin and with topical corticosteroid drops. R17 Acute uveitis entities should always be treated with corticosteroids first. Corticosteroids are thus the first line of therapy for all uveitic syndromes. For situations in which topical, periocular, or systemic corticosteroids are ineffective or need to be tapered for steroid-associated side effects, immunosuppressive medica tions may be useful. E. . GLAUCOMA GLAUCOMA R1 Indentation tonometry gives falsely low readings under all of the following c onditions: a. High myopia. b. Decreased central corneal thickness. c. Little fluorescein. d. Greater than 3 D of with-the-rule astigmatism. R2 A pupillary diameter of less than 3 mm can cause general depression of the fi eld. It is best to test the field with a pupil larger than 3 mm. Patients taking pilocarpine may need to refrain from taking th e medication for 24 hours before the test or be dilated at the time of their examination. R3 The following are well established early signs of glaucomatous damage. a. Focal enlargement of the cup appears as localized notching of the rim. The cu p can become vertically oval if narrowing of the rim occurs at either the superior or inferior pole of t he disc. b. Splinter hemorrhages usually clear over several weeks but are often followed by localized notching of the rim. c. Glaucomatous optic atrophy is associated with loss of axons in the nerve fibe r layer, which can be best evaluated with red-free illumination. d. Peripapillary atrophy is not considered to be a sign of early glaucomatous da mage. Other conditions such as ocular histoplasmosis or myopia can result in peripapillary atrophy. R4 The small image obtained using indirect ophthalmoscopy does not allow for ade quate evaluation of the optic nerve details. Careful examination with the direct ophthalmoscope can provide importan t information about the pallor of the optic cup; however, the most effective methods include stereoscopic examination using the slit lamp in combination with a posterior-pole contact lens, a 90-D lens, or a Hruby lens. R5 Malignant melanomas can be associated with normal, elevated, or depressed IOP . Elevated IOP occurs more frequently with melanomas of the iris/ciliary body than with choroidal melanomas . Glaucoma may occur by a variety of mechanisms, including 1) obstruction of the trabecular meshwork by melanin-conta ining macrophages (melanomalytic glaucoma), 2) direct extension of tumor into the trabecular meshwork, 3) angle c losure from anterior displacement of the

lens-iris diaphragm or peripheral anterior synechiae, 4) inflammation, and 5) ne ovascularization of the angle. Neovascularization of the angle appears to be the most common cause of elevated IOP, especially among eyes treated with radiation. R6 Sodium hydroxide is the most common agent causing glaucoma. Glaucoma is most often associated with alkali burns but can also be seen after severe alkali burns. IOP rises initially because of s cleral shrinkage and release of prostaglandins. Later, IOP rises because of inflammation, posterior synechiae ca using pupillary block, or acute lens swelling. Finally, IOP may continue to be high due to direct injury to the trabe cular meshwork. Filtering surgery may be needed but can be difficult due to conjunctival scarring. A cyclodestructive pro cedure or Seton valve may be required. R7 Patients at particular risk for aqueous misdirection are those with crowded a nterior segments (i.e., angle closure, nanophthalmos). Postoperative inflammation may cause swelling of the ciliary bod y and ciliary processes leading to aqueous misdirection. Myopia is not associated with an increased risk of aqueous misdirection. R8 Hyperosmotic agents may aggravate congestive heart failure by an increase in extracellular volume. Backache; headache; mental confusion; and subdural, even subarachnoid hemorrhages have als o been reported. R9 Prostaglandin analogs such as latanoprost (Xalatan) more commonly produce con junctival hyperemia than true allergic conjunctivitis. From 1% to 5% of patients taking dorzolamide (Trusopt) may have allergic symptoms. Up to 20% of patients taking brimonidine (Alphagan) may show such symptoms. R10 The most frequent cause of failure after filtration surgery is bleb scarring due to episcleral fibrosis. This excessive healing response is largely due to the proliferation of fibroblasts and the prod uction of collagen and glycosaminoglycans. The antimetabolite 5-FU has been used to modulate wound heal ing after filtration surgery. 5-FU inhibits fibroblast proliferation to prevent episcleral fibrosis. R11 In addition to inhibiting fibroblast proliferation, 5-FU also inhibits the g rowth of epithelial cells of the conjunctiva and cornea. It is associated with several undesirable complications including conjun ctival wound leaks, corneal epithelial defects, thin-walled ischemic blebs, hypotony, and suprachoroidal hemorrhage. It has not been associated with increased risk of retinal detachment. R12 The combined effect of timolol and a miotic or timolol and a carbonic anhydr ase inhibitor is significantly greater than the effect of any of the medications alone.

R13 The Collaborative Initial Glaucoma Treatment Study (CIGTS). a. The study was designed to address the question of medical therapy versus earl y filtration surgery on the long-term progression of glaucoma. b. Patients in the early surgery group were more likely to lose visual acuity an d visual field during the first few years of follow-up study. c. After 4 years of follow-up, both groups (medical versus early surgery group) were similar in visual acuity and visual field. d. The CIGTS found that increased IOP was significantly decreased in both the me dically and surgically treated groups, with the surgery group having a larger decrease. However, in the surgery group, the need for subsequent cataract surgery was significantly higher, and patients were more likely to lose visual acuity and visual field during the first few years of follow-up study. After 4 years, p atients in both groups were similar in visual acuity and visual field. Few patients developed serious vision loss from glaucoma after either treatment. e. The CIGTS investigators concluded that the study results provided no reason t o change current treatment approaches to glaucoma. The CIGTS researchers also compared the impact of these two treatments on the patients' health-related quality of life. Their findings provided no reason to c hange current treatment approaches to glaucoma. R14 The prostaglandin analogs are a relatively new class of anti-glaucoma agents . Unlike latanoprost and travoprost, which lower IOP by increasing uveoscleral outflow, bimatoprost decreases IOP by increasing uveoscleral and trabecular outflow. Unoprostone appears to lower IOP by increasing trabecular ou tflow only. Latanoprost and travoprost are pro-drugs that penetrate the cornea and become biologically activ e after being hydrolyzed by corneal esterase. Neither bimatoprost nor unoprostone appears to be a pro-drug. An ocula r side effect unique to this class of drugs is the darkening of the iris and periocular skin. Other side effects in clude hypertrichosis of the eyelashes, conjunctival hyperemia, and exacerbation of herpes keratitis, CME, an d uveitis. R15 The continuous-wave argon laser was the unit most commonly used for creating iridotomies in the early days of laser surgery; however, the pulsed Nd: YAG laser is probably the more commonly used to day. Iridotomies created with an argon laser have more extensive early edema and tissue destruction at the margin s of treatment histologically as compared with those created with the Nd: YAG laser. Argon laser has the disadvan tage of more iritis, pupillary distortion, and late closure of the iridotomy. Clinically, the Nd: YAG laser has the disadvantage of frequent bleeding. In general, Nd: YAG laser iridotomies require fewer total applications with a marke d reduction in total energy as compared with argon laser iridotomies. In some cases, it may be advantageous to use both lasers: the argon for its coagulative effects and the Nd: YAG for its disruptive properties. R16 Nonpenetrating glaucoma surgery includes deep sclerectomy with collagen impl ant and deep sclerectomy with

injection of viscoelastic into Schlemm's canal (viscocanalostomy). The surgery i nvolves creating a superficial scleral flap and a deeper scleral dissection underneath to leave behind a thin layer of scler a and Descemet's membrane. Preliminary data comparing nonpenetrating procedures to standard trabeculectomy shows better IOP reduction after standard trabeculectomy but a lower incidence of postoperative complications suc h as hypotony after nonpenetrating procedures. However, the nonpenetrating surgeries are technically more difficult . R17 Drainage of choroidal effusions after trabeculotomy should also be considere d with: a. Impending failure of the bleb. b. Kissing choroidals. c. Flat anterior chamber with corneal decompensation. d. Worsening cataract or corneal decompensation. R18 Transscleral cyclophotocoagulation is useful in many types of refractory gla ucoma, such as glaucoma in aphakia or pseudophakia, neovascular glaucoma, glaucoma associated with inflammation, an d glaucoma in eyes with multiple failed filtering procedures. Observations in animal and human eyes suggest that the most likely mechanism of IOPlowering is reduced aqueous production through destruction of ciliary epithelium . R19 PXF syndrome is bilateral in 50% of patients and manifests itself in older-a ge patients. It is recognized by the presence of dandruff-like particles on the pupillary border, anterior lens capsu le, zonules, and other areas in the anterior segment. The material is distributed widely, including the conjunctiva, orbital tissues, skin, and viscera, supporting the concept that PXF is a systemic disease. Associated eye findings include a Kruken berg spindle, decreased corneal endothelial cell density, a heavily pigmented trabecular meshwork, narrow angle, poor pupillary dilation (iris muscle degeneration and/or lack of iris stroma elasticity due to accumulation of PXF), nuclear sclerotic cataract, zonular weakness leading to forward subluxation or dislocation of the lens, and peripupi llary (not peripheral) transillumination defects. An accumulation of pigment may also be seen along Schwalbe's line (Samp aolesi's line). From 20% to 60% of patients can have an associated open angle glaucoma. R20 The candy stripe is a tip-off to ghost cell glaucoma. Two conditions are nec essary for the development of this unique form of glaucoma: vitreous hemorrhage (red blood cells degenerate and become rig id in the vitreous) and a break in the anterior hyaloid face (to allow the cells to enter the anterior chamber). The cr enated khaki-colored ghost cells layer out in the anterior chamber and can be distinguished from fresher red cells, creatin g the effect of a candy stripe. Ghost cells cannot escape easily from the meshwork and hence produce intertrabecular obstruc tion and raise the IOP. In hemolytic glaucoma, hemoglobin-laden macrophages block the meshwork, whereas in phacolytic glaucoma, macrophages are

engorged with lens protein. Hyphema can also produce elevated IOP, especially in the setting of sickle cell hemoglobinopathies, which include sickle cell trait. R21 Delayed suprachoroidal hemorrhages after filtering surgery typically present during the first few postoperative days with severe pain, occasional nausea, and a marked reduction i n vision. The IOP is usually elevated, the anterior chamber is shallow or flat, and large choroidal detachmen ts are present. Risk factors associated with this condition include aphakia, pseudophakia, myopia, previous v itrectomy, and preoperative IOP greater than 30 mmHg. (Risk factors are also bleeding disorders and the presence of elevated blood pressure. Yanoff). R22 Aqueous shunt devices are reserved for those glaucoma cases in which standar d filtration surgery would fail or has already failed. Therefore, glaucoma implant surgery is indicated in the followin g situations: a. Failed trabeculectomy. b. Active uveitis. c. Neovascular glaucoma. d. Inadequate conjunctiva. e. Impending need for PK. f. A tube shunt can be placed in the presence of a scleral buckle. In post vitre ctomy cases, the tube can be placed through the pars plana. It may not be wise to perform incisional surgery in an eye with poor visual potential given the risks and, at times, complicated postoperative course of tub e shunt surgery. R23 The following are complications of glaucoma implant procedure: hypotony, sha llow chamber, migration/expulsion of tube, conjunctival melting, corneal edema, diplopia, and elevated IOP. R24 The Baerveldt implant does not have a pressure-sensitive valve, so hypotony in the early postoperative period has to be managed by other means. Hypotony is common when drainage implants are install ed in a one-stage procedure without complete tube occlusion. The absence of any resistance to aqueous outflo w invariably results in reduction of IOP to below physiologic levels. A two-stage procedure has been recommended to l imit early hypotony. During the first operation, the scleral plate is sutured to the globe without connecting the tube into the anterior chamber. This procedure is followed by a second operation 2 to 8 weeks later, during which the tube is i nserted into the anterior chamber. An alternative to limit overfiltration after one-stage installation involves the te mporary occlusion of the tube lumen with a ligature, or with semipermeable, biodegradable collagen lacrimal plugs. F. . CORNEA CORNEA R1 Stevens-Johnson syndrome is a systemic autoimmune disease that causes sloughi ng of the mucous membranes (including the conjunctiva). This can be a reaction to medication from sulfonami des, Dilantin, and others. In milder cases, patients may present with a papillary conjunctivitis. R2 VDRL and RPR tests detect antilipoidal antibodies produced by the host during treponemal infection. Autoimmune diseases, such as rheumatoid arthritis and lupus erythematosus, have similar ant ibodies that may give a false-positive reading. Wegener's granulomatosis is associated with the antineutrophil cytoplas mic antibody.

R3 Megalocornea is usually an X-linked, isolated, nonprogressive congenital corn eal enlargement with a horizontal corneal diameter of greater than 13 mm. It has been associated with systemic con ditions such as Down, Marfan's, and Alport's syndromes, craniosynostosis, and facial hemiatrophy. R4 Peter's anomaly is sporadic. R5 The differential diagnosis of blue sclera. a. Hurler's syndrome. b. Osteogenesis imperfecta. c. Turner's syndrome. d. Although Marfan's disease is a disorder of collagen synthesis, blue sclera is not seen in this condition. R6 Sclerocornea is a nonprogressive, noninflammatory scleralization of the corne a. Ninety percent of cases are bilateral with no gender predilection. Half are sporadic. The remaining may be either domi nant or recessive. R7 Enlarged corneal nerves may be found in Refsum's disease, ichthyosis, congeni tal glaucoma, and other diseases. Multiple endocrine neoplasia, type IIb (Sipple-Gorlin syndrome), is associated w ith enlarged corneal nerves. R8 Basal cell carcinoma is the most common malignant tumor of the eyelid skin, b ut squamous cell carcinoma is the most frequently occurring malignancy of the conjunctiva. Squamous papillomas are benign lesions that can, on rare occasions, undergo malignant transformation. R9 Mucoepidermoid carcinoma of the conjunctiva is a more aggressive variant of s quamous cell carcinoma. It should be suspected in cases involving recurrence after primary excision or in which no in vasion into the globe occurs. It typically occurs in patients over age 60 and involves cells that are able to produce malig nant mucus secreting cells (goblet cells). R10 Terrien's marginal degeneration. Peripheral thinning occurs superiorly first , then circumferentially. Unlike Mooren's ulcer or peripheral ulcerative keratitis of autoimmune disease, thinning occurs with an intact epithelium in an essentially quiet eye. Thinning to perforation is rare. Against-the-rule astigmatism is ofte n induced. Thinning that is more apparent than real is a characteristic of furrow degeneration, a benign condition that do es not affect vision. R11 Mooren's ulcer. Note the conjunctival injection, ulceration of the periphera l cornea, and undermined leading edge. Pain may be severe and accompanied by photophobia. An autoimmune process likely plays a role as immunoglobulin, complement, and plasma cells are found in the adjacent conjunctiva. Two clinical types have been described. The type found in older adults is usually unilateral accompanied by mild pain, and it is more responsive to therapy such as topical

steroids. Corneal perforation is rare. The other type is bilateral, often found in younger black males, and rapidly progressive. These lesions respond poorly to therapy, and systemic immunosuppres sives are often necessary. R12 Circumscribed posterior keratoconus is seen mostly in women and is character ized by a localized central indentation of the posterior cornea with variable amounts of stromal haze. Loss of stromal substance occurs, but Descemet's membrane and endothelium are intact. Occurrence is sporadic, usually unilateral and nonprogressive. Amblyopia may occur. R13 Recurrent erosions are uncommon in granular dystrophy as opposed to lattice dystrophy, in which they occur much more frequently. R14 Causes of secondary acquired conjunctival melanosis include Addison's diseas e, radiation, and pregnancy. Black conjunctival adrenochrome deposits result from the oxidative byproducts of epinephrine compounds. R15 EDTA is able to chelate the calcium found in band keratopathy. It is necessa ry to scrape to epithelium to expose the calcium to the EDTA. R16 Congenital hereditary stromal dystrophy is a rare, autosomal dominant condit ion presenting at birth as a central, anterior stromal feathery opacity that may cause reduced visual acuity. The corn eal periphery is clear. There is no pain, photophobia, or tearing because IOP is normal. R17 The most common stromal dystrophy is lattice, followed by granular. R18 Macular corneal dystrophy, the least common of the classic stromal dystrophi es. It usually leads to symptoms at an earlier age than either lattice or granular dystrophy and is caused by an error in the synthesis of keratan sulfate, leading to unsulfated keratan that is not degraded effectively. These mucopolysaccharide deposits accumulate throughout the cornea (including the periphery) and stain with colloidal iron and alcian blue. R19 Recurrence in the graft is common in both Reis-Bucklers' dystrophy and latti ce dystrophy. R20 Fuchs' dystrophy. Treatment measures are aimed at limiting visual dysfunctio n and discomfort due to epithelial breakdown and ruptured bullae. Hypertonic solutions and ointments may provide so me relief, but they do little when the edema is advanced. Some feel that lowering IOP may help control edema. Both hype rtonic solutions and IOP-lowering agents, however, are only temporizing measures and are not long-term solutions. Ruptured bullae may be treated with patching or bandage contact lens. Penetrating keratoplasty carries a good progno sis in these patients. Anterior stromal puncture is not indicated in Fuchs' dystrophy, but it may be helpful in treating cases of recurrent erosion. R21 Thygeson's superficial punctate keratitis is a bilateral (although it may be asymmetrical) disease consisting of a course punctate keratitis without an accompanying conjunctivitis. These lesions may or may not stain with fluorescein. Corneal sensation is normal. It is very responsive to topical corti costeroids, but this may increase the rate of recurrence. Topical cyclosporine may be beneficial. No clear etiology has bee n established, but a viral etiology has been postulated (although antiviral medications have not proven beneficial). Peo ple possessing the histocompatibility antigen HLA-DR3 have a 5.65 greater relative risk of having Thygeson's superfici

al punctate keratitis. R22 Tuberculosis, herpes simplex, and acquired syphilis would more likely cause a unilateral interstitial keratitis. Congenital syphilis causes a bilateral interstitial keratitis that may not becom e manifest until the patient is 10-20 years of age. Cogan's syndrome affects middle-aged adults with hearing loss, vertigo, and interstitial keratitis. R23 You obtain a cornea for an elective penetrating keratoplasty and you notice that the color of the storage medium is yellow-orange instead of the usual pink. This indicates possible microbial conta mination. R24 Peters' anomaly is the most frequent indication for penetrating keratoplasty in children. R25 Penetrating keratoplasty in children can be very challenging. The eye wall i s extremely flaccid, and scleral support is often crucial. Anterior bulging of the lens-iris diaphragm is common, and many s urgeons prefer to use a small-sized graft to reduce this problem as well as to reduce the chance of peripheral synec hiae formation. Frequent postoperative follow-up and early suture removal (as early as 2 to 4 weeks postoperatively in neonates) help reduce the incidence of neovascularization of the graft. R26 Corneal graft survival is excellent with current techniques (90%). The incid ence is even higher for conditions such as keratoconus. R27 Overall, the chance of an endothelial graft rejection episode is 20% to 25% according to most studies. If the rejection is recognized early, intensive treatment may save the graft from failure. R28 The most interesting conclusion of Collaborative Corneal Transplant Study wa s that even for high-risk keratoplasties, HLA tissue matching was neither clearly advantageous nor cost-effective. The pos sibility also existed of a correlation of risk and ABO blood type incompatibility (ABO blood type incompatibility was show n to be a possible risk factor). Further study is needed to decide whether this is a necessary screening test. R29 Most contact lens wearers are very reluctant to discontinue use of their len ses for any significant amount of time. Stability of refraction and topography are important when deciding on a surgical plan. Most surgeons require at least 2 to 3 weeks of a contact lens holiday to verify stability. Some individuals require a longer period before they show stability. R30 Most surgeons will rarely perform arcuate incisions greater than 90° because o f decreased efficacy and increased instability of effect. R31 The inferotemporal quadrant is the thinnest in most patients (38%). Of the p rovided choices, the thinnest quadrants (in descending order) are the temporal (28%), inferior (19%), nasal (11%), and s uperior (4%). Variation exists in individual patients; therefore, multiple paracentral measurements are important to determine the thinnest region.

Determination of the thinnest paracentral corneal measurement is used to set the blade depth for radial keratotomy as well as to determine which quadrant to incise first. G. . LENS/CATARACT LENS/CATARACT R1 The following syndrome is associated with retained lens nuclei. a. Trisomy 13. b. Rubella. c. Lowe's syndrome. R2 Amiodarone and phenothiazine cause stellate cataracts in the anterior lens ca psule. All forms of steroids have been associated with cataract formation. Although echothiophate has been associated w ith progressive cataract formation in adults, this has not been reported in children. R3 Marfan's syndrome is a genetic abnormality in fibrillin, a structural protein in collagen. Patients with Marfan's are tall; have long, thin fingers; have hyperextensible joints; and have aneurysms of the aorta. Ocular manifestations include subluxation of the lenses superotemporally and a high risk of retinal detachment . Given the high risk of retinal detachment, the increased risk of complications with routine cataract surgery, a nd the generally nonprogressive nature of the subluxation, spectacles, or contact lenses should be tried first. Most cases are amenable to spectacles or contact lenses. Pupil dilation is sometimes helpful because it allows the pat ient to see around the subluxed lens. A reading adds is needed secondary to poor accommodation of the lens due to zonula r dehiscence. R4 In cortical cataracts, there is hydropic swelling of lens fibers. The eosinop hilic, globular material between lens fibers is called morgagnian globules. R5 Anterior polar cataracts are usually small and nonprogressive, do not usually impair vision, and may be seen in association with microphthalmos, a persistent pupillary membrane, and anterior l enticonus. Posterior polar cataracts cause more visual impairment and tend to be larger than anterior polar cataracts . Both can be autosomal dominant or sporadic. Posterior polar cataracts may be associated with posterior lenticon us or a remnant of the tunica vasculosa lentis. R6 Toric lenses are designed to counteract the corneal astigmatism. When the len s rotates off axis, the amount of astigmatism correction decreases. When it rotates 90° off axis, the astigmatism me asured will be greater than the corneal astigmatism. The lens should be rotated back to the correct position wit hin the several weeks after surgery before the anterior and posterior capsule fuse together and make it more difficu lt to manipulate the lens. The lens should not be placed in the ciliary sulcus because it will not be stable in this location. R7 Alport's syndrome, congenital rubella, and Weill-Marchesani have been reporte d to be associated with microspherophakia. R8 Three kinds of aspiration systems exist in phacoemulsification machines. The peristaltic pump has rollers that move along tubing and create a relatively rapid rise in vacuum. The diaphragm pump ha s valves over both the inlet and outlet of a fluid chamber covered by a diaphragm. This system allows a slower bu ild of vacuum. The Venturi pump

produces the most rapid increase in vacuum. This, however, can be the most dange rous because it allows almost instantaneous engagement of unwanted tissues such as capsule or iris. R9 Iris prolapse associated with an excessively deep anterior chamber suggests t oo high a bottle height, which can be remedied by simply lowering the bottle. If iris prolapse is associated with a no rmal or shallow anterior chamber depth, this may be caused by wound leak around the phacoemulsification tip or possibly suprachoroidal hemorrhage. If leak around the phaco tip is evident, a temporary suture may be placed. Peripheral ir idectomy at the site of prolapse may also help reposit the iris. Excessive phacoemulsification power does not result in iris prolapse. R10 When traumatic capsular rupture is present with lens subluxation and disrupt ion of the anterior hyaloid face, the pars plana approach for lensectomy and vitrectomy is the most appropriate. The presen ce of a hard nucleus, if present, may make this technically difficult. Extracapsular cataract extraction would not be indicated with loss of zonular integrity and presence of free vitreous. Intracapsular cataract extraction is contraindicated when vitreous is present and the capsule is ruptured. R11 PMMA lenses, when packaged, may pick up static charges that attract dust and debris when opened. Therefore, the lens may be rinsed with balanced salt solution before insertion. Silicone lenses do not require this and, in fact, may be more difficult to handle once they have gotten wet. Application of a viscoelasti c, such as sodium hyaluronate, may facilitate insertion of a lens, especially a foldable one, through a small incis ion. R12 The indications for cataract surgery in nanophthalmic patients are similar t o those in other patients. However, the chances for complications and a poor visual outcome are significantly higher. Th ese complications include retinal detachment, choroidal effusion, postoperative angle-closure glaucoma, flat anter ior chamber, cystoid macular edema, corneal decompensation, and malignant glaucoma. Extracapsular cataract ex traction with posterior chamber lens insertion is generally the procedure of choice, using the smallest incision possible. Some cases with refractory positive vitreous pressure and anterior segment crowding may do best without IOL implantation. In eyes with significant glaucoma, small incision cataract surgery may be combined with trabe culectomy. When the anterior chamber is shallow preoperatively and the choroid is thickened, anterior sclerotomies ar e indicated at the time of surgery. R13 Risk factors for expulsive choroidal hemorrhage include myopia, glaucoma, at herosclerotic vascular disease, hypertension, and previous expulsive hemorrhage in the opposite eye. Using as sm all an incision as possible along with being ready to perform a sclerotomy is important. Decompression before opening t he globe with digital pressure or

other device may be helpful. Close the wound with nonabsorbable, preferably nylo n, sutures to prevent delayed hemorrhage. It is also beneficial to keep blood pressure controlled and level of anesthesia deep (if general anesthesia is used) during surgery. Shelved or self-sealing incisions allow more rapid closure and repressurization of the globe should bleeding occur. R14 The posterior capsule is capable of moving more posterior than normal withou t support of the vitreous. Zonular dehiscence may result from attempted manual expression of the nucleus during ext racapsular cataract extraction. The posterior capsule is also more likely to move forward with the lens during phaco emulsification and aspiration in extracapsular cataract extraction via phacoemulsification. As a result, capsular tears may be more likely. Collapse of the globe is usually not problematic with a well-formed phaco incision. Positive pre ssure from the posterior segment occurs more frequently when the vitreous cavity is occupied by the vitreous. R15 Lane and colleagues evaluated IOP elevation associated with three commonly u sed viscoelastic agents (sodium hyaluronate, chondroitin sulfate, and hydroxymethylcellulose) and all produced s ignificant pressure elevations at 4 ± 1 hours postoperatively. Removing the viscoelastic did not eliminate significant p ostoperative IOP elevation, although when chondroitin sulfate was removed, the pressure elevation was slightly less. R16 Retained cortical material can incite inflammation in the months following c ataract surgery, but remote inflammation due to the cortical lens material is unusual. Proliferation of lens epithelium ( Elschnig pearls and Soemmering's ring) is noninflammatory. R17 The patient has with-the-rule astigmatism that has no obvious lenticular com ponent. A 6.0-mm scleral incision placed superiorly may help decrease the patient's preoperative astigmatism 1.5 ± 0.5 D. A 3.5-mm incision would be more astigmatically neutral with 0.5 ± 0.3 D drift against-the-rule. R18 By placing a scleral-sutured posterior chamber IOL, The needle should be pas sed 0.75 mm posterior to the limbus. R19 Among the causes of corneal edema the day following cataract extraction are elevated IOP (from inflammation, glaucoma, debris clogging the trabecular meshwork), corneal decompensation (low endothelial counts as in Fuchs' dystrophy, endothelial chemical toxicity), or trauma to the endothelium. Epithel ial downgrowth would not present immediately after surgery but would take weeks to develop. The corneal decompens ation would overlie the area of the downgrowth, and a membrane may be seen on the endothelium and iris. R20 The incidence of clinical cystoid macular edema is less than 1% following un complicated cataract extraction. The incidence of angiographic evidence of cystoid macular edema is approximately 10 times this. H. . RETINA AND VITREOUS RETINA AND VITREOUS R1 CMV retinitis can lead to significant atrophy of the retina and subsequent re tinal detachment. Oftentimes, multiple retinal defects are present, and the patients need long-term internal tamponade with sil icone oil to prevent recurrent detachments. R2 A 7-year-old girl reports having poor vision for 2 weeks. This history is sug gestive of Leber's idiopathic stellate

neuroretinitis. The exact etiology of neuroretinitis is unknown but has been lin ked to viral infections (mumps, influenza, and varicella) and other diseases (cat-scratch fever, leptospirosis). R3 The natural course of Leber's stellate neuroretinitis is spontaneous resoluti on over several months. The prognosis is excellent, and over 80% of patients have visual acuity better than 20/40. R4 Coats' disease (congenital retinal telangiectasias) tends to occur unilateral ly in otherwise healthy boys. The majority of boys have the juvenile form, with a peak incidence within the end of the firs t decade. An adult form occurs after age 16 and may be associated with hypercholesterolemia. R5 Histopathologically, CRVO occurs as a result of thrombosis at the lamina crib rosa. Risk factors for CRVO include hypertension, glaucoma, diabetes, hyperopia, hypercoagulable states, and older a ge. Emboli are associated with arterial occlusion rather than venous obstruction. R6 Although the exact etiologies of AMPPE, MEWDS, and Leber's stellate neuroreti nitis have not been determined, all have been linked with viral illnesses. DUSN is thought to be caused by migration of a nematode under the retina. R7 Presence of three or more of the following characteristics indicates high ris k for PDR as outlined by the Diabetic Retinopathy Study: a. Any NV. b. NV on or within 1 DD of the optic disc. c. NVD greater than 1/3 disc area. d. NVE greater than 1/2 disc area. e. Vitreous or preretinal hemorrhage. R8 No effective treatment for dry ARMD has been found. Antioxidant vitamins and nutritional supplements may play a role in the prevention of ARMD, but once the loss of the RPE and photoreceptors has o ccurred, they are of no benefit. Eyes with the dry form of ARMD may rarely develop exudative changes, and monitoring w ith an Amsler grid is appropriate. Laser and subretinal surgery may be indicated if a choroidal neovascular membran e is present; however, they are not appropriate for the nonexudative form of ARMD. R9 Subretinal hemorrhage, along with a gray-green crescent, subretinal exudates, and a localized serous retinal detachment, may be indicative of an active choroidal neovascular membrane. Photo coagulation scars may indicate previous treatment for exudative ARMD, placing this eye at a higher risk of recu rrence of a CNVM. Approximately 10% of patients with dry ARMD will progress onto the wet form.

R10 Incontinentia pigmenti presents with vascular abnormalities in the periphera l retina; these abnormalities include capillary nonperfusion, arteriovenous shunts, and fibrovascular proliferation an d neovascularization. Pathologic myopia and chorioretinitis sclopetaria have breaks in Bruch's membrane, predisposing th ese eyes to formation of choroidal neovascular membranes. Juxtafoveal telangiectasis, pathologic myopia, and chorio retinitis sclopetaria are associated with neovascularization in the posterior pole. R11 The causes of CRAO in children and young adults differ from those of the old er population. In one study, one third of the patients had a history of migraine. Other factors include trauma (especially in males), hypercoagulable states (oral contraceptives, pregnancy), cardiac emboli, collagen-vascular disorders, and IV drug abuse. In the elderly population, atherosclerotic disease is much more common. R12 Conditions that may have CME without leakage of fluorescein include Goldmann -Favre, retinitis pigmentosa, and nicotinic acid maculopathy. Epiretinal membranes may cause a CME that leaks from traction and distortion of paramacular capillaries. R13 A choroidal neovascular membrane will cause subretinal hemorrhage or intrare tinal hemorrhage. Preretinal hemorrhage would be very unusual. Sickle cell retinopathy, trauma (e.g., shaken baby syndrome), and macroaneurysms may cause hemorrhages at all levels of the retina. R14 Pars plana cysts are common and occur in 16% to 18% of autopsied eyes. They contain mucopolysaccharides on histopathology and can be seen in otherwise normal eyes. Patients with multiple myeloma may have several large pars plana cysts that contain myeloma proteins. R15 RPE degenerations have been identified in patients with Hunter's, Hurler's, Sanfilippo's, and Scheie's mucopolysaccharidoses. Maroteaux-Lamy syndrome is not usually associated with RP E degeneration. R16 True silent choroid is defined as the blockage of underlying choroidal fluor escence during fluorescein angiography by material in the RPE cells. This occurs classically in patients who have Stargard t's disease. Lipofuscin accumulation within RPE cells results in the blockage of underlying choroidal fluorescence an d gives the appearance of a very dark or silent choroid. Systemic argyrosis, which occurs in patients who are receiving s ystemic silver (e.g., from tanning agents), can also cause an angiographically dark choroid. R17 Rhegmatogenous detachments occur in up to 25% of patients with CMV retinitis . They are associated with a diffusely necrotic peripheral retina with numerous small retinal holes. These de tachments cannot be repaired by conventional scleral buckling surgery, and they require pars plana vitrectomy wi th internal silicone oil tamponade and endolaser. R18 Although exudative retinal detachments may occur in CMV retinitis, it is ver y uncommon. Myopia is not associated with exudative retinal detachments. R19 Intraocular copper foreign bodies may result in two distinct disease process es. One is a mild form called chalcosis. The other is a more severe suppurative form of endophthalmitis. The severity of intraocular inflammation is directly proportional to the concentration of copper within the foreign body. Chalcosis o

ccurs in copper amalgams that contain significant amounts of nickel or other metals. Chalcosis is characterized by mil d intraocular inflammation, development of a sunflower cataract, and Kayser-Fleischer rings. Characteristically, early r emoval of the intraocular foreign body in chalcosis can improve the amplitudes of the ERG, which have been suppressed from the presence of intraocular copper ions. R20 Uveal effusion syndrome usually occurs in eyes with abnormally short axial l ength. These eyes often will have thickened sclera, which are thought to impede vortex venous outflow from the eye , resulting in recurrent choroidal effusions. Treatment is often unrewarding but involves the placement of partial thickness scleral windows near the vortex vein exit sites in at least three or four quadrants of the eye. Choroidal effusions or hemorrhage can occur during or after intraocular surgery and are thought to be caused by rapid changes in IO P-shearing choroidal perforating arteries. Hypertension and atherosclerosis are risk factors for choroidal effusi ons. R21 Traumatic macular holes usually result from pre-existing commotio in the reg ion of the macula. They are characterized by disruption and necrosis of retinal photoreceptors and subsequen t loss of retinal tissue. Unlike idiopathic macular holes, which are caused by tangential traction on the macula where vitrectomy and gas-fluid exchange have successfully closed the hole, surgery is not, in general, the trea tment for traumatic holes. There have been several reports of improvement in vision in traumatic holes with vitre ctomy, gas-fluid exchange, and application of TGF-. R22 Inert foreign bodies, such as glass, plastic, sand, stone, or ceramic, are w ell-tolerated in the eye. Wood incites a brisk inflammatory reaction and may harbor harmful microorganisms. Brass contain s copper, which may lead to chalcosis and retinal degeneration. Iron is tolerated poorly within the eye. Int raocular iron causes siderosis, resulting in photoreceptor and RPE degeneration. R23 The most common traumatic retinal tear is an inferotemporal retinal dialysis . The vast majority of retinal dialyses occur in this quadrant. After trauma, the most common retinal tear is a superona sal dialysis, followed by giant tears, flap tears, and tears around lattice. These statistics are true only for blunt ocular injury. R24 The original description by Terson consisted of both retinal and vitreous he morrhages in patients who have subarachnoid and subdural hemorrhages. About 20% of patients with spontaneous or traumatic subarachnoid hemorrhages will present with intraocular hemorrhages.

R25 Fluorescein leakage is not a feature of either solar maculopathy or photic i njury. Both may be associated with intense staining of the damaged RPE, particularly in the acute phases of the injury. As the injured RPE heals, the fluorescein angiogram would be characterized by persistent window defects. R26 White-centered retinal hemorrhages (Roth's spots) can be found in conditions with septic emboli (endocarditis, Candida bacteremia), leukemia, and collagen-vascular diseases. Measles causes wh ite-centered hemorrhages (Koplik's spots) on the buccal mucosa, but retinal hemorrhages are not found. R27 In measles (rubeola) virus infection, the classic triad of postnatally acqui red measles cough, coryza, and follicular conjunctivitis-can be accompanied by Koplik spots on the conjunctiva and a mild epithelial keratitis. R28 PIC is an entity that is slightly more common in myopic women in the third a nd fourth decades of life. It may be associated with recurrent choroidal neovascular membranes. It is not, however, a ssociated with a viral prodrome as is acute posterior multifocal placoid pigment epitheliopathy. R29 Fluorescein angiography of APMPPE is characterized by early hypofluorescence of lesions, followed by late hyperfluorescence of the entire lesion. Rarely, perivascular staining may be see n. Because of pigmentary disturbances that occur in the healed cases, choroidal neovascular membranes can develop, alt hough it is extremely rare. R30 Fluorescein angiography in MEWDS is characterized by early punctate hyperflu orescence, often in a wreath-like configuration, followed by late staining of the same punctate areas of hyperfluo rescence. These areas of hyperfluorescence correspond to the white spots seen clinically. Late disc stain ing is also a common feature. R31 Choroidal neovascularization is by far the most common macular complication of multifocal choroiditis. This is the major cause of vision loss in most patients with multifocal choroiditis. R32 Adult onset foveomacular vitelliform dystrophy is not associated with an abn ormal EOG. (The EOG is these individuals tend to be normal or only mildly subnormal. AAO). R33 Leber's congenital amaurosis is a disorder of congenital blindness that ofte n presents with an initially normalappearing fundus in the neonatal period. However, an ERG done at this time is ch aracteristically flat and diagnostic. R34 Juvenile X-linked retinoschisis may be associated with vitreous hemorrhage a nd rhegmatogenous retinal detachments. Although macular changes of a spoke wheel-type splitting of the ner ve fiber layer can occur, it is not true CME. R35 Gyrate atrophy is a metabolic disorder that is seen mainly in Scandinavian L aplanders. It is associated with a deficiency in the ornithine aminotransferase enzyme, critical in the urea cycle. This results in an accumulation of serum ornithine. The end result is that of an RPE degeneration that begins in the peri phery and is characterized by scalloped areas of RPE loss with eventual loss of choriocapillaris and medium-sized choroi dal vessels. Recent research has suggested that the ornithine aminotransferase gene is located on chromosome 10; therefore, abnormalities on chromosome 10 may result in gyrate atrophy. R36 ERG amplitudes in carriers of choroideremia are typically normal, unlike car riers of juvenile X-linked retinitis

pigmentosa. However, female carriers of choroideremia do show midperipheral pigm entary changes and choroidal atrophy. R37 Hermansky-Pudlak syndrome and Chediak-Higashi syndrome are both potentially lethal autosomal recessive diseases that present with albinism. Hermansky-Pudlak syndrome has abnormal plat elets that may lead to a bleeding diathesis. In Chediak-Higashi syndrome, a disorder in microtubule formation resu lts in leukocytes that cannot release enzymes from lysosomes. This disorder increases the risk of recurrent pyogenic i nfections. R38 The highest risk for retinal detachment is Retinal detachment in the fellow eye. Although myopia, lattice degeneration, and a family history of retinal detachments are important factors and are associated with higher-thannormal risk of retinal detachment, retinal detachment in the fellow eye increase s the chance of retinal detachment to approximately 10% to 15%. Myopia, especially moderate myopia, is the next highes t risk factor (7% to 8%), followed by family history and lattice degeneration. R39 Patients with a giant retinal tear in one eye are at very high risk of retin al detachment in the fellow eye about 50%. R40 Retinal tears are much more frequently located between the 10- and 2-o'clock meridians when associated with a posterior vitreous detachment. Nearly 70% of tears occur within these meridians. R41 The most important factor that determines visual outcome in the presence of a retinal detachment is whether the macula is attached or detached at the time of presentation. The duration of macu lar detachment may also play a role in determining final visual outcome, especially if the macula has been detached for less than 24 hours. Vitreous hemorrhage and pigment cells in the vitreous increase the risk of proliferative vitreoretinopathy, but these are not factors in determining visual outcome. R42 Proliferative vitreoretinopathy is the most common reason for failure of scl eral buckling surgery. A 5% to 10% chance of failure of primary scleral buckling exists in cases of rhegmatogenous retinal detachment. R43 Intraocular gases can provide tamponade of retinal breaks. The longest lasti ng gas is C3F8 in which over 50% of the gas will still be present in 3 weeks. R44 SF6 has the highest (fastest) expansile rate for any intraocular gas. It can cause dramatic increases in IOP in the early postoperative period. R45 Surface tension of silicone oil is significantly less than the surface tensi on of all gases, including air. R46 Silicone oil is buoyant when placed in a fluid-filled eye. The specific grav ity of gases, such as perfluoro-octane, is much less than that of water or any liquid.

R47 Multiple evanescent white dot syndromes: This disorder typically occurs in f emales in the third and fourth decades of life. It is often associated with symptoms of shimmering photopsias a nd paracentral scotomas. Some patients may have a flu-like illness preceding their symptoms. These scotomas of ten correspond to an enlarged physiologic blind spot on visual field testing. The disorder is typically self-l imited, with resolution occurring between 3 and 10 weeks after onset. Usually, no treatment is necessary. However, patients may have decreased vision despite resolution of the syndrome as a result of pigmentary changes in the fovea that h ave a characteristic granular appearance. In addition, 10% to 15% of patients may have recurrent episodes in t he same eye or in the fellow eye. R48 In cases of cone dystrophy, electrophysiologic findings suggest a diminished cone ERG response. R49 A hot spot is a well-defined small bright area that appears within 3 to 5 mi nutes and lasts for 20 minutes but loses definition late. Stippled hyperfluorescence is a term used to describe a feature of occult CNVM seen on fluorescein angiography, NOT on ICG. Well-defined hyperfluorescence is an area of homogenous fluorescence with well-defined, easily determined borders that appears within 5 minutes, lasts for 15 minutes, a nd fades thereafter. Hypofluorescence represents areas of diminished fluorescence either caused by a blockage defect o r by some insult to the integrity of the choriocapillaris in that area. R50 Louis-Bar, or ataxia-telangiectasia, does not have characteristic retinal ch anges. 4) PRETEST.iv A. NEURO-OPHTHALMOLOGY. P1 A patient with trochlear fourth nerve palsy who looks binocularly at a horizo ntal line will perceive two lines. The two images will not be parallel; one will be intorted, forming a V pointing towa rd the side of the lesion. The normal eye will perceive a horizontal line. The paralyzed eye is hypertropic and extorted; therefore, its image will be lower and intorted. P2 An RAPD associated with normal visual field is most likely to be seen in comp lete destruction of one brachium of the superior colliculus (contralateral RAPD). P3 Retrobulbar pain radiating to a tender spot behind his ear for a long period, the diagnosis is greater occipital neuralgia. P4 Syphilis may cause third nerve palsy. Myasthenia always spares the pupil. P5 Parinaud's syndrome has accommodative spasm and skew deviation. P6 Binasal visual defects are most frequently due to glaucoma. P7 Causes of downbeat nystagmus include spinocerebellar degeneration, cervicomed ullary junction lesions, and alcohol intoxication. P8 VER. a. AION characterized by low amplitude but occurs at normal latency. b. Acute optic neuritis characterized by low amplitude and increased latency. c. Old optic neuritis characterized by latency remains increased but amplitude i s often normal. B. PEDIATRIC OPHTHALMOLOGY AND STRABISMUS. P1 Treatment of consecutive esotropia includes prisms, hyperopic correction, mio tics, possibly reoperation, and Botulinum postoperatively. C. ORBIT, EYELID AND LACRIMAL SYSTEM.

P1 Pain with mouth opening and mastication is a sign of zygomaticomaxillary trip od fracture. P2 Tissue divided during lateral canthotomy includes skin and lateral raphe of t he orbicularis oculi. P3 Blepharochalasis onset is between 10-18 years of age, it is due to developmen tal deficiency of in eyelid elastic tissue. It is associated with Ehlers-Danlos syndrome. P4 Bacitracin is a bacteriocidal and effective against gram positive bacteria. P5 Although rare, Phlyctenulosis may lead to corneal perforation. P6 Pseudodendrite has been reported as a complication of contact lens wear. P7 Surgical intervention in hyphaema. Earlier intervention is required in patien t with preexisting glaucomatous optic nerve changes and those with sickle cell disease. P8 Patient's age are important in radial keratotomy, older patient gets more cor rection. P9 Spread of mucormycosis to the cranial cavity via the ophthalmic artery. P10 Partial obstruction of the nasolacrimal duct may resolve successfully with a ntibiotic and steroid. D. UVEITIS. P1 TOXOCARIASIS. The worm doesnot complete its cycle in humans, rendering stool examination unnecessary. Ocular inflammation is due to the reaction to the dead worm, so steroid, but not antihelminthics, may be helpful. P2 Intraoperatively, posterior Synechiolysis and removal of cyclitic membranes w ith vitrectomy may be necessary. P3 SO= HLA A11. Macular histoplasmosis= HLA B7. Posner Schlossman HLA Dw54. E. GLAUCOMA. P1 10% of patients don t respond initially to topical timolol treatment. P2 Ocular hypertension is present in 5-10% of the general population. P3 Carotid stenosis. It may produce florid RI with a surprisingly normal IOP pro bably caused by ciliary hypoperfusion. P4 Lens particle glaucoma usually seen after cataract extraction when residual c ortical material is left in the eye. P5 A contact lens with a convex button is useful when performing a laser iridoto my because it does: a. Reduce corneal burn.

b. Reduce eye movement. c. Act as a prism. d. Act as a heat sink. P6 Diarrhea has been reported with timolol. P7 Causes of hypotony after trabeculectomy include: a. Cyclodialysis. b. Wound leak. c. Overfiltration. d. Inflammation. P8 Bleb encapsulation appears as a tense, high, white bleb and is associated wit h elevation of IOP. Often with conservative management the bleb will function better over several months. The r esponse to needle revision is variable. P9 The following may block sclerostomy after trabeculectomy. a. Iris. b. Fibrin. c. Vitreous. d. Ciliary process. F. THE LENS AND CATARACT. P1 Corneal ulceration and melting is a described complication following cataract surgery in patients with KCS and RA. P2 Metabolic conditions associated with cataract in infants and children include hypoglycemia and hypocalcemia. P3 Honan balloon. Its use may avoid expulsive choroidal hemorrhage. A pressure o f 30 mmHg for 10-15 min is sufficient. Bradycardia is a complication. P4 Nonabsorbable sutures include silk, polyester (Mersilene), nylon and polyprop ylene (Prolene). Nylon nonabsorbable monofilament is the suture material of choice for wound closure because of its t ensile strength, elasticity, and minimal tissue reaction. Although defined as non-absorbable, it may undergo degradation over long periods. Both polyester and polypropylene are better defined as nonabsorbable; they are used as sutures on a permanent basis, including suturing of PCIOL. P5 Concomitant Late fluorescein staining of the optic nerve is the distinguishin g features of CME associated with cataract surgery than other causes of CME. P6 Causes of long axial length. a. Posterior staphyloma. b. Excess fluid between probe and the cornea. c. Measurement of scleral rather than retinal spikes. d. Ultrasonic velocity that is too fast. P7 Causes of short axial length. a. Excessive pressure on the cornea. b. Choroidal thickening or effusion and vitreous opacity. G. POSTERIOR SEGMENT. P1 Cherry red spot is present in both CRAO and OAO. Prolonged choroidal filling on FA is present in OAO. The RPE is undisturbed in a pure CRAO, but it develops pigmentary changes after the ischemi c insults of an OAO. P2 Insertion of intraocular lens during surgery does not affect the incidence of CMO. P3 The cystic retinal tuft is a second only to lattice degeneration as a visible peripheral retinal lesion associated with RRD. P4 RD is present in about 1% of patients with lattice. P5 AAO. Cystic and zonular traction retinal tufts, both with firm vitreoretinal adhesions, may predispose to retinal detachment.

P6 Ophthalmomyiasis causing criss crossing and railroad tracking in the retina. P7 NVI may occur following nonischaemic CRVO (5% of cases). P8 Complications of PRP includes: a. Exudative RD. b. Permanent posttreatment mydriasis. c. VH. d. Permanent lens opacities. P9 Infectious endophthalmitis. National eye trauma registry. a. The removal of IOFB within 24 hour of penetrating injury was shown to signifi cantly reduce the incidence. b. Increased risk in patient older than 50 years of age, particularly with delay in surgery. c. This study shows no significant different in risk with composition of an IOFB . d. Positive culture was found in approximately two third of cases. 5) 1500.v A. NEURO-OPHTHALMOLOGY. 1. Giant cell arteritis is associated with amaurosis fugax and usually not TVOs. 2. Kanski. Transient visual obscurations last only a few seconds and are charact erized by a 'greying out' or 'darkening' of vision in one or both eyes. They classically occur in patients wi th papilloedema and are often

precipitated by changes in posture. They may also precede anterior ischaemic opt ic neuropathy in patients with giant cell arteritis. 3. AAO. Transient visual obscurations, episodes of unilateral or bilateral visua l loss lasting seconds and often described as "grayouts," "whiteouts," "smokiness or fogginess," or "blackouts" o f vision. These obscurations often occur with orthostatic changes and may be associated with headache, nausea, and vomiting. 4. AAO. Patients with orbital masses such as hemangioma or meningioma-especially those with an intraconal mass accompanied by disc swelling-may experience transient obscurations of vision in certain fields of gaze, especially downgaze. These obscurations presumably result from positional vascular obstruct ion. There is often a clue to orbital involvement (eg, proptosis, restriction in motility). 5. AAO. Amaurosis fugax ("fleeting blindness") denotes a subtype of transient mo nocular visual loss attributed to ischemia or vascular insufficiency. Amaurosis fugax is characterized as sudden, painless, temporary visual loss lasting 2-30 minutes, followed by complete recovery. Between episodes, ocular ex aminations reveal normal anatomy or abnormalities confined to the retinal vasculature. 6. AAO. With amaurosis fugax, the patient typically reports that a curtain of da rkness descends over one eye, resulting in loss of vision lasting 2-30 minutes. In patients with some residual circulati on from a retinal or cilioretinal artery, the curtain may extend only partially. At resolution, the curtain may either ascend or dissolve like a clearing fog. 7. Papilloedema, optic disc drusen, and vertebral basilar insufficiency are asso ciated with TVOs. 8. The VER is mostly a measure of macular function. Correct refraction must be w orn during the pattern VER test. The VER may be performed on infants 6 months or older. 9. The menace test is the least useful test for suspected functional visual loss . This test is moving an object quickly toward the patient's eye in an attempt to cause functional patient to blink. Thi s test is easily invalidated because air movement may induce the corneal reflex. 10. The ganglion cells at the retina synapse at the lateral geniculate body at t he thalamus. 11. Acute papilloedema does not have a RAPD, because the effect on the optic ner ve usually is symmetric. 12. Typically, the earliest loss of visual field loss in chronic papilloedema is inferior nasal field defect. 13. Pseudotumour cerebri has been associated with pregnancy but not birth contro l pills. 14. Temporal artery biopsy specimen remains histopathologically positive while a patient is being treated with high dose corticosteroids for 14 days. 15. Optic disc drusen does not contain amyloid but have shown to contain amino a cid, mucopolysaccharides, calcium, and hemosiderin. 16. Optic disc drusen. Subretinal neovascular membrane may occur. Loss of vision is extremely rare. There is no retinal folds associated with drusen. Peripapillary retinal folds ar e associated with disc oedema and known as Paten's lines. 17. Anomia is inability to name objects. Agnosia is the inability to recognize o

bjects by sight, though one may be able to recognize object by touch. Prosopagnosia is the inability to recognize f aces. Palinopsia is a persistent visual image after the image is removed. Alexia is the inability to read despite normal vision. 18. Alexia with agraphia occurs with left parietal lobe involving the angular gy rus. Alexia without agraphia occurs with left occipital lobe and the splenium of corpus callosum. 19. The major cause of death in patients with retinal emboli is not stroke but c ardiovascular disease (MI). 20. Superior oblique myokymia has not been associated with any disease of the ne rvous system. It may be associated with blurred vision. 21. The lesion in congenital apraxia is unknown, whereas the lesion in acquired apraxia localizes to the frontoparietal cortices. 22. The fast phase is opposite the lesion in peripheral nystagmus, and unidirect ional or bidirectional in central nystagmus. Vertical nystagmus is never present in peripheral nystagmus. 23. Any lesion distal to the geniculate ganglia will not affect the function of the lacrimal nerve. 24. Percentage of Bell's palsy completely recover in 3 months is 75%. 25. Bell's palsy is idiopathic. Aberrant regeneration is common in patients who don t recover. Impairment of lacrimation, dysacousis, and advanced age are poor prognostic signs for complete recovery. 26. Milkersson-Rosenthal syndrome is recurrent facial paralysis is accompanied b y chronic facial swelling and lingua plicata (furrowing of the tongue). The etiology is unknown. 27. Carbamazepine and phenytoin usually are beneficial in facial myokymia but no t in hemifacial spasm. 28. Deep-tendon reflexes are absent in 90% of cases in Adie's pupil. Adie' pupil is associated with herpes zoster. 29. MS. Peripheral nerves are not affected. At 10 years follow up, a general rul e of thumb is one third is dead, one third is disabled, and one third suffers no disabilities. 30. MG and CPEO are least likely to be associated with positive forced duction t est unless chronic. B. . PEDIATRICS PEDIATRICS 1. Congenital glaucoma associated with NF. Lisch's nodules develop less frequent ly in the eye involved with glaucoma. 2. Congenital open angle glaucoma is the most likely to respond to goniotomy.

3. Features of aniridia are small cornea. Typically the angle is normal during i nfancy but becomes progressively obstructed with age. The glaucoma associated with aniridia typically occurs in t he second decade of life; its occurrence at birth or during infancy is unusual. 4. Fabry's disease is associated with deficiency of the alpha-galactosidase. 5. ROP. It is unlikely that ROP will be detected as early as 2 weeks old. Typica lly, ROP is seen at between 3 and 5 weeks of age. 6. Thalidomide has been associated with microphthalmos with coloboma. LSD associ ated with microphthalmos without coloboma. 7. Gonococci and herpes simplex virus has been associated with corneal ulceratio n and perforation. 8. An orbital abscess requires prompt surgical drainage. 9. Waardenburg's syndrome associated with hypochromic iris heterochromia. 10. Pierre-Robin syndrome associated with high myopia and megalocornea. 11. Seizures are associated with tuberous sclerosis, Wyburn-Mason syndrome, and Sturge-Weber syndrome but not with NF. 12. Tuberous sclerosis. 50% associated with mental retardation. Reduced vision o ccurs because of vitreous haemorrhage and serous retinal detachment. 13. Wyburn-Mason syndrome. The retinal AVMs are non-pulsatile. The intracranial AVMs are most common in the midbrain and may cause obstructive hydrocephalus and papilledema. There may be involvement of the sinuses, resulting in severe epistaxis. 14. The most common site of metastasis of retinoblastoma is the bone marrow. 15. Enzymatic assay helpful in the diagnosis of retinoblastoma is lactic acid de hydrogenase. 16. Trilateral retinoblastoma carries the highest mortality despite of treatment . Metastatic retinoblastoma once was uniformly fatal, but chemotherapy has resulted in long term survival. 17. Treatment of retinoblastoma. Following external beam radiotherapy, the tumou r may fail to continue to grow; this is considered a success for radiotherapy, whereas the other modalities requ ire regression of the tumour to be considered successful. 18. The most common manifestation of child abuse is retinal hemorrhage. 19. It is extremely uncommon to see retinal hemorrhage from head trauma in an in fant. 20. Coat's disease, typically there is no racial, familial or genetic predisposi tion. 21. Lowe's syndrome has miosed pupil that will not dilate. 22. Dulcitol is the substance that is accumulates in the lens of a patient with galactosemia. 23. Patient with achromatopsia have normal pupillary reaction, normal scotopic b ut an abnormal photopic ERG. 24. In congenital retinoschisis, the b wave is markedly reduced as is the b/a wa ve ratio. 25. Source of vitreous hemorrhage in congenital retinoschisis are neovasculariza tion of the retina, stretched retinal vessels crossing the inner and outer layer breaks. 26. Mechanism of RD in retinoschisis. There must be a hole in both inner and out er layer of the retina to allow fluid to get underneath the retina. C. . STRABISMUS STRABISMUS

1. Whenever there is fusion in the presence of strabismus, then harmonious ARC i s present. 2. In after-image crossed image indicate esotropia, whereas uncrossed image indi cate exotropia. 3. The medial and lateral rectus muscles have three antagonists each, whereas th e other muscles have only the two. 4. Convergence insufficiency should not have exotropia. Accommodative spasm is a sign of Convergence insufficiency. 5. V pattern is present in inferior oblique overaction but not in DVD. 6. Bilateral superior oblique tenotomies produce up to 35-45 prism diopters of a n eso shift in downgaze. 7. Deafness and eight nerve palsy is not a features of Mobius syndrome. D. . OCULOPLASTICS OCULOPLASTICS 1. The volume of the orbit is 30 cc. the entrance height is less than the entran ce width. 2. The average diameter of the optic foramina is about 5.5 mm. Greater than 1 mm of asymmetry between the two eyes also is considered abnormal. Possible lesions include glioma and optic nerve she ath meningiomas. 3. Indications of treatment of child of a capillary hemangioma are amblyopia, st rabismus, and anisometropia. Severe disfigurement is not an absolute indication. 4. Rhabdomyosarcoma most commonly located superonasally with the exception of al veolar types (inferiorly located). 5. Strap cells are associated with rhabdomyosarcoma. 6. Capillary hemangioma also may change with position or Valsalva maneuvers, but orbital varix is the most classic. 7. Lacrimal gland lesions 50% are epithelial tumour and 50% are inflammatory. 8. Approximately 50% of epithelial tumors are benign mixed tumors (pleomorphic a denomas) and about 50% are carcinomas. Approximately half of the carcinomas are adenoid cystic tumors, and the remainders are composed of malignant mixed tumors, primary adenocarcinomas, mucoepidermoid carcinomas, and squamous carcinomas. 9. Malignant lymphoma is much more likely to have a monoclonal proliferation, wh ereas benign lymphoid proliferation is likely to have a polyclonal proliferation. 10. The nerve that supplies sensation to the medial canthal area is the nasocili ary nerve.

11. Wei's procedure was described first for cicatricial entropion repair, but it also has application in involutional entropion repair. 12. It is more common to overcorrect an acquired ptosis than a congenital ptosis . Congenital ptosis repair usually remains undercorrected. 13. Destruction of the meibomian gland orifices is seen only with malignant neop lasm. 14. Acanthosis nigricans is seen primarily in African Americans. They may be sec ondary to obesity, endocrinopathy, or an internal malignancy. They also may be hereditary. 15. Basal cell carcinoma occurs more commonly on the lower eyelid near the media l canthus, possibly because of reflected sunlight from the nose. 16. Lid sharing procedure should be avoided in children because of the risk of a mblyopia. 17. The effect of botulinum toxin injection in EBS is 1-3 months. 18. 33% of the NLD obstruction is bilateral. 19. Floppy eyelid syndrome is associated with chronic papillary conjunctivitis. E. . CORNEA CORNEA 1. Herpes zoster ophthalmicus may cause Horner syndrome. 2. Iris atrophy and profound corneal anaesthesia are common in herpes zoster. 3. Interstitial keratitis is a response to treponemal antigens and not live orga nisms. 4. Thygeson superficial punctate keratitis stains with fluorescein and rose Beng al. 5. Necrotizing scleritis with inflammation is associated with high risk of morta lity because of an associated autoimmune disease and complications of vasculitis. 6. Diffuse anterior scleritis is the most benign form of scleritis. 7. Corneal intraepithelial neoplasia may metastasize if it invades the trabecula r meshwork. Local invasion is more common than metastasis. 8. Most eye bank don t set an upper limit on donor age but make the decision based on the overall quality of the tissue and leave the ultimate decision to the surgeon. 9. The use of infant donor cornea is best in patient with aphakic bullous kerato pathy, because infant cornea is flaccid, it is usually lead to a myopic correction and may best be used aphakic conditions a s an alternative to secondary IOL implantation. 10. Primary graft failure. Corticosteroids typically don t help but should be trie d before immediate regrafting. 11. Interrupted sutures only in keratoplasty are best performed for vascularized cornea. 12. Allograft rejection rarely occurs before 2 weeks but may occur as late as 20 years after transplantation. 13. Increased corneal thickness is an early sign of allograft rejection. 14. Conjunctival flap don t reform the AC because the fluid percolates beneath the flap in the case of corneal perforation. 15. Giant papillae are more specific than are small papillae. 16. Sattler's veils typically result from intracellular epithelial oedema. 17. Corynebacterium typically arranged in Chinese-letter formation. 18. Blood agar enriched with vitamin K is specific for Actinomyces. 19. Blood agar enriched with NAD is specific for Haemophilus. 20. Canada= candida. Filamentous= tropical= hot climate. Yeast= temperate. 21. Topical antiviral. Most toxic is idoxuridine. Least toxic acyclovir.

22. Viral papillomas usually are seen on the palpebral conjunctiva, whereas noni nfectious papillomas are seen on the bulbar conjunctiva. 23. GPC is more prevalent with soft contact lens followed by gas-permeable and t hen PMMA lenses. 24. The epithelium over the phlyctenulosis begins to break down and may, in rare cases, results in corneal perforation. 25. Adult chlamydial conjunctivitis does not have a conjunctival membrane, where as adenoviral conjunctivitis does. 26. Posterior embryotoxon is seen in 8-30% of normal eyes and is inherited in do minant trait. 27. In von Hippel's corneal ulcer, the iris adhesions are extensive and arise fr om the stroma as well as the collarette, whereas Peters' anomaly has iris adhesions from the iris collarettes only. 28. STUMPED: sclerocornea, Trauma (forceps delivery), Ulcer, Metabolic, Peter's anomaly, Endothelial dystrophy (CHED), Dermoid. Congenital glaucoma is included. 29. Arcus juvenilis (a congenital anomaly) is not associated with serum lipid ab normalities. It is usually involves only the sector of the peripheral cornea. 30. Terrien's marginal degeneration leaves intact epithelium whereas Mooren ulce r involves epithelium. 31. 10% of patient with map dot fingerprint dystrophy will have recurrent cornea l erosion. 50% of patient with recurrent corneal erosion will have map dot dystrophy. 32. Meesmann's dystrophy is not associated with recurrent epithelial erosions. 33. HLA B8 is associated with primary Sj.gren syndrome. 34. The acute phase of Steven Jonson syndrome last approximately for 6 weeks.

35. Fabry's disease as opposed to other Sphingolipidoses, the retina and CNS are normal. The female carriers show cornea verticillata only and are asymptomatic. 36. The K-F ring has multiple appearance, may appear brown, red or green. The di sease is diagnosed by decreased level of ceruloplasmin. F. . UVEITIS UVEITIS 1. JXG need no treatment, regress spontaneously. 2. Lack of KPs and lack of posterior synechiae are characteristic of phacolytic glaucoma, as it is not a true uveitis but glaucoma secondary to a clogged TM from leaky lens material. 3. CMO rarely occurs with Fuchs heterochromic iridocyclitis. 4. Sarcoidosis may present as a granulomatous or nongranulomatous disease. 5. Syphilitic uveitis diagnosed only with positive serology. Periarterial sheath ing and papillitis is a common finding. 6. Lyme disease. Keratitis is present in the late stages of the disease. It is p resent with follicular conjunctivitis. 7. Photophobia, pain, and red eyes typically absent in pars planitis. 8. Postoperative endophthalmitis incidence is 0.1-0.4%. Posttraumatic endophthal mitis incidence is 3-8%. 9. Previous intraocular infection in the fellow eye does not predispose the pati ent to a second case of endophthalmitis. 10. Severe postoperative endophthalmitis usually present within 1-4 days. 11. In most of the cases of fungal endogenous endophthalmitis, there is no evide nce of fungemia. Candida endophthalmitis is rare in AIDS. 12. Ivermectin is the treatment of choice for onchocerciasis. Cause of blindness is glaucoma, cataract, and optic atrophy. 13. Vision and ERG usually are not affected by the classic pigmentary retinopath y seen in rubella retinitis. Retinal blood vessels are normal. It is present in up to half of infants with maternal r ubella syndrome. 14. Thiabendazole is the treatment of systemic toxocariasis; it has not been eff ective for ocular toxocariasis. 15. Early signs of SO includes near vision is affected early, mild photophobia, red eye and mild vitritis. 16. The skin and hair findings in VKH syndrome usually occur weeks to months aft er the ocular inflammation. 17. SO is associated with HLA A11. 18. Cotton wool spot in AIDS. It is believed that the retina is infected with HI V but not cytomegalovirus. 19. Behçet disease is type IV HSR. Idiopathic uveitis is type III HSR. Mooren's ul cer is type II HSR. Giant papilla is type I HSR. 20. Chronic uveitis is inflammation lasting longer than 3 months. 21. The conjunctivitis in Kawasaki disease spares the limbus and has no discharg e. G. GLAUCOMA. 1. Complete turnover of the AC takes about 100 minutes. 2. The prelaminar and laminar optic nerve is supplied by short posterior ciliary artery. 3. Up to 40-50% of the optic nerve axon may be destroyed before the Goldmann vis ual field becomes abnormal. Kanski 20%. 4. Glaucomatous cupping is shallower in myopic eyes. 5. The fellow eye of angle closure glaucoma. Pilocarpine is a good choice until

laser iridotomy can be accomplished. The risk of bilaterality is 50% within 5 years. 6. In Chandler's syndrome, the corneal endothelium is dysfunctional and may caus e corneal oedema at normal IOP. 7. The best treatment for patient with malignant glaucoma 2 days after cataract surgery is disruption of the vitreous with the Nd: YAG laser. 8. Mechanisms of raise IOP after scleral buckling. a. Placement of the encircling band over the vortex veins. b. Shallowing of the AC angle by the buckling effect. c. Anterior rotation of the ciliary body. 9. Too much fluorescein, vertical misalignment, and increased corneal thickness will result in high tonometry readings. 10. With the rule astigmatism will result in low tonometry readings. 11. Increased scleral thickness does not affect applanation tonometry. 12. The most commonly presenting symptoms in iridocorneal endothelial syndrome a re iris abnormalities, blurred vision, and pain. 13. Oral Furosemide has not been noted to have an effect on IOP. 14. Oral digoxin (decrease aqueous production by 45%), propranolol, timolol, and Nadolol decrease IOP. 15. The treatment of ghost cell glaucoma is initially medically, followed by an AC washout or possible vitrectomy to clear remaining degenerated RBC. 16. Cautions to CAIs. Those with chronic liver failure are at risk of increased serum ammonia. Hypokalemia may worsen if the patient is concurrently on diuretics. DK acidosis may be worse by carbonic anhydrase inhibitors. Patients with COPD may retain carbon dioxide while on CAIs. 17. Most common cause of glaucoma in patient with retinoblastoma is neovasculari zation (73%). Most common cause of glaucoma in patient with malignant melanoma is neovascularizatio n.

18. Betaxolol has an additive effect with epinephrine. Betaxolol has 10 times gr eater b1 blocker than b2 blocker. Therefore, epinephrine's stimulation of b2 receptors is not blocked. 19. Iridotomy is not helpful for malignant glaucoma but it may be helpful for di stinguishing malignant glaucoma from pupillary block. H. . LENS AND TRAUMA LENS AND TRAUMA 1. Posterior lenticonus is seen in Lowe syndrome. 2. Zimmermann tumour is a phakomatous choristoma. 3. Alport's syndrome. The most likely to have a spontaneous rupture of the anter ior lens capsule. The area of the capsule involved in the anterior lenticonus is extremely thin and fragile. 4. Chlorpromazine is associated with pigmented anterior subcapsular cataract. 5. Zones of inflammation in phacoanaphylactic endophthalmitis from inner adjacen t to the lens, 1st PMNs, 2nd epithelioid cells giant cells and granulation tissue, the outermost zone consist of lymphocytes, plasma cells, and some eosinophils. 6. Trauma is associated with 50% of cases of lens dislocation. 7. Projection light test may be helpful in detecting an occult RD but is not sen sitive enough for detecting macular disease from cataract. I. . RETINA RETINA 1. Retinal neovascularization occur with ischemic event, retinal neovascularizat ion also is seen in inflammation (sarcoidosis) and intraocular tumor. 2. Diffuse confluent drusen material is located in inner collagenous layer of Br uch membrane. 3. AMD is diagnosed only when visual loss develops. With disciform scar in one e ye, the risk for CNV in the other eye is 20%. 4. According to MPS study. Extrafoveal CNV (>200 micrometer from center of the f ovea) will experience severe visual loss (>6 lines). Untreated 60%. Treated 25%. 5. Epiretinal membrane. Vitrectomy is reserved for eyes 20/60 or worse. 6. Macular hole are more common in women. Bilateral in 25-30% of cases. 7. BRVO is more common in hyperopia. 8. The incidence of NG after ischemic CRVO is 33%; the incidence of RI after isc hemic CRVO is 60%. 9. Causes of CME but no leakage include: a. Goldmann-Favre syndrome. b. Juvenile retinoschisis. c. Usher syndrome. d. Nicotinic-acid maculopathy. 10. The peak incidence of Irvin-Gass syndrome is 6-10 weeks. 11. Treatment of cavernous haemangioma is not indicated unless recurrent vitreou s hemorrhage occurs. 12. All mucopolysaccharidoses is AR except Hunter is X-linked recessive. Hunter uses a cross (X) bow. 13. Patient with a symptomatic PVD without a retinal break should be reexamined in a few weeks, because a retinal tear may take some time to develop. 14. Patient with a symptomatic PVD without an obvious retinal break but with vit reous hemorrhage should be followed closely. Daily examination may be overkill but this is the best choice provided. 15. Lesions predisposing to a RD are lattice degeneration, vitreoretinal tufts,

and meridional folds. 16. The RDs with the best prognosis are retinal dialysis or small holes, detachm ent with demarcation line and detachment with minimal subretinal fluid. 17. Stickler's syndrome has hyperextendable joint. 18. Percentage of eyes of diabetic patients undergoing vitrectomy for nonclearin g vitreous hemorrhage has a final visual acuity of no light perception is 20%. 19. Features of occult scleral rupture include: a. Decreased VA. b. Marked decrease in duction. c. Conjunctival chemosis. d. Severe vitreous hemorrhage. e. Deep AC due to vitreous collapse. 20. Corneal laceration may be deep or shallow AC. 21. Mizuo-Nakamura effect have increased incidence of RD. 22. ERG, b-wave is larger in scotopic than photopic ERG. Oscillatory potential i s located with the b-wave (regarded as part of the b-wave). The oscillatory potential is generated by bipo lar cells. 23. Only about 7% of cones are in the macula; the rest are distributed about the fundus. 24. Acquired optic nerve disease tend to cause red green defect except glaucoma and AD optic neuropathy which is initially cause blue yellow. 25. Acquired retinal disease tends to cause blue-yellow defects except cone dyst rophy and stargardt's disease which cause predominantly red green defect.

26. MEWDS and birdshot occur more commonly in women than men. 27. MEWDS and APMPPE both usually are preceded by a viral illness. 28. Most common association with angioid streak is the Pseudoxanthoma Elasticum. 29. Thioridazine is associated with pigmentary retinopathy. 30. Gaucher's, Krabbe's and Fabry's disease (among others) don t have a macular ch erry-red spot. Gaucher's disease have a normal fundus, Krabbe's have optic atrophy, and Fabry's disease h as a tortuous retinal vessels. 31. Cavernous haemangioma classically grow faster during pregnancy. 32. 25% of patient with AS will develop uveitis. 33. A positive angle kappa simulates an exotropia or makes an esotropia appear o rthophoric. 34. Opsoclonus seen most commonly with postviral encephalopathic syndrome. It is also seen with neuroblastoma in children and visceral carcinoma in adults. 35. Collier's sign is a lid retraction that worsens with upgaze and improves wit h downgaze. 36. There is an increased risk of malignant melanoma in patient with neurofibrom atosis. 37. Wyburn-Mason syndrome classically has a retinal AVM and a midbrain AVM. The midbrain AVM may cause Parinaud's dorsal midbrain syndrome. 38. The incidence of disc drusen is 20-50 times greater in patients with pseudox anthoma elasticum than in normal individuals. 39. Posner-Schlossman has been reported to occur after relief of emotional stres s. 6) Board review. 1. DDX of gaze evoked TVL are orbital cavernous haemangioma and optic nerve shea th meningioma. Other orbital lesion may cause it. 2. Indication for treatment of partial accommodative esotropia is greater than 1 0 PD; surgery is indicated to reduce the deviation to enhance the development of binocular vision. 3. Three most common causes of unilateral cataract are PFV, trauma and lenticonu s. 4. Anterior ciliary vessels do not re-establish perfusion to the anterior segmen t after removing a rectus muscle during strabismus surgery. 5. DDX of pulsatile proptosis after blunt head/orbit trauma include traumatic ar terio-venous fistula and transmitted meningeal arterial pulses with orbit roof fractures. 7) PROVISION. 1. The incidence of postoperative IOP rise after second trabeculoplasty is highe r than with the initial trabeculoplasty. 2. Topical steroid is ineffective in the treatment of SLK. 3. Characteristic features distinguishing adult inclusion conjunctivitis (AIC) f rom adenoviral KC is the presence of micropannus which may extend 1 to 3 mm from the limbus into the superior cornea in AIC. Adenoviral disease is not characterized by corneal neovascularization. Subepithelial infiltrate that occur in AIC occur in the superior cornea. 4. Pupil size 4.5 mm or less carries 5 times risk of vitreous loss than the pupi l size more than 4.5 mm. 8) Other. 1. Contraceptive pill may cause poor tear secretion and interfere with contact l ens wear. Rifampicin causes discoloration of secretion including the tear which may stain the contact lens.

2. Isografts refer to transplant between two identical twins. 3. Indications of drainage of SRF. a. Difficulty in localization of breaks in bullous detachments, particularly if the breaks are post-equatorial. b. Immobile retina (e.g. PVR) because a non-drainage procedure will be successfu l only if the detached retina is sufficiently mobile to move back against the buckle during the postoperative per iod. c. Long-standing RD because the SRF is viscid and may take months to absorb. Dra inage may therefore be necessary, even if the break can be closed without it. d. Inferior RD associated with equatorial tears should ideally be drained becaus e, when the patient assumes the upright position postoperatively, any residual SRF will gravitate inferiorly and may reopen the break. e. Deep SRF beneath the retinal break. f. Intraocular pressure increases due to displaced volume from the buckling effe ct, fish-mouthing of large retinal breaks, and bullous retinal detachments. 4. Intravitreal gas injection: C3F8 has a longer half-life than SF6 and also mor e expansile. C3F8 typically expands to 4 times its volume whereas SF6 only 2 to 3 times and therefore C3F8 also causes mo re significant increase in intraocular pressure than SF6. Intravitreal gas injection can cause retinal tear and catarac t. 5. Argon laser trabeculoplasty: a. High power can lead to significant inflammation and anterior synechiae. b. Better in those over 50 years of age. c. Reduced pressure is due to alternation of the aqueous outflow. d. Produces a drop of pressure of between 5 to 10 mmHg. e. Used when the patient is not responsive to maximal medical therapy for glauco ma. 6. Deafness and pigmentary retinopathy are seen in the following conditions: a. Congenital syphilis. b. Congenital rubella.

i lamkin ii AAO iii review iv pretest v 1500 c. d. e. f. g. h. i. Usher's syndrome. Cockayne's syndrome. Alstroms syndrome. Leber's amaurosis. Alport's syndrome. Hunter's syndrome (MP II). San Fillipo's disease (MP III).

Sponsor Documents

Or use your account on DocShare.tips

Hide

Forgot your password?

Or register your new account on DocShare.tips

Hide

Lost your password? Please enter your email address. You will receive a link to create a new password.

Back to log-in

Close